模块 | 周 | 年级 | 问题 | 答案 | 正确答案 |
---|---|---|---|---|---|
2 | 1 | 2007 | 1. 下列哪一项关于d-tubocurarine的叙述是正确的?(2.0分) | A.是一种除极化型肌松药 B.可引起高血钾症 C.可促进运动神经末梢释放乙酰胆碱 D.过量可用新斯的明对抗 E.也可用于治疗支气管哮喘 |
D.过量可用新斯的明对抗 |
2 | 1 | 2007 | 2. Pilocarpine 对眼的作用 (2.0分) | A.扩瞳、降低眼内压、调节麻痹 B.缩瞳、降低眼内压、调节麻痹 C.缩瞳、升高眼内压、调节痉挛 D.扩瞳、升高眼内压、调节痉挛 E.缩瞳、降低眼内压、调节痉挛 |
E.缩瞳、降低眼内压、调节痉挛 |
2 | 1 | 2007 | 3. 下述哪一组织对neostigmine不敏感?(2.0分) | A.膀胱平滑肌 B.胃肠平滑肌 C.骨骼肌 D.中枢神经 E.以上都不是 |
D.中枢神经 |
2 | 1 | 2007 | 4. 关于有机磷酸酯类中毒的特点,下述哪一项是错误的? (2.0分) | A.形成磷酰化胆碱酯酶,使胆碱酯酶活性过强 B.形成磷酰化胆碱酯酶,使胆碱酯酶失活 C.严重中毒时可出现N样症状及中枢症状 D.应当及时抢救,以免使磷酰化胆碱酯酶"老化" E.抢救时应当合用atropine和pyraloxime methoiodide (PAM) |
A.形成磷酰化胆碱酯酶,使胆碱酯酶活性过强 |
2 | 1 | 2007 | 5. 治疗中、重度有机磷酸酯类中毒应当采用(2.0分) | A.Atropine B.Atroping + pyraloxime Methiodide(PAM) C.Atropine + neostigmine D.Atropine + scopolamine E.Pyraloxime methiodide(PAM) |
B.Atroping + pyraloxime Methiodide(PAM) |
2 | 1 | 2007 | 6. Duchenne's muscular dystrophy is a lethal childhood disease that involves skeletal muscle cells. Which of the following statements best describes the pathophysiology of this devastating disease? (2.0分) | A.Structural changes in the costomere results in damage to the muscle cell membrane (sarcolemma) during both concentric and eccentric contraction. B.Structural changes in the production of collagen disrupt the packaging of the muscle fibers and results in loss of connection with α-motor neurons at the neuromuscular junctions. C.Structural changes in the contractile proteins in the myocyte (muscle fiber) results in loss of the cell's ability to shorten and produce movement. D.Structural changes in the myocytes results in injured cells that are destroyed and replaced by mitosis and differentiation of satellite cells. E.Structural changes at the enthesis as a result of defective fibrocartilage insertions results in loss of range of motion of the muscle at the joint. |
A.Structural changes in the costomere results in damage to the muscle cell membrane (sarcolemma) during both concentric and eccentric contraction. |
2 | 1 | 2007 | 7. Both of Mary's sons have the X-linked disorder Duchenne muscular dystrophy. A deletion in the DMD gene is found in each of the boys, but no mutation in DMD is found when a blood sample from Mary is analyzed. What is the most likely explanation? (2.0分) | A.The phenotype is not penetrant in Mary B.The boys were affected by separate mutation events in DMD C.The mutation was inherited from the boys' father D.Mary is germline mosaic for a DMD deletion E.The phenotype is X-linked recessive |
D.Mary is germline mosaic for a DMD deletion |
2 | 1 | 2007 | 8. A newborn is noted to maintain a frog-leg position with a weak cry and minimal spontaneous movement. A muscular dystrophy is suspected, and a serum enzyme is measured that reflects the abundance of a high energy storage molecule in muscle. Which of the following enzymes was measured? (2.0分) | A.Creatine phosphokinase B.Adenosine triphosphatase C.Glucose-1-phosphate dehydrogenase D.Pyrophosphatase |
A.Creatine phosphokinase |
2 | 1 | 2007 | 9. 生理科学实验中观察和记录的生物信号的基本特性为 (2.0分) | A.振幅低、频率低、源阻抗大 B.振幅高、频率高、源阻抗大 C.振幅低、频率高、源阻抗大 D.振幅低、频率高、源阻抗小 |
A.振幅低、频率低、源阻抗大 |
2 | 1 | 2007 | 10. 关于时间常数,描述正确的是: (2.0分) | A.时间常数越大,下限截止频率越高,对低频成分的滤波程度越小。 B.时间常数越小,下限截止频率越高,对低频成分的滤波程度越大。 C.时间常数越小,下限截止频率越低,对低频成分的滤波程度越大。 D.时间常数越大,下限截止频率越低,对低频成分的滤波程度越大。 |
B.时间常数越小,下限截止频率越高,对低频成分的滤波程度越大。 |
2 | 1 | 2007 | 11. 时间常数 =0.1s时,高通滤波器的下限转折频率约为(2.0分) | A.FL = 1.60Hz B.FL = 0.10Hz C.FL=0.16Hz D.FL = 1.00Hz |
A.FL = 1.60Hz |
2 | 1 | 2007 | 12. 使用生物信号采集处理系统,下列参数设置或操作正确的是(2.0分) | A.记录蟾蜍坐骨神经AP,灵敏度和低通滤波分别设置为0.5V和30Hz B.动脉血压记录完毕后在保存数据前须进行零点设置 C.记录骨骼肌收缩张力时,耦合输入方式选择交流耦合 D.记录离体豚鼠回肠收缩张力时,时间常数设置为直流 |
D.记录离体豚鼠回肠收缩张力时,时间常数设置为直流 |
2 | 1 | 2007 | 13. 进行低温对小鼠耐受缺氧的影响实验,正确的分组方法是(2.0分) | A.随机挑选两只小鼠,一只作为对照组、一只作为实验组 B.将两只小鼠随机分配给对照组和实验组 C.随机挑选体重相近的两只小鼠,一只作为对照组、一只作为实验组 D.将两只体重相近性别相同的小鼠随机分配给对照组和实验组 |
D.将两只体重相近性别相同的小鼠随机分配给对照组和实验组 |
2 | 1 | 2007 | 14. 任氏液的理化特性与蟾蜍的(2.0分) | A.组织液相似 B.血液相似 C.体液相似 D.细胞内液相似 |
A.组织液相似 |
2 | 1 | 2007 | 15. Succinylcholine过量引起呼吸肌麻痹时,最有效的解救方法是 (2.0分) | A.注射neostigmine B.注射atropine C.注射nikethamide D.注射lobeline E.用人工呼吸机 |
E.用人工呼吸机 |
2 | 1 | 2007 | 16. A mutation causing an abnormality in presenilin that disrupts amyloid production or function has been associated with (2.0分) | A.Marfan syndrome B.Duchenne muscular dystrophy C.cystic fibrosis D.none of the above |
D.none of the above |
2 | 1 | 2007 | 17. 具有中枢抑制作用的M胆碱受体阻断药是(2.0分) | A.Scopolamine B. Atropine C.Anisodamine D.Propantheline E.Homatropine |
A.Scopolamine |
2 | 1 | 2007 | 18. 假设一个神经元的突触上同时表达AMPAR和与Gs偶联的mGluR,当突触前释放神经递质谷氨酸时,下列哪种神经元反应排序符合时间先后次序? (2.0分) | A.细胞内cAMP含量改变 跨膜电位改变 G蛋白激活 基因表达改变 B.跨膜电位改变 G蛋白激活 细胞内cAMP含量改变 基因表达改变 C.G蛋白激活 跨膜电位改变 细胞内cAMP含量改变 基因表达改变 D.G蛋白激活 细胞内cAMP含量改变 跨膜电位改变 基因表达改变 |
B.跨膜电位改变 G蛋白激活 细胞内cAMP含量改变 基因表达改变 |
2 | 1 | 2007 | 19. 受体或离子通道为跨膜蛋白质。下列哪些方法有助于确定跨膜蛋白质的膜拓扑学结构? (2.0分) | A.利用特异性抗体的免疫印迹检测(Western Blot) B.蛋白质氨基酸序列的疏水性分析(hydrophilicity plot) C.放射性同位素标记配体的结合试验(Binding Assay) D.膜片钳(Patch-clamp)电生理记录 |
B.蛋白质氨基酸序列的疏水性分析(hydrophilicity plot) |
2 | 1 | 2007 | 20. 电压门控离子通道离子选择性取决于: (2.0分) | A.电压感受器 B.组成通道的亚基数目 C.羧基端的磷酸化 D.选择性滤器 |
D.选择性滤器 |
2 | 1 | 2007 | 21. 在化学突触传递的特征中,错误的是(2.0分) | A.后放 B.总和 C.双向性传递 D.兴奋节律的改变 E.对内环境变化敏感 |
C.双向性传递 |
2 | 1 | 2007 | 22. 细胞外液中Ca2+浓度↓,对突触传递的影响是(2.0分) | A.突触前膜释放的递质↓ B.突触前轴突末梢动作电位的幅度↓ C.突触前膜释放的递质↑ D.突触前轴突末梢动作电位的幅度↑ |
A.突触前膜释放的递质↓ |
2 | 1 | 2007 | 23. EPSP属于(2.0分) | A.局部电位 B.AP C.终板电位 D.阈电位 |
A.局部电位 |
2 | 1 | 2007 | 24. 24.下列关于神经胶质细胞的描述,正确的是(2.0分) | A.与相邻细胞有突触联系 B.既有树突又有轴突 C.细胞间普遍存在缝隙连接 D.有随细胞外Na+浓度改变的膜电位 E.有产生动作电位的能力 |
C.细胞间普遍存在缝隙连接 |
2 | 1 | 2007 | 25. 神经冲动抵达末梢时,引起递质释放主要有赖于哪种离子的作用?(2.0分) | A.Ca2+ B.Cl‐ C.Mg2+ D.Na+ E.K+ |
A.Ca2+ |
2 | 1 | 2007 | 26. 在复合收缩时,肌肉的动作电位 (2.0分) | A.仍独立存在 B.发生复合 C.幅值变大 D.幅值变小 E.频率变低 |
A.仍独立存在 |
2 | 1 | 2007 | 27. 局部反应的时间总和是指(2.0分) | A.同一部位连续的阈下刺激引起的去极化反应的叠加 B.同一部位连续的阈上刺激引起的去极化反应的叠加 C.同一时间不同部位的阈上刺激引起的去极化反应的叠加 D.同一时间不同部位的阈下刺激引起的去极化反应的叠加 E.同一部位一个足够大的刺激引起的去极化反应 |
A.同一部位连续的阈下刺激引起的去极化反应的叠加 |
2 | 1 | 2007 | 28. 神经‐肌肉接头处的化学递质是 (2.0分) | A.乙酰胆碱 B.去甲肾上腺素 C.γ‐氨基丁酸 D.肾上腺素 E.5‐羟色胺 |
A.乙酰胆碱 |
2 | 1 | 2007 | 29. 在神经‐肌接头传递过程中,ACh 与ACh 门控通道结合使终板膜(2.0分) | A.仅K+通透性增加,发生超极化 B.对Na+、K+通透性增加,发生去极化 C.对Na+、K+通透性增加,发生超极化 D.对Ca2+通透性增加,发生去极化 E.对Cl‐通透性增加,发生超极化 |
B.对Na+、K+通透性增加,发生去极化 |
2 | 1 | 2007 | 30. 神经‐肌接头兴奋传递的阻断剂是(2.0分) | A.胆碱酯酶 B.阿托品 C.α‐银环蛇毒 D.六烃季铵 E.四乙胺 |
C.α‐银环蛇毒 |
2 | 1 | 2007 | 31. 重症肌无力患者的骨骼肌对运动神经冲动的反应降低是由于(2.0分) | A.递质释放量减少 B.递质含量减少 C.胆碱酯酶活性增高 D.受体数目减少或功能障碍 E.微终板电位减小 |
D.受体数目减少或功能障碍 |
2 | 1 | 2007 | 32. 多单位平滑肌的主要特点是(2.0分) | A.静息电位不稳定 B.不受外来神经的支配 C.具有自律性 D.不受化学递质的影响 E.各细胞活动时各自独立 |
E.各细胞活动时各自独立 |
2 | 1 | 2007 | 33. 不含有副交感节前纤维的神经是 (2.0分) | A.舌咽神经 B.三叉神经 C.面神经 D.动眼神经 E.迷走神经 |
B.三叉神经 |
2 | 1 | 2007 | 34. 支配虎口皮肤的神经是(2.0分) | A.肌皮神经 B.尺神经 C.正中神经 D.腋神经 E.桡神经 |
E.桡神经 |
2 | 1 | 2007 | 35. 支配胸锁乳突肌的神经是 (2.0分) | A.副神经 B.耳大神经 C.面神经颈支 D.枕小神经 E.颈丛的肌支 |
A.副神经 |
2 | 1 | 2007 | 36. 关于锥体外系统受损害症状和体征,那一项不正确?(2.0分) | A.不自主运动 B.肌张力减低或升高 C.动作迟缓和减少 D.姿势反射不良 E.病理反射阳性 |
E.病理反射阳性 |
2 | 1 | 2007 | 37. 关于上运动神经元瘫痪那项是正确?(2.0分) | A.表现为偏瘫,单瘫,截瘫或四肢瘫,肌张力增高,腱反射亢进,病理反射阳性,并早期明显肌肉萎缩 B.表现为偏瘫,单瘫,截瘫或四肢瘫,肌张力减低,腱反射减弱或消失,病理反射阳性。 C.表现为偏瘫,单瘫,截瘫或四肢瘫,肌张力增高,腱反射减弱或消失,病理反射阳性。 D.表现为偏瘫,单瘫,截瘫或四肢瘫,肌张力增高,腱反射亢进,病理反射阳性,肌电图和神经传导速度正常 |
D.表现为偏瘫,单瘫,截瘫或四肢瘫,肌张力增高,腱反射亢进,病理反射阳性,肌电图和神经传导速度正常 |
2 | 1 | 2007 | 38. 肌病的临床表现和辅助检查特点那项错误?(2.0分) | A.肌无力和萎缩,肌酶升高,肌电图肌原性改变,可伴有假性肌肉肥大 B.肌无力和萎缩,肌酶升高,肌电图肌原性改变,可伴有肌痛和压痛 C.肌无力和萎缩,肌酶升高,肌电图肌原性改变,可伴有强直和myokymia D.肌无力和萎缩,肌酶升高,肌电图肌原性改变,,可伴有局部感觉减退 |
D.肌无力和萎缩,肌酶升高,肌电图肌原性改变,,可伴有局部感觉减退 |
2 | 1 | 2007 | 39. 关于皮肌炎的描述那项错误?(2.0分) | A.对称性近端肌肌无力,肌酶升高,肌电图肌原性改变,可伴有关节痛和关节炎 B.对称性近端肌肌无力,肌酶升高,肌电图肌原性改变,可伴有肺间质改变 C.对称性近端肌肌无力,肌酶升高,肌电图肌原性改变,可伴有心肌受累, D.对称性近端肌肌无力,肌酶升高,肌电图肌原性改变,可伴有皮疹 E.对称性近端肌肌无力,肌酶升高,肌电图肌原性改变,可伴有恶性肿瘤和其它结缔组织病 F.对称性近端肌肌无力,肌酶升高,肌电图肌原性改变,肌活检特点肌细胞的胞浆和胞核内查到嗜酸性包涵体 |
F.对称性近端肌肌无力,肌酶升高,肌电图肌原性改变,肌活检特点肌细胞的胞浆和胞核内查到嗜酸性包涵体 |
2 | 1 | 2007 | 40. 以运动诱发肌无力为特点的疾病,下面哪个最不可能。(2.0分) | A.脂质沉积性肌病 B.周期性麻痹 C.线粒体肌病 D.风湿多肌痛 E.重症肌无力 F.糖原累积病 |
D.风湿多肌痛 |
2 | 1 | 2007 | 41. 患者,男,18岁 参加运动会400米跑步后次日,全身肌肉疼痛,痉挛,无力,检查发现低血糖,高血脂,肌红蛋白尿.为明确病因诊断最有价值的辅助检查是? (2.0分) | A.血钾和心电图 B.肌肉病理和酶学检测 C.肌酶 D.肌电图 E.血清抗肌球蛋白抗体 |
B.肌肉病理和酶学检测 |
2 | 1 | 2007 | 42. 患者男性,53岁,出现进行性的四肢肌肉无力和萎缩3年,家族中无类似发病者。肌电图检查提示肌源性损害,肌酶检查发现CK轻度升高,给予肾上腺皮质激素治疗无效, 下列对诊断最有帮助的检查是?(2.0分) | A.血常规生化检查 B.神经传导速度检查 C.肌肉活检 D.肌电图重频电刺激 E.血免疫学检查 |
C.肌肉活检 |
2 | 1 | 2007 | 43. 重症肌无力病人出现危象时,其主要的临床征兆是? (2.0分) | A.吞咽困难 B.眼球运动障碍 C.构音障碍 D.呼吸肌无力,不能维持正常通气和换气功能 E.四肢无力加重 |
D.呼吸肌无力,不能维持正常通气和换气功能 |
2 | 1 | 2007 | 44. 线粒体病最好发的部位是?(2.0分) | A.眼,皮肤和肾 B.脑,心脏和内分泌系统 C.肌肉,胃肠和周围神经系统 D.脑,肌肉和眼 E.视,听觉系统和中枢神经系统 |
D.脑,肌肉和眼 |
2 | 1 | 2007 | 45. 关于引起神经-肌肉接头(NMJ)传递障碍描述那项错误?(2.0分) | A.突触后膜乙酰胆碱受体数量减少,功能抑制 B.突触间隙胆碱酯酶活性改变 C.突触前膜释放乙酰胆碱障碍 D.骨骼肌细胞膜的离子通道功能失常 E.高镁血症可影响突触前膜释放乙酰胆碱 |
D.骨骼肌细胞膜的离子通道功能失常 |
2 | 1 | 2007 | 46. 重症肌无力危象的抢救最重要是? (2.0分) | A.积极控制感染 B.区分肌无力性危象,胆碱能危象或反拗危象,以便对因治疗 C.大剂量免疫球蛋白或血浆置换 D.保持呼吸道通畅,呼吸机支持 E.激素治疗 |
B.区分肌无力性危象,胆碱能危象或反拗危象,以便对因治疗 |
2 | 1 | 2007 | 47. 重症肌无力辅助诊断那项不需要?(2.0分) | A.乙酰胆碱受体抗体(AchR-ab)测定 B.胸腺影像检查 C.药物试验 D.疲劳试验 E.普通肌电图和神经传导 F.重复电刺激(RNS)单纤维肌电图(SFEMG) |
E.普通肌电图和神经传导 |
2 | 1 | 2007 | 48. 针对重症肌无力免疫机制药物治疗起效最快的方法是?(2.0分) | A.大剂量免疫球蛋白或血浆置换 B.激素治疗 C.免疫抑制剂 D.抗胆碱酯酶 |
A.大剂量免疫球蛋白或血浆置换 |
2 | 1 | 2007 | 49. 32岁,男性,暴食后次日早晨双下肢无力伴大腿酸胀;体检,意识清楚,颅神经正常,下肢对称性迟缓性瘫痪,大小便正常,感觉正常,反射正常。无家族遗传病史。最有可能的诊断是(2.0分) | A.高钾型周期性麻痹 B.肉毒毒素中毒 C.吉兰-巴雷综合征 D.重症肌无力 E.低钾型周期性麻痹 F.正常血钾型周期性麻痹 |
E.低钾型周期性麻痹 |
2 | 1 | 2007 | 50. Duchenne 型为X连锁隐性遗传 和Becker 型呈常染色体显性遗传 (2.0分) | A.Duchenne 型发病年龄相对早 B.Duchenne 型进展快,预后差 C.Duchenne 型易伴发心肌和认知功能损害 D.Duchenne 型为X连锁隐性遗传 和Becker 型呈常染色体显性遗传 E.肌肉病理Duchenne 型dystrophin染色缺失Becker dystrophin染色减少 |
D.Duchenne 型为X连锁隐性遗传 和Becker 型呈常染色体显性遗传 |
2 | 1 | 2008 | 1. Pilocarpine 对眼的作用(2.0分) | A.缩瞳、降低眼内压、调节麻痹 B.扩瞳、降低眼内压、调节麻痹 C.扩瞳、升高眼内压、调节痉挛 D.缩瞳、升高眼内压、调节痉挛 E.缩瞳、降低眼内压、调节痉挛 |
E.缩瞳、降低眼内压、调节痉挛 |
2 | 1 | 2008 | 2. Succinylcholine过量引起呼吸肌麻痹时,最有效的解救方法是(2.0分) | A.注射neostigmine B.注射atropine C.注射nikethamide D.注射lobeline E.用人工呼吸机 |
E.用人工呼吸机 |
2 | 1 | 2008 | 3. 治疗中、重度有机磷酸酯类中毒应当采用 (2.0分) | A.Atroping + pyraloxime Methiodide(PAM) B.Atropine C.Atropine + scopolamine D.Atropine + neostigmine E.Pyraloxime methiodide(PAM) |
A.Atroping + pyraloxime Methiodide(PAM) |
2 | 1 | 2008 | 4. 关于有机磷酸酯类中毒的特点,下述哪一项是错误的?(2.0分) | A.形成磷酰化胆碱酯酶,使胆碱酯酶活性过强 B.形成磷酰化胆碱酯酶,使胆碱酯酶失活 C.严重中毒时可出现N样症状及中枢症状 D.应当及时抢救,以免使磷酰化胆碱酯酶"老化" E.抢救时应当合用atropine和pyraloxime methoiodide (PAM) |
A.形成磷酰化胆碱酯酶,使胆碱酯酶活性过强 |
2 | 1 | 2008 | 5. 在突触的描述中,哪一项错误? (2.0分) | A.突触分电突触和化学性突触两类 B.突触也是神经元与肌细胞、腺细胞等之间特化的细胞连接 C.突触是神经元与神经元之间特化的细胞连接 D.化学性突触由突触前成分、突触间隙和突触后成分组成 E.突触小泡内含神经递质的受体 |
E.突触小泡内含神经递质的受体 |
2 | 1 | 2008 | 6. 关于时间常数,描述正确的是: (2.0分) | A.时间常数越小,下限截止频率越高,对低频成分的滤波程度越大。 B.时间常数越小,下限截止频率越低,对低频成分的滤波程度越大。 C.时间常数越大,下限截止频率越高,对低频成分的滤波程度越小。 D.时间常数越大,下限截止频率越低,对低频成分的滤波程度越大。 |
A.时间常数越小,下限截止频率越高,对低频成分的滤波程度越大。 |
2 | 1 | 2008 | 7. 重症肌无力危象的抢救最重要是? (2.0分) | A.区分肌无力性危象,胆碱能危象或反拗危象,以便对因治疗 B.保持呼吸道通畅,呼吸机支持 C.大剂量免疫球蛋白或血浆置换 D.积极控制感染 E.激素治疗 |
A.区分肌无力性危象,胆碱能危象或反拗危象,以便对因治疗 |
2 | 1 | 2008 | 8. 下述哪一组织对neostigmine不敏感?(2.0分) | A.中枢神经 B.胃肠平滑肌 C.膀胱平滑肌 D.骨骼肌 E.以上都不是 |
A.中枢神经 |
2 | 1 | 2008 | 9. 下列关于神经胶质细胞的叙述,错误的是 (2.0分) | A.具有转运代谢物质的作用 B.没有轴突 C.具有许多突起 D.具有支持的作用 E.没有细胞分裂能力 |
E.没有细胞分裂能力 |
2 | 1 | 2008 | 10. 支配胸锁乳突肌的神经是(2.0分) | A.耳大神经 B.枕小神经 C.面神经颈支 D.副神经 E.颈丛的肌支 |
D.副神经 |
2 | 1 | 2008 | 11. 在化学突触传递的特征中,错误的是 (2.0分) | A.后放 B.总和 C.双向性传递 D.兴奋节律的改变 E.对内环境变化敏感 |
C.双向性传递 |
2 | 1 | 2008 | 12. Duchenne's muscular dystrophy is a lethal childhood disease that involves skeletal muscle cells. Which of the following statements best describes the pathophysiology of this devastating disease? (2.0分) | A.Structural changes in the costomere results in damage to the muscle cell membrane (sarcolemma) during both concentric and eccentric contraction. B.Structural changes in the contractile proteins in the myocyte (muscle fiber) results in loss of the cell's ability to shorten and produce movement. C.Structural changes in the myocytes results in injured cells that are destroyed and replaced by mitosis and differentiation of satellite cells. D.Structural changes in the production of collagen disrupt the packaging of the muscle fibers and results in loss of connection with α-motor neurons at the neuromuscular junctions. E.Structural changes at the enthesis as a result of defective fibrocartilage insertions results in loss of range of motion of the muscle at the joint. |
A.Structural changes in the costomere results in damage to the muscle cell membrane (sarcolemma) during both concentric and eccentric contraction. |
2 | 1 | 2008 | 13. 使用生物信号采集处理系统,下列参数设置或操作正确的是(2.0分) | A.动脉血压记录完毕后在保存数据前须进行零点设置 B.记录骨骼肌收缩张力时,耦合输入方式选择交流耦合 C.记录蟾蜍坐骨神经AP,灵敏度和低通滤波分别设置为0.5V和30Hz D.记录离体豚鼠回肠收缩张力时,时间常数设置为直流 |
D.记录离体豚鼠回肠收缩张力时,时间常数设置为直流 |
2 | 1 | 2008 | 14. 时间常数 =0.1s时,高通滤波器的下限转折频率约为(2.0分) | A.FL = 1.60Hz B.FL=0.16Hz C.FL = 0.10Hz D.FL = 1.00Hz |
C.FL = 0.10Hz |
2 | 1 | 2008 | 15. 递质在神经元之间的传递方向为 (2.0分) | A.突触小泡→突触后膜→突触间隙 B.突触小泡→突触间隙→突触后膜 C.突触小泡→突触后膜→突触前膜 D.突触后膜→突触间隙→突触前膜 |
B.突触小泡→突触间隙→突触后膜 |
2 | 1 | 2008 | 16. 生物医学实验设计的三大因素为(2.0分) | A.受试对象、实验技术、实验效应 B.实验动物、实验技术、实验指标 C.受试对象、处理因素、实验效应 D.实验动物、实验分组、观察指标 |
C.受试对象、处理因素、实验效应 |
2 | 1 | 2008 | 17. 关于锥体外系统的解剖生理描述那一项不正确?(2.0分) | A.属于上运动神经元 B.维持和调整体态姿势 C.调节肌张力,协调肌肉运动 D.担负半自动的刻板动作及反射性的运动 |
A.属于上运动神经元 |
2 | 1 | 2008 | 18. 任氏液的理化特性与蟾蜍的(2.0分) | A.血液相似 B.体液相似 C.组织液相似 D.细胞内液相似 |
C.组织液相似 |
2 | 1 | 2008 | 19. 关于atropine的叙述,下列哪一项是错误的?(2.0分) | A.可用于治疗缓慢型心律失常 B.可用于治疗感染性休克 C.可用于治疗过敏性休克 D.可用于全身麻醉前给药 E.可用于缓解内脏绞痛 |
C.可用于治疗过敏性休克 |
2 | 1 | 2008 | 20. 进行低温对小鼠耐受缺氧的影响的实验,设生理盐水对照组的目的是(2.0分) | A.消除溶剂对实验的影响 B.消除非处理因素对实验的影响 C.消除实验条件对实验的影响 D.消除动物对实验的影响 |
B.消除非处理因素对实验的影响 |
2 | 1 | 2008 | 21. 患者男性,53岁,出现进行性的四肢肌肉无力和萎缩3年,家族中无类似发病者。肌电图检查提示肌源性损害,肌酶检查发现CK轻度升高,给予肾上腺皮质激素治疗无效。最有可能诊断是?(2.0分) | A.皮肌炎 B.包涵体肌炎 C.线粒体肌病 D.多发性肌炎 E.进行性肌营养不良 F.糖原累积病 |
B.包涵体肌炎 |
2 | 1 | 2008 | 22. 32岁,男性,暴食后次日早晨双下肢无力伴大腿酸胀;体检,意识清楚,颅神经正常,下肢对称性迟缓性瘫痪,大小便正常,感觉正常,反射正常。无家族遗传病史。最有可能的诊断是(2.0分) | A.吉兰-巴雷综合征 B.肉毒毒素中毒 C.低钾型周期性麻痹 D.重症肌无力 E.高钾型周期性麻痹 F.正常血钾型周期性麻痹 |
C.低钾型周期性麻痹 |
2 | 1 | 2008 | 23. 生理科学实验中观察和记录的生物信号的基本特性为(2.0分) | A.振幅低、频率高、源阻抗大 B.振幅低、频率低、源阻抗大 C.振幅高、频率高、源阻抗大 D.振幅低、频率高、源阻抗小 |
B.振幅低、频率低、源阻抗大 |
2 | 1 | 2008 | 24. 当神经冲动到达运动神经末梢时可引起接头前膜的(2.0分) | A.Ca2+通道开放 B.Na+通道关闭 C.K+通道开放 D.Cl‐通道开放 E.Cl‐通道关闭 |
A.Ca2+通道开放 |
2 | 1 | 2008 | 25. 在神经‐肌接头传递过程中,ACh 与ACh门控通道结合使终板膜 (2.0分) | A.对Ca2+通透性增加,发生去极化 B.对Na+、K+通透性增加,发生去极化 C.仅K+通透性增加,发生超极化 D.对Na+、K+通透性增加,发生超极化 E.对Cl‐通透性增加,发生超极化 |
B.对Na+、K+通透性增加,发生去极化 |
2 | 1 | 2008 | 26. 肌丝滑行理论的直接证据是骨骼肌收缩时(2.0分) | A.明带、暗带和H 带长度均缩短 B.明带缩短,暗带和H 带长度不变 C.暗带缩短,明带和H 带长度不变 D.明带和H 带缩短,暗带长度不变 E.明带、暗带和H 带长度均不变 |
D.明带和H 带缩短,暗带长度不变 |
2 | 1 | 2008 | 27. 关于小脑病变那项描述不正确?(2.0分) | A.肌张力升高 B.轮替动作不协调 C.平衡不良 D.精细活动困难 |
A.肌张力升高 |
2 | 1 | 2008 | 28. 下列关于神经胶质细胞的描述,正确的是(2.0分) | A.与相邻细胞有突触联系 B.有随细胞外Na+浓度改变的膜电位 C.既有树突又有轴突 D.细胞间普遍存在缝隙连接 E.有产生动作电位的能力 |
D.细胞间普遍存在缝隙连接 |
2 | 1 | 2008 | 29. 神经冲动抵达末梢时,引起递质释放主要有赖于哪种离子的作用?(2.0分) | A.Mg2+ B.Ca2+ C.Cl‐ D.Na+ E.K+ |
B.Ca2+ |
2 | 1 | 2008 | 30. 下列哪一项关于d-tubocurarine的叙述是正确的?(2.0分) | A.过量可用新斯的明对抗 B.是一种除极化型肌松药 C.可促进运动神经末梢释放乙酰胆碱 D.可引起高血钾症 E.也可用于治疗支气管哮喘 |
A.过量可用新斯的明对抗 |
2 | 1 | 2008 | 31. 神经元产生动作电位的部位是(2.0分) | A.树突 B.胞体 C.轴丘 D.轴索 E..始段 |
E..始段 |
2 | 1 | 2008 | 32. You see a 25 year old ambulatory male with a history of a dystrophic myopathy. He presented at 6 years of age with muscle weakness. Routine laboratory studies revealed elevated circulating muscle enzymes, and molecular testing of his DNA showed an in-fram(2.0分) | A.Becker muscular dystrophy B.Duchenne muscular dystrophy C.Becker muscular dystrophy carrier D.Duchenne muscular dystrophy carrier E.Duchenne muscular dystrophy that he outgrew |
A.Becker muscular dystrophy |
2 | 1 | 2008 | 33. 生物医学实验设计的原则是 (2.0分) | A.随机、重复、均衡 B.随机、重复、齐同 C.对照、随机、均衡 D.随机、对照、重复 |
D.随机、对照、重复 |
2 | 1 | 2008 | 34. A mutation causing an abnormality in presenilin that disrupts amyloid production or function has been associated with (2.0分) | A.Marfan syndrome B.Duchenne muscular dystrophy C.cystic fibrosis D.none of the above |
D.none of the above |
2 | 1 | 2008 | 35. 具有中枢抑制作用的M胆碱受体阻断药是(2.0分) | A.Atropine B.Propantheline C.Anisodamine D.Scopolamine E.Homatropine |
D.Scopolamine |
2 | 1 | 2008 | 36. Duchenne muscular dystrophy 和Becker muscular dystrophy 的鉴别点那项错误。(2.0分) | A.Duchenne 型发病年龄相对早 B.Duchenne 型为X连锁隐性遗传 和Becker 型呈常染色体显性遗传 C.Duchenne 型进展快,预后差 D.Duchenne 型易伴发心肌和认知功能损害 E.肌肉病理Duchenne 型dystrophin染色缺失Becker dystrophin染色减少 |
B.Duchenne 型为X连锁隐性遗传 和Becker 型呈常染色体显性遗传 |
2 | 1 | 2008 | 37. 关于下运动神经元瘫痪那项是正确?(2.0分) | A.可表现为四肢远端或肢体的某一部位无力,肌张力增高,腱反射亢进,病理反射阳性,肌电图和神经传导速度正常 B.可表现为四肢远端或肢体的某一部位无力,肌张力减低,腱反射减弱或消失,病理反射阳性。 C.可表现为四肢远端或肢体的某一部位无力,肌张力增高,腱反射亢进,病理反射阳性,并早期明显肌肉萎缩 D.可表现为四肢远端或肢体的某一部位无力,肌张力增高,腱反射减弱或消失,病理反射阴性。 E.可表现为四肢远端或肢体的某一部位无力, 肌张力减低,腱反射减弱或消失,病理反射阴性,常伴感觉障碍和受累部位肌肉萎缩。 |
E.可表现为四肢远端或肢体的某一部位无力, 肌张力减低,腱反射减弱或消失,病理反射阴性,常伴感觉障碍和受累部位肌肉萎缩。 |
2 | 1 | 2008 | 38. 关于多发性肌炎的描述那项错误?(2.0分) | A.对称性近端肌肌无力,肌酶升高,肌电图肌原性改变,可伴有肌痛和压痛 B.对称性近端肌肌无力,肌酶升高,肌电图肌原性改变,可伴有肌萎缩, C.对称性近端肌肌无力,肌酶升高,肌电图肌原性改变,可伴有可伴有强直, D.对称性近端肌肌无力,肌酶升高,肌电图肌原性改变,可伴有呼吸表浅、呼吸困难, E.对称性近端肌肌无力,肌酶升高,肌电图肌原性改变,可伴有吞咽和构音障碍 |
C.对称性近端肌肌无力,肌酶升高,肌电图肌原性改变,可伴有可伴有强直, |
2 | 1 | 2008 | 39. 进行低温对小鼠耐受缺氧的影响实验,正确的分组方法是(2.0分) | A.将两只小鼠随机分配给对照组和实验组 B.随机挑选体重相近的两只小鼠,一只作为对照组、一只作为实验组 C.随机挑选两只小鼠,一只作为对照组、一只作为实验组 D.将两只体重相近性别相同的小鼠随机分配给对照组和实验组 |
D.将两只体重相近性别相同的小鼠随机分配给对照组和实验组 |
2 | 1 | 2008 | 40. 患者,男,18岁 参加运动会400米跑步后次日,全身肌肉疼痛,痉挛,无力,检查发现低血糖,高血脂,肌红蛋白尿.可能最大的疾病诊断是?(2.0分) | A.周期性麻痹 B.线粒体肌病 C.脂质沉积性肌病 D.糖原累积病 E.重症肌无力 |
D.糖原累积病 |
2 | 1 | 2008 | 41. 关于进行性肌营养不良描述那项错误?(2.0分) | A.遗传性肌肉变性疾病 B.电生理为肌源性损害,神经传导速度正常 C.不能耐受运动和近端肌无力,肌肉痉挛和疼痛为主要表现 D.遗传方式有常染色体显性和隐性或X连锁隐性遗传 E.病理特点:进行性的肌纤维坏死,再生和脂肪及结缔组织增生,无异常代谢产物堆积 |
C.不能耐受运动和近端肌无力,肌肉痉挛和疼痛为主要表现 |
2 | 1 | 2008 | 42. 线粒体病脑肌病(MELAS)的主要临床表现: (2.0分) | A.Stroke-like episodes,ragged-red fibres and/or lactic acidosis and cardiomyopathy B.Stroke-like episodes,ragged-red fibres and/or lactic acidosis and Diabetes mellitus C.Stroke-like episodes,ragged-red fibres and/or lactic acidosis and bilateral deafness D.Stroke-like episodes, ragged-red fibres and/or lactic acidosis and seizures and/or dementia E.Stroke-like episodes,ragged-red fibres and/or lactic acidosis and cerebellar ataxia |
D.Stroke-like episodes, ragged-red fibres and/or lactic acidosis and seizures and/or dementia |
2 | 1 | 2008 | 43. 支配下斜肌的神经是(2.0分) | A.动眼神经 B.展神经 C.滑车神经 D.面神经 E.三叉神经 |
C.滑车神经 |
2 | 1 | 2008 | 44. 骨骼肌细胞中横管的功能是(2.0分) | A.钙离子进出肌纤维的通道 B.钙离子的贮存库 C.营养物质进出肌细胞的通道 D.兴奋传向肌细胞深部 E.钙离子和肌钙蛋白结合 |
D.兴奋传向肌细胞深部 |
2 | 1 | 2008 | 45. 关于引起神经-肌肉接头(NMJ)传递障碍描述那项错误? (2.0分) | A.突触后膜乙酰胆碱受体数量减少,功能抑制 B.突触前膜释放乙酰胆碱障碍 C.骨骼肌细胞膜的离子通道功能失常 D.突触间隙胆碱酯酶活性改变 E.高镁血症可影响突触前膜释放乙酰胆碱 |
C.骨骼肌细胞膜的离子通道功能失常 |
2 | 1 | 2008 | 46. 关于皮肌炎和多发性肌炎肌酶升高描述那项错误(2.0分) | A.绝大多数病人在病程某一阶段可出现肌酶活性增高,晚期肌萎缩后肌酶不再释放 B.绝大多数病人在病程某一阶段可出现肌酶活性增高,CK最敏感。 C.绝大多数病人在病程某一阶段可出现肌酶活性增高,是诊断、疗效监测及预后的评价指标 D.绝大多数病人在病程某一阶段可出现肌酶活性增高,CK特异强,只有在肌肉病变时升高 E.绝大多数病人在病程某一阶段可出现肌酶活性增高,包括肌酸激酶(CK)、醛缩酶(ALD)、乳酸脱氢酶(LDH)、门冬氨酸氨基转移酶(AST)。 |
D.绝大多数病人在病程某一阶段可出现肌酶活性增高,CK特异强,只有在肌肉病变时升高 |
2 | 1 | 2008 | 47. 重症肌无力辅助诊断那项不需要? (2.0分) | A.胸腺影像检查 B.药物试验 C.乙酰胆碱受体抗体(AchR-ab)测定 D.疲劳试验 E.普通肌电图和神经传导 F.重复电刺激(RNS)单纤维肌电图(SFEMG) |
E.普通肌电图和神经传导 |
2 | 1 | 2008 | 48. 针对重症肌无力免疫机制药物治疗起效最快的方法是? (2.0分) | A.激素治疗 B.免疫抑制剂 C.大剂量免疫球蛋白或血浆置换 D.抗胆碱酯酶 |
C.大剂量免疫球蛋白或血浆置换 |
2 | 1 | 2008 | 49. 三角肌的支配神经是 (2.0分) | A.腋神经 B.肌皮神经 C.正中神经 D.尺神经 E.桡神经 |
B.肌皮神经 |
2 | 1 | 2008 | 50. 低钾型麻痹经正规补充氯化钾数天后,低钾仍不能纠正最需要鉴别疾病是? (2.0分) | A.原发性醛固酮增多症和肾小管性酸中毒: B.甲亢伴周期性麻痹 C.低钾性周期性麻痹 D.过度胃肠道丢失后:呕吐、腹泻等 |
A.原发性醛固酮增多症和肾小管性酸中毒: |
2 | 1 | 2009 | 1. You see a 25 year old ambulatory male with a history of a dystrophic myopathy. He presented at 6 years of age with muscle weakness. Routine laboratory studies revealed elevated circulating muscle enzymes, and molecular testing of his DNA showed an in-frame deletion of the dystrophin gene. What is the most likely diagnosis of this patient? (2.0分) | A.Becker muscular dystrophy B.Becker muscular dystrophy carrier C.Duchenne muscular dystrophy D.Duchenne muscular dystrophy carrier E.Duchenne muscular dystrophy that he outgrew |
A.Becker muscular dystrophy |
2 | 1 | 2009 | 2. Duchenne's muscular dystrophy is a lethal childhood disease that involves skeletal muscle cells. Which of the following statements best describes the pathophysiology of this devastating disease? (2.0分) | A.Structural changes in the production of collagen disrupt the packaging of the muscle fibers and results in loss of connection with α-motor neurons at the neuromuscular junctions. B.Structural changes in the costomere results in damage to the muscle cell membrane (sarcolemma) during both concentric and eccentric contraction. C.Structural changes in the contractile proteins in the myocyte (muscle fiber) results in loss of the cell's ability to shorten and produce movement. D.Structural changes in the myocytes results in injured cells that are destroyed and replaced by mitosis and differentiation of satellite cells. E.Structural changes at the enthesis as a result of defective fibrocartilage insertions results in loss of range of motion of the muscle at the joint. |
B.Structural changes in the costomere results in damage to the muscle cell membrane (sarcolemma) during both concentric and eccentric contraction. |
2 | 1 | 2009 | 3. A mutation causing an abnormality in presenilin that disrupts amyloid production or function has been associated with (2.0分) | A.cystic fibrosis B.Duchenne muscular dystrophy C.Marfan syndrome D.Wilson disease E.none of the above |
E.none of the above |
2 | 1 | 2009 | 4. Which of the following genetic diseases results from a mutation that prevents a protein from forming? (2.0分) | A.Duchenne muscular dystrophy B.Hemophilia A C.Epidermolysis bullosa D.Huntington disease E.Fragile X syndrome |
A.Duchenne muscular dystrophy |
2 | 1 | 2009 | 5. 生理科学实验中观察和记录的生物信号的基本特性为 (2.0分) | A.振幅低、频率低、源阻抗大 B.振幅低、频率高、源阻抗大 C.振幅高、频率高、源阻抗大 D.振幅低、频率高、源阻抗小 |
A.振幅低、频率低、源阻抗大 |
2 | 1 | 2009 | 6. 关于时间常数,描述正确的是(2.0分) | A.时间常数越小,下限截止频率越低,对低频成分的滤波程度越大。 B.时间常数越小,下限截止频率越高,对低频成分的滤波程度越大。 C.时间常数越大,下限截止频率越高,对低频成分的滤波程度越小。 D.时间常数越大,下限截止频率越低,对低频成分的滤波程度越大。 |
B.时间常数越小,下限截止频率越高,对低频成分的滤波程度越大。 |
2 | 1 | 2009 | 7. 时间常数 =0.1s时,高通滤波器的下限转折频率约为(2.0分) | A.FL=0.16Hz B.FL = 0.10Hz C.FL = 1.60Hz D.FL = 1.00Hz |
C.FL = 1.60Hz |
2 | 1 | 2009 | 8. 使用生物信号采集处理系统,下列参数设置或操作正确的是(2.0分) | A.记录骨骼肌收缩张力时,耦合输入方式选择交流耦合 B.动脉血压记录完毕后在保存数据前须进行零点设置 C.记录蟾蜍坐骨神经AP,灵敏度和低通滤波分别设置为0.5V和30Hz D.记录离体豚鼠回肠收缩张力时,时间常数设置为直流 |
D.记录离体豚鼠回肠收缩张力时,时间常数设置为直流 |
2 | 1 | 2009 | 9. 生物医学实验设计的原则是 (2.0分) | A.对照、随机、均衡 B.随机、重复、齐同 C.随机、重复、均衡 D.随机、对照、重复 |
D.随机、对照、重复 |
2 | 1 | 2009 | 10. 进行低温对小鼠耐受缺氧的影响实验,正确的分组方法是(2.0分) | A.随机挑选两只小鼠,一只作为对照组、一只作为实验组 B.随机挑选体重相近的两只小鼠,一只作为对照组、一只作为实验组 C.将两只小鼠随机分配给对照组和实验组 D.将两只体重相近性别相同的小鼠随机分配给对照组和实验组 |
D.将两只体重相近性别相同的小鼠随机分配给对照组和实验组 |
2 | 1 | 2009 | 11. 关于胸椎的特点是(2.0分) | A.横突上有横突孔 B.棘突分叉 C.上、下关节突不明显 D.棘突水平伸向后方 E.椎体侧面后部有肋凹 |
E.椎体侧面后部有肋凹 |
2 | 1 | 2009 | 12. 关于腰椎的正确说法是 (2.0分) | A.椎体粗壮,横切面呈三角形 B.椎孔呈圆形 C.各棘突的间隙较宽 D.上关节面是冠状位 E.上关节面呈水平位 |
C.各棘突的间隙较宽 |
2 | 1 | 2009 | 13. 上颌骨是(2.0分) | A.长骨 B.短骨 C.扁骨 D.不规则骨 E.以上都不是 |
D.不规则骨 |
2 | 1 | 2009 | 14. 关节腔内有关节盘的关节是(2.0分) | A.肩关节 B.胸锁关节 C.肘关节 D.髋关节 E.踝关节 |
B.胸锁关节 |
2 | 1 | 2009 | 15. 通过圆孔的结构是(2.0分) | A.眶上神经 B.眼神经 C.上颌神经 D.上颌动脉 E.脑膜中动脉 |
C.上颌神经 |
2 | 1 | 2009 | 16. 关于锥体外系统的解剖生理描述那一项不正确?(2.0分) | A.调节肌张力,协调肌肉运动 B.维持和调整体态姿势 C.属于上运动神经元 D..担负半自动的刻板动作及反射性的运动 |
C.属于上运动神经元 |
2 | 1 | 2009 | 17. 关于小脑病变那项描述不正确?(2.0分) | A.肌无力 B.平衡不良 C.轮替动作不协调 D.精细活动困难 |
A.肌无力 |
2 | 1 | 2009 | 18. 关于下运动神经元瘫痪那项是正确(2.0分) | A.可表现为四肢远端或肢体的某一部位无力,肌张力增高,腱反射减弱或消失,病理反射阴性。 B.可表现为四肢远端或肢体的某一部位无力,肌张力减低,腱反射减弱或消失,病理反射阳性。 C.可表现为四肢远端或肢体的某一部位无力,肌张力增高,腱反射亢进,病理反射阳性,并早期明显肌肉萎缩 D.可表现为四肢远端或肢体的某一部位无力,肌张力增高,腱反射亢进,病理反射阳性,肌电图和神经传导速度正常 E.可表现为四肢远端或肢体的某一部位无力, 肌张力减低,腱反射减弱或消失,病理反射阴性,常伴感觉障碍和受累部位肌肉萎缩。 |
E.可表现为四肢远端或肢体的某一部位无力, 肌张力减低,腱反射减弱或消失,病理反射阴性,常伴感觉障碍和受累部位肌肉萎缩。 |
2 | 1 | 2009 | 19. 关于多发性肌炎的描述那项错误?(2.0分) | A.对称性近端肌无力,肌酶升高,肌电图肌原性改变,可伴有肌痛和压痛 B.对称性近端肌无力,肌酶升高,肌电图肌原性改变,可伴有肌萎缩, C.对称性近端肌无力,肌酶升高,肌电图肌原性改变,可伴有呼吸表浅、呼吸困难, D.对称性近端肌无力,肌酶升高,肌电图肌原性改变,可伴有肌阵挛, E.对称性近端肌无力,肌酶升高,肌电图肌原性改变,可伴有吞咽和构音障碍 |
D.对称性近端肌无力,肌酶升高,肌电图肌原性改变,可伴有肌阵挛, |
2 | 1 | 2009 | 20. 关于皮肌炎和多发性肌炎肌酶升高描述那项错误?(2.0分) | A.绝大多数病人在病程某一阶段可出现肌酶活性增高,CK特异强,只有在肌肉病变时升高 B.绝大多数病人在病程某一阶段可出现肌酶活性增高,CK最敏感。 C.绝大多数病人在病程某一阶段可出现肌酶活性增高,是诊断、疗效监测及预后的评价指标 D.绝大多数病人在病程某一阶段可出现肌酶活性增高,晚期肌萎缩后肌酶不再释放 E.绝大多数病人在病程某一阶段可出现肌酶活性增高,包括肌酸激酶(CK)、醛缩酶(ALD)、乳酸脱氢酶(LDH)、门冬氨酸氨基转移酶(AST)。 |
A.绝大多数病人在病程某一阶段可出现肌酶活性增高,CK特异强,只有在肌肉病变时升高 |
2 | 1 | 2009 | 21. 患者,男,18岁 参加运动会400米跑步后次日,全身肌肉疼痛,痉挛,无力,检查发现低血糖,高血脂,肌红蛋白尿.可能最大的疾病诊断是?(2.0分) | A.脂质沉积性肌病 B.糖原累积病 C.线粒体肌病 D.周期性麻痹 E.重症肌无力 |
B.糖原累积病 |
2 | 1 | 2009 | 22. 关于进行性肌营养不良描述那项错误?(2.0分) | A.遗传性肌肉变性疾病 B.遗传方式有常染色体显性和隐性或X连锁隐性遗传 C.不能耐受运动和近端肌无力,肌肉痉挛和疼痛为主要表现 D.电生理为肌源性损害,神经传导速度正常 E.病理特点.进行性的肌纤维坏死,再生和脂肪及结缔组织增生,无异常代谢产物堆积 |
C.不能耐受运动和近端肌无力,肌肉痉挛和疼痛为主要表现 |
2 | 1 | 2009 | 23. 患者男性,53岁,出现进行性的四肢肌肉无力和萎缩3年,家族中无类似发病者。肌电图检查提示肌源性损害,肌酶检查发现CK轻度升高,给予肾上腺皮质激素治疗无效。最有可能诊断是? (2.0分) | A.皮肌炎 B.多发性肌炎 C.线粒体肌病 D.包涵体肌炎 E.进行性肌营养不良 F.糖原累积病 |
D.包涵体肌炎 |
2 | 1 | 2009 | 24. Duchenne muscular dystrophy 和Becker muscular dystrophy 的鉴别点那项错误。(2.0分) | A.Duchenne 型为X连锁隐性遗传 和Becker 型呈常染色体显性遗传 B.Duchenne 型发病年龄相对早 C.Duchenne 型进展快,预后差 D.Duchenne 型易伴发心肌和认知功能损害 E.肌肉病理Duchenne 型dystrophin染色缺失Becker dystrophin染色减少 |
A.Duchenne 型为X连锁隐性遗传 和Becker 型呈常染色体显性遗传 |
2 | 1 | 2009 | 25. 线粒体病脑肌病(MELAS)的主要临床表现:(2.0分) | A.Stroke-like episodes,ragged-red fibres and/or lactic acidosis and bilateral deafness B.Stroke-like episodes,ragged-red fibres and/or lactic acidosis and Diabetes mellitus C.Stroke-like episodes,ragged-red fibres and/or lactic acidosis and cardiomyopathy D.Stroke-like episodes, ragged-red fibres and/or lactic acidosis and seizures and/or dementia E.Stroke-like episodes,ragged-red fibres and/or lactic acidosis and cerebellar ataxia |
D.Stroke-like episodes, ragged-red fibres and/or lactic acidosis and seizures and/or dementia |
2 | 1 | 2009 | 26. 关于引起神经-肌肉接头(NMJ)传递障碍描述那项错误?(2.0分) | A.突触前膜释放乙酰胆碱障碍 B.突触间隙胆碱酯酶活性改变 C.突触后膜乙酰胆碱受体数量减少,功能抑制 D.骨骼肌细胞膜的离子通道功能失常 E.高镁血症可影响突触前膜释放乙酰胆碱 |
D.骨骼肌细胞膜的离子通道功能失常 |
2 | 1 | 2009 | 27. 重症肌无力危象的抢救最重要是?(2.0分) | A.区分肌无力性危象,胆碱能危象或反拗危象,以便对因治疗 B.保持呼吸道通畅,呼吸机支持 C.积极控制感染 D.大剂量免疫球蛋白或血浆置换 E.激素治疗 |
A.区分肌无力性危象,胆碱能危象或反拗危象,以便对因治疗 |
2 | 1 | 2009 | 28. 重症肌无力辅助诊断那项不常规检查?(2.0分) | A.疲劳试验,药物试验 B.免疫学指标和甲状腺功能检查 C.乙酰胆碱受体抗体(AchR-ab)测定 D.胸腺影像检查 E.普通肌电图和神经传导 F.重复电刺激(RNS)单纤维肌电图(SFEMG) |
E.普通肌电图和神经传导 |
2 | 1 | 2009 | 29. 针对重症肌无力免疫机制药物治疗起效最快的方法是?(2.0分) | A.大剂量免疫球蛋白或血浆置换 B.免疫抑制剂 C.激素治疗 D.抗胆碱酯酶 |
A.大剂量免疫球蛋白或血浆置换 |
2 | 1 | 2009 | 30. 32岁,男性,暴食后次日早晨双下肢无力伴大腿酸胀;体检,意识清楚,颅神经正常,下肢对称性迟缓性瘫痪,大小便正常,感觉正常,反射正常。无家族遗传病史。最有可能的诊断是(2.0分) | A.吉兰-巴雷综合征 B.肉毒毒素中毒 C.重症肌无力 D.低钾型周期性麻痹 E.高钾型周期性麻痹 F.正常血钾型周期性麻痹 |
D.低钾型周期性麻痹 |
2 | 1 | 2009 | 31. 线粒体病最好发的部位是?(2.0分) | A.脑,肌肉和眼 B.脑,心脏和内分泌系统 C.肌肉,胃肠和周围神经系统 D.眼,皮肤和肾 E.视,听觉系统和中枢神经系统 |
A.脑,肌肉和眼 |
2 | 1 | 2009 | 32. 患者,男,18岁 参加运动会400米跑步后次日,全身肌肉疼痛,痉挛,无力,检查发现低血糖,高血脂,肌红蛋白尿.为明确病因诊断最有价值的辅助检查是?(2.0分) | A.肌酶 B.肌电图 C.肌肉病理和酶学检测 D.血钾和心电图 E.血清抗肌球蛋白抗体 |
C.肌肉病理和酶学检测 |
2 | 1 | 2009 | 33. 肌病的临床表现和辅助检查特点那项错误?(2.0分) | A.肌无力和萎缩,肌酶升高,肌电图肌原性改变,可伴有肌痛和压痛 B.肌无力和萎缩,肌酶升高,肌电图肌原性改变,可伴有假性肌肉肥大 C.肌无力和萎缩,肌酶升高,肌电图肌原性改变,可伴有强直和myokymia D.肌无力和萎缩,肌酶升高,肌电图肌原性改变,,受累部位感觉减退 |
D.肌无力和萎缩,肌酶升高,肌电图肌原性改变,,受累部位感觉减退 |
2 | 1 | 2009 | 34. 在神经‐肌接头传递过程中,ACh 与ACh门控通道结合使终板膜(2.0分) | A.对Na+、K+通透性增加,发生超极化 B.对Na+、K+通透性增加,发生去极化 C.仅K+通透性增加,发生超极化 D.对Ca2+通透性增加,发生去极化 E.对Cl‐通透性增加,发生超极化 |
B.对Na+、K+通透性增加,发生去极化 |
2 | 1 | 2009 | 35. 引发微终板电位的原因是 (2.0分) | A.神经末梢连续兴奋 B.神经末梢一次兴奋 C.几百个突触小泡释放的ACh D. 一个突触小泡释放的ACh E.自发释放的一个ACh 分子 |
D. 一个突触小泡释放的ACh |
2 | 1 | 2009 | 36. 神经‐肌接头兴奋传递的阻断剂是 (2.0分) | A.阿托品 B. 胆碱酯酶 C.α‐银环蛇毒 D.六烃季铵 E. 四乙胺 |
C.α‐银环蛇毒 |
2 | 1 | 2009 | 37. 神经‐肌接头传递中,消除乙酰胆碱的酶是(2.0分) | A.磷酸二酯酶 B.腺苷酸环化酶 C. 胆碱酯酶 D.ATP 酶 E.胆碱乙酰化酶 |
C. 胆碱酯酶 |
2 | 1 | 2009 | 38. 下列哪种毒素或药物能阻断骨骼肌终板膜上的乙酰胆碱受体(2.0分) | A.河豚毒素 B.阿托品 C.筒箭毒 D.心得安(普萘洛尔) E. 四乙胺 |
C.筒箭毒 |
2 | 1 | 2009 | 39. 骨骼肌收缩和舒张的基本功能单位是 (2.0分) | A.肌原纤维 B. 肌小节 C.肌纤维 D.粗肌丝 E. 细肌丝 |
B. 肌小节 |
2 | 1 | 2009 | 40. 骨骼肌收缩时,下列哪一结构的长度不变?(2.0分) | A.明带 B.暗带 C. H 带 D.肌小节 E.肌原纤维 |
B.暗带 |
2 | 1 | 2009 | 41. 肌细胞中的三联管指的是(2.0分) | A.每个横管及其两侧的肌小节 B.每个横管及其两侧的终末池 C.横管、纵管和肌质网 D. 每个纵管及其两侧的横管 E.每个纵管及其两侧的肌小节 |
B.每个横管及其两侧的终末池 |
2 | 1 | 2009 | 42. 下列哪种肌肉蛋白质具有ATP 酶活性(2.0分) | A.肌动蛋白 B.肌钙蛋白 C.肌球蛋白 D.原肌球蛋白 E. 调节蛋白 |
C.肌球蛋白 |
2 | 1 | 2009 | 43. 神经冲动抵达末梢时,引起递质释放主要有赖于哪种离子的作用?(2.0分) | A.Cl‐ B.Ca2+ C.Mg2+ D. Na+ E.K+ |
B.Ca2+ |
2 | 1 | 2009 | 44. 下列关于神经胶质细胞的叙述,错误的是 (2.0分) | A.具有许多突起 B.具有转运代谢物质的作用 C.具有支持的作用 D.没有轴突 E.没有细胞分裂能力 |
E.没有细胞分裂能力 |
2 | 1 | 2009 | 45. Pilocarpine(毛果芸香碱)对眼的调节作用是 (2.0分) | A.睫状肌松弛,悬韧带拉紧,晶状体变扁平 B.睫状肌松弛,悬韧带放松,晶状体变凸 C.睫状肌收缩,悬韧带拉紧,晶状体变凸 D.睫状肌收缩,悬韧带拉紧,晶状体变扁平 E.睫状肌收缩,悬韧带放松,晶状体变凸 |
E.睫状肌收缩,悬韧带放松,晶状体变凸 |
2 | 1 | 2009 | 46. 全身麻醉前给atropine(阿托品)的目的是 (2.0分) | A.镇静 B.增强麻醉药的作用 C.减少患者对术中的不良刺激的记忆 D.减少呼吸道腺体分泌 E.松弛骨骼肌 |
D.减少呼吸道腺体分泌 |
2 | 1 | 2009 | 47. 琥珀胆碱(succinylcholine)松弛骨骼肌的主要机制是 (2.0分) | A. 抑制中枢多突触反射 B.抑制脊髓丁运动神经元 C.与ACh竞争运动终板膜上的NM受体 D.使运动终板膜产生持久的去极化 E.减少运动神经末梢ACh的释放 |
D.使运动终板膜产生持久的去极化 |
2 | 1 | 2009 | 48. 易逆性抗胆碱酯酶药的作用不包括 (2.0分) | A.兴奋骨骼肌 B.兴奋胃肠道平滑肌 C.兴奋心脏 D.促进腺体分泌 E.缩瞳 |
C.兴奋心脏 |
2 | 1 | 2009 | 49. Neostigmine不用于下列哪一种情况? (2.0分) | A.阵发性室上性心动过速 B.重症肌无力 C.手术后肠胀气 D.支气管哮喘 E.非除极化型肌松药过量中毒 |
D.支气管哮喘 |
2 | 1 | 2009 | 50. 碘解磷定(pyraloxime methiodide)治疗有机磷酸酯类中毒的作用机制是 (2.0分) | A.能阻断M胆碱受体的作用 B.能阻断N胆碱受体的作用 C.能直接对抗体内积聚的ACh的作用 D.能恢复AChE水解ACh的活性 E.能与AChE结合而抑制酶的活性 |
D.能恢复AChE水解ACh的活性 |
2 | 1 | 2010 | 1. You see a 25 year old ambulatory male with a history of a dystrophic myopathy. He presented at 6 years of age with muscle weakness. Routine laboratory studies revealed elevated circulating muscle enzymes, and molecular testing of his DNA showed an in-frame deletion of the dystrophin gene. What is the most likely diagnosis of this patient? (2.0分) | A.Becker muscular dystrophy B.Becker muscular dystrophy carrier C.Duchenne muscular dystrophy D.Duchenne muscular dystrophy carrier E.Duchenne muscular dystrophy that he outgrew |
A.Becker muscular dystrophy |
2 | 1 | 2010 | 2. Duchenne's muscular dystrophy is a lethal childhood disease that involves skeletal muscle cells. Which of the following statements best describes the pathophysiology of this devastating disease?(2.0分) | A.Structural changes in the production of collagen disrupt the packaging of the muscle fibers and results in loss of connection with α-motor neurons at the neuromuscular junctions. B.Structural changes in the costomere results in damage to the muscle cell membrane (sarcolemma) during both concentric and eccentric contraction. C.Structural changes in the contractile proteins in the myocyte (muscle fiber) results in loss of the cell's ability to shorten and produce movement. D.Structural changes in the myocytes results in injured cells that are destroyed and replaced by mitosis and differentiation of satellite cells. E.Structural changes at the enthesis as a result of defective fibrocartilage insertions results in loss of range of motion of the muscle at the joint. |
B.Structural changes in the costomere results in damage to the muscle cell membrane (sarcolemma) during both concentric and eccentric contraction. |
2 | 1 | 2010 | 3. A mutation causing an abnormality in presenilin that disrupts amyloid production or function has been associated with(2.0分) | A.cystic fibrosis B.Duchenne muscular dystrophy C.Marfan syndrome D.Wilson disease E.none of the above |
E.none of the above |
2 | 1 | 2010 | 4. 生理科学实验中观察和记录的生物信号的基本特性为 (2.0分) | A.振幅低、频率低、源阻抗大 B.振幅低、频率高、源阻抗大 C.振幅高、频率高、源阻抗大 D.振幅低、频率高、源阻抗小 |
A.振幅低、频率低、源阻抗大 |
2 | 1 | 2010 | 5. 关于时间常数,描述正确的是:(2.0分) | A.时间常数越小,下限截止频率越低,对低频成分的滤波程度越大。 B.时间常数越小,下限截止频率越高,对低频成分的滤波程度越大。 C.时间常数越大,下限截止频率越高,对低频成分的滤波程度越小。 D.时间常数越大,下限截止频率越低,对低频成分的滤波程度越大。 |
B.时间常数越小,下限截止频率越高,对低频成分的滤波程度越大。 |
2 | 1 | 2010 | 6. 时间常数?=0.1s时,高通滤波器的下限转折频率约为 (2.0分) | A.FL=0.16Hz B.FL = 1.60Hz C.FL = 0.10Hz D.FL = 1.00Hz |
B.FL = 1.60Hz |
2 | 1 | 2010 | 7. 使用RM6240多道生理信号采集处理系统,下列参数设置或操作正确的是 (2.0分) | A.记录骨骼肌收缩张力时,耦合输入方式选择交流耦合 B.动脉血压记录完毕后在保存数据前须进行零点设置 C.记录蟾蜍坐骨神经AP,灵敏度和低通滤波分别设置为0.5V和30Hz D.记录离体豚鼠回肠收缩张力时,时间常数设置为直流 |
D.记录离体豚鼠回肠收缩张力时,时间常数设置为直流 |
2 | 1 | 2010 | 8. 生物医学实验设计的三大原则是 (2.0分) | A.对照、随机、均衡 B.随机、重复、均衡 C.随机、重复、齐同 D.随机、对照、重复 |
D.随机、对照、重复 |
2 | 1 | 2010 | 9. 进行低温对小鼠耐受缺氧的影响实验,正确的分组方法是 (2.0分) | A.随机挑选两只小鼠,一只作为对照组、一只作为实验组 B.随机挑选体重相近的两只小鼠,一只作为对照组、一只作为实验组 C.将两只小鼠随机分配给对照组和实验组 D.将两只体重相近性别相同的小鼠随机分配给对照组和实验组 |
D.将两只体重相近性别相同的小鼠随机分配给对照组和实验组 |
2 | 1 | 2010 | 10. 给l0只高血压动物服用某药前舒张压为115.5±7.0mmHg,服药10天后的舒张压为95.7±10.1mmHg,统计结果为p<0.01,表明该药对降低高血压动物的舒张压 (2.0分) | A.有一定作用 B.有非常好的作用 C.没有作用 D.不能确定 |
A.有一定作用 |
2 | 1 | 2010 | 11. 下列关于时间常数的叙述,正确的是: (2.0分) | A.时间常数越小,下限截止频率越低,对低频成分的滤波程度越大。 B.时间常数越小,下限截止频率越高,对低频成分的滤波程度越大。 C.时间常数越大,下限截止频率越高,对低频成分的滤波程度越小。 D.时间常数越大,下限截止频率越低,对低频成分的滤波程度越大。 |
B.时间常数越小,下限截止频率越高,对低频成分的滤波程度越大。 |
2 | 1 | 2010 | 12. 进行镇痛实验,小鼠的痛阈提高百分率及差异是否显著应采用的统计描述、显著性检验方法是 (2.0分) | A.平均数±标准差、t检验 B.平均数±标准差、?2检验 C.率±标准误、?2检验 D.率±标准误、t检验 |
A.平均数±标准差、t检验 |
2 | 1 | 2010 | 13. 给家兔静脉注射麻醉,下列哪项静脉麻醉的叙述是错误的 (2.0分) | A.快速推注麻醉剂,直至麻醉剂注射完毕 B.快速推注三分之二的麻醉剂后,视兔角膜反射情况酌情注射后续药物 C.注射器按5mL/kg体重剂量抽取200g/L的氨基甲酸乙酯麻醉剂 D.补充麻醉时,一次注射的剂量不超过总量的五分之一 |
A.快速推注麻醉剂,直至麻醉剂注射完毕 |
2 | 1 | 2010 | 14. 当神经冲动到达运动神经末梢时可引起接头前膜的(2.0分) | A.Na+通道关闭 B.Ca2+通道开放 C.K+通道开放 D.Cl‐通道开放 E.Cl‐通道关闭 |
B.Ca2+通道开放 |
2 | 1 | 2010 | 15. 在神经‐肌接头传递过程中,ACh 与ACh门控通道结合使终板膜 (2.0分) | A.对Na+、K+通透性增加,发生超极化 B.对Na+、K+通透性增加,发生去极化 C.仅K+通透性增加,发生超极化 D.对Ca2+通透性增加,发生去极化 E.对Cl‐通透性增加,发生超极化 |
B.对Na+、K+通透性增加,发生去极化 |
2 | 1 | 2010 | 16. 引发微终板电位的原因是 (2.0分) | A.神经末梢连续兴奋 B.神经末梢一次兴奋 C.几百个突触小泡释放的ACh D. 一个突触小泡释放的ACh E.自发释放的一个ACh 分子 |
D. 一个突触小泡释放的ACh |
2 | 1 | 2010 | 17. 当神经冲动到达运动神经末梢时可引起接头前膜的 (2.0分) | A.Na+通道关闭 B.Ca2+通道开放 C. K+通道开放 D.Cl‐通道开放 E.Cl‐通道关闭 |
B.Ca2+通道开放 |
2 | 1 | 2010 | 18. 在神经‐肌接头传递过程中,ACh 与ACh门控通道结合使终板膜 (2.0分) | A.对Na+、K+通透性增加,发生超极化 B.对Na+、K+通透性增加,发生去极化 C.仅K+通透性增加,发生超极化 D.对Ca2+通透性增加,发生去极化 E.对Cl‐通透性增加,发生超极化 |
B.对Na+、K+通透性增加,发生去极化 |
2 | 1 | 2010 | 19. 引发微终板电位的原因是 (2.0分) | A.神经末梢连续兴奋 B.神经末梢一次兴奋 C.几百个突触小泡释放的ACh D.一个突触小泡释放的ACh E.自发释放的一个ACh 分子 |
B.神经末梢一次兴奋 |
2 | 1 | 2010 | 20. 下列关于神经胶质细胞的描述,正确的是 (2.0分) | A.既有树突又有轴突 B.与相邻细胞有突触联系 C.细胞间普遍存在缝隙连接 D.有随细胞外Na+浓度改变的膜电位 E.有产生动作电位的能力 |
C.细胞间普遍存在缝隙连接 |
2 | 1 | 2010 | 21. 神经冲动抵达末梢时,引起递质释放主要有赖于哪种离子的作用(2.0分) | A.Cl‐ B.Ca2+ C.Mg2+ D.Na+ E.K+ |
B.Ca2+ |
2 | 1 | 2010 | 22. 下列关于神经胶质细胞的叙述,错误的是 (2.0分) | A.具有许多突起 B. 具有转运代谢物质的作用 C.具有支持的作用 D.没有轴突 E.没有细胞分裂能力 |
E.没有细胞分裂能力 |
2 | 1 | 2010 | 23. 下列哪种神经递质释放前储存于分泌颗粒中: (2.0分) | A.谷氨酸 B.多巴胺 C.内啡肽 D.甘氨酸 |
C.内啡肽 |
2 | 1 | 2010 | 24. 关于神经递质,下列哪项说法是错误的: (2.0分) | A.氨基酸类和单胺类神经递质常常是在突触前末梢合成的 B.氨基酸类和肽类神经递质可共存于同一个神经元 C.肽类神经递质的释放往往需要一连串的动作电位 D.神经递质的量子释放指的是神经末梢总是大量释放递质 |
D.神经递质的量子释放指的是神经末梢总是大量释放递质 |
2 | 1 | 2010 | 25. 当突触前释放神经递质谷氨酸时,下列哪种兴奋性突触后反应排序符合时间先后次序? (2.0分) | A.突触后膜去极化? AMPA受体激活? Ca2+内流? NMDA受体激活 B.Ca2+内流活?突触后膜去极化?AMPA受体激活?NMDA受体激活 C.AMPA受体激活?突触后膜去极化?NMDA受体兴奋?Ca2+内流 D.NMDA受体激活? Ca2+内流?突触后膜去极化?AMPA受体激活 |
C.AMPA受体激活?突触后膜去极化?NMDA受体兴奋?Ca2+内流 |
2 | 1 | 2010 | 26. 关于锥体外系统的解剖生理描述那一项不正确? (2.0分) | A.调节肌张力,协调肌肉运动 B.维持和调整体态姿势 C.属于上运动神经元 D.担负半自动的刻板动作及反射性的运动 |
C.属于上运动神经元 |
2 | 1 | 2010 | 27. 关于小脑病变那项描述不正确? (2.0分) | A.不同程度肌肉无力 B.共济平衡不良 C.轮替动作不协调 D.精细活动困难 E. |
A.不同程度肌肉无力 |
2 | 1 | 2010 | 28. 关于下运动神经元瘫痪那项是正确? (2.0分) | A.可表现为四肢远端或肢体的某一部位无力,肌张力增高,腱反射减弱或消失,病理反射阴性。 B.可表现为四肢远端或肢体的某一部位无力,肌张力减低,腱反射减弱或消失,病理反射阳性。 C.可表现为四肢远端或肢体的某一部位无力,肌张力增高,腱反射亢进,病理反射阳性,并早期明显肌肉萎缩 D.可表现为四肢远端或肢体的某一部位无力,肌张力增高,腱反射亢进,病理反射阳性,肌电图和神经传导速度正常 E.可表现为四肢远端或肢体的某一部位无力, 肌张力减低,腱反射减弱或消失,病理反射阴性,常伴感觉障碍和受累部位肌肉萎缩。 F. |
E.可表现为四肢远端或肢体的某一部位无力, 肌张力减低,腱反射减弱或消失,病理反射阴性,常伴感觉障碍和受累部位肌肉萎缩。 |
2 | 1 | 2010 | 29. 关于多发性肌炎的描述那项错误? (2.0分) | A.对称性近端肌无力,肌酶升高,肌电图肌原性改变,可伴有肌痛和压痛 B.对称性近端肌无力,肌酶升高,肌电图肌原性改变,可伴有肌萎缩, C.对称性近端肌无力,肌酶升高,肌电图肌原性改变,可伴有呼吸表浅、呼吸困难, D.对称性近端肌无力,肌酶升高,肌电图肌原性改变,可伴有肌阵挛, E.对称性近端肌无力,肌酶升高,肌电图肌原性改变,可伴有吞咽和构音障碍 F. |
D.对称性近端肌无力,肌酶升高,肌电图肌原性改变,可伴有肌阵挛, |
2 | 1 | 2010 | 30. 关于皮肌炎和多发性肌炎肌酶升高描述那项错误? (2.0分) | A.绝大多数病人在病程某一阶段可出现肌酶活性增高,CK特异强,只有在肌肉病变时升高 B.绝大多数病人在病程某一阶段可出现肌酶活性增高,CK最敏感。 C.绝大多数病人在病程某一阶段可出现肌酶活性增高,是诊断、疗效监测及预后的评价指标 D.绝大多数病人在病程某一阶段可出现肌酶活性增高,晚期肌萎缩后肌酶不再释放 E.绝大多数病人在病程某一阶段可出现肌酶活性增高,包括肌酸激酶(CK)、醛缩酶(ALD)、乳酸脱氢酶(LDH)、门冬氨酸氨基转移酶(AST)。 F. |
A.绝大多数病人在病程某一阶段可出现肌酶活性增高,CK特异强,只有在肌肉病变时升高 |
2 | 1 | 2010 | 31. 患者,男,18岁 参加运动会400米跑步后次日,全身肌肉疼痛,痉挛,无力,检查发现低血糖,高血脂,肌红蛋白尿.可能最大的疾病诊断是? (2.0分) | A.脂质沉积性肌病 B.糖原累积病 C.线粒体肌病 D.周期性麻痹 E.重症肌无力 F.单纯的横纹肌溶解症 |
B.糖原累积病 |
2 | 1 | 2010 | 32. 关于进行性肌营养不良描述那项错误? (2.0分) | A.遗传性肌肉变性疾病 B.遗传方式有常染色体显性和隐性或X连锁隐性遗传 C.不能耐受运动和近端肌无力,肌肉痉挛和疼痛为主要表现 D.电生理为肌源性损害,神经传导速度正常 E.病理特点:进行性的肌纤维坏死,再生和脂肪及结缔组织增生,无异常代谢产物堆积 F. |
C.不能耐受运动和近端肌无力,肌肉痉挛和疼痛为主要表现 |
2 | 1 | 2010 | 33. 患者男性,53岁,出现进行性的四肢肌肉无力和萎缩3年,家族中无类似发病者。肌电图检查提示肌源性损害,肌酶检查发现CK轻度升高,给予肾上腺皮质激素治疗无效。最有可能诊断是? (2.0分) | A.皮肌炎 B.多发性肌炎 C.线粒体肌病 D.包涵体肌炎 E.Lambert-Eaton 综合征 F.糖原累积病 |
D.包涵体肌炎 |
2 | 1 | 2010 | 34. Duchenne muscular dystrophy 和Becker muscular dystrophy 的鉴别点那项错误。(2.0分) | A.Duchenne 型为X连锁隐性遗传 和Becker 型呈常染色体显性遗传 B.Duchenne 型发病年龄相对早 C.Duchenne 型进展快,预后差 D.Duchenne 型易伴发心肌和认知功能损害 E.肌肉病理Duchenne 型dystrophin染色缺失Becker dystrophin染色减少 F. |
A.Duchenne 型为X连锁隐性遗传 和Becker 型呈常染色体显性遗传 |
2 | 1 | 2010 | 35. 线粒体病脑肌病(MELAS)的主要临床表现:(2.0分) | A.Stroke-like episodes,ragged-red fibres and/or lactic acidosis and bilateral deafness B.Stroke-like episodes,ragged-red fibres and/or lactic acidosis and Diabetes mellitus C.Stroke-like episodes,ragged-red fibres and/or lactic acidosis and cardiomyopathy D.Stroke-like episodes, ragged-red fibres and/or lactic acidosis and seizures and/or dementia E.Stroke-like episodes,ragged-red fibres and/or lactic acidosis and cerebellar ataxia F. |
D.Stroke-like episodes, ragged-red fibres and/or lactic acidosis and seizures and/or dementia |
2 | 1 | 2010 | 36. 关于引起神经-肌肉接头(NMJ)传递障碍描述那项错误?(2.0分) | A.突触前膜释放乙酰胆碱障碍 B.突触间隙胆碱酯酶活性改变 C.突触后膜乙酰胆碱受体数量减少,功能抑制 D.骨骼肌细胞膜的离子通道功能失常 E.高镁血症可影响突触前膜释放乙酰胆碱 F. |
D.骨骼肌细胞膜的离子通道功能失常 |
2 | 1 | 2010 | 37. 重症肌无力危象的抢救最重要是?(2.0分) | A.区分肌无力性危象,胆碱能危象或反拗危象,以便对因治疗 B.保持呼吸道通畅,呼吸机支持 C.积极控制感染 D.大剂量免疫球蛋白或血浆置换 E.大剂量激素冲击治疗 F. |
B.保持呼吸道通畅,呼吸机支持 |
2 | 1 | 2010 | 38. 重症肌无力辅助诊断那项最具价值?(2.0分) | A.疲劳试验和药物试验 B.系统免疫学指标和甲状腺功能检查 C.乙酰胆碱受体抗体(AchR-ab)和重复电刺激(RNS)单纤维肌电图(SFEMG)测定 D.胸腺CT和MRI影像检查 E.普通肌电图和神经传导 F.重复电刺激(RNS)和 药物试验 |
C.乙酰胆碱受体抗体(AchR-ab)和重复电刺激(RNS)单纤维肌电图(SFEMG)测定 |
2 | 1 | 2010 | 39. 针对重症肌无力免疫机制药物治疗起效最快的方法是? (2.0分) | A.大剂量免疫球蛋白或血浆置换 B.免疫抑制剂 C.激素治疗 D.抗胆碱酯酶 E. |
A.大剂量免疫球蛋白或血浆置换 |
2 | 1 | 2010 | 40. 32岁,亚洲男性,运动后次日早晨醒来发现双下肢无力伴大腿酸胀;体检,意识清楚,颅神经正常,下肢对称性迟缓性瘫痪,大小便正常,感觉正常,反射正常。无家族遗传病史。最有可能的诊断是 (2.0分) | A.吉兰-巴雷综合征 B.肉毒毒素中毒 C.甲亢伴低钾型周期性麻痹 D.线粒体肌病 E.高钾型周期性麻痹 F.正常血钾型周期性麻痹 |
C.甲亢伴低钾型周期性麻痹 |
2 | 1 | 2010 | 41. 一病人前臂受伤后屈腕力明显减弱,无名指和小指不能屈曲,拇指不能内收,手掌小鱼际区和小指皮肤感觉丧失,试问可能是什么神经受到了损伤 (2.0分) | A.正中神经 B.腋神经 C.桡神经 D.尺神经 E.肌皮神经 |
D.尺神经 |
2 | 1 | 2010 | 42. 小腿上段骨折后,病人出现足不能背屈、不能外翻、不能伸趾,行走困难,出现"跨阈步态";小腿前外侧、足背和趾背外侧区感觉障碍。试问什么神经的损伤导致了以上症状的发生(2.0分) | A.胫神经 B.臀上神经 C.腓浅神经 D.臀下神经 E.腓总神经 F. |
E.腓总神经 |
2 | 1 | 2010 | 43. 乳腺癌手术清扫淋巴结时,最容易损伤一条神经,此神经损伤后手臂不能上举,并出现"翼状肩",该神经是 (2.0分) | A.胸背神经 B.胸长神经 C.胸内侧神经 D.胸外侧神经 E.肩胛下神经 |
B.胸长神经 |
2 | 1 | 2010 | 44. Pilocarpine(毛果芸香碱)对眼的调节作用是 (2.0分) | A.睫状肌松弛,悬韧带拉紧,晶状体变扁平 B.睫状肌松弛,悬韧带放松,晶状体变凸 C.睫状肌收缩,悬韧带拉紧,晶状体变凸 D.睫状肌收缩,悬韧带拉紧,晶状体变扁平 E.睫状肌收缩,悬韧带放松,晶状体变凸 |
E.睫状肌收缩,悬韧带放松,晶状体变凸 |
2 | 1 | 2010 | 45. 全身麻醉前给atropine(阿托品)的目的是 (2.0分) | A.镇静 B.增强麻醉药的作用 C.减少患者对术中的不良刺激的记忆 D.减少呼吸道腺体分泌 E.松弛骨骼肌 |
D.减少呼吸道腺体分泌 |
2 | 1 | 2010 | 46. 下述哪一类病人禁用atropine? (2.0分) | A.有机磷酸酯类中毒 B.窦性心动过缓 C.胃肠绞痛 D.前列腺肥大 E.虹膜睫状体炎 |
D.前列腺肥大 |
2 | 1 | 2010 | 47. Anisodamine(山莨菪碱)的特点是 (2.0分) | A.可治疗青光眼 B.可治疗晕动病 C.可治疗感染性休克 D.可用于麻醉前给药 E.可缓解震颤麻痹的症状 |
C.可治疗感染性休克 |
2 | 1 | 2010 | 48. 下列哪一项关于succinylcholine(scoline)的叙述是正确的? (2.0分) | A.是一种非除极化型肌松药 B.一次给药作用可维持2小时以上 C.可引起高血钾症 D.过量可用新斯的明对抗 E.可口服维持肌松作用 |
C.可引起高血钾症 |
2 | 1 | 2010 | 49. 易逆性抗胆碱酯酶药的作用不包括 (2.0分) | A.兴奋骨骼肌 B.兴奋胃肠道平滑肌 C.兴奋心脏 D.促进腺体分泌 E.缩瞳 |
C.兴奋心脏 |
2 | 1 | 2010 | 50. 碘解磷定(pyraloxime methiodide)治疗有机磷酸酯类中毒的作用机制是 (2.0分) | A.能阻断M胆碱受体的作用 B.能阻断N胆碱受体的作用 C.能直接对抗体内积聚的ACh的作用 D.能恢复AChE水解ACh的活性 E.能与AChE结合而抑制酶的活性 |
D.能恢复AChE水解ACh的活性 |
2 | 1 | 2011 | 1. 下列突触传递相关的电化学时间哪些耗时最长?(2.0分) | A.轴突上的动作电位传导 B.突触囊泡与突触前末梢的膜融合 C.递质在突触间隙的清除 D.突触后递质门控通道的开放 |
C.递质在突触间隙的清除 |
2 | 1 | 2011 | 2. 关于突触整合,下列哪项描述是错误的:(2.0分) | A.同时兴奋的钾通道有利于EPSP向胞体扩散 B.同时多个突触兴奋产生的EPSP相加称为时间相加 C.与放电起始区(轴丘)距离近的兴奋性突触,更易触发动作电位发放 D.抑制性突触的活动不利于动作电位的产生 |
A.同时兴奋的钾通道有利于EPSP向胞体扩散 |
2 | 1 | 2011 | 3. 下列哪种神经递质不是通过重摄取途径清除的?(2.0分) | A.谷氨酸(Glutamate) B.5-羟色胺(Serotonin) C.乙酰胆碱(Acetylcholine) D.多巴胺(Dopamine) |
C.乙酰胆碱(Acetylcholine) |
2 | 1 | 2011 | 4. 下列哪种神经递质释放前储存于分泌颗粒中(2.0分) | A.谷氨酸 B.多巴胺 C.内啡肽 D.甘氨酸 |
C.内啡肽 |
2 | 1 | 2011 | 5. Your patient is a 5-year-old girl who appears to have Duchenne muscular dystrophy. What is the most likely genetic explanation for this disease in a girl? (2.0分) | A.She has a 46, XY karyotype with sex reversal. B.She has two independent DMD mutations. C.She has skewed X inactivation. D.She has a dominant negative mutation. E.This diagnosis is impossible. F. |
C.She has skewed X inactivation. |
2 | 1 | 2011 | 6. A mutation causing an abnormality in presenilin(早老素) that disrupts amyloid(淀粉样蛋白) production or function has been associated with(2.0分) | A.cystic fibrosis B.Duchenne muscular dystrophy C.Marfan syndrome D.Wilson disease E.none of the above F. |
E.none of the above |
2 | 1 | 2011 | 7. Which of the following genetic diseases results from a mutation that prevents a protein from forming? (2.0分) | A.Duchenne muscular dystrophy B.Hemophilia A C.Epidermolysis bullosa(大疱性表皮松解症) D.Huntington disease E.Fragile X syndrome F. |
A.Duchenne muscular dystrophy |
2 | 1 | 2011 | 8. Which one of the following types of mutation is most likely to lead to premature termination of translation? (2.0分) | A.Insertion of a single base into DNA B.Deletion of three bases from DNA C.Deletion of an entire gene D.Single base change in a promoter E.Exon skip F. |
A.Insertion of a single base into DNA |
2 | 1 | 2011 | 9. 生物医学实验设计的三大原则是 (2.0分) | A.对照、随机、均衡 B.随机、重复、均衡 C.随机、重复、齐同 D.随机、对照、重复 |
D.随机、对照、重复 |
2 | 1 | 2011 | 10. 学术论文的期刊参考文献正确书写格式为(2.0分) | A.作者.文题[J].刊名.年份,卷(期):起止页. B.文题[J].刊名.年份,卷(期):起止页.作者. C.作者.文题[J].刊名.年份:起止页. D.文题[J].刊名.年份,卷(期). |
A.作者.文题[J].刊名.年份,卷(期):起止页. |
2 | 1 | 2011 | 11. 生理科学实验中观察和记录的生物信号的基本特性为(2.0分) | A.振幅低、频率低、源阻抗大 B.振幅低、频率高、源阻抗大 C.振幅高、频率高、源阻抗大 D.振幅低、频率高、源阻抗小 |
A.振幅低、频率低、源阻抗大 |
2 | 1 | 2011 | 12. 关于时间常数,描述正确的是(2.0分) | A.时间常数越小,下限截止频率越低,对低频成分的滤波程度越大。 B.时间常数越小,下限截止频率越高,对低频成分的滤波程度越大。 C.时间常数越大,下限截止频率越高,对低频成分的滤波程度越小。 D.时间常数越大,下限截止频率越低,对低频成分的滤波程度越大。 |
B.时间常数越小,下限截止频率越高,对低频成分的滤波程度越大。 |
2 | 1 | 2011 | 13. 任氏液的理化特性与蟾蜍的(2.0分) | A.体液相似 B.血液相似 C.组织液相似 D.细胞内液相似 |
C.组织液相似 |
2 | 1 | 2011 | 14. Pilocarpine对眼的调节作用是 (2.0分) | A.睫状肌松弛,悬韧带拉紧,晶状体变扁平 B.睫状肌松弛,悬韧带放松,晶状体变凸 C.睫状肌收缩,悬韧带拉紧,晶状体变凸 D.睫状肌收缩,悬韧带拉紧,晶状体变扁平 E.睫状肌收缩,悬韧带放松,晶状体变凸 |
E.睫状肌收缩,悬韧带放松,晶状体变凸 |
2 | 1 | 2011 | 15. 全身麻醉前给atropine的目的是 (2.0分) | A.镇静 B.增强麻醉药的作用 C.减少患者对术中的不良刺激的记忆 D.减少呼吸道腺体分泌 E.松弛骨骼肌 |
D.减少呼吸道腺体分泌 |
2 | 1 | 2011 | 16. 下述哪一类病人禁用atropine? (2.0分) | A.有机磷酸酯类中毒 B.窦性心动过缓 C.胃肠绞痛 D.前列腺肥大 E.虹膜睫状体炎 |
D.前列腺肥大 |
2 | 1 | 2011 | 17. 关于atropine的叙述,下列哪一项是错误的? (2.0分) | A.可用于治疗过敏性休克 B.可用于治疗感染性休克 C.可用于治疗缓慢型心律失常 D.可用于全身麻醉前给药 E.可用于缓解内脏绞痛 |
A.可用于治疗过敏性休克 |
2 | 1 | 2011 | 18. 下列哪一项关于succinylcholine(scoline)的叙述是正确的? (2.0分) | A.是一种非除极化型肌松药 B.一次给药作用可维持2小时以上 C.可引起高血钾症 D.过量可用新斯的明对抗 E.可口服维持肌松作用 |
C.可引起高血钾症 |
2 | 1 | 2011 | 19. 易逆性抗胆碱酯酶药的作用不包括 (2.0分) | A.兴奋骨骼肌 B.兴奋胃肠道平滑肌 C.兴奋心脏 D.促进腺体分泌 E.缩瞳 |
C.兴奋心脏 |
2 | 1 | 2011 | 20. Neostigmine不用于下列哪一种情况 (2.0分) | A.阵发性室上性心动过速 B.重症肌无力 C.手术后肠胀气 D.支气管哮喘 E.非除极化型肌松药过量中毒 |
D.支气管哮喘 |
2 | 1 | 2011 | 21. 碘解磷定(pyraloxime methiodide)治疗有机磷酸酯类中毒的作用机制是 (2.0分) | A.能阻断M胆碱受体的作用 B.能阻断N胆碱受体的作用 C.能直接对抗体内积聚的ACh的作用 D.能恢复AChE水解ACh的活性 E.能与AChE结合而抑制酶的活性 |
D.能恢复AChE水解ACh的活性 |
2 | 1 | 2011 | 22. 关于有机磷酸酯类中毒的特点,下述哪一项是错误的? (2.0分) | A.形成磷酰化胆碱酯酶,使胆碱酯酶失活 B.形成磷酰化胆碱酯酶,使胆碱酯酶活性过强 C.严重中毒时可出现N样症状及中枢症状 D.应当及时抢救,以免使磷酰化胆碱酯酶"老化" E.应当早期大量重复使用atropine进行抢救 |
B.形成磷酰化胆碱酯酶,使胆碱酯酶活性过强 |
2 | 1 | 2011 | 23. 重症肌无力患者的骨骼肌对运动神经冲动的反应降低是由于(2.0分) | A.递质含量减少 B.递质释放量减 C.胆碱酯酶活性增高 D.受体数目减少或功能障碍 E.微终板电位减小 |
D.受体数目减少或功能障碍 |
2 | 1 | 2011 | 24. 肌丝滑行理论的直接证据是骨骼肌收缩时(2.0分) | A.明带缩短,暗带和H 带长度不变 B.明带和H 带缩短,暗带长度不变 C.暗带缩短,明带和H 带长度不变 D.明带、暗带和H 带长度均缩短 E.明带、暗带和H 带长度均不变 |
B.明带和H 带缩短,暗带长度不变 |
2 | 1 | 2011 | 25. 在骨骼肌细胞兴奋‐收缩耦联过程中,胞质内的Ca2+来自(2.0分) | A.横管膜上电压门控Ca2+通道引起的胞外Ca2+内流 B.细胞膜上NMDA 受体通道开放引起的胞外Ca2+内流 C.肌质网上Ca2+释放通道开放引起的胞内Ca2+释放 D.肌质网上Ca2+泵的反向转运 E.线粒体内Ca2+的释放 |
C.肌质网上Ca2+释放通道开放引起的胞内Ca2+释放 |
2 | 1 | 2011 | 26. 下列关于神经兴奋传导的叙述,错误的是(2.0分) | A.动作电位可沿细胞膜传导到整个细胞 B.传导的方式是通过产生局部电流来刺激未兴奋部位,使之也出现动作电位 C.动作电位的幅度随传导距离增加而衰减 D.传导速度与神经纤维的直径有关 E.传导速度与温度有关 |
C.动作电位的幅度随传导距离增加而衰减 |
2 | 1 | 2011 | 27. 神经冲动抵达末梢时,引起递质释放主要有赖于哪种离子的作用?(2.0分) | A.Cl‐ B.Ca2+ C.Mg2+ D.Na+ E.K+ |
B.Ca2+ |
2 | 1 | 2011 | 28. 下列哪种肌肉蛋白质具有ATP 酶活性(2.0分) | A.肌动蛋白 B.肌钙蛋白 C.肌球蛋白 D.原肌球蛋白 E.调节蛋白 |
C.肌球蛋白 |
2 | 1 | 2011 | 29. 神经‐骨骼肌接头处兴奋传递的特点不包括(2.0分) | A.1 :1 的关系 B.时间延搁 C.双向传递 D.易受环境因素的影响 E.易受药物的影响 |
C.双向传递 |
2 | 1 | 2011 | 30. 骨骼肌细胞的钙释放通道主要位于下列何处膜结构上?(2.0分) | A.连接肌质网 B.纵形肌质网 C.横管 D.运动终板 E.线粒体 |
A.连接肌质网 |
2 | 1 | 2011 | 31. 关于锥体外系统的解剖生理描述那一项不正确?(2.0分) | A.调节肌张力,协调肌肉运动 B.维持和调整体态姿势 C.属于上运动神经元 D.担负半自动的刻板动作及反射性的运动 E. |
C.属于上运动神经元 |
2 | 1 | 2011 | 32. 关于小脑病变那项描述不正确?(2.0分) | A.不同程度肌肉无力 B.共济平衡不良 C.轮替动作不协调 D.精细活动困难 E. |
A.不同程度肌肉无力 |
2 | 1 | 2011 | 33. 关于下运动神经元瘫痪那项是正确?(2.0分) | A.可表现为四肢远端或肢体的某一部位无力,肌张力增高,腱反射减弱或消失,病理反射阴性。 B.可表现为四肢远端或肢体的某一部位无力,肌张力减低,腱反射减弱或消失,病理反射阳性。 C.可表现为四肢远端或肢体的某一部位无力,肌张力增高,腱反射亢进,病理反射阳性,并早期明显肌肉萎缩 D.可表现为四肢远端或肢体的某一部位无力,肌张力增高,腱反射亢进,病理反射阳性,肌电图和神经传导速度正常 E.可表现为四肢远端或肢体的某一部位无力, 肌张力减低,腱反射减弱或消失,病理反射阴性,常伴感觉障碍和受累部位肌肉萎缩。 F. |
E.可表现为四肢远端或肢体的某一部位无力, 肌张力减低,腱反射减弱或消失,病理反射阴性,常伴感觉障碍和受累部位肌肉萎缩。 |
2 | 1 | 2011 | 34. 关于多发性肌炎的描述那项错误?(2.0分) | A.对称性近端肌无力,肌酶升高,肌电图肌原性改变,可伴有肌痛和压痛 B.对称性近端肌无力,肌酶升高,肌电图肌原性改变,可伴有肌萎缩, C.对称性近端肌无力,肌酶升高,肌电图肌原性改变,可伴有呼吸表浅、呼吸困难, D.对称性近端肌无力,肌酶升高,肌电图肌原性改变,可伴有肌阵挛, E.对称性近端肌无力,肌酶升高,肌电图肌原性改变,可伴有吞咽和构音障碍 F. |
D.对称性近端肌无力,肌酶升高,肌电图肌原性改变,可伴有肌阵挛, |
2 | 1 | 2011 | 35. 关于皮肌炎和多发性肌炎肌酶升高描述那项错误?(2.0分) | A.绝大多数病人在病程某一阶段可出现肌酶活性增高,CK特异强,只有在肌肉病变时升高 B.绝大多数病人在病程某一阶段可出现肌酶活性增高,CK最敏感。 C.绝大多数病人在病程某一阶段可出现肌酶活性增高,是诊断、疗效监测及预后的评价指标 D.绝大多数病人在病程某一阶段可出现肌酶活性增高,晚期肌萎缩后肌酶不再释放 E.绝大多数病人在病程某一阶段可出现肌酶活性增高,包括肌酸激酶(CK)、醛缩酶(ALD)、乳酸脱氢酶(LDH)、门冬氨酸氨基转移酶(AST)。 |
A.绝大多数病人在病程某一阶段可出现肌酶活性增高,CK特异强,只有在肌肉病变时升高 |
2 | 1 | 2011 | 36. 患者,男,18岁 参加运动会400米跑步后次日全身肌肉疼痛,痉挛,无力,检查发现低血糖,高血脂,肌红蛋白尿.可能最大的疾病诊断是?(2.0分) | A.脂质沉积性肌病 B.糖原累积病 C.线粒体肌病 D.周期性麻痹 E.重症肌无力 F.单纯的横纹肌溶解症 |
B.糖原累积病 |
2 | 1 | 2011 | 37. 关于进行性肌营养不良描述那项错误?(2.0分) | A.遗传性肌肉变性疾病 B.遗传方式有常染色体显性和隐性或X连锁隐性遗传 C.不能耐受运动和近端肌无力,肌肉痉挛和疼痛为主要表现 D.电生理为肌源性损害,神经传导速度正常 E.病理特点:进行性的肌纤维坏死,再生和脂肪及结缔组织增生,无异常代谢产物堆积 F. |
C.不能耐受运动和近端肌无力,肌肉痉挛和疼痛为主要表现 |
2 | 1 | 2011 | 38. 患者男性,53岁,出现进行性的四肢肌肉无力和萎缩3年,家族中无类似发病者。肌电图检查提示肌源性损害,肌酶检查发现CK轻度升高,给予肾上腺皮质激素治疗无效。最有可能诊断是?(2.0分) | A.皮肌炎 B.多发性肌炎 C.线粒体肌病 D.包涵体肌炎 E.Lambert-Eaton 综合征 F.糖原累积病 |
D.包涵体肌炎 |
2 | 1 | 2011 | 39. Duchenne muscular dystrophy 和Becker muscular dystrophy 的鉴别点那项错误。 (2.0分) | A.Duchenne 型为X连锁隐性遗传 和Becker 型呈常染色体显性遗传 B.Duchenne 型发病年龄相对早 C.Duchenne 型进展快,预后差 D.Duchenne 型易伴发心肌和认知功能损害 E.肌肉病理Duchenne 型dystrophin染色缺失Becker dystrophin染色减少 F. |
A.Duchenne 型为X连锁隐性遗传 和Becker 型呈常染色体显性遗传 |
2 | 1 | 2011 | 40. 线粒体病脑肌病(MELAS)的主要临床表现(2.0分) | A.Stroke-like episodes,ragged-red fibres and/or lactic acidosis and bilateral deafness B.Stroke-like episodes,ragged-red fibres and/or lactic acidosis and Diabetes mellitus C.Stroke-like episodes,ragged-red fibres and/or lactic acidosis and cardiomyopathy D.Stroke-like episodes, ragged-red fibres and/or lactic acidosis and seizures and/or dementia E.Stroke-like episodes,ragged-red fibres and/or lactic acidosis and cerebellar ataxia F. |
D.Stroke-like episodes, ragged-red fibres and/or lactic acidosis and seizures and/or dementia |
2 | 1 | 2011 | 41. 关于引起神经-肌肉接头(NMJ)传递障碍描述那项错误?(2.0分) | A.突触前膜释放乙酰胆碱障碍 B.突触间隙胆碱酯酶活性改变 C.突触后膜乙酰胆碱受体数量减少,功能抑制 D.骨骼肌细胞膜的离子通道功能失常 E.高镁血症可影响突触前膜释放乙酰胆碱 F. |
D.骨骼肌细胞膜的离子通道功能失常 |
2 | 1 | 2011 | 42. 重症肌无力危象的抢救最重要是?(2.0分) | A.区分肌无力性危象,胆碱能危象或反拗危象,以便对因治疗 B.保持呼吸道通畅,呼吸机支持 C.积极控制感染 D.大剂量免疫球蛋白或血浆置换 E.大剂量激素冲击治疗 F. |
B.保持呼吸道通畅,呼吸机支持 |
2 | 1 | 2011 | 43. 管理小指皮肤感觉的神经是(2.0分) | A.正中神经 B.桡神经 C.尺神经 D.腋神经 E.肌皮神经 |
C.尺神经 |
2 | 1 | 2011 | 44. 脐平面的皮肤感觉神经是(2.0分) | A.第8对胸神经 B.第9对胸神经 C.第10对胸神经 D.第1l对胸神经 E.第12对胸神经 |
C.第10对胸神经 |
2 | 1 | 2011 | 45. 病人出现呼吸困难症状,伴有窒息感,经检查发现腹式呼吸明显减弱。试问可能是什么神经受到了损伤 (2.0分) | A.膈神经 B.迷走神经 C.舌咽神经 D.内脏大神经 E.肋间神经 |
A.膈神经 |
2 | 1 | 2011 | 46. 确定某因素与某种疾病的关联强度的指标是(2.0分) | A.相对危险度 B. 统计学检验的χ2值 C.某人群某病的患病率 D.某人群某病的发病率 E.某人群某病的死亡率 |
A.相对危险度 |
2 | 1 | 2011 | 47. 下列哪种研究方法可以研究人群某疾病的自然史 (2.0分) | A.现况研究 B.生态学研究 C. 队列研究 D.病例对照研究 E. 以上都不是 |
C. 队列研究 |
2 | 1 | 2011 | 48. 对于队列研究的错误观点是(2.0分) | A.人力、物力花费较大 B.通常要考虑失访和混杂偏倚的影响 C.可以直接计算暴露与疾病的关联强度 D.若疾病罕见而暴露因素常见,则适宜进行队列研究 E.若疾病常见且暴露因素常见,则适宜进行队列研究 F. |
D.若疾病罕见而暴露因素常见,则适宜进行队列研究 |
2 | 1 | 2011 | 49. 在一次队列研究中,暴露组和非暴露组各5000人,暴露组40人发病,非暴露组20人发病,则相对危险度为(2.0分) | A.1.0 B.2.0 C.3.0 D. 4.0 E. 无法计算 |
B.2.0 |
2 | 1 | 2011 | 50. 队列研究中的调查对象应选择(2.0分) | A.在患病人群中,选择有、无某种暴露因素的两个组 B.在患病人群中,选择有某种暴露因素的为一组,在未患病人群中选择无某种暴露因素的为另一组 C.在未患病人群中选择有某种暴露因素的为一组,在患病人群中选择无某种暴露因素的为另一组 D.在未患病人群中,选择有、无某种暴露因素的两组 E.任选有、无暴露的两个组 |
D.在未患病人群中,选择有、无某种暴露因素的两组 |
2 | 1 | 2012 | 1. Your patient is a 5-year-old girl who appears to have Duchenne muscular dystrophy(DMD). What is the most likely genetic explanation for this disease in a girl? (2.0分) | A.She has a 46, XY karyotype with sex reversal. B.She has two independent DMD mutations. C.She has skewed X inactivation. D.She has a dominant negative mutation. E.This diagnosis is impossible. |
C.She has skewed X inactivation. |
2 | 1 | 2012 | 2. A mutation causing an abnormality in presenilin(早老素) that disrupts amyloid(淀粉样蛋白) production or function has been associated with (2.0分) | A.cystic fibrosis B.Duchenne muscular dystrophy C.Marfan syndrome D.Wilson disease E.none of the above |
E.none of the above |
2 | 1 | 2012 | 3. Which of the following genetic diseases results from a mutation that prevents a protein from forming? (2.0分) | A.Duchenne muscular dystrophy B.Hemophilia A C.Epidermolysis bullosa(大疱性表皮松解症) D.Huntington disease E.Fragile X syndrome |
A.Duchenne muscular dystrophy |
2 | 1 | 2012 | 4. Which one of the following types of mutation is most likely to lead to premature termination of translation?(哪种类型的突变可导致翻译提前终止?)(2.0分) | A.Insertion of a single base into DNA B.Deletion of three bases from DNA C.Deletion of an entire gene D.Single base change in a promoter E.Exon skip |
A.Insertion of a single base into DNA |
2 | 1 | 2012 | 5. Atropine对眼睛的作用是(2.0分) | A.扩瞳,升高眼内压,视远物模糊 B.扩瞳,升高眼内压,视近物模糊 C.扩瞳,降低眼内压,视近物模糊 D.扩瞳,降低眼内压,视远物模糊 E.缩瞳,升高眼内压,视近物模糊 |
B.扩瞳,升高眼内压,视近物模糊 |
2 | 1 | 2012 | 6. 有机磷酸酯类急性中毒时,atropine不能缓解下列哪一项症状? (2.0分) | A.瞳孔缩小 B.出汗 C.恶心,呕吐 D.呼吸困难 E.肌肉颤动 |
E.肌肉颤动 |
2 | 1 | 2012 | 7. 中枢作用较强的胆碱受体阻断药是 (2.0分) | A.atropine B.anisodamine C.scopolamine D.homatropine E.propantheline |
C.scopolamine |
2 | 1 | 2012 | 8. Succinylcholine松弛骨骼肌的主要机制是 (2.0分) | A.抑制中枢多突触反射 B.抑制脊髓丁运动神经元 C.与ACh竞争运动终板膜上的NM受体 D.使运动终板膜产生持久的去极化 E.减少运动神经末梢ACh的释放 |
D.使运动终板膜产生持久的去极化 |
2 | 1 | 2012 | 9. 筒箭毒碱(d-tubocurarine)骨骼肌松弛作用的机制是 (2.0分) | A.阻断M受体 B.阻断NN受体 C.阻断NM受体 D.阻断α受体 E.引起骨骼肌运动终板持久除极化 |
C.阻断NM受体 |
2 | 1 | 2012 | 10. 关于neostigmine的描述,错误的是 (2.0分) | A.口服吸收差,可皮下注射给药 B.易进人中枢神经系统 C.可促进胃肠道的运动 D.常用于治疗重症肌无力 E.禁用于机械性肠梗阻 |
B.易进人中枢神经系统 |
2 | 1 | 2012 | 11. 有机磷酸酯类中毒时产生M样症状的原因是 (2.0分) | A.胆碱能神经递质释放减少 B.胆碱能神经递质释放增加 C.胆碱能神经递质破坏减慢 D.药物排泄减慢 E.M受体的敏感性增加 |
C.胆碱能神经递质破坏减慢 |
2 | 1 | 2012 | 12. 治疗中、重度有机磷酸酯类中毒应当采用 (2.0分) | A.Atropine + pyraloxime chloride B.Atropine + neostigmine C.Atropine + scopolamine D.Atropine E.Pyraloxime chloride |
A.Atropine + pyraloxime chloride |
2 | 1 | 2012 | 13. Neostigmine的药理作用特点描述正确的是 (2.0分) | A.对腺体的兴奋作用强 B.对支气管平滑肌兴奋作用强 C.对胃肠道平滑肌兴奋作用弱 D.对膀胱平滑肌兴奋作用弱 E.对骨骼肌兴奋作用强 |
E.对骨骼肌兴奋作用强 |
2 | 1 | 2012 | 14. 运动神经兴奋时,何种离子进入轴突末梢的量与囊泡释放量呈正变关系? (2.0分) | A.Mg2+ B.Ca2+ C.Na+ D.K+ E.Cl‐ |
B.Ca2+ |
2 | 1 | 2012 | 15. 在神经‐骨骼肌接头部位,囊泡释放ACh所引起的膜电位变化称(2.0分) | A.突触后电位 B.接头后电位 C.局部电位 D.微终板电位 E.终板电位 |
E.终板电位 |
2 | 1 | 2012 | 16. 神经‐骨骼肌接头处兴奋传递的特点不包括 (2.0分) | A.1 :1 的关系 B.时间延搁 C.双向传递 D.易受环境因素的影响 E.易受药物的影响 |
C.双向传递 |
2 | 1 | 2012 | 17. 筒箭毒可被作为肌松剂应用,是由于能在终板膜部位(2.0分) | A.激活胆碱酯酶 B.与ACh 竞争结合位点 C.与ACh 结合成复合物 D.抑制神经末梢Ca2+内流 E.减少囊泡释放Ach |
B.与ACh 竞争结合位点 |
2 | 1 | 2012 | 18. 肌丝滑行理论的直接证据是骨骼肌收缩时(2.0分) | A.明带缩短,暗带和H 带长度不变 B.明带和H 带缩短,暗带长度不变 C.暗带缩短,明带和H 带长度不变 D.明带、暗带和H 带长度均缩短 E.明带、暗带和H 带长度均不变 |
B.明带和H 带缩短,暗带长度不变 |
2 | 1 | 2012 | 19. 下列哪种肌肉蛋白质具有ATP 酶活性(2.0分) | A.肌动蛋白 B.肌钙蛋白 C.肌球蛋白 D.原肌球蛋白 E.调节蛋白 |
C.肌球蛋白 |
2 | 1 | 2012 | 20. 骨骼肌收缩时释放到肌浆中的Ca2+被何处的钙泵转运? (2.0分) | A.横管 B.肌膜 C.线粒体膜 D.肌质网膜 E.粗面内质网 |
D.肌质网膜 |
2 | 1 | 2012 | 21. 下列关于神经胶质细胞的描述,正确的是 (2.0分) | A.既有树突又有轴突 B.细胞间普遍存在缝隙连接 C.有随细胞外Na+浓度改变的膜电位 D.有产生动作电位的能力 E.细胞间具有突触连接 |
B.细胞间普遍存在缝隙连接 |
2 | 1 | 2012 | 22. 下列哪种神经胶质细胞可在中枢神经系统中形成神经纤维的髓鞘 (2.0分) | A.星形胶质细胞 B.少突胶质细胞 C.施万细胞 D.卫星细胞 E.小胶质细胞 |
B.少突胶质细胞 |
2 | 1 | 2012 | 23. 有髓神经纤维的传导特点是(2.0分) | A.传导速度慢 B.跳跃式传导 C.衰减性传导 D.单向传导 E.电缆式传导 |
B.跳跃式传导 |
2 | 1 | 2012 | 24. 关于锥体外系统受损害症状和体征,那一项不正确?(2.0分) | A.肌张力减低或升高 B.不自主运动 C.动作迟缓和减少 D.姿势反射不良 E.病理反射阳性 |
E.病理反射阳性 |
2 | 1 | 2012 | 25. 关于上运动神经元瘫痪那项是正确?(2.0分) | A.表现为偏瘫,单瘫,截瘫或四肢瘫,肌张力增高,腱反射减弱或消失,病理反射阳性。 B.表现为偏瘫,单瘫,截瘫或四肢瘫,肌张力减低,腱反射减弱或消失,病理反射阳性。 C.表现为偏瘫,单瘫,截瘫或四肢瘫,肌张力增高,腱反射亢进,病理反射阳性,并早期明显肌肉萎缩 D.表现为偏瘫,单瘫,截瘫或四肢瘫,肌张力增高,腱反射亢进,病理反射阳性,肌电图和神经传导速度正常 |
D.表现为偏瘫,单瘫,截瘫或四肢瘫,肌张力增高,腱反射亢进,病理反射阳性,肌电图和神经传导速度正常 |
2 | 1 | 2012 | 26. 肌病的临床表现和辅助检查特点那项错误?(2.0分) | A.肌无力和萎缩,肌酶升高,肌电图肌原性改变,可伴有肌痛和压痛 B.肌无力和萎缩,肌酶升高,肌电图肌原性改变,可伴有假性肌肉肥大 C.肌无力和萎缩,肌酶升高,肌电图肌原性改变,可伴有强直和myokymia D.肌无力和萎缩,肌酶升高,肌电图肌原性改变,,受累部位感觉减退 |
D.肌无力和萎缩,肌酶升高,肌电图肌原性改变,,受累部位感觉减退 |
2 | 1 | 2012 | 27. 关于皮肌炎的描述那项错误?(2.0分) | A.对称性近端肌肌无力,肌酶升高,肌电图肌原性改变,可伴有皮疹 B.对称性近端肌肌无力,肌酶升高,肌电图肌原性改变,可伴有关节痛和关节炎 C.对称性近端肌肌无力,肌酶升高,肌电图肌原性改变,可伴有肺间质改变 D.对称性近端肌肌无力,肌酶升高,肌电图肌原性改变,可伴有心肌受累, E.对称性近端肌肌无力,肌酶升高,肌电图肌原性改变,可伴有恶性肿瘤和其它结缔组织病 F.对称性近端肌肌无力,肌酶升高,肌电图肌原性改变,肌活检特点肌细胞的胞浆和胞核内查到嗜酸性包涵体 |
F.对称性近端肌肌无力,肌酶升高,肌电图肌原性改变,肌活检特点肌细胞的胞浆和胞核内查到嗜酸性包涵体 |
2 | 1 | 2012 | 28. 以运动诱发活动障碍为特点的疾病,下面哪个最不可能(2.0分) | A.脂质沉积性肌病 B.风湿多肌痛 C.线粒体肌病 D.周期性麻痹 E.重症肌无力 F.糖原累积病 |
B.风湿多肌痛 |
2 | 1 | 2012 | 29. 患者,男,18岁 参加运动会400米跑步后次日,全身肌肉疼痛,痉挛,无力,检查发现低血糖,高血脂,肌红蛋白尿.为明确病因诊断最有价值辅助检查是?(2.0分) | A.肌酶 B.肌电图 C.肌肉病理和酶学检测 D.血钾和心电图 E.血清抗肌球蛋白抗体 F.线粒体病相关基因检测 |
C.肌肉病理和酶学检测 |
2 | 1 | 2012 | 30. 患者男性,53岁,出现进行性的四肢肌肉无力和萎缩3年,家族中无类似发病者。肌电图检查提示肌源性损害,肌酶检查发现CK轻度升高,给予肾上腺皮质激素治疗无效。下列对病因诊断最有帮助的检查是?(2.0分) | A.肌电图和重频电刺激 B.神经传导速度检查 C.肌肉活检 D.血常规生化检查 E.血免疫学检查 |
C.肌肉活检 |
2 | 1 | 2012 | 31. 什么临床征兆特别警示该重症肌无力患者发生危象可能较大?(2.0分) | A.上睑下垂和复视 B.严重吞咽困难和构音障碍 C.乙酰胆碱受体抗体(AchR-ab)高滴度 D.明显呼吸困难,口唇紫绀,意识不清 E.四肢无力加重 |
B.严重吞咽困难和构音障碍 |
2 | 1 | 2012 | 32. 线粒体病最好发的部位是?(2.0分) | A.脑,肌肉和眼 B.脑,心脏和内分泌系统 C.肌肉,胃肠和周围神经系统 D.眼,皮肤和肾 E.视,听觉系统和中枢神经系统 |
A.脑,肌肉和眼 |
2 | 1 | 2012 | 33. 线粒体病的描述那项错误?(2.0分) | A.mtDNA是裸露的,缺乏组蛋白和DNA结合蛋白的保护 B.mtDNA 复制快速且不具有校读功能,复制错误率高 C.父系遗传 D.细胞中可同时存在正常mtDNA和突变mtDNA E.mtDNA 突变基因的表型表达具有阈值效应 F.mtDNA 编码基因排列紧密,无内含子,所以mtDNA的任何突变可影响到其基因组内的一些重要功能区域 |
C.父系遗传 |
2 | 1 | 2012 | 34. 重症肌无力重要的阴性症状与体征,那项描述错误?(2.0分) | A.感觉障碍 B.肢体肌肉萎缩 C.腱反射亢进,病理征阳性 D.吞咽困难 E.肌肉跳动除非应用较大剂量胆碱酯酶抑制剂 |
D.吞咽困难 |
2 | 1 | 2012 | 35. 不首先推荐胸腺治疗(手术或放疗) 的重症肌无力类型是?(2.0分) | A.胸腺肥大 B.胸腺瘤 C.年轻女性特别病程短于6月-2年 D.patients with MuSK antibody-associated MG without thymoma E.patients with generalized MG have no detectable AChR or MuSK antibodies F.高AchR抗体 |
D.patients with MuSK antibody-associated MG without thymoma |
2 | 1 | 2012 | 36. 关于 Lambert-Eaton 综合征那项描述不正确?(2.0分) | A.2/3伴发癌肿,尤其是小细胞肺癌 B.发病机制:突触前膜释放乙酰胆碱障碍 C.四肢受累,颅神经支配肌肉很少受累 D.低频和高频重复电刺激波幅均增高 E.抗胆碱酯酶药无明显疗效,盐酸胍可能有效 F.50%伴自主神经症状 |
D.低频和高频重复电刺激波幅均增高 |
2 | 1 | 2012 | 37. 32岁,亚洲男性,暴食后次日早晨双下肢无力伴大腿酸胀;体检,意识清楚,颅神经正常,下肢对称性迟缓性瘫痪,大小便正常,感觉正常,反射正常。无家族遗传病史。最有意义的辅助检查是 (2.0分) | A.肌电图和神经传导 B.重复电刺激(RNS)单纤维肌电图(SFEMG) C.肌肉病理 D.血钾和心电图 E.甲状腺功能 |
D.血钾和心电图 |
2 | 1 | 2012 | 38. 低钾型麻痹经正规补充氯化钾数天后,低钾仍不能纠正最需要鉴别疾病是?(2.0分) | A.甲亢伴周期性麻痹 B.原发性醛固酮增多症和肾小管性酸中毒: C.低钾性周期性麻痹 D.过度胃肠道丢失后:呕吐、腹泻等 |
B.原发性醛固酮增多症和肾小管性酸中毒: |
2 | 1 | 2012 | 39. 在一项扁桃体切除术是否同其后发生何杰金病有联系的调查中发现:事先做过扁桃体切除者发生何杰金病的相对危险度为2.9,据此可推断: (2.0分) | A.事先做过扁桃体切除者中病死率较高 B.事先做过扁桃体切除者发生何杰金病的发病率较低 C.扁桃体切除术可以防止何杰金病的发生 D.事先做过扁桃体切除者何杰金病发病率为未切除者的2.9倍 E.相对危险度为2.9时意义不大 |
D.事先做过扁桃体切除者何杰金病发病率为未切除者的2.9倍 |
2 | 1 | 2012 | 40. 一种新治疗方法可以延长寿命,但不能治愈该病,则会出现(2.0分) | A.该病患病率将降低 B.该病发病率将升高 C.该病发病率将降低 D.该病患病率将升高 |
D.该病患病率将升高 |
2 | 1 | 2012 | 41. 下列指标中,用于测定发病率不准确且病死率极低的传染病的流行强度的指标是(2.0分) | A.病死率 B.现患率 C.超额死亡率 D.罹患率 E.累积死亡率 |
D.罹患率 |
2 | 1 | 2012 | 42. 小腿上段骨折后,病人出现足不能背屈、不能外翻、不能伸趾,行走困难,出现"跨阈步态";小腿前外侧、足背和趾背外侧区感觉障碍。试问什么神经的损伤导致了以上症状的发生 (2.0分) | A.胫神经 B.臀上神经 C.腓浅神经 D.臀下神经 E.腓总神经 |
E.腓总神经 |
2 | 1 | 2012 | 43. 乳腺癌手术清扫淋巴结时,最容易损伤一条神经,此神经损伤后手臂不能上举,并出现"翼状肩",该神经是(2.0分) | A.胸背神经 B.胸长神经 C.胸内侧神经 D.胸外侧神经 E.肩胛下神经 |
B.胸长神经 |
2 | 1 | 2012 | 44. 某孕妇随着胎儿的生长,逐渐到两下肢交叉困难,大腿不能内收,提示子宫压迫了一条神经,该神经是 (2.0分) | A.股神经 B.闭孔神经 C.盆内脏神经 D.骶丛 E.腰丛 |
B.闭孔神经 |
2 | 1 | 2012 | 45. 下列哪种神经递质释放前储存于分泌颗粒中:(2.0分) | A.谷氨酸 B.多巴胺 C.内啡肽 D.甘氨酸 |
C.内啡肽 |
2 | 1 | 2012 | 46. 以下哪项有关化学突触形态学的描述是错误的: (2.0分) | A.突触前后膜分化不对称的通常是兴奋性突触 B.存在轴突-轴突之间形成的突触 C.中枢神经系统化学突触的大小差别很大 D.神经-肌肉接头的运动终板膜平展光滑 |
D.神经-肌肉接头的运动终板膜平展光滑 |
2 | 1 | 2012 | 47. 以下哪种神经递质、突触后受体及突触后反应的组合是错误的(2.0分) | A.GABA-GABAA受体-IPSP B.谷氨酸-AMPA受体- EPSP C.乙酰胆碱-nAchR-G蛋白激活 D.肾上腺素-β受体-cAMP增高 |
C.乙酰胆碱-nAchR-G蛋白激活 |
2 | 1 | 2012 | 48. 关于突触整合,下列哪项描述是错误的(2.0分) | A.同时兴奋的钾通道有利于EPSP向胞体扩散 B.同时多个突触兴奋产生的EPSP相加称为时间相加 C.与放电起始区(轴丘)距离近的兴奋性突触,更易触发动作电位发放 D.抑制性突触的活动不利于动作电位的产生 |
A.同时兴奋的钾通道有利于EPSP向胞体扩散 |
2 | 1 | 2012 | 49. 下列哪种方法可以用来分析离子通道的三维结构?(2.0分) | A.蛋白质氨基酸序列的疏水性分析(hydrophilicity plot) B.单颗粒冷冻电镜技术(Single particle cryo-EM) C.放射性同位素标记配体的结合试验(Binding Assay) D.膜片钳(Patch-clamp)电生理记录 |
B.单颗粒冷冻电镜技术(Single particle cryo-EM) |
2 | 1 | 2012 | 50. 下列突触传递相关的电化学时间哪些耗时最长?(2.0分) | A.轴突上的动作电位传导 B.突触囊泡与突触前末梢的膜融合 C.递质在突触间隙的清除 D.突触后递质门控通道的开放 |
C.递质在突触间隙的清除 |
2 | 1 | 2013 | 1. Your patient is a 5-year-old girl who appears to have Duchenne muscular dystrophy(DMD). What is the most likely genetic explanation for this disease in a girl? (2.0分) | A.She has a 46, XY karyotype with sex reversal. B.She has two independent DMD mutations. C.She has skewed X inactivation. D.She has a dominant negative mutation. E.This diagnosis is impossible. F. |
C.She has skewed X inactivation. |
2 | 1 | 2013 | 2. A mutation causing an abnormality in presenilin(早老素) that disrupts amyloid(淀粉样蛋白) production or function has been associated with(2.0分) | A.cystic fibrosis B.Duchenne muscular dystrophy C.Marfan syndrome D.Wilson disease E.none of the above F. |
E.none of the above |
2 | 1 | 2013 | 3. Which of the following genetic diseases results from a mutation that prevents a protein from forming? (2.0分) | A.Duchenne muscular dystrophy B.Hemophilia A C.Epidermolysis bullosa(大疱性表皮松解症) D.Huntington disease E.Fragile X syndrome F. |
A.Duchenne muscular dystrophy |
2 | 1 | 2013 | 4. Which one of the following types of mutation is most likely to lead to premature termination of translation?(哪种类型的突变可导致翻译提前终止?)(2.0分) | A.Insertion of a single base into DNA B.Deletion of three bases from DNA C.Deletion of an entire gene D.Single base change in a promoter E.Exon skip |
A.Insertion of a single base into DNA |
2 | 1 | 2013 | 5. Pilocarpine对眼的调节作用是 (2.0分) | A.睫状肌松弛,悬韧带拉紧,晶状体变扁平 B.睫状肌松弛,悬韧带放松,晶状体变凸 C.睫状肌收缩,悬韧带拉紧,晶状体变凸 D.睫状肌收缩,悬韧带拉紧,晶状体变扁平 E.睫状肌收缩,悬韧带放松,晶状体变凸 |
E.睫状肌收缩,悬韧带放松,晶状体变凸 |
2 | 1 | 2013 | 6. 全身麻醉前给atropine的目的是 (2.0分) | A.镇静 B.增强麻醉药的作用 C.减少患者对术中的不良刺激的记忆 D.减少呼吸道腺体分泌 E.松弛骨骼肌 |
D.减少呼吸道腺体分泌 |
2 | 1 | 2013 | 7. 下述哪一类病人禁用atropine? (2.0分) | A.有机磷酸酯类中毒 B.窦性心动过缓 C.胃肠绞痛 D.前列腺肥大 E.虹膜睫状体炎 |
D.前列腺肥大 |
2 | 1 | 2013 | 8. 关于atropine的叙述,下列哪一项是错误的(2.0分) | A.可用于治疗过敏性休克 B.可用于治疗感染性休克 C.可用于治疗缓慢型心律失常 D.可用于全身麻醉前给药 E.可用于缓解内脏绞痛 |
A.可用于治疗过敏性休克 |
2 | 1 | 2013 | 9. 下列哪一项关于succinylcholine的叙述是正确的(2.0分) | A.是一种非除极化型肌松药 B.一次给药作用可维持2小时以上 C.可引起高血钾症 D.过量可用新斯的明对抗 E.可口服维持肌松作用 |
C.可引起高血钾症 |
2 | 1 | 2013 | 10. Neostigmine的作用不包括 (2.0分) | A.兴奋骨骼肌 B.兴奋胃肠道平滑肌 C.兴奋心脏 D.促进腺体分泌 E.缩瞳 |
C.兴奋心脏 |
2 | 1 | 2013 | 11. Neostigmine不用于下列哪一种情况? (2.0分) | A.阵发性室上性心动过速 B.重症肌无力 C.手术后肠胀气 D.支气管哮喘 E.非除极化型肌松药过量中毒 |
D.支气管哮喘 |
2 | 1 | 2013 | 12. 碘解磷定(pyraloxime methiodide)治疗有机磷酸酯类中毒的作用机制是 (2.0分) | A.能阻断M胆碱受体的作用 B.能阻断N胆碱受体的作用 C.能直接对抗体内积聚的ACh的作用 D.能恢复AChE水解ACh的活性 E.能与AChE结合而抑制酶的活性 |
D.能恢复AChE水解ACh的活性 |
2 | 1 | 2013 | 13. 关于有机磷酸酯类中毒的特点,下述哪一项是错误的? (2.0分) | A.形成磷酰化胆碱酯酶,使胆碱酯酶失活 B.形成磷酰化胆碱酯酶,使胆碱酯酶活性过强 C.严重中毒时可出现N样症状及中枢症状 D.应当及时抢救,以免使磷酰化胆碱酯酶"老化" E.应当早期大量重复使用atropine进行抢救 |
B.形成磷酰化胆碱酯酶,使胆碱酯酶活性过强 |
2 | 1 | 2013 | 14. 运动神经兴奋时,何种离子进入轴突末梢的量与囊泡释放量呈正变关系? (2.0分) | A.Mg2+ B.Na+ C.Ca2+ D.K+ E.Cl‐ |
C.Ca2+ |
2 | 1 | 2013 | 15. 在神经‐骨骼肌接头部位,大量囊泡释放ACh所引起的膜电位变化称 (2.0分) | A.突触后电位 B.接头后电位 C.局部电位 D.微终板电位 E.终板电位 |
E.终板电位 |
2 | 1 | 2013 | 16. 筒箭毒可被作为肌松剂应用,是由于能在终板膜部位(2.0分) | A.激活胆碱酯酶 B.与ACh竞争结合位点 C.与ACh 结合成复合物 D.抑制神经末梢Ca2+内流 E.减少囊泡释放Ach |
B.与ACh竞争结合位点 |
2 | 1 | 2013 | 17. 重症肌无力患者的骨骼肌对运动神经冲动的反应降低是由于(2.0分) | A.递质含量减少 B.递质释放量减少 C.胆碱酯酶活性增高 D.受体数目减少或功能障碍 E.微终板电位减小 |
D.受体数目减少或功能障碍 |
2 | 1 | 2013 | 18. 在骨骼肌细胞兴奋‐收缩耦联过程中,胞质内的Ca2+来自(2.0分) | A.横管膜上电压门控Ca2+通道引起的胞外Ca2+内流 B.肌质网上Ca2+释放通道开放引起的胞内Ca2+释放 C.细胞膜上NMDA 受体通道开放引起的胞外Ca2+内流 D.肌质网上Ca2+泵的反向转运 E.线粒体内Ca2+的释放 |
B.肌质网上Ca2+释放通道开放引起的胞内Ca2+释放 |
2 | 1 | 2013 | 19. 下列关于神经胶质细胞的描述,正确的是 (2.0分) | A.既有树突又有轴突 B.细胞间普遍存在缝隙连接 C.有随细胞外Na+浓度改变的膜电位 D.有产生动作电位的能力 E.细胞间具有突触连接 |
B.细胞间普遍存在缝隙连接 |
2 | 1 | 2013 | 20. 神经元产生动作电位的部位是(2.0分) | A.胞体 B.树突 C.轴丘 D.始段 E.轴索 |
D.始段 |
2 | 1 | 2013 | 21. 有髓神经纤维的传导特点是(2.0分) | A.传导速度慢 B.跳跃式传导 C.衰减性传导 D.单向传导 E.电缆式传导 |
B.跳跃式传导 |
2 | 1 | 2013 | 22. 下列不属于星形胶质细胞功能的是 (2.0分) | A.营养作用 B.参与血-脑屏障的形成 C.吞噬作用 D.免疫应答作用 E.修复作用 |
C.吞噬作用 |
2 | 1 | 2013 | 23. 神经‐肌接头传递中,消除乙酰胆碱的酶是(2.0分) | A.胆碱酯酶 B.腺苷酸环化酶 C.磷酸二酯酶 D.ATP 酶 E.胆碱乙酰化酶 |
A.胆碱酯酶 |
2 | 1 | 2013 | 24. 流行病学研究方法包括观察法与实验法两大类,它们的根本区别在于: (2.0分) | A.是否设立对照组 B.研究现场的不同 C.是否人为控制研究的条件 D.是否盲法观察 E.是否进行统计学检验 |
C.是否人为控制研究的条件 |
2 | 1 | 2013 | 25. 下列不属于观察性研究的是 (2.0分) | A.临床试验 B.病例对照研究 C.临床病例报告 D.队列研究 |
A.临床试验 |
2 | 1 | 2013 | 26. 23名在妊娠期每天吸烟一包以上的孕妇,她们所生的头胎婴儿平均出生体重比16名从不吸烟妇女所生的头胎婴儿的平均体重轻2000g差异有统计学意义(P<0.05),这表明 (2.0分) | A.妊娠期吸烟有碍胎儿生长 B.平均出生体重的差异很大,不能仅用随机误差解释 C.平均出生体重的差异很容易由随机误差造成 D.研究孕妇吸烟与出生低体重的关系意义不大 E.妊娠期间吸烟不影响胎儿生长 F.比特异危险度更具有疾病预防和公共卫生意义 |
A.妊娠期吸烟有碍胎儿生长 |
2 | 1 | 2013 | 27. 相对危险度 (2.0分) | A.是暴露比之于未暴露情况下增加超额疾病的数量 B.是非暴露组与暴露组发病或死亡危险之差的绝对值 C.是非暴露组发病或死亡的危险为暴露组的多少倍 D.比特异危险度更具有病因学意义 |
C.是非暴露组发病或死亡的危险为暴露组的多少倍 |
2 | 1 | 2013 | 28. 某研究小组在2000年的时候对某社区的人群进行调查,该社区30-60岁的非高血压人群都进行调查,总共有1500人被纳入到该研究中,其中发现320位居民抽样,随访10年,抽烟和非抽烟居民发生高血压的分别为120人和180人。该项研究用的是什么抽样方法? (2.0分) | A.简单随机抽样 B.分层抽样 C.系统抽样 D.整群抽样 |
D.整群抽样 |
2 | 1 | 2013 | 29. 关于锥体外系统的解剖生理描述那一项不正确? (2.0分) | A.调节肌张力,协调肌肉运动 B.维持和调整体态姿势 C.属于上运动神经元 D.担负半自动的刻板动作及反射性运动 E. |
C.属于上运动神经元 |
2 | 1 | 2013 | 30. 关于小脑病变那项描述不正确? (2.0分) | A.不同程度肌肉无力 B.共济平衡不良 C.轮替动作不协调 D.精细活动困难 E. |
A.不同程度肌肉无力 |
2 | 1 | 2013 | 31. 关于下运动神经元瘫痪那项是正确?(2.0分) | A.可表现为四肢远端或肢体的某一部位无力,肌张力增高,腱反射减弱或消失,病理反射阴性。 B.可表现为四肢远端或肢体的某一部位无力,肌张力减低,腱反射减弱或消失,病理反射阳性。 C.可表现为四肢远端或肢体的某一部位无力,肌张力增高,腱反射亢进,病理反射阳性,并早期明显肌肉萎缩 D.可表现为四肢远端或肢体的某一部位无力,肌张力增高,腱反射亢进,病理反射阳性,肌电图和神经传导速度正常 E.可表现为四肢远端或肢体的某一部位无力, 肌张力减低,腱反射减弱或消失,病理反射阴性,常伴感觉障碍和受累部位肌肉萎缩。 F. |
E.可表现为四肢远端或肢体的某一部位无力, 肌张力减低,腱反射减弱或消失,病理反射阴性,常伴感觉障碍和受累部位肌肉萎缩。 |
2 | 1 | 2013 | 32. 关于皮肌炎的描述那项错误?(2.0分) | A.对称性近端肌肌无力,肌酶升高,肌电图肌原性改变,可伴有皮疹 B.对称性近端肌肌无力,肌酶升高,肌电图肌原性改变,可伴有关节痛和关节炎 C.对称性近端肌肌无力,肌酶升高,肌电图肌原性改变,可伴有肺间质改变 D.对称性近端肌肌无力,肌酶升高,肌电图肌原性改变,可伴有心肌受累, E.对称性近端肌肌无力,肌酶升高,肌电图肌原性改变,可伴有恶性肿瘤和其它结缔组织病 F.对称性近端肌肌无力,肌酶升高,肌电图肌原性改变,肌活检特点肌细胞的胞浆和胞核内查到嗜酸性包涵体 |
F.对称性近端肌肌无力,肌酶升高,肌电图肌原性改变,肌活检特点肌细胞的胞浆和胞核内查到嗜酸性包涵体 |
2 | 1 | 2013 | 33. 患者,男,18岁 参加运动会400米跑步后次日,全身肌肉疼痛,痉挛,无力,检查发现低血糖,高血脂,肌红蛋白尿.为明确病因诊断最有价值辅助检查是?(2.0分) | A.肌酶 B.肌电图 C.肌肉病理和酶学检测 D.血钾和心电图 E.血清抗肌球蛋白抗体 F.线粒体病相关基因检测 |
C.肌肉病理和酶学检测 |
2 | 1 | 2013 | 34. 什么临床征兆特别警示该重症肌无力患者发生危象可能较大?(2.0分) | A.上睑下垂和复视 B.严重吞咽困难和构音障碍 C.乙酰胆碱受体抗体(AchR-ab)高滴度 D.明显呼吸困难,口唇紫绀,意识不清 E.四肢无力加重 F. |
B.严重吞咽困难和构音障碍 |
2 | 1 | 2013 | 35. 重症肌无力危象的首要抢救步骤?(2.0分) | A.区分肌无力性危象,胆碱能危象或反拗危象,以便对因治疗 B.保持呼吸道通畅,呼吸机支持 C.积极控制感染 D.大剂量免疫球蛋白或血浆置换 E.大剂量激素冲击治疗 F. |
B.保持呼吸道通畅,呼吸机支持 |
2 | 1 | 2013 | 36. 32岁,亚洲男性,暴食后次日早晨双下肢无力伴大腿酸胀;体检,意识清楚,颅神经正常,下肢对称性迟缓性瘫痪,大小便正常,感觉正常,反射正常。无家族遗传病史。最有意义的辅助检查是(2.0分) | A.肌电图和神经传导 B.重复电刺激(RNS)单纤维肌电图(SFEMG) C.肌肉病理 D.血钾和心电图 E.甲状腺功能 F. |
D.血钾和心电图 |
2 | 1 | 2013 | 37. 低钾型麻痹经正规补充氯化钾数天后,低钾仍不能纠正最需要鉴别疾病是?(2.0分) | A.甲亢伴周期性麻痹 B.原发性醛固酮增多症和肾小管性酸中毒: C.低钾性周期性麻痹 D.过度胃肠道丢失后:呕吐、腹泻等 E. |
B.原发性醛固酮增多症和肾小管性酸中毒: |
2 | 1 | 2013 | 38. 管理小指皮肤感觉的神经是(2.0分) | A.正中神经 B.桡神经 C.尺神经 D.腋神经 E.肌皮神经 |
C.尺神经 |
2 | 1 | 2013 | 39. 脐平面的皮肤感觉神经是 (2.0分) | A.第8对胸神经 B.第9对胸神经 C.第10对胸神经 D.第1l对胸神经 E.第12对胸神经 |
C.第10对胸神经 |
2 | 1 | 2013 | 40. 病人出现呼吸困难症状,伴有窒息感,经检查发现腹式呼吸明显减弱。试问可能是什么神经受到了损伤 (2.0分) | A.膈神经 B.迷走神经 C.舌咽神经 D.内脏大神经 E.肋间神经 |
A.膈神经 |
2 | 1 | 2013 | 41. 患者,男,18岁 参加运动会400米跑步后次日,全身肌肉疼痛,痉挛,无力,检查发现低血糖,高血脂,肌红蛋白尿.可能最大的疾病诊断是? (2.0分) | A.脂质沉积性肌病 B.原累积病 C.线粒体肌病 D.周期性麻痹 E.重症肌无力 F.单纯的横纹肌溶解症 |
B.原累积病 |
2 | 1 | 2013 | 42. 动作电位沿运动神经纤维传导抵达神经‐肌接头部位时,轴突末梢释放Ach,使终板膜(2.0分) | A.产生终板电位,然后在什么部位引发动作电位 B.肌细胞膜 C.接头后膜 D.终板膜 E.横管膜 F.三联管膜 |
A.产生终板电位,然后在什么部位引发动作电位 |
2 | 1 | 2013 | 43. 实验中,如果同时刺激神经纤维的两端,产生的两个动作电位 (2.0分) | A.将各自通过中点后传导到另一端 B.将在中点相遇,然后传回到起始点 C.只有较强的动作电位通过中点而到达另一端 D.将在中点相遇后停止传导 E.到达中点后将复合成一个更大的动作电位 F. |
D.将在中点相遇后停止传导 |
2 | 1 | 2013 | 44. 细胞膜上不衰减形式传播的电活动是 (2.0分) | A.动作电位 B.静息电位 C.终板电位 D.感受器电位 E.突触后电位 F. |
A.动作电位 |
2 | 1 | 2013 | 45. 下列哪项在突触囊泡释放中与感受Ca2+浓度直接相关: (2.0分) | A.活性区(Active Zone) B.线粒体(mitochondria) C.SNARE蛋白复合体 D.Synaptotagmin和Complexin |
D.Synaptotagmin和Complexin |
2 | 1 | 2013 | 46. 关于神经递质,下列哪项说法是错误的(2.0分) | A.氨基酸类和单胺类神经递质常常是在突触前末梢合成的 B.氨基酸类和肽类神经递质可共存于同一个神经元 C.肽类神经递质的释放往往需要一连串的动作电位 D.神经递质的量子释放指的是神经末梢总是大量释放递质 |
D.神经递质的量子释放指的是神经末梢总是大量释放递质 |
2 | 1 | 2013 | 47. 下列哪种方法可以测量单个离子通道的电流?(2.0分) | A.离子通道的三维结构分析 B.同位素标记的配体结合分析 C.用特异抗体做免疫细胞化学染色 D.膜片钳(Patch-clamp)电生理记录 |
D.膜片钳(Patch-clamp)电生理记录 |
2 | 1 | 2013 | 48. 当突触前释放神经递质谷氨酸时,下列哪种兴奋性突触后反应排序符合时间先后次序(2.0分) | A.突触后膜去极化? AMPA受体激活? Ca2+内流? NMDA受体激活 B.Ca2+内流活?突触后膜去极化?AMPA受体激活?NMDA受体激活 C.AMPA受体激活?突触后膜去极化?NMDA受体兴奋?Ca2+内流 D.NMDA受体激活? Ca2+内流?突触后膜去极化?AMPA受体激活 |
C.AMPA受体激活?突触后膜去极化?NMDA受体兴奋?Ca2+内流 |
2 | 1 | 2013 | 49. 下列哪种方法可以用来分析离子通道的三维结构?(2.0分) | A.蛋白质氨基酸序列的疏水性分析(hydrophilicity plot) B.单颗粒冷冻电镜技术(Single particle cryo-EM) C.放射性同位素标记配体的结合试验(Binding Assay) D.膜片钳(Patch-clamp)电生理记录 E. |
B.单颗粒冷冻电镜技术(Single particle cryo-EM) |
2 | 1 | 2013 | 50. 电压门控离子通道对跨膜电位的敏感性取决于:(2.0分) | A.组成通道的亚基数目 B.电压感受器 C.羧基端的磷酸化 D.选择性滤器 |
B.电压感受器 |
2 | 1 | 2014 | 1. 细胞膜上不衰减形式传播的电活动是(2.0分) | A.动作电位 B.静息电位 C.终板电位 D.感受器电位 E.突触后电位 |
A.动作电位 |
2 | 1 | 2014 | 2. 临床上普鲁卡因局部麻醉药的应用是由于阻断了局部神经冲动的传导,其神经细胞发生的变化是(2.0分) | A.细胞变性 B.结构完整性破坏 C.功能完整性破坏 D.细胞膜电压门控钾通道破坏 E.细胞膜化学门控钠通道破坏 |
C.功能完整性破坏 |
2 | 1 | 2014 | 3. 动作电位沿运动神经纤维传导抵达神经‐肌肉接头部位时,轴突末梢释放Ach,使终板膜产生终板电位,然后在什么部位引发动作电位(2.0分) | A.肌细胞膜 B.接头后膜 C.终板膜 D.横管膜 E.三联管膜 |
A.肌细胞膜 |
2 | 1 | 2014 | 4. 人工减小细胞浸浴液中的Na+浓度,所记录的动作电位出现(2.0分) | A.幅度变小 B.幅度变大 C.时程缩短 D.时程延长 E.复极相延长 |
A.幅度变小 |
2 | 1 | 2014 | 5. 关于肌电图,以下说法正确的是(2.0分) | A.肌电图所记录到的肌电是单根肌纤维的电活动 B.肌电图是骨骼肌在收缩时的电兴奋活动,经过引导、放大、记录而成的图形 C.肌电图是骨骼肌在收缩后的电兴奋活动,经过引导、放大、记录而成的图形 D.肌电图是骨骼肌在收缩前的电兴奋活动,经过引导、放大、记录而成的图形 |
D.肌电图是骨骼肌在收缩前的电兴奋活动,经过引导、放大、记录而成的图形 |
2 | 1 | 2014 | 6. 关于神经纤维的传导速度,以下说法错误的是(2.0分) | A.有髓鞘神经纤维比无髓鞘神经纤维传导速度要快得多 B.与髓鞘厚度有关 C.与纤维直径成正比 D.与刺激强度成正比 E.神经纤维传导速度与温度呈正相关 |
D.与刺激强度成正比 |
2 | 1 | 2014 | 7. You see a 25 year old ambulatory (可行走的) male with a history of a dystrophic myopathy(肌营养不良性肌病). He presented at 6 years of age with muscle weakness. Routine laboratory studies(常规实验室检查)revealed elevated circulating muscle enzymes, and molecular testing of his DNA showed an in-frame deletion(整码缺失突变) of the dystrophin gene. What is the most likely diagnosis of this patient? (2.0分) | A.Becker muscular dystrophy(BMD) B.BMD carrier C.Duchenne muscular dystrophy(DMD) D.DMD carrier E.DMD that he outgrew(长大) |
A.Becker muscular dystrophy(BMD) |
2 | 1 | 2014 | 8. Both of Mary's sons have the X-linked disorder Duchenne muscular dystrophy(DMD). A deletion(缺失) in the DMD gene is found in each of the boys, but no mutation in DMD is found when a blood sample from Mary is analyzed. What is the most likely explanation?(2.0分) | A.The boys were affected by separate mutation events in DMD B.The mutation was inherited from the boys' father C.Mary is germline mosaic(种系嵌合体) for a DMD deletion(缺失) D.The phenotype is not penetrant(不外显) in Mary E.The phenotype is X-linked recessive |
C.Mary is germline mosaic(种系嵌合体) for a DMD deletion(缺失) |
2 | 1 | 2014 | 9. A newborn is noted to maintain a frog-leg position with a weak cry and minimal spontaneous movement. A muscular dystrophy(肌营养不良) is suspected, and a serum enzyme is measured that reflects the abundance(丰度。即在给定的生物组织细胞中,某特异大分子的相对含量) of a high energy storage molecule in muscle. Which of the following enzymes was measured? (2.0分) | A.Adenosine triphosphatase(腺苷三磷酸酶) B.Creatine phosphokinase(肌酸磷酸激酶) C.Glucose-1-phosphate dehydrogenase(葡糖-1-磷酸脱氢酶) D.Pyrophosphatase(焦磷酸酶) E.Glucose-6-phosphate dehydrogenase(G6PD。葡糖-6-磷酸脱氢酶) |
B.Creatine phosphokinase(肌酸磷酸激酶) |
2 | 1 | 2014 | 10. A 7 year old male tires easily and can be outrun(超过) by his 5 year old brother even though the 7 year old's gastrocnemius muscles(腓肠肌) appear well-developed. His serum creatine kinase(肌酸激酶) is 1000 U/L (normal about 100 U/L). A gastrocnemius muscle biopsy(活检) shows marked interstitial fibrosis(间质纤维化)with atrophy(萎缩)of many fibers and increase in size of others. Immuno-histochemical staining(免疫组化染色) with an antibody to dystrophin reveals a lack of staining under the sarcolemma(肌膜) of the fibers. Over the next 5 years, he becomes progressively weaker until he is confined to a wheelchair(轮椅). Which of the following is the most likely diagnosis for this child? (2.0分) | A.An autoimmune disease B.Duchenne muscular dystrophy(DMD) C.Becker muscular dystrophy(BMD) D.Myasthenia gravis(重症肌无力) E.spinal muscular atrophy 1 (SMA-1) |
B.Duchenne muscular dystrophy(DMD) |
2 | 1 | 2014 | 11. DMD(假性肥大型肌营养不良)为X-连锁隐性遗传病(XR)。临床上有一例男性患者,他的哪一位亲属不存在因遗传患病的可能 (2.0分) | A.同胞兄弟 B.姨表兄弟 C.舅舅 D.姑表兄弟 E.外甥 |
D.姑表兄弟 |
2 | 1 | 2014 | 12. 一项子宫内膜癌与雌激素应用之间关系的匹配病例对照研究,共计73对,病例和对照两组均有雌激素暴露史者27对,两组均无暴露史者4对,病例组暴露而对照组未暴露者29对,其余为对照组暴露而病例组未暴露者。据此计算OR值为:(2.0分) | A.6.75 B.9.67 C.2.23 D.2.72 E.3.49 |
C.2.23 |
2 | 1 | 2014 | 13. 神经‐肌肉接头处的化学递质是(2.0分) | A.肾上腺素 B.去甲肾上腺素 C.γ‐氨基丁酸 D.乙酰胆碱 E.5‐羟色胺 |
D.乙酰胆碱 |
2 | 1 | 2014 | 14. 在神经‐骨骼肌接头部位,大量囊泡释放ACh所引起的膜电位变化称(2.0分) | A.突触后电位 B.接头后电位 C.局部电位 D.微终板电位 E.终板电位 |
E.终板电位 |
2 | 1 | 2014 | 15. 在神经‐肌接头传递过程中,ACh与ACh门控通道结合使终板膜(2.0分) | A.对Na+、K+通透性增加,发生超极化 B.对Na+、K+通透性降低,发生超极化 C.对Na+、K+通透性增加,发生超极化 D.对Na+、K+通透性增加,发生去极化 E.对Ca2+通透性增加,发生去极化 |
D.对Na+、K+通透性增加,发生去极化 |
2 | 1 | 2014 | 16. 骨骼肌收缩和舒张的基本功能单位是(2.0分) | A.肌原纤维 B.肌小节 C.肌纤维 D.粗肌丝 E.细肌丝 |
B.肌小节 |
2 | 1 | 2014 | 17. 关于骨骼肌收缩机制,下列哪条是错误的?(2.0分) | A.引起兴奋收缩偶联的离子是Ca2+ 离子 B.细肌丝向粗肌丝滑行 C.Ca2+与横桥结合 D.肌小节缩短 |
C.Ca2+与横桥结合 |
2 | 1 | 2014 | 18. 在骨骼肌细胞兴奋‐收缩耦联过程中,胞质内的Ca2+来自(2.0分) | A.横管膜上电压门控Ca2+通道引起的胞外Ca2+内流 B.肌质网上Ca2+释放通道开放引起的胞内Ca2+释放 C.细胞膜上NMDA 受体通道开放引起的胞外Ca2+内流 D.肌质网上Ca2+泵的反向转运 E.线粒体内Ca2+的释放 |
B.肌质网上Ca2+释放通道开放引起的胞内Ca2+释放 |
2 | 1 | 2014 | 19. 下列关于神经胶质细胞的描述,正确的是(2.0分) | A.既有树突又有轴突 B.细胞间普遍存在缝隙连接 C.有随细胞外Na+浓度改变的膜电位 D.有产生动作电位的能力 E.细胞间具有突触连接 |
B.细胞间普遍存在缝隙连接 |
2 | 1 | 2014 | 20. 神经纤维的传导速度(2.0分) | A.与阈电位成正比 B.与髓鞘化的程度成反比 C.与纤维直径成负相关 D.与刺激强度成正比 E.温度降低,传导速度变慢 |
E.温度降低,传导速度变慢 |
2 | 1 | 2014 | 21. 下列哪种神经胶质细胞可在外周神经系统中形成神经纤维的髓鞘(2.0分) | A.星形胶质细胞 B.少突胶质细胞 C.施万细胞 D.卫星细胞 E.小胶质细胞 |
C.施万细胞 |
2 | 1 | 2014 | 22. 有髓神经纤维的传导特点是(2.0分) | A.传导速度慢 B.跳跃式传导 C.衰减性传导 D.单向传导 E.电缆式传导 |
B.跳跃式传导 |
2 | 1 | 2014 | 23. 关于锥体外系统的解剖生理描述那一项不正确?(2.0分) | A.调节肌张力,协调肌肉运动 B.维持和调整体态姿势 C.属于上运动神经元 D.担负半自动的刻板动作及反射性的运动 |
C.属于上运动神经元 |
2 | 1 | 2014 | 24. 关于小脑病变那项描述不正确?(2.0分) | A.不同程度肌肉无力 B.共济平衡不良 C.轮替动作不协调 D.精细活动困难 |
A.不同程度肌肉无力 |
2 | 1 | 2014 | 25. 关于下运动神经元瘫痪那项是正确?(2.0分) | A.可表现为四肢远端或肢体的某一部位无力,肌张力增高,腱反射减弱或消失,病理反射阴性。 B.可表现为四肢远端或肢体的某一部位无力,肌张力减低,腱反射减弱或消失,病理反射阳性。 C.可表现为四肢远端或肢体的某一部位无力,肌张力增高,腱反射亢进,病理反射阳性,并早期明显肌肉萎缩 D.可表现为四肢远端或肢体的某一部位无力,肌张力增高,腱反射亢进,病理反射阳性,肌电图和神经传导速度正常 E.可表现为四肢远端或肢体的某一部位无力, 肌张力减低,腱反射减弱或消失,病理反射阴性,常伴感觉障碍和受累部位肌肉萎缩。 |
E.可表现为四肢远端或肢体的某一部位无力, 肌张力减低,腱反射减弱或消失,病理反射阴性,常伴感觉障碍和受累部位肌肉萎缩。 |
2 | 1 | 2014 | 26. 关于多发性肌炎的描述那项错误?(2.0分) | A.对称性近端肌无力,肌酶升高,肌电图肌原性改变,可伴有肌痛和压痛 B.对称性近端肌无力,肌酶升高,肌电图肌原性改变,可伴有肌萎缩, C.对称性近端肌无力,肌酶升高,肌电图肌原性改变,可伴有呼吸表浅、呼吸困难, D.对称性近端肌无力,肌酶升高,肌电图肌原性改变,可伴有肌阵挛, E.对称性近端肌无力,肌酶升高,肌电图肌原性改变,可伴有吞咽和构音障碍 |
D.对称性近端肌无力,肌酶升高,肌电图肌原性改变,可伴有肌阵挛, |
2 | 1 | 2014 | 27. 以运动诱发活动障碍为特点的疾病,下面哪个最不可能(2.0分) | A.脂质沉积性肌病 B.风湿多肌痛 C.线粒体肌病 D.周期性麻痹 E.重症肌无力 |
B.风湿多肌痛 |
2 | 1 | 2014 | 28. 患者,男,18岁 参加运动会400米跑步后次日,全身肌肉疼痛,痉挛,无力,检查发现低血糖,高血脂,肌红蛋白尿.可能最大的疾病诊断是?(2.0分) | A.脂质沉积性肌病 B.糖原累积病 C.线粒体肌病 D.周期性麻痹 E.重症肌无力 |
B.糖原累积病 |
2 | 1 | 2014 | 29. 关于进行性肌营养不良描述那项错误?(2.0分) | A.遗传性肌肉变性疾病 B.遗传方式有常染色体显性和隐性或X连锁隐性遗传 C.不能耐受运动和近端肌无力,肌肉痉挛和疼痛为主要表现 D.电生理为肌源性损害,神经传导速度正常 E.病理特点:进行性的肌纤维坏死,再生和脂肪及结缔组织增生,无异常代谢产物堆积 |
C.不能耐受运动和近端肌无力,肌肉痉挛和疼痛为主要表现 |
2 | 1 | 2014 | 30. 患者男性,53岁,出现进行性的四肢肌肉无力和萎缩3年,家族中无类似发病者。肌电图检查提示肌源性损害,肌酶检查发现CK轻度升高,给予肾上腺皮质激素治疗无效。最有可能诊断是?(2.0分) | A.皮肌炎 B.多发性肌炎 C.线粒体肌病 D.包涵体肌炎 E.Lambert-Eaton 综合征 |
D.包涵体肌炎 |
2 | 1 | 2014 | 31. Duchenne muscular dystrophy 和Becker muscular dystrophy 的鉴别点那项错误。(2.0分) | A.Duchenne 型为X连锁隐性遗传 和Becker 型呈常染色体显性遗传 B.Duchenne 型发病年龄相对早 C.Duchenne 型进展快,预后差 D.Duchenne 型易伴发心肌和认知功能损害 E.肌肉病理Duchenne 型dystrophin染色缺失Becker dystrophin染色减少 |
A.Duchenne 型为X连锁隐性遗传 和Becker 型呈常染色体显性遗传 |
2 | 1 | 2014 | 32. 线粒体病脑肌病(MELAS)的主要临床表现(2.0分) | A.Stroke-like episodes,ragged-red fibres and/or lactic acidosis and bilateral deafness B.Stroke-like episodes,ragged-red fibres and/or lactic acidosis and Diabetes mellitus C.Stroke-like episodes,ragged-red fibres and/or lactic acidosis and cardiomyopathy D.Stroke-like episodes, ragged-red fibres and/or lactic acidosis and seizures and/or dementia E.Stroke-like episodes,ragged-red fibres and/or lactic acidosis and cerebellar ataxia |
D.Stroke-like episodes, ragged-red fibres and/or lactic acidosis and seizures and/or dementia |
2 | 1 | 2014 | 33. 关于引起神经-肌肉接头(NMJ)传递障碍描述那项错误?(2.0分) | A.突触前膜释放乙酰胆碱障碍 B.突触间隙胆碱酯酶活性改变 C.突触后膜乙酰胆碱受体数量减少,功能抑制 D.骨骼肌细胞膜的离子通道功能失常 E.高镁血症可影响突触前膜释放乙酰胆碱 |
D.骨骼肌细胞膜的离子通道功能失常 |
2 | 1 | 2014 | 34. 重症肌无力危象的抢救最重要是?(2.0分) | A.区分肌无力性危象,胆碱能危象或反拗危象,以便对因治疗 B.保持呼吸道通畅,呼吸机支持 C.积极控制感染 D.大剂量免疫球蛋白或血浆置换 E.大剂量激素冲击治疗 |
B.保持呼吸道通畅,呼吸机支持 |
2 | 1 | 2014 | 35. 重症肌无力辅助诊断那项最具价值?(2.0分) | A.疲劳试验和药物试验 B.系统免疫学指标和甲状腺功能检查 C.乙酰胆碱受体抗体(AchR-ab)和重复电刺激(RNS)单纤维肌电图(SFEMG)测定 D.胸腺CT和MRI影像检查 E.普通肌电图和神经传导 |
C.乙酰胆碱受体抗体(AchR-ab)和重复电刺激(RNS)单纤维肌电图(SFEMG)测定 |
2 | 1 | 2014 | 36. 关于 Lambert-Eaton 综合征那项描述不正确?(2.0分) | A.2/3伴发癌肿,尤其是小细胞肺癌 B.发病机制:突触前膜释放乙酰胆碱障碍 C.四肢受累,颅神经支配肌肉很少受累 D.低频和高频重复电刺激波幅均增高 E.抗胆碱酯酶药无明显疗效,盐酸胍可能有效 |
D.低频和高频重复电刺激波幅均增高 |
2 | 1 | 2014 | 37. 32岁,亚洲男性,暴食后次日早晨双下肢无力伴大腿酸胀;体检,意识清楚,颅神经正常,下肢对称性迟缓性瘫痪,大小便正常,感觉正常,反射正常。无家族遗传病史。最有意义的辅助检查是 (2.0分) | A.普通肌电图和神经传导 B.重复电刺激(RNS)单纤维肌电图(SFEMG) C.肌肉病理 D.血钾和心电图 E.甲状腺功能 |
D.血钾和心电图 |
2 | 1 | 2014 | 38. 低钾型麻痹经正规补充氯化钾数天后,低钾仍不能纠正最需要鉴别疾病是?(2.0分) | A.甲亢伴周期性麻痹 B.原发性醛固酮增多症和肾小管性酸中毒: C.低钾性周期性麻痹 D.过度胃肠道丢失后:呕吐、腹泻等 |
B.原发性醛固酮增多症和肾小管性酸中毒: |
2 | 1 | 2014 | 39. 32岁,亚洲男性,运动后次日早晨醒来发现双下肢无力伴大腿酸胀;体检,意识清楚,颅神经正常,下肢对称性迟缓性瘫痪,大小便正常,感觉正常,反射正常。无家族遗传病史。最有可能的诊断是(2.0分) | A.吉兰-巴雷综合征 B.肉毒毒素中毒 C.甲亢伴低钾型周期性麻痹 D.线粒体肌病 E.高钾型周期性麻痹 |
C.甲亢伴低钾型周期性麻痹 |
2 | 1 | 2014 | 40. 不首先推荐胸腺治疗(手术或放疗) 的重症肌无力类型是?(2.0分) | A.胸腺肥大 B.胸腺瘤 C.年轻女性特别病程短于6月-2年 D.patients with MuSK antibody-associated MG without thymoma E.patients with generalized MG have no detectable AChR or MuSK antibodies |
D.patients with MuSK antibody-associated MG without thymoma |
2 | 1 | 2014 | 41. 关于皮肌炎和多发性肌炎肌酶升高描述那项错误?(2.0分) | A.绝大多数病人在病程某一阶段可出现肌酶活性增高,CK特异强,只有在肌肉病变时升高 B.绝大多数病人在病程某一阶段可出现肌酶活性增高,CK最敏感。 C.绝大多数病人在病程某一阶段可出现肌酶活性增高,是诊断、疗效监测及预后的评价指标 D.绝大多数病人在病程某一阶段可出现肌酶活性增高,晚期肌萎缩后肌酶不再释放 E.绝大多数病人在病程某一阶段可出现肌酶活性增高,包括肌酸激酶(CK)、醛缩酶(ALD)、乳酸脱氢酶(LDH)、门冬氨酸氨基转移酶(AST)。 |
A.绝大多数病人在病程某一阶段可出现肌酶活性增高,CK特异强,只有在肌肉病变时升高 |
2 | 1 | 2014 | 42. 下列哪项不是离子通道激活的因素?(2.0分) | A.离子通道的内化(internalization) B.化学配体的结合(ligand) C.牵拉力(stretch) D.跨膜电位(voltage) |
B.化学配体的结合(ligand) |
2 | 1 | 2014 | 43. 关于神经递质,下列哪项说法是错误的(2.0分) | A.氨基酸类和单胺类神经递质常常是在突触前末梢合成的 B.氨基酸类和肽类神经递质可共存于同一个神经元 C.肽类神经递质的释放往往需要一连串的动作电位 D.神经递质的量子释放指的是神经末梢总是大量释放递质 |
D.神经递质的量子释放指的是神经末梢总是大量释放递质 |
2 | 1 | 2014 | 44. Atropine对眼睛的作用是 (2.0分) | A.扩瞳,升高眼内压,视远物模糊 B.扩瞳,升高眼内压,视近物模糊 C.扩瞳,降低眼内压,视近物模糊 D.扩瞳,降低眼内压,视远物模糊 E.缩瞳,升高眼内压,视近物模糊 |
B.扩瞳,升高眼内压,视近物模糊 |
2 | 1 | 2014 | 45. 有机磷酸酯类急性中毒时,atropine不能缓解下列哪一项症状? (2.0分) | A.瞳孔缩小 B.出汗 C.恶心,呕吐 D.呼吸困难 E.肌肉颤动 |
E.肌肉颤动 |
2 | 1 | 2014 | 46. 关于neostigmine的描述,错误的是 (2.0分) | A.口服吸收差,可皮下注射给药 B.易进人中枢神经系统 C.可促进胃肠道的运动 D.常用于治疗重症肌无力 E.禁用于机械性肠梗阻 |
B.易进人中枢神经系统 |
2 | 1 | 2014 | 47. 碘解磷定(pyraloxime methiodide)治疗有机磷酸酯类中毒的作用机制是 (2.0分) | A.能阻断M胆碱受体的作用 B.能阻断N胆碱受体的作用 C.能直接对抗体内积聚的ACh的作用 D.能恢复AChE水解ACh的活性 E.能与AChE结合而抑制酶的活性 |
D.能恢复AChE水解ACh的活性 |
2 | 1 | 2014 | 48. 脐平面的皮肤感觉神经是(2.0分) | A.第8对胸神经 B.第9对胸神经 C.第10对胸神经 D.第1l对胸神经 E.第12对胸神经 |
C.第10对胸神经 |
2 | 1 | 2014 | 49. 一病人下肢受伤后出现了足不能跖屈,内翻力减弱,小腿后面和足底皮肤感觉障碍;早期出现"钩状足"畸形,随着时间的推移逐渐出现"马蹄内翻足"畸形。试问什么神经的损伤导致了以上症状的发生(2.0分) | A.坐骨神经 B.臀下神经 C.胫神经 D.腓总神经 E.腓深神经 |
C.胫神经 |
2 | 1 | 2014 | 50. 某孕妇随着胎儿的生长,逐渐到两下肢交叉困难,大腿不能内收,提示子宫压迫了一条神经,该神经是(2.0分) | A.股神经 B.闭孔神经 C.盆内脏神经 D.骶丛 E.腰丛 |
B.闭孔神经 |
2 | 1 | 2015 | 1. Your patient is a 5-year-old girl who appears to have Duchenne muscular dystrophy(DMD). What is the most likely genetic explanation for this disease in a girl? (2.0分) | A.She has a 46, XY karyotype with sex reversal. B.She has two independent DMD mutations. C.She has skewed X inactivation. D.She has a dominant negative mutation. E.This diagnosis is impossible. F. |
C.She has skewed X inactivation. |
2 | 1 | 2015 | 2. A mutation causing an abnormality in presenilin(早老素) that disrupts amyloid(淀粉样蛋白) production or function has been associated with (2.0分) | A.cystic fibrosis B.Duchenne muscular dystrophy C.Marfan syndrome D.Wilson disease E.none of the above F. |
E.none of the above |
2 | 1 | 2015 | 3. Which of the following genetic diseases results from a mutation that prevents a protein from forming? (2.0分) | A.Duchenne muscular dystrophy B.Hemophilia A C.Epidermolysis bullosa(大疱性表皮松解症) D.Huntington disease E.Fragile X syndrome F. |
A.Duchenne muscular dystrophy |
2 | 1 | 2015 | 4. Which one of the following types of mutation is most likely to lead to premature termination of translation?(哪种类型的突变可导致翻译提前终止?)(2.0分) | A.Insertion of a single base into DNA B.Deletion of three bases from DNA C.Deletion of an entire gene D.Single base change in a promoter E.Exon skip |
A.Insertion of a single base into DNA |
2 | 1 | 2015 | 5. Pilocarpine对眼的调节作用是 (2.0分) | A.睫状肌松弛,悬韧带拉紧,晶状体变扁平 B.睫状肌松弛,悬韧带放松,晶状体变凸 C.睫状肌收缩,悬韧带拉紧,晶状体变凸 D.睫状肌收缩,悬韧带拉紧,晶状体变扁平 E.睫状肌收缩,悬韧带放松,晶状体变凸 |
E.睫状肌收缩,悬韧带放松,晶状体变凸 |
2 | 1 | 2015 | 6. 全身麻醉前给atropine的目的是 (2.0分) | A.镇静 B.增强麻醉药的作用 C.减少患者对术中的不良刺激的记忆 D.减少呼吸道腺体分泌 E.松弛骨骼肌 |
D.减少呼吸道腺体分泌 |
2 | 1 | 2015 | 7. 下述哪一类病人禁用atropine?(2.0分) | A.有机磷酸酯类中毒 B.窦性心动过缓 C.胃肠绞痛 D.前列腺肥大 E.虹膜睫状体炎 |
D.前列腺肥大 |
2 | 1 | 2015 | 8. 关于atropine的叙述,下列哪一项是错误的? (2.0分) | A.可用于治疗过敏性休克 B.可用于治疗感染性休克 C.可用于治疗缓慢型心律失常 D.可用于全身麻醉前给药 E.可用于缓解内脏绞痛 |
A.可用于治疗过敏性休克 |
2 | 1 | 2015 | 9. 下列哪一项关于succinylcholine的叙述是正确的? (2.0分) | A.是一种非除极化型肌松药 B.一次给药作用可维持2小时以上 C.可引起高血钾症 D.过量可用新斯的明对抗 E.可口服维持肌松作用 |
C.可引起高血钾症 |
2 | 1 | 2015 | 10. Neostigmine的作用不包括(2.0分) | A.兴奋骨骼肌 B.兴奋胃肠道平滑肌 C.兴奋心脏 D.促进腺体分泌 E.缩瞳 |
C.兴奋心脏 |
2 | 1 | 2015 | 11. Neostigmine不用于下列哪一种情况? (2.0分) | A.阵发性室上性心动过速 B.重症肌无力 C.手术后肠胀气 D.支气管哮喘 E.非除极化型肌松药过量中毒 |
D.支气管哮喘 |
2 | 1 | 2015 | 12. 碘解磷定(pyraloxime methiodide)治疗有机磷酸酯类中毒的作用机制是(2.0分) | A.能阻断M胆碱受体的作用 B.能阻断N胆碱受体的作用 C.能直接对抗体内积聚的ACh的作用 D.能恢复AChE水解ACh的活性 E.能与AChE结合而抑制酶的活性 |
D.能恢复AChE水解ACh的活性 |
2 | 1 | 2015 | 13. 关于有机磷酸酯类中毒的特点,下述哪一项是错误的? (2.0分) | A.形成磷酰化胆碱酯酶,使胆碱酯酶失活 B.形成磷酰化胆碱酯酶,使胆碱酯酶活性过强 C.严重中毒时可出现N样症状及中枢症状 D.应当及时抢救,以免使磷酰化胆碱酯酶"老化" E.应当早期大量重复使用atropine进行抢救 |
B.形成磷酰化胆碱酯酶,使胆碱酯酶活性过强 |
2 | 1 | 2015 | 14. 运动神经兴奋时,何种离子进入轴突末梢的量与囊泡释放量呈正变关系? (2.0分) | A.Mg2+ B.Na+ C.Ca2+ D.K+ E.Cl‐ |
C.Ca2+ |
2 | 1 | 2015 | 15. 在神经‐骨骼肌接头部位,大量囊泡释放ACh所引起的膜电位变化称(2.0分) | A.突触后电位 B.接头后电位 C.局部电位 D.微终板电位 E.终板电位 |
E.终板电位 |
2 | 1 | 2015 | 16. 筒箭毒可被作为肌松剂应用,是由于能在终板膜部位(2.0分) | A.激活胆碱酯酶 B.与ACh竞争结合位点 C.与ACh 结合成复合物 D.抑制神经末梢Ca2+内流 E.减少囊泡释放Ach |
B.与ACh竞争结合位点 |
2 | 1 | 2015 | 17. 重症肌无力患者的骨骼肌对运动神经冲动的反应降低是由于 (2.0分) | A.递质含量减少 B.递质释放量减少 C.胆碱酯酶活性增高 D.受体数目减少或功能障碍 E.微终板电位减小 |
D.受体数目减少或功能障碍 |
2 | 1 | 2015 | 18. 在骨骼肌细胞兴奋‐收缩耦联过程中,胞质内的Ca2+来自(2.0分) | A.横管膜上电压门控Ca2+通道引起的胞外Ca2+内流 B.肌质网上Ca2+释放通道开放引起的胞内Ca2+释放 C.细胞膜上NMDA 受体通道开放引起的胞外Ca2+内流 D.肌质网上Ca2+泵的反向转运 E.线粒体内Ca2+的释放 |
B.肌质网上Ca2+释放通道开放引起的胞内Ca2+释放 |
2 | 1 | 2015 | 19. 下列关于神经胶质细胞的描述,正确的是 (2.0分) | A.既有树突又有轴突 B.细胞间普遍存在缝隙连接 C.有随细胞外Na+浓度改变的膜电位 D.有产生动作电位的能力 E.细胞间具有突触连接 |
B.细胞间普遍存在缝隙连接 |
2 | 1 | 2015 | 20. 神经元产生动作电位的部位是 (2.0分) | A.胞体 B.树突 C.轴丘 D.始段 E.轴索 |
D.始段 |
2 | 1 | 2015 | 21. 有髓神经纤维的传导特点是(2.0分) | A.传导速度慢 B.跳跃式传导 C.衰减性传导 D.单向传导 E.电缆式传导 |
B.跳跃式传导 |
2 | 1 | 2015 | 22. 下列不属于星形胶质细胞功能的是(2.0分) | A.营养作用 B.参与血-脑屏障的形成 C.吞噬作用 D.免疫应答作用 E.修复作用 |
C.吞噬作用 |
2 | 1 | 2015 | 23. 神经‐肌接头传递中,消除乙酰胆碱的酶是(2.0分) | A.胆碱酯酶 B.腺苷酸环化酶 C.磷酸二酯酶 D.ATP 酶 E.胆碱乙酰化酶 |
A.胆碱酯酶 |
2 | 1 | 2015 | 24. 流行病学研究方法包括观察法与实验法两大类,它们的根本区别在于(2.0分) | A.是否设立对照组 B.研究现场的不同 C.是否人为控制研究的条件 D.是否盲法观察 E.是否进行统计学检验 |
C.是否人为控制研究的条件 |
2 | 1 | 2015 | 25. 下列不属于观察性研究的是(2.0分) | A.临床试验 B.病例对照研究 C.临床病例报告 D.队列研究 |
A.临床试验 |
2 | 1 | 2015 | 26. 23名在妊娠期每天吸烟一包以上的孕妇,她们所生的头胎婴儿平均出生体重比16名从不吸烟妇女所生的头胎婴儿的平均体重轻2000g差异有统计学意义(P<0.05),这表明 (2.0分) | A.妊娠期吸烟有碍胎儿生长 B.平均出生体重的差异很大,不能仅用随机误差解释 C.平均出生体重的差异很容易由随机误差造成 D.研究孕妇吸烟与出生低体重的关系意义不大 E.妊娠期间吸烟不影响胎儿生长 |
A.妊娠期吸烟有碍胎儿生长 |
2 | 1 | 2015 | 27. 相对危险度 (2.0分) | A.是暴露比之于未暴露情况下增加超额疾病的数量 B.是非暴露组与暴露组发病或死亡危险之差的绝对值 C.是非暴露组发病或死亡的危险为暴露组的多少倍 D.比特异危险度更具有病因学意义 |
C.是非暴露组发病或死亡的危险为暴露组的多少倍 |
2 | 1 | 2015 | 28. 某研究小组在2000年的时候对某社区的人群进行调查,该社区30-60岁的非高血压人群都进行调查,总共有1500人被纳入到该研究中,其中发现320位居民抽样,随访10年,抽烟和非抽烟居民发生高血压的分别为120人和180人。该项研究用的是什么抽样方法? (2.0分) | A.简单随机抽样 B.分层抽样 C.系统抽样 D.整群抽样 |
D.整群抽样 |
2 | 1 | 2015 | 29. 关于锥体外系统的解剖生理描述那一项不正确? (2.0分) | A.调节肌张力,协调肌肉运动 B.维持和调整体态姿势 C.属于上运动神经元 D.担负半自动的刻板动作及反射性运动 E. |
C.属于上运动神经元 |
2 | 1 | 2015 | 30. 关于小脑病变那项描述不正确? (2.0分) | A.不同程度肌肉无力 B.共济平衡不良 C.轮替动作不协调 D.精细活动困难 E. |
A.不同程度肌肉无力 |
2 | 1 | 2015 | 31. 关于下运动神经元瘫痪那项是正确?(2.0分) | A.可表现为四肢远端或肢体的某一部位无力,肌张力增高,腱反射减弱或消失,病理反射阴性。 B.可表现为四肢远端或肢体的某一部位无力,肌张力减低,腱反射减弱或消失,病理反射阳性。 C.可表现为四肢远端或肢体的某一部位无力,肌张力增高,腱反射亢进,病理反射阳性,并早期明显肌肉萎缩 D.可表现为四肢远端或肢体的某一部位无力,肌张力增高,腱反射亢进,病理反射阳性,肌电图和神经传导速度正常 E.可表现为四肢远端或肢体的某一部位无力, 肌张力减低,腱反射减弱或消失,病理反射阴性,常伴感觉障碍和受累部位肌肉萎缩。 F. |
E.可表现为四肢远端或肢体的某一部位无力, 肌张力减低,腱反射减弱或消失,病理反射阴性,常伴感觉障碍和受累部位肌肉萎缩。 |
2 | 1 | 2015 | 32. 关于皮肌炎的描述那项错误?(2.0分) | A.对称性近端肌肌无力,肌酶升高,肌电图肌原性改变,可伴有皮疹 B.对称性近端肌肌无力,肌酶升高,肌电图肌原性改变,可伴有关节痛和关节炎 C.对称性近端肌肌无力,肌酶升高,肌电图肌原性改变,可伴有肺间质改变 D.对称性近端肌肌无力,肌酶升高,肌电图肌原性改变,可伴有心肌受累, E.对称性近端肌肌无力,肌酶升高,肌电图肌原性改变,可伴有恶性肿瘤和其它结缔组织病 F.对称性近端肌肌无力,肌酶升高,肌电图肌原性改变,肌活检特点肌细胞的胞浆和胞核内查到嗜酸性包涵体 |
F.对称性近端肌肌无力,肌酶升高,肌电图肌原性改变,肌活检特点肌细胞的胞浆和胞核内查到嗜酸性包涵体 |
2 | 1 | 2015 | 33. 患者,男,18岁 参加运动会400米跑步后次日,全身肌肉疼痛,痉挛,无力,检查发现低血糖,高血脂,肌红蛋白尿.为明确病因诊断最有价值辅助检查是?(2.0分) | A.肌酶 B.肌电图 C.肌肉病理和酶学检测 D.血钾和心电图 E.血清抗肌球蛋白抗体 F.线粒体病相关基因检测 |
C.肌肉病理和酶学检测 |
2 | 1 | 2015 | 34. 什么临床征兆特别警示该重症肌无力患者发生危象可能较大?(2.0分) | A.上睑下垂和复视 B.严重吞咽困难和构音障碍 C.乙酰胆碱受体抗体(AchR-ab)高滴度 D.明显呼吸困难,口唇紫绀,意识不清 E.四肢无力加重 F. |
B.严重吞咽困难和构音障碍 |
2 | 1 | 2015 | 35. 重症肌无力危象的首要抢救步骤?(2.0分) | A.区分肌无力性危象,胆碱能危象或反拗危象,以便对因治疗 B.保持呼吸道通畅,呼吸机支持 C.积极控制感染 D.大剂量免疫球蛋白或血浆置换 E.大剂量激素冲击治疗 F. |
B.保持呼吸道通畅,呼吸机支持 |
2 | 1 | 2015 | 36. 32岁,亚洲男性,暴食后次日早晨双下肢无力伴大腿酸胀;体检,意识清楚,颅神经正常,下肢对称性迟缓性瘫痪,大小便正常,感觉正常,反射正常。无家族遗传病史。最有意义的辅助检查是 (2.0分) | A.肌电图和神经传导 B.重复电刺激(RNS)单纤维肌电图(SFEMG) C.肌肉病理 D.血钾和心电图 E.甲状腺功能 F. |
D.血钾和心电图 |
2 | 1 | 2015 | 37. 低钾型麻痹经正规补充氯化钾数天后,低钾仍不能纠正最需要鉴别疾病是?(2.0分) | A.甲亢伴周期性麻痹 B.原发性醛固酮增多症和肾小管性酸中毒: C.低钾性周期性麻痹 D.过度胃肠道丢失后:呕吐、腹泻等 E. |
B.原发性醛固酮增多症和肾小管性酸中毒: |
2 | 1 | 2015 | 38. 管理小指皮肤感觉的神经是(2.0分) | A.正中神经 B.桡神经 C.尺神经 D.腋神经 E.肌皮神经 |
C.尺神经 |
2 | 1 | 2015 | 39. 脐平面的皮肤感觉神经是 (2.0分) | A.第8对胸神经 B.第9对胸神经 C.第10对胸神经 D.第1l对胸神经 E.第12对胸神经 |
C.第10对胸神经 |
2 | 1 | 2015 | 40. 病人出现呼吸困难症状,伴有窒息感,经检查发现腹式呼吸明显减弱。试问可能是什么神经受到了损伤 (2.0分) | A.膈神经 B.迷走神经 C.舌咽神经 D.内脏大神经 E.肋间神经 |
A.膈神经 |
2 | 1 | 2015 | 41. 患者,男,18岁 参加运动会400米跑步后次日,全身肌肉疼痛,痉挛,无力,检查发现低血糖,高血脂,肌红蛋白尿.可能最大的疾病诊断是? (2.0分) | A.脂质沉积性肌病 B.糖原累积病 C.线粒体肌病 D.周期性麻痹 E.重症肌无力 F.单纯的横纹肌溶解症 |
B.糖原累积病 |
2 | 1 | 2015 | 42. 细胞膜上不衰减形式传播的电活动是(2.0分) | A.动作电位 B.静息电位 C.终板电位 D.感受器电位 E.突触后电位 F. |
A.动作电位 |
2 | 1 | 2015 | 43. 临床上普鲁卡因局部麻醉药的应用是由于阻断了局部神经冲动的传导,其神经细胞发生的变化是 (2.0分) | A.细胞变性 B.结构完整性破坏 C.功能完整性破坏 D.细胞膜电压门控钾通道破坏 E.细胞膜化学门控钠通道破坏 |
C.功能完整性破坏 |
2 | 1 | 2015 | 44. 动作电位沿运动神经纤维传导抵达神经‐肌肉接头部位时,轴突末梢释放Ach,使终板膜产生终板电位,然后在什么部位引发动作电位(2.0分) | A.肌细胞膜 B.接头后膜 C.终板膜 D.横管膜 E.三联管膜 |
A.肌细胞膜 |
2 | 1 | 2015 | 45. 人工减小细胞浸浴液中的Na+浓度,所记录的动作电位出现 (2.0分) | A.幅度变小 B.幅度变大 C.时程缩短 D.时程延长 E.复极相延长 |
A.幅度变小 |
2 | 1 | 2015 | 46. 关于肌电图,以下说法正确的是 (2.0分) | A.肌电图所记录到的肌电是单根肌纤维的电活动 B.肌电图是骨骼肌在收缩时的电兴奋活动,经过引导、放大、记录而成的图形 C.肌电图是骨骼肌在收缩后的电兴奋活动,经过引导、放大、记录而成的图形 D.肌电图是骨骼肌在收缩前的电兴奋活动,经过引导、放大、记录而成的图形 |
D.肌电图是骨骼肌在收缩前的电兴奋活动,经过引导、放大、记录而成的图形 |
2 | 1 | 2015 | 47. 关于神经纤维的传导速度,以下说法错误的是 (2.0分) | A.有髓鞘神经纤维比无髓鞘神经纤维传导速度要快得多 B.与髓鞘厚度有关 C.与纤维直径成正比 D.与刺激强度成正比 E.神经纤维传导速度与温度呈正相关 |
D.与刺激强度成正比 |
2 | 1 | 2015 | 48. 下列哪种方法可以测量单个离子通道的电流?(2.0分) | A.离子通道的三维结构分析 B.同位素标记的配体结合分析 C.用特异抗体做免疫细胞化学染色 D.膜片钳(Patch-clamp)电生理记录 |
D.膜片钳(Patch-clamp)电生理记录 |
2 | 1 | 2015 | 49. 当突触前释放神经递质谷氨酸时,下列哪种兴奋性突触后反应排序符合时间先后次序(2.0分) | A.突触后膜去极化? AMPA受体激活? Ca2+内流? NMDA受体激活 B.Ca2+内流活?突触后膜去极化?AMPA受体激活?NMDA受体激活 C.AMPA受体激活?突触后膜去极化?NMDA受体兴奋?Ca2+内流 D.NMDA受体激活? Ca2+内流?突触后膜去极化?AMPA受体激活 |
C.AMPA受体激活?突触后膜去极化?NMDA受体兴奋?Ca2+内流 |
2 | 1 | 2015 | 50. 下列哪种方法可以用来分析离子通道的三维结构?(2.0分) | A.蛋白质氨基酸序列的疏水性分析(hydrophilicity plot) B.单颗粒冷冻电镜技术(Single particle cryo-EM) C.放射性同位素标记配体的结合试验(Binding Assay) D.膜片钳(Patch-clamp)电生理记录 |
B.单颗粒冷冻电镜技术(Single particle cryo-EM) |
2 | 2 | 2007 | 1. Bankart损伤是(2.0分) | A.肩关节脱位常见的一种病理改变 B.肩袖损伤常见的病理改变 C.肩峰撞击综合症 D.以上都不是 |
A.肩关节脱位常见的一种病理改变 |
2 | 2 | 2007 | 2. 疼痛弧试验用于检查下列那种疾病(2.0分) | A.肩关节脱位 B.膝关节脱位 C.肩袖损伤 D.膝关节交叉韧带损伤 |
C.肩袖损伤 |
2 | 2 | 2007 | 3. 急性运动损伤处理的一般原則(RICE原则)不包括:(2.0分) | A.休息 B.冰敷 C.压迫 D.缝合 E.抬高患处 |
D.缝合 |
2 | 2 | 2007 | 4. 关节镜手术适应症不包括(2.0分) | A.滑膜大部切除术 B.关节灌洗清创术、 C.人工关节置换术 D.膝关节撕裂半月板部份或全部切除术、 E.肱二头肌腱粘连松解术 |
C.人工关节置换术 |
2 | 2 | 2007 | 5. Finkelstein 试验是用于检查下列病变 (2.0分) | A.半月板损伤 B.狭窄性腱鞘炎 C.交叉韧带损伤 D.侧副韧带损伤 |
B.狭窄性腱鞘炎 |
2 | 2 | 2007 | 6. 关于肩袖正确的是(2.0分) | A.联合肌腱同关节囊之间有滑囊间隔 B.维持肩关节的稳定 C.肩关节外展的主要结构 D.喙肩韧带是肩袖的组成部分 |
D.喙肩韧带是肩袖的组成部分 |
2 | 2 | 2007 | 7. 关于肩袖损伤表现错误的是 (2.0分) | A.肩外展疼痛,肩外展或伴内外旋时,疼痛加重。 B.压痛局限于二头肌间沟 C.痛弧试验阳性。 D.外展外旋试验阳性 |
B.压痛局限于二头肌间沟 |
2 | 2 | 2007 | 8. 膝关节抽屉试验是检查(2.0分) | A.半月板损伤 B.交叉韧带损伤 C.关节侧副韧带损伤 D.软骨破碎 E.软骨过度增殖 |
E.软骨过度增殖 |
2 | 2 | 2007 | 9. 下列哪项不是半月板损伤主要表现(2.0分) | A.膝关节疼痛、肿胀、弹响 B.关节间隙压痛 C.膝前痛 D.活动障碍、交锁 |
C.膝前痛 |
2 | 2 | 2007 | 10. 膝关节损伤常见三联症 (2.0分) | A.前交叉韧带 内侧副韧带 内侧半月板 B.后交叉韧带 内侧副韧带 内侧半月板 C.前交叉韧带 外侧副韧带 内侧半月板 D.前交叉韧带 内侧副韧带 外侧半月板 |
A.前交叉韧带 内侧副韧带 内侧半月板 |
2 | 2 | 2007 | 11. McMurray-Fouche Test (2.0分) | A.检查内侧副韧带损伤 B.检查外侧副韧带损伤 C.检查半月板损伤 D.检查交叉韧带损伤 |
C.检查半月板损伤 |
2 | 2 | 2007 | 12. Guillain-Barre综合征出现呼吸明显无力,痰多时应采取的治疗为:(2.0分) | A.用呼吸兴奋剂 B.吸痰 C.吸氧 D.立即人工辅助呼吸,必要时气管切开, E.注射免疫抑制剂 |
D.立即人工辅助呼吸,必要时气管切开, |
2 | 2 | 2007 | 13. Guillain-Barre综合征最严重的症状是:(2.0分) | A.吞咽困难 B.呼吸肌麻痹 C.肺部感染 D.心力衰竭 E.四肢瘫痪 |
B.呼吸肌麻痹 |
2 | 2 | 2007 | 14. Guillain-Barre综合征自主神经病变可出现(2.0分) | A.血管舒缩功能障碍 B.汗腺活动功能障碍 C.心动过速 D.括约肌功能障碍 E.以上都是 |
E.以上都是 |
2 | 2 | 2007 | 15. 患者男,23岁,病前有低热不适,2天前出现双下肢进行性瘫痪,伴小便障碍,检查发现剑突水平以下所有感觉均障碍,双下肢迟缓性瘫痪,有尿潴留。其最可能的诊断是(2.0分) | A.脊髓出血 B.周期性麻痹 C.急性脊髓炎 D.脊髓肿瘤 E.急性多发性神经根神经炎 |
C.急性脊髓炎 |
2 | 2 | 2007 | 16. 患者女性,60岁,1年前出现左侧胸背部疼痛,左下肢无力,右下肢痛温觉减退,近2月出现双下肢均无力,近1个月伴排尿困难和便秘。查体:颅神经和双上肢正常,双下肢肌力2级,肌张力增高,膝反射亢进,双侧Babinski征(+),双T4以下痛觉温度觉和深感觉减退。最可能的诊断是: (2.0分) | A.脊髓炎 B.重症肌无力 C.脊髓空洞症 D.脊髓压迫症 E.Guillain-Barre综合征 |
D.脊髓压迫症 |
2 | 2 | 2007 | 17. 17.患者男性19岁,双足有踩棉花感,睁眼时站立稍不稳,闭目时加重,其病变部位最可能在: (2.0分) | A.丘脑 B.小脑 C.后索 D.额叶 E.侧索 |
C.后索 |
2 | 2 | 2007 | 18. 分离性感觉障碍最易出现于: (2.0分) | A.周围神经病变 B.脊神经节病变 C.脊髓前角病变 D.脊髓中央病变 E.脊髓横贯性损害 |
D.脊髓中央病变 |
2 | 2 | 2007 | 19. The classic fragile X syndrome phenotype is caused by:(2.0分) | A.expansion of the CGG repeat into the full mutation range B.abnormal RNA splicing C.hypermethylation of the CGG repeat D.all of the above E.A and B only F.A and C only |
F.A and C only |
2 | 2 | 2007 | 20. Fragile X syndrome and Huntington chorea exemplify a new category of genetic disease that involves triplet repeat instability. Boys with fragile X syndrome have over 500-1000 triplet repeats in region proximal to the fragile X gene coding sequence, while normal males have repeat numbers of less than 60. The figure below diagrams the results of genetic analysis of a fragile X syndrome family as conducted by which of the following techniques? (2.0分) | A.Immunoblotting B.Northern blotting C.Southern blotting D.Western blotting E.Reverse transcriptase-PCR amplification |
C.Southern blotting |
2 | 2 | 2007 | 21. Jane is concerned about her risk of Huntington disease because her maternal grandfather died of the disease. Her mother is 45 year old and has not exhibited signs of the disease so Jane’s brother Mike tells her she’s worried for nothing. Which of the following properties of mutations in the Huntington disease gene suggests that Mike says might not be true? (2.0分) | A.High rate of mutation reversion B.X-linked inheritance C.Incomplete penetrance D.Age-dependent penetrance E.It is a sex-limited phenotype |
D.Age-dependent penetrance |
2 | 2 | 2007 | 22. Which of the following trinucleotide repeat disorders is inherited in an autosomal recessive fashion?(2.0分) | A.fragile X syndrome B.Friedreich ataxia C.Huntington disease D.myotonic dystrophy E.spinocerebellar ataxia |
B.Friedreich ataxia |
2 | 2 | 2007 | 23. The CAG repeat in the Huntington disease gene encodes:(2.0分) | A.A signal to methylate the promoter. B.A signal to alter patterns of splicing. C.A RNA protein binding segment. D.A polyglutamine repeat. E.A transcription factor |
D.A polyglutamine repeat. |
2 | 2 | 2007 | 24. 流行病学研究中,疾病的三间分布指的是什么? (2.0分) | A.空间、病因和人群; B.时间、地点和人群; C.传染源、传播途径和易感人群; D.病原体、易感人群和地点; |
B.时间、地点和人群; |
2 | 2 | 2007 | 25. 以下哪项指标只有横断面研究可以获得? (2.0分) | A.发病率; B.率差(AR); C.率差%; D.患病率; |
D.患病率; |
2 | 2 | 2007 | 26. 与抽样调查相比,下列那一条是普查所特有的: (2.0分) | A.属于描述性研究 B.研究疾病分布特征 C.通过研究,可以提供病因线索 D.疾病的早期发现和早期诊断 |
D.疾病的早期发现和早期诊断 |
2 | 2 | 2007 | 27. 下列哪项是队列研究的不足之处 (2.0分) | A.不易确立因果关系 B.易发生回忆偏倚 C.易发生失访偏倚 D.论证强度不高 |
C.易发生失访偏倚 |
2 | 2 | 2007 | 28. 队列研究比较的是(2.0分) | A.不同组人群发病/死亡人数的差别 B.不同组人群患病/死亡人数的差别 C.不同组人群发病/死亡率的差别 D.不同组人群患病/死亡率的差别 |
C.不同组人群发病/死亡率的差别 |
2 | 2 | 2007 | 29. 拟在某工厂进行他汀类降血脂药物的降脂效果评价,该工厂男性职工2000人;女性职工2300人。根据既往调查发现,男性的甘油三酯水平为3.0±2.5mmol/L,女性的甘油三酯水平为3.2±1.0mmol/L。拟采用抽样调查的方式进行抽样,考虑到不同性别甘油三酯的变化,最好采用哪种抽样方法。 (2.0分) | A.系统抽样 B.最优分配分层随机抽样 C.等比例分层随机抽样; D.整群抽样; |
B.最优分配分层随机抽样 |
2 | 2 | 2007 | 30. 以下关于回顾性队列研究(又称历史性队列研究)的描述,不正确的是: (2.0分) | A.节省时间、物力; B.需要随访; C.资料收集迅速、出结果快; D.由果到因的时间顺序 |
B.需要随访; |
2 | 2 | 2007 | 31. 弗洛伊德将人的精神划分为若干部分,但日常生活中对我们影响较深的是(2.0分) | A.潜意识 B.前意识 C.意识 D.无意识 |
C.意识 |
2 | 2 | 2007 | 32. 有人指出在弗洛伊德理论中,人的性腺与人脑更大的理由是弗洛伊德过分强调了(2.0分) | A.力比多 B.客体 C.主体 D.利他 |
A.力比多 |
2 | 2 | 2007 | 33. 经典精神分析理论认为由于意识到自己的思想行为不符合道德规范而产生的良心不安、羞耻感和有罪感而产生的焦虑是 (2.0分) | A.现实的焦虑 B.神经症性焦虑 C.道德焦虑 D.以上都不是 |
C.道德焦虑 |
2 | 2 | 2007 | 34. 精神分析师可以通过释梦了解病人的潜意识,更好地为下列疾病提供治疗依据 (2.0分) | A.焦虑症 (或旧称神经症) B.精神分裂症 C.皮肤冬季瘙痒症 D.痴呆症 |
A.焦虑症 (或旧称神经症) |
2 | 2 | 2007 | 35. 依据马斯洛需要层次理论,中国知识分子的需要多停留在(2.0分) | A.生理需要 B.社交需要 C.安全需要 D.尊重的需要 |
D.尊重的需要 |
2 | 2 | 2007 | 36. 来访者中心治疗的根本原则就是人为地创造一种尊重气氛,且这种气氛满足下列条件(2.0分) | A.绝对的 B.无条件的 C.积极的 D.以上都是 |
D.以上都是 |
2 | 2 | 2007 | 37. 公元一世纪时,古希腊哲学家Epictetus就写到,打扰人们的不是事情本身,而是(2.0分) | A.人们对事情所持有的观点 B.人们的经历 C.邻居的情绪 D.骗子所设的骗局 |
A.人们对事情所持有的观点 |
2 | 2 | 2007 | 38. 研究心身疾病时,学者们愿意借用(2.0分) | A.微积分理论 B.星象学说 C.经济发展学说 D.心理生物学理论 |
D.心理生物学理论 |
2 | 2 | 2007 | 39. 颈膨大以上颈髓损伤出现何种瘫痪: (2.0分) | A.双上肢周围性瘫,双下肢中枢性瘫 B.四肢中枢性瘫 C.双下肢周围性瘫 D.双下肢中枢性瘫 E.四肢周围性瘫 |
B.四肢中枢性瘫 |
2 | 2 | 2007 | 40. 有关血管源性脑水肿,下列说法错误的是:(2.0分) | A.病理生理基础是血脑屏障破坏,血浆进入细胞和间隔。 B.主要发生在脑皮质。 C.可见于肿瘤、出血、炎症等。 D.在T1加权像显示不明显。 E.Gd-DTPA增强后无异常对比增强。 |
B.主要发生在脑皮质。 |
2 | 2 | 2007 | 41. 脑脓肿壁短T2低信号最可能是:(2.0分) | A.含铁血黄素沉着。 B.胶原结构。 C.血管影。 D.钙化。 E.铁沉积 |
B.胶原结构。 |
2 | 2 | 2007 | 42. MRI可提供多种信息,其中描述错误的是 (2.0分) | A.组织T1值 B.组织T2值 C.质子密度 D.组织密度值 E.组织代谢信息 |
D.组织密度值 |
2 | 2 | 2007 | 43. 表皮样囊肿MR信号特征不包括: (2.0分) | A.T1WI低信号 B.T2WI高信号 C.增强扫描明显强化 D.FLAIR高信号 E.DWI高信号 |
C.增强扫描明显强化 |
2 | 2 | 2007 | 44. SWI可以有效应用于如下疾病检查不包括(2.0分) | A.鉴别脑梗塞中出血灶 B.发现静脉畸形 C.Parkinson病 D.Alzheimer 病 E.脑炎 |
E.脑炎 |
2 | 2 | 2007 | 45. ASL属于那种检查(2.0分) | A.Diffusion B.Perfusion C.MR波谱 D.SWI E.Fmri |
B.Perfusion |
2 | 2 | 2007 | 46. 中央前回为:(2.0分) | A.运动中枢 B.感觉中枢 C.视觉中枢 D.听觉中枢 E.嗅觉中枢 |
A.运动中枢 |
2 | 2 | 2007 | 47. FLAIR序列常用于脑室旁病变显示,其主要特点是:(2.0分) | A.扫描速度快。 B.很好的T1对比。 C.很好的T2对比。 D.脑脊液的信号得到有效抑制。 |
D.脑脊液的信号得到有效抑制。 |
2 | 2 | 2007 | 48. 视神经鞘膜常发生的病变为:(2.0分) | A.胶质瘤。 B.神经鞘瘤 C.脑膜瘤。 D.炎症。 E.神经纤维瘤。 |
C.脑膜瘤。 |
2 | 2 | 2007 | 49. Gd-DTPA增强可用于(2.0分) | A.鉴别水肿与病变组织 B.碘过敏不能行CT增强者 C.在一定过程上区分肿瘤性病变与非肿瘤性病变 D.发现脑膜病变 E.以上均对 |
E.以上均对 |
2 | 2 | 2007 | 50. 桥脑胶质瘤的常见表现不包括:(2.0分) | A.T1加权像低信号 B.T2加权像高信号 C.桥脑肿胀 D.可包绕基底动脉 E.增强明显 |
E.增强明显 |
2 | 2 | 2008 | 1. 第2题中用的是什么抽样方法? (3.0分) | A.系统抽样 B.分层抽样 C.简单随机抽样 D.整群抽样 |
D.整群抽样 |
2 | 2 | 2008 | 2. 脑脓肿壁短T2低信号最可能是:(3.0分) | A.钙化。 B.胶原结构。 C.血管影。 D.含铁血黄素沉着。 E.铁沉积。 |
B.胶原结构。 |
2 | 2 | 2008 | 3. MRI可提供多种信息,其中描述错误的是(3.0分) | A.质子密度 B.组织T2值 C.组织T1值 D.组织密度值 E.组织代谢信息 |
D.组织密度值 |
2 | 2 | 2008 | 4. 有人指出在弗洛伊德理论中,人的性腺与人脑更大的理由是弗洛伊德过分强调了(3.0分) | A.主体 B.客体 C.力比多 D.利他 |
C.力比多 |
2 | 2 | 2008 | 5. FLAIR序列常用于脑室旁病变显示,其主要特点是: (3.0分) | A.脑脊液的信号得到有效抑制。 B.很好的T1对比。 C.很好的T2对比。 D.扫描速度快。 E.采用短的TI。 |
A.脑脊液的信号得到有效抑制。 |
2 | 2 | 2008 | 6. A 3-year-old boy is evaluated for symptoms of autism including poor eye contact and severe speech delay. Family history indicates that his mother had trouble in school due to "attention deficit", that he has an older brother with mental disability, and that another older brother and older sister are normal. The figure below diagrams the results of genetic analysis of the boy (lane 4), his mother (lane 5), and his siblings (lanes 1-3) for fragile X syndrome. Which of the following best describes the results of this genetic analysis? (4.0分) | A.Immunoblot analysis reveals the boy is not affected with fragile X syndrome but one brother is. B.Southern blot analysis reveals the boy is affected with fragile X syndrome as one brother. C.Northern blot analysis reveals the boy is affected with fragile X syndrome and that his mother is a carrier. D.Western blot analysis reveals the boy and his siblings do not have fragile X syndrome. E.Reverse transcriptase-PCR amplification reveals no aberrant RNA transcription in this family. |
B.Southern blot analysis reveals the boy is affected with fragile X syndrome as one brother. |
2 | 2 | 2008 | 7. 分离性感觉障碍最易出现于: (3.0分) | A.脊髓中央病变 B.脊神经节病变 C.脊髓前角病变 D.周围神经病变 E.脊髓横贯性损害 |
A.脊髓中央病变 |
2 | 2 | 2008 | 8. 在确定某因素是否为某病的病因时,与队列研究相比,病例对照研究的主要缺点是: (3.0分) | A.费用高、费时长 B.获得病例困难 C.在确定有无某病发生时,可能有困难 D.在确定有无可疑因素时,可能有偏性 E.所需样本较大 |
D.在确定有无可疑因素时,可能有偏性 |
2 | 2 | 2008 | 9. 精神分析师可以通过释梦了解病人的潜意识,更好地为下列疾病提供治疗依据(3.0分) | A.精神分裂症 B.焦虑症 (或旧称神经症) C.皮肤冬季瘙痒症 D.痴呆症 |
B.焦虑症 (或旧称神经症) |
2 | 2 | 2008 | 10. Guillain-Barre综合征自主神经病变可出现(3.0分) | A.血管舒缩功能障碍 B.心动过速 C.汗腺活动功能障碍 D.括约肌功能障碍 E.以上都是 |
E.以上都是 |
2 | 2 | 2008 | 11. Guillain-Barre综合征辅助检查可能有阳性表现的是:(3.0分) | A.肌电图(神经传导速度) B.X片 C.脊髓磁共振 D.PET E.以上都不是 |
A.肌电图(神经传导速度) |
2 | 2 | 2008 | 12. Guillain-Barre综合征最严重的症状是:(3.0分) | A.肺部感染 B.呼吸肌麻痹 C.心力衰竭 D.吞咽困难 E.四肢瘫痪 |
B.呼吸肌麻痹 |
2 | 2 | 2008 | 13. 经典精神分析理论认为由于意识到自己的思想行为不符合道德规范而产生的良心不安、羞耻感和有罪感而产生的焦虑是 (3.0分) | A.道德焦虑 B.神经症性焦虑 C.现实的焦虑 D.以上都不是 |
A.道德焦虑 |
2 | 2 | 2008 | 14. 有关血管源性脑水肿,下列说法错误的是: (3.0分) | A.可见于肿瘤、出血、炎症等。 B.主要发生在脑皮质。 C.病理生理基础是血脑屏障破坏,血浆进入细胞和间隔。 D.在T1加权像显示不明显。 E.Gd-DTPA增强后无异常对比增强。 |
B.主要发生在脑皮质。 |
2 | 2 | 2008 | 15. 表皮样囊肿MR信号特征不包括: (3.0分) | A.增强扫描明显强化 B.T1WI低信号 C.FLAIR高信号 D.T2WI高信号 E.DWI高信号 |
A.增强扫描明显强化 |
2 | 2 | 2008 | 16. 患者男性19岁,双足有踩棉花感,睁眼时站立稍不稳,闭目时加重,其病变部位最可能在: (3.0分) | A.小脑 B.丘脑 C.后索 D.额叶 E.侧索 |
C.后索 |
2 | 2 | 2008 | 17. 患者女性,60岁,1年前出现左侧胸背部疼痛,左下肢无力,右下肢痛温觉减退,近2月出现双下肢均无力,近1个月伴排尿困难和便秘。查体:颅神经和双上肢正常,双下肢肌力2级,肌张力增高,膝反射亢进,双侧Babinski征(+),双T4以下痛觉温度觉和深感觉减退。最可能的诊断是: (3.0分) | A.脊髓压迫症 B.重症肌无力 C.脊髓空洞症 D.脊髓炎 E.Guillain-Barre综合征 |
A.脊髓压迫症 |
2 | 2 | 2008 | 18. 视神经鞘膜常发生的病变为:(3.0分) | A.炎症。 B.脑膜瘤。 C.胶质瘤。 D.神经鞘瘤 E.神经纤维瘤。 |
B.脑膜瘤。 |
2 | 2 | 2008 | 19. 中央前回为:(3.0分) | A.感觉中枢 B.运动中枢 C.视觉中枢 D.听觉中枢 E.嗅觉中枢 |
B.运动中枢 |
2 | 2 | 2008 | 20. Guillain-Barre综合征出现呼吸明显无力,痰多时应采取的治疗为:(3.0分) | A.立即人工辅助呼吸,必要时气管切开, B.吸痰 C.吸氧 D.用呼吸兴奋剂 E.注射免疫抑制剂 |
A.立即人工辅助呼吸,必要时气管切开, |
2 | 2 | 2008 | 21. 患者男,23岁,病前有低热不适,2天前出现双下肢进行性瘫痪,伴小便障碍,检查发现剑突水平以下所有感觉均障碍,双下肢迟缓性瘫痪,有尿潴留。其最可能的诊断是: (3.0分) | A.脊髓出血 B.脊髓肿瘤 C.急性脊髓炎 D.周期性麻痹 E.急性多发性神经根神经炎 |
C.急性脊髓炎 |
2 | 2 | 2008 | 22. SWI可以有效应用于如下疾病检查不包括(3.0分) | A.D.Alzheimer 病 B.B.发现静脉畸形 C.C.Parkinson病 D.A.鉴别脑梗塞中出血灶 E.E.脑炎 |
E.E.脑炎 |
2 | 2 | 2008 | 23. 依据马斯洛需要层次理论,中国知识分子的需要多停留在(3.0分) | A.安全需要 B.社交需要 C.生理需要 D.尊重的需要 |
D.尊重的需要 |
2 | 2 | 2008 | 24. ASL属于那种检查(3.0分) | A.Diffusion B.MR波谱 C.Perfusion D.SWI E.fMRI |
C.Perfusion |
2 | 2 | 2008 | 25. 公元一世纪时,古希腊哲学家Epictetus就写到,打扰人们的不是事情本身,而是 (3.0分) | A.人们的经历 B.人们对事情所持有的观点 C.邻居的情绪 D.骗子所设的骗局 |
B.人们对事情所持有的观点 |
2 | 2 | 2008 | 26. 来访者中心治疗的根本原则就是人为地创造一种尊重气氛,且这种气氛满足下列条件(3.0分) | A.积极的 B.无条件的 C.绝对的 D.以上都是 |
D.以上都是 |
2 | 2 | 2008 | 27. The classic fragile X syndrome phenotype is caused by: (3.0分) | A.hypermethylation of the CGG repeat B.abnormal RNA splicing C.A and B only D.expansion of the CGG repeat into the full mutation range E.all of the above F.A and C only |
F.A and C only |
2 | 2 | 2008 | 28. 临床流行病学研究方法包括观察法与实验法两大类,它们的根本区别在于: (3.0分) | A.是否盲法观察 B.是否设立对照组 C.研究现场的不同 D.是否人为控制研究的条件 E.是否进行统计学检验 |
D.是否人为控制研究的条件 |
2 | 2 | 2008 | 29. Gd-DTPA增强可用于:(3.0分) | A.在一定过程上区分肿瘤性病变与非肿瘤性病变 B.碘过敏不能行CT增强者 C.鉴别水肿与病变组织 D.发现脑膜病变 E.以上均对 |
E.以上均对 |
2 | 2 | 2008 | 30. 弗洛伊德将人的精神划分为若干部分,但日常生活中对我们影响较深的是(3.0分) | A.潜意识 B.意识 C.前意识 D.无意识 |
B.意识 |
2 | 2 | 2008 | 31. In Huntington's disease: (3.0分) | A.The chorea observed is unique to Huntington's disease. B.Onset is typically insidious and occurs in adolescents. C.Personality changes including depression and suicide may be prominent. D.Females serve as carriers but rarely are affected. E.The family history is generally easy to obtain and aids greatly in diagnosis. |
C.Personality changes including depression and suicide may be prominent. |
2 | 2 | 2008 | 32. 假定某一筛选计划检查了1000人,某病的患病率为2%,而这种筛选试验的敏感度为95%特异度为90%,据此资料,被筛选出来的阳性者中,某病人数为 (3.0分) | A.20人 B.1人 C.117人 D.19人 E.98人 |
D.19人 |
2 | 2 | 2008 | 33. Jane is concerned about her risk of Huntington disease because her maternal grandfather died of the disease. Her mother is 45 year old and has not exhibited signs of the disease so Jane's brother Mike tells her she's worried for nothing. Which of the foll(3.0分) | A.High rate of mutation reversion B.Age-dependent penetrance C.Incomplete penetrance D.X-linked inheritance E.It is a sex-limited phenotype |
B.Age-dependent penetrance |
2 | 2 | 2009 | 1. 颅内最常见的原发肿瘤是: (2.0分) | A.脑膜瘤 B.胶质瘤 C.转移瘤 D.听神经瘤 |
B.胶质瘤 |
2 | 2 | 2009 | 2. 临床上最多见的脑血管畸形是(2.0分) | A.动脉瘤 B.海绵状血管瘤 C.动静脉畸形 D.毛细血管扩张症 |
C.动静脉畸形 |
2 | 2 | 2009 | 3. 尾状核头部占位首先引起下列什么部位受压改变: (2.0分) | A.侧脑室枕角 B.侧脑室三角部 C.颞叶 D.侧脑室额角 |
D.侧脑室额角 |
2 | 2 | 2009 | 4. 对于脑外伤后怀疑蛛网膜下腔出血的病人,首选的检查方法应为: (2.0分) | A.MRI弥散加权成像 B.PET 18F-FDG成像 C.CT或MR脑灌注成像 D.CT平扫 E.MR波谱分析 |
D.CT平扫 |
2 | 2 | 2009 | 5. T2-FLAIR序列可以改善脑室旁病变的显示,其主要原因是: (2.0分) | A.抑制了水分子弥散 B.很好的T1对比 C.很好的T2对比 D.脑脊液的信号得到有效抑制 E.扫描速度快 |
D.脑脊液的信号得到有效抑制 |
2 | 2 | 2009 | 6. 关于磁共振血管造影,下列说法正确的是:(2.0分) | A.可以使用TOF成像 B.只能二维显示,不能进行三维重建 C.血管和周围组织的对比比较差 D.血流速度和方向不影响成像效果 E.必须使用对比剂 |
A.可以使用TOF成像 |
2 | 2 | 2009 | 7. 以下关于水分子弥散的说法不正确的是(2.0分) | A.DWI是目前能够在活体上进行水分子弥散测量和成像的唯一方法 B.通过对水分子弥散方向的测定,可以了解神经纤维的走行方向 C.血管源性水肿导致局部水分子弥散受限 D.细胞内水肿导致局部水分子弥散受限 E.组织细胞密集导致水分子弥散受限 |
C.血管源性水肿导致局部水分子弥散受限 |
2 | 2 | 2009 | 8. 以下关于DTI定量参数的说法,正确的是: (2.0分) | A.FA表示弥散各向异性分数,数值范围在-1~1之间 B.FA数值越大表示弥散各向异性约小 C.FA等于1时,表示弥散各向同性 D.MD数值越大代表弥散越不受限 E.在数量上λ2总是大于λ1 |
D.MD数值越大代表弥散越不受限 |
2 | 2 | 2009 | 9. 急性缺血性脑卒中患者行CT脑灌注成像检查后,以下哪条强烈提示存在缺血半暗带(2.0分) | A.局部CBV显著降低 B.局部CBF和CBV均显著降低 C.局部MTT显著延长 D.局部CBV显著降低而CBF轻微减低 E.局部CBF显著降低而CBV轻微减低 |
E.局部CBF显著降低而CBV轻微减低 |
2 | 2 | 2009 | 10. ASL可以提供以下哪方面信息: (2.0分) | A.水分子弥散 B.脑血流灌注 C.脑神经元活动 D.脑组织代谢 E.血脑屏障的完整性 |
B.脑血流灌注 |
2 | 2 | 2009 | 11. 目前能够进行活体组织内化学物质无创性检测的方法是: (2.0分) | A.PWI B.DWI C.MRS D.MR动态增强 E.SWI |
C.MRS |
2 | 2 | 2009 | 12. 关于氢质子波谱分析,以下说法错误的是: (2.0分) | A.能在分子水平反应组织代谢情况 B.能够通过计算波峰下面积,对各代谢产物的浓度进行相对定量分析 C.所有导致神经元损伤和丢失的病变都可以表现有NAA峰的降低 D.在正常情况下,1.3ppm出可以测得Lac峰 E.Cho峰增高通常为肿瘤细胞增殖所致 |
D.在正常情况下,1.3ppm出可以测得Lac峰 |
2 | 2 | 2009 | 13. 脑内胶质瘤术后放疗后复查,增强扫描局部明显强化,下列哪种影像学技术无助于鉴别强化区域是肿瘤复发还是放射性坏死。(2.0分) | A.DWI B.PWI C.MRS D.SWI E.BOLD-fMRI |
E.BOLD-fMRI |
2 | 2 | 2009 | 14. SWI检查可以达到以下目的,但不包括: (2.0分) | A.鉴别脑梗塞中出血灶 B.发现脑静脉畸形 C.提示脑变性疾病中铁等顺磁性金属的沉积 D.发现脑外伤病人的脑内微出血 E.鉴别脑炎和低级别脑胶质瘤 |
E.鉴别脑炎和低级别脑胶质瘤 |
2 | 2 | 2009 | 15. 关于BOLD-fMRI技术,下列说法正确的是: (2.0分) | A.基于神经元活动继发的脑血流变化来成像 B.基于神经元活动继发的脑内葡萄糖含量改变来成像 C.可以用于某些神经功能的脑部定位 D.对于神经元活动的时间分辨率超过脑电记录 E.无法对大脑活动的整体性进行分析 |
C.可以用于某些神经功能的脑部定位 |
2 | 2 | 2009 | 16. Guillain-Barre综合征出现呼吸明显无力,痰多时应采取的治疗为: (2.0分) | A.用呼吸兴奋剂 B.吸痰 C.吸氧 D.立即人工辅助呼吸,必要时气管切开, E.注射免疫抑制剂 |
D.立即人工辅助呼吸,必要时气管切开, |
2 | 2 | 2009 | 17. Guillain-Barre综合征起病一周内最常见的表现是: (2.0分) | A.四肢弛缓性瘫 B.肌萎缩 C.尿潴留 D.腱反射亢进 E.以上都不是 |
E.以上都不是 |
2 | 2 | 2009 | 18. Guillain-Barre综合征最严重的症状是: (2.0分) | A.吞咽困难 B.肺部感染 C.呼吸肌麻痹 D.心力衰竭 E.四肢瘫痪 |
C.呼吸肌麻痹 |
2 | 2 | 2009 | 19. Guillain-Barre综合征神经病变的病理特征:(2.0分) | A.节段性脱髓鞘 B.轴索损伤 C.坏死 D.空泡样变 E.以上都不是 |
A.节段性脱髓鞘 |
2 | 2 | 2009 | 20. Guillain-Barre综合征自主神经病变可出现 (2.0分) | A.血管舒缩功能障碍 B.汗腺活动功能障碍 C.心动过速 D.括约肌功能障碍 E.以上都是 |
E.以上都是 |
2 | 2 | 2009 | 21. Guillain-Barre综合征免疫疗法包括 (2.0分) | A.抗生素 B.静脉注射免疫球蛋白 C.机械通气 D.输血 E.维生素B12 |
B.静脉注射免疫球蛋白 |
2 | 2 | 2009 | 22. 分离性感觉障碍最易出现于 (2.0分) | A.周围神经病变 B.脊神经节病变 C.脊髓前角病变 D.脊髓中央病变 E.脊髓横贯性损害 |
D.脊髓中央病变 |
2 | 2 | 2009 | 23. 前角损害可出现: (2.0分) | A.上运动神经元瘫痪 B.下运动神经元瘫痪 C.共济失调 D.Babinski征阳性 E.节段性感觉障碍 |
B.下运动神经元瘫痪 |
2 | 2 | 2009 | 24. 患者男性19岁,双足有踩棉花感,睁眼时站立稍不稳,闭目时加重,其病变部位最可能在: (2.0分) | A.丘脑 B.小脑 C.后索 D.额叶 E.侧索 |
C.后索 |
2 | 2 | 2009 | 25. 颈膨大以上颈髓损伤出现何种瘫痪: (2.0分) | A.双上肢周围性瘫,双下肢中枢性瘫 B.四肢中枢性瘫 C.双下肢周围性瘫 D.双下肢中枢性瘫 E.四肢周围性瘫 |
B.四肢中枢性瘫 |
2 | 2 | 2009 | 26. 患者女性,60岁,1年前出现左侧胸背部疼痛,左下肢无力,右下肢痛温觉减退,近2月出现双下肢均无力,近1个月伴排尿困难和便秘。查体:颅神经和双上肢正常,双下肢肌力2级,肌张力增高,膝反射亢进,双侧Babinski征(+),双T4以下痛觉温度觉和深感觉减退。最可能的诊断是: (2.0分) | A.脊髓炎 B.重症肌无力 C.脊髓空洞症 D.脊髓压迫症 E.Guillain-Barre综合征 |
D.脊髓压迫症 |
2 | 2 | 2009 | 27. 急性脊髓炎急性期可出现: (2.0分) | A.脊髓休克 B.尿便障碍 C.截瘫或四肢瘫 D.损伤平面以下深、浅感觉障碍 E.以上都是 |
E.以上都是 |
2 | 2 | 2009 | 28. 群体是流行病学研究的一大特征。这里的群体是指: (2.0分) | A.不患某病的人,包括正常人和患其他疾病的人 B.在一个人群中某种疾病的所有病人 C.某一特定范围的人群,包括病人和非病人 D.以上都不对 |
C.某一特定范围的人群,包括病人和非病人 |
2 | 2 | 2009 | 29. 描述短期的急性疾病的严重程度,以下哪个指标最好? (2.0分) | A.死亡专率; B.5年生存率; C.病死率; D.标化死亡率; |
C.病死率; |
2 | 2 | 2009 | 30. 一种新治疗方法可以延长寿命,但不能治愈该病,则会出现(2.0分) | A.该病患病率将降低 B.该病发病率将升高 C.该病发病率将降低 D.该病患病率将升高 |
D.该病患病率将升高 |
2 | 2 | 2009 | 31. 年龄调整的死亡率使用的目的是: (2.0分) | A.用来判断在一个人群中特定的年龄段发生死亡的实际数量; B.用来调整没有年龄信息的研究对象的死亡率; C.比较相同年龄组人群的死亡率; D.在比较死亡率的时候,用来排除年龄因素对死亡率分布的影响; |
D.在比较死亡率的时候,用来排除年龄因素对死亡率分布的影响; |
2 | 2 | 2009 | 32. 弗洛伊德将人的精神划分为若干部分,但日常生活中对我们影响较深的是 (2.0分) | A.潜意识 B.前意识 C.意识 D.无意识 |
C.意识 |
2 | 2 | 2009 | 33. 弗洛伊德有关人格结构模式较为独特,其中主导我们现实抉择的是 (2.0分) | A.本我 B.自我 C.超我 D.忘我 |
B.自我 |
2 | 2 | 2009 | 34. 有人指出在弗洛伊德理论中,人的性腺与人脑更大的理由是弗洛伊德过分强调了(2.0分) | A.力比多 B.客体 C.主体 D.利他 |
A.力比多 |
2 | 2 | 2009 | 35. 弗洛伊德认为婴幼儿期的人格停留在 (2.0分) | A.口腔期 B.肛门期 C.性器期 D.潜伏期 |
A.口腔期 |
2 | 2 | 2009 | 36. 经典精神分析理论认为由于意识到自己的思想行为不符合道德规范而产生的良心不安、羞耻感和有罪感而产生的焦虑是 (2.0分) | A.现实的焦虑 B.神经症性焦虑 C.道德焦虑 D.以上都不是 |
C.道德焦虑 |
2 | 2 | 2009 | 37. 自我防御机制的形成是为了 (2.0分) | A.减轻或消除人格内部的冲突 B.降低或避免焦虑 C.保持人格的完整和统一 D.以上都是 |
D.以上都是 |
2 | 2 | 2009 | 38. 精神分析师可以通过释梦了解病人的潜意识,更好地为下列疾病提供治疗依据(2.0分) | A.焦虑症 (或旧称神经症) B.精神分裂症 C.皮肤冬季瘙痒症 D.痴呆症 |
A.焦虑症 (或旧称神经症) |
2 | 2 | 2009 | 39. 新精神分析学派强调了社会文化因素对人格形成的重要作用,进而反对了弗洛伊德的 (2.0分) | A.本能论 B.泛性论 C.两者都是 D.两者都不是 |
C.两者都是 |
2 | 2 | 2009 | 40. 依据马斯洛需要层次理论,中国知识分子的需要多停留在 (2.0分) | A.生理需要 B.社交需要 C.安全需要 D.尊重的需要 |
D.尊重的需要 |
2 | 2 | 2009 | 41. 罗杰斯尤为提倡尊重人的客观权利,同时注重培养 (2.0分) | A.人格 B.创造精神 C.自我发展 D.以上都是 |
D.以上都是 |
2 | 2 | 2009 | 42. 来访者中心治疗的根本原则就是人为地创造一种尊重气氛,且这种气氛满足下列条件 (2.0分) | A.绝对的 B.无条件的 C.积极的 D.以上都是 |
D.以上都是 |
2 | 2 | 2009 | 43. 个体接受一个刺激之后,它行为的结果是消极刺激增加,从而使行为反应逐渐减弱,这种行为反射模式是 (2.0分) | A.正强化 B.负强化 C.消退 D.惩罚 |
D.惩罚 |
2 | 2 | 2009 | 44. A 3-year-old boy is evaluated for symptoms of autism including poor eye contact and severe speech delay. Family history indicates that his mother had trouble in school due to "attention deficit", that he has an older brother with mental disability, and that another older brother and older sister are normal. The figure below diagrams the results of genetic analysis of the boy (lane 4), his mother (lane 5), and his siblings (lanes 1-3) for fragile X syndrome. Which of the following best describes the results of this genetic analysis? (2.0分) | A.Immunoblot analysis reveals the boy is not affected with fragile X syndrome but one brother is. B.Northern blot analysis reveals the boy is affected with fragile X syndrome and that his mother is a carrier. C.Southern blot analysis reveals the boy is affected with fragile X syndrome as one brother. D.Western blot analysis reveals the boy and his siblings do not have fragile X syndrome. E.Reverse transcriptase-PCR amplification reveals no aberrant RNA transcription in this family. |
C.Southern blot analysis reveals the boy is affected with fragile X syndrome as one brother. |
2 | 2 | 2009 | 45. In Huntington disease: (2.0分) | A.Personality changes including depression and suicide may be prominent. B.Onset is typically insidious and occurs in adolescents. C.The chorea observed is unique to Huntington disease. D.Females serve as carriers but rarely are affected. E.The family history is generally easy to obtain and aids greatly in diagnosis. |
A.Personality changes including depression and suicide may be prominent. |
2 | 2 | 2009 | 46. The classic fragile X syndrome phenotype is caused by: (2.0分) | A.expansion of the CGG repeat into the full mutation range B.abnormal RNA splicing C.hypermethylation of the CGG repeat D.all of the above E.A and C only |
E.A and C only |
2 | 2 | 2009 | 47. Which of the following trinucleotide repeat disorders is likely to be more severe when inherited from the father? (2.0分) | A.fragile X syndrome B.Friedreich ataxia C.Huntington disease D.myotonic dystrophy E.none of the above |
C.Huntington disease |
2 | 2 | 2009 | 48. The following statements about molecular tests are NOT correct: (2.0分) | A.Northern blotting involves analysis of RNA B.Western blotting involves analysis of proteins C.Southern blotting identifies all point mutations D.PCR can detect large triplet repeat expansions E.PCR can detect large rearrangements |
C.Southern blotting identifies all point mutations |
2 | 2 | 2009 | 49. The CAG repeat in the Huntington disease gene encodes: (2.0分) | A.A signal to methylate the promoter. B.A signal to alter patterns of splicing. C.A RNA protein binding segment. D.A polyglutamine repeat. E.A transcription factor |
D.A polyglutamine repeat. |
2 | 2 | 2009 | 50. A 22-year-old African-American international female student of ZJU presents with a complaint of muscle weakness following exercise. Neurological examination reveals that muscles supplied by cranial nerves are most affected. You suspect myasthenia gravis, a diagnosis confirmed by antibodies against which of the following in the patient's blood? (2.0分) | A.Acetylcholinesterase B.Muscle mitochondrial membranes C.Cranial nerve synaptic membranes D.Cranial nerve presynaptic membranes E.Acetylcholine receptor |
E.Acetylcholine receptor |
2 | 2 | 2010 | 1. The CAG repeat in the Huntington disease gene encodes: (3.0分) | A.A signal to methylate the promoter. B.A signal to alter patterns of splicing. C.A RNA protein binding segment. D.A polyglutamine repeat. E.A transcription factor |
D.A polyglutamine repeat. |
2 | 2 | 2010 | 2. A husband and wife have 3 children, two boys and one girl. The husband was diagnosed with Huntington disease in his mid-fifties, as was his father. The wife has no symptoms and no family history of Huntington disease. Assuming 100% penetrance, what is the probability that all 3 of his children will eventually develop Huntington disease? (3.0分) | A.41647 B.41643 C.41641 D.41702 E.Virtually 0 |
A.41647 |
2 | 2 | 2010 | 3. In Huntington disease:(3.0分) | A.Personality changes including depression and suicide may be prominent. B.Onset is typically insidious and occurs in adolescents. C.The chorea observed is unique to Huntington disease. D.Females serve as carriers but rarely are affected. E.The family history is generally easy to obtain and aids greatly in diagnosis. |
A.Personality changes including depression and suicide may be prominent. |
2 | 2 | 2010 | 4. Which of the following trinucleotide repeat disorders is inherited in an autosomal recessive fashion? (3.0分) | A.fragile X syndrome B.Friedreich ataxia C.Huntington disease D.myotonic dystrophy E.spinocerebellar ataxia |
B.Friedreich ataxia |
2 | 2 | 2010 | 5. The classic fragile X syndrome phenotype is caused by: (3.0分) | A.expansion of the CGG repeat into the full mutation range B.abnormal RNA splicing C.hypermethylation of the CGG repeat D.all of the above E.A and C only |
E.A and C only |
2 | 2 | 2010 | 6. 颅内最常见的原发肿瘤是(3.0分) | A.脑膜瘤 B.胶质瘤 C.转移瘤 D.听神经瘤 |
B.胶质瘤 |
2 | 2 | 2010 | 7. 临床上最多见的脑血管畸形是:(3.0分) | A.动脉瘤 B.海绵状血管瘤 C.动静脉畸形 D.毛细血管扩张症 |
C.动静脉畸形 |
2 | 2 | 2010 | 8. 尾状核头部占位首先引起下列什么部位受压改变:(3.0分) | A.侧脑室枕角 B.侧脑室三角部 C.颞叶 D.侧脑室额角 |
D.侧脑室额角 |
2 | 2 | 2010 | 9. 在一项研究中,发现年龄在30-35岁年龄组中,男性的偏头痛有0.5%,而女性的偏头痛有1%。因此推测在这个年龄段中,女性发生偏头痛风险是男性的2倍。这样的推测对吗? (3.0分) | A.正确; B.不对,因为没有考虑到性别之间的年龄队列效应; C.不对,因为采用的只是比值来比较男女之间的差别; D.不对,因为无法区别偏头痛发生的比率是发病率还是患病率; E.不对,因为没有考虑检出率的差异; |
D.不对,因为无法区别偏头痛发生的比率是发病率还是患病率; |
2 | 2 | 2010 | 10. 年龄调整的死亡率使用的目的是: (3.0分) | A.用来判断在一个人群中特定的年龄段发生死亡的实际数量; B.在比较死亡率的时候,用来排除年龄因素对死亡率分布的影响; C.比较相同年龄组人群的死亡率; D.用来调整没有年龄信息的研究对象的死亡率; E.用来调整不同来源数据的偏倚; |
B.在比较死亡率的时候,用来排除年龄因素对死亡率分布的影响; |
2 | 2 | 2010 | 11. 有关移民流行病学研究的叙述,错误的是: (4.0分) | A.是疾病或健康状况在时间、空间和人群分布上的综合描述; B.可以区分遗传因素或环境因素对疾病或健康状况影响的大小; C.可以对研究因素与疾病或健康状况是否存在关联进行检验; D.通过比较移民、原居地与现居地本土人群疾病或健康状况发生率的差异来进行研究; |
C.可以对研究因素与疾病或健康状况是否存在关联进行检验; |
2 | 2 | 2010 | 12. Guillain-Barre综合征出现呼吸明显无力,痰多时应采取的治疗为:(3.0分) | A.用呼吸兴奋剂 B.吸痰 C.吸氧 D.立即人工辅助呼吸,必要时气管切开, E.注射免疫抑制剂 |
D.立即人工辅助呼吸,必要时气管切开, |
2 | 2 | 2010 | 13. Guillain-Barre综合征起病一周内最常见的表现是:(3.0分) | A.四肢弛缓性瘫 B.肌萎缩 C.尿潴留 D.腱反射亢进 E.以上都不是 |
E.以上都不是 |
2 | 2 | 2010 | 14. Guillain-Barre综合征最严重的症状是(3.0分) | A.吞咽困难 B.肺部感染 C.呼吸肌麻痹 D.心力衰竭 E.四肢瘫痪 |
C.呼吸肌麻痹 |
2 | 2 | 2010 | 15. Guillain-Barre综合征神经病变的病理特征:(3.0分) | A.节段性脱髓鞘 B.轴索损伤 C.坏死 D.空泡样变 E.以上都不是 |
A.节段性脱髓鞘 |
2 | 2 | 2010 | 16. Guillain-Barre综合征自主神经病变可出现(3.0分) | A.血管舒缩功能障碍 B.汗腺活动功能障碍 C.心动过速 D.括约肌功能障碍 E.以上都是 |
E.以上都是 |
2 | 2 | 2010 | 17. Guillain-Barre综合征免疫疗法包括 (3.0分) | A.抗生素 B.静脉注射免疫球蛋白 C.机械通气 D.输血 E.维生素B12 |
B.静脉注射免疫球蛋白 |
2 | 2 | 2010 | 18. Guillain-Barre综合征出现呼吸明显无力,痰多时应采取的治疗为:(3.0分) | A.用呼吸兴奋剂 B.吸痰 C.吸氧 D.立即人工辅助呼吸,必要时气管切开, E.注射免疫抑制剂 |
D.立即人工辅助呼吸,必要时气管切开, |
2 | 2 | 2010 | 19. Guillain-Barre综合征起病一周内最常见的表现是:(3.0分) | A.四肢弛缓性瘫 B.肌萎缩 C.尿潴留 D.腱反射亢进 E.以上都不是 |
E.以上都不是 |
2 | 2 | 2010 | 20. Guillain-Barre综合征最严重的症状是:(3.0分) | A.吞咽困难 B.肺部感染 C.呼吸肌麻痹 D.心力衰竭 E.四肢瘫痪 |
C.呼吸肌麻痹 |
2 | 2 | 2010 | 21. Guillain-Barre综合征神经病变的病理特征:(3.0分) | A.节段性脱髓鞘 B.轴索损伤 C.坏死 D.空泡样变 E.以上都不是 |
A.节段性脱髓鞘 |
2 | 2 | 2010 | 22. Guillain-Barre综合征自主神经病变可出现(3.0分) | A.血管舒缩功能障碍 B.汗腺活动功能障碍 C.心动过速 D.括约肌功能障碍 E.以上都是 |
E.以上都是 |
2 | 2 | 2010 | 23. Guillain-Barre综合征免疫疗法包括 (3.0分) | A.抗生素 B.静脉注射免疫球蛋白 C.机械通气 D.输血 E.维生素B12 |
B.静脉注射免疫球蛋白 |
2 | 2 | 2010 | 24. 弗洛伊德将人的精神划分为若干部分,但日常生活中对我们影响较深的是 (3.0分) | A.潜意识 B.前意识 C.意识 D.无意识 |
C.意识 |
2 | 2 | 2010 | 25. 有人指出在弗洛伊德理论中,人的性腺与人脑更大的理由是弗洛伊德过分强调了(3.0分) | A.力比多 B.客体 C.主体 D.利他 |
A.力比多 |
2 | 2 | 2010 | 26. 经典精神分析理论认为由于意识到自己的思想行为不符合道德规范而产生的良心不安、羞耻感和有罪感而产生的焦虑是 (3.0分) | A.现实的焦虑 B.神经症性焦虑 C.道德焦虑 D.以上都不是 |
C.道德焦虑 |
2 | 2 | 2010 | 27. 精神分析师可以通过释梦了解病人的潜意识,更好地为下列疾病提供治疗依据 (3.0分) | A.焦虑症 (或旧称神经症) B.精神分裂症 C.皮肤冬季瘙痒症 D.痴呆症 |
A.焦虑症 (或旧称神经症) |
2 | 2 | 2010 | 28. 依据马斯洛需要层次理论,中国知识分子的需要多停留在(3.0分) | A.生理需要 B.社交需要 C.安全需要 D.尊重的需要 |
D.尊重的需要 |
2 | 2 | 2010 | 29. 来访者中心治疗的根本原则就是人为地创造一种尊重气氛,且这种气氛满足下列条件 (3.0分) | A.绝对的 B.无条件的 C.积极的 D.以上都是 |
D.以上都是 |
2 | 2 | 2010 | 30. 公元一世纪时,古希腊哲学家Epictetus就写到,打扰人们的不是事情本身,而是(3.0分) | A.人们对事情所持有的观点 B.人们的经历 C.邻居的情绪 D.骗子所设的骗局 |
A.人们对事情所持有的观点 |
2 | 2 | 2010 | 31. 研究心身疾病时,学者们愿意借用 (3.0分) | A.微积分理论 B.星象学说 C.经济发展学说 D.心理生物学理论 |
D.心理生物学理论 |
2 | 2 | 2010 | 32. 个体接受一个刺激之后,它行为的结果是消极刺激增加,从而使行为反应逐渐减弱,这种行为反射模式是(3.0分) | A.正强化 B.负强化 C.消退 D.惩罚 |
D.惩罚 |
2 | 2 | 2010 | 33. 罗杰斯尤为提倡尊重人的客观权利,同时注重培养 (3.0分) | A.人格 B.创造精神 C.自我发展 D.以上都是 |
D.以上都是 |
2 | 2 | 2011 | 1. 用方形电脉冲刺激组织,方形电脉冲的刺激"三要素"是(2.0分) | A.方形电脉冲的幅度、波宽、方波前沿的强度变化率 B.方形电脉冲的幅度、波宽、方波后沿的强度变化率 C.方形电脉冲的幅度、频率、方波前沿的强度变化率 D.方形电脉冲的幅度、波宽、频率 |
A.方形电脉冲的幅度、波宽、方波前沿的强度变化率 |
2 | 2 | 2011 | 2. 用方形电脉冲刺激组织,在一定的刺激波宽下,刚能引起组织发生兴奋的刺激称为(2.0分) | A.阈下刺激 B.阈刺激 C.阈上刺激 D.最大刺激 |
B.阈刺激 |
2 | 2 | 2011 | 3. 对蟾蜍坐骨神经干动作电位引导实验,下列叙述正确的是(2.0分) | A.神经干动作电位的振幅与刺激强度成正比 B.在3cm范围内增加引导电极间距,BAP振幅增大 C.3 mol/L KCl处理神经干,KCl使神经纤维产生超极化抑制 D.4g/L普如卡因处理神经干,普如卡因使神经纤维产生超极化抑制 |
B.在3cm范围内增加引导电极间距,BAP振幅增大 |
2 | 2 | 2011 | 4. 下列叙述正确的是(2.0分) | A.神经干的神经冲动传导速度与刺激强度有关 B.神经干的神经冲动传导速度与神经干的兴奋性有关 C.神经干的神经冲动传导速度与实验环境温度无关 D.神经干的神经冲动传导速度与神经干受刺激和引导的方向有关 |
B.神经干的神经冲动传导速度与神经干的兴奋性有关 |
2 | 2 | 2011 | 5. A husband and wife have 3 children, two boys and one girl. The husband was diagnosed with Huntington disease in his mid-fifties, as was his father. The wife has no symptoms and no family history of Huntington disease. Assuming 100% penetrance, what is the probability that all 3 of his children will eventually develop Huntington disease?(2.0分) | A.42012 B.42008 C.42006 D.42067 E.Virtually 0 |
A.42012 |
2 | 2 | 2011 | 6. Which of the following trinucleotide repeat disorders is inherited in an autosomal recessive fashion?(2.0分) | A.fragile X syndrome B.Friedreich ataxia C.Huntington disease D.myotonic dystrophy E.spinocerebellar ataxia |
B.Friedreich ataxia |
2 | 2 | 2011 | 7. Which of the following trinucleotide repeat disorders is likely to be more severe when inherited from the father?(2.0分) | A.fragile X syndrome B.Friedreich ataxia C.Huntington disease D.myotonic dystrophy E.none of the above |
C.Huntington disease |
2 | 2 | 2011 | 8. Currently, a novel significant modifier of Alzheimer disease risk was identified by whole-exome and whole-genome sequencing in families with multiple individuals affected with Alzheimer disease. It was(2.0分) | A.APP B.PSEN1 C.PSEN2 D.APOE E.TREM2 |
E.TREM2 |
2 | 2 | 2011 | 9. 下面关于姿势的描述错误的是:(2.0分) | A.人体的姿势与身体的健康状态以及个人习惯有着密切关系 B.姿势举止的状态,不受某些疾病的影响 C.观察姿势可提供重要的疾病线索 D.观察姿势对诊断有参考意义 |
B.姿势举止的状态,不受某些疾病的影响 |
2 | 2 | 2011 | 10. 下面关于步态的描述正确的是(2.0分) | A.步态是一种简单的运动过程 B.步态不涉及脊髓反射和大、小脑的调节 C.步态要求神经和肌肉系统的高度协调 D.步态不涉及大、小脑的调节 |
C.步态要求神经和肌肉系统的高度协调 |
2 | 2 | 2011 | 11. 下面关于醉酒步态的描述正确的是(2.0分) | A.步行时两腿间距正常 B.可以走直线 C.重心不易控制 D.可以稳定站立 |
C.重心不易控制 |
2 | 2 | 2011 | 12. 下面关于感觉性共济失调步态的描述正确的是(2.0分) | A.由浅感觉障碍引起 B.行走时步幅较大 C.两腿间距正常 D.以大脑病变的可能性较大 |
B.行走时步幅较大 |
2 | 2 | 2011 | 13. 下面关于痉挛性偏瘫步态的描述错误的是(2.0分) | A.患侧下肢屈曲困难 B.又称划圈样步态 C.多见于小脑肿瘤, D.是由一侧锥体束损害引起 |
C.多见于小脑肿瘤, |
2 | 2 | 2011 | 14. 关于慌张步态的描述错误的是(2.0分) | A.全身肌张力增高 B.行走时重心后移 C.走路时步伐细小 D.多见于震颤麻痹 |
B.行走时重心后移 |
2 | 2 | 2011 | 15. 关于摇摆步态的描述正确的是(2.0分) | A.骨盆带肌及腰肌肌力增高 B.站立时脊柱前凸以维持身体重心平衡 C.行走臀部前后摇摆 D.多见于小脑肿瘤 |
B.站立时脊柱前凸以维持身体重心平衡 |
2 | 2 | 2011 | 16. 关于舞蹈步态的描述错误的是(2.0分) | A.步行时肢体有大幅度的、不规则的、不自主运动 B.行路不稳,呈跳跃式或舞蹈样 C.下肢突然外甩,上肢扭曲 D.多见于小脑肿瘤 |
D.多见于小脑肿瘤 |
2 | 2 | 2011 | 17. 有关血管源性脑水肿,下列说法错误的是(2.0分) | A.病理生理基础是血脑屏障破坏,血浆进入细胞和间隔。 B.主要发生在脑皮质。 C.可见于肿瘤、出血、炎症等。 D.在T1加权像显示不明显。 E.Gd-DTPA增强后无异常对比增强。 |
B.主要发生在脑皮质。 |
2 | 2 | 2011 | 18. 目前能够进行活体组织内化学物质无创性检测的方法是(2.0分) | A.PWI B.DWI C.MR波谱 D.MR动态增强 E.MR |
C.MR波谱 |
2 | 2 | 2011 | 19. 脑内环形强化占位,DWI弥散受限高信号,邻近脑实质水肿,需考虑诊断(2.0分) | A.转移瘤 B.星形细胞瘤 C.脑脓肿 D.脑囊虫病 E.蛛网膜囊肿 |
C.脑脓肿 |
2 | 2 | 2011 | 20. SWI可以有效应用于如下疾病检查不包括(2.0分) | A.鉴别脑梗塞中出血灶 B.发现静脉畸形 C.Parkinson病 D.Alzheimer 病 E.脑炎 |
E.脑炎 |
2 | 2 | 2011 | 21. Gd-DTPA增强可用于(2.0分) | A.鉴别水肿与病变组织 B.碘过敏不能行CT增强者 C.在一定过程上区分肿瘤性病变与非肿瘤性病变 D.发现脑膜病变 E.以上均对 |
E.以上均对 |
2 | 2 | 2011 | 22. 以下何种脊髓损伤预后最好(2.0分) | A.脊髓中央损伤综合征 B.前脊髓症候群 C.脊髓半侧横断综合征 D.脊髓后索综合征 E.脊髓离断 |
D.脊髓后索综合征 |
2 | 2 | 2011 | 23. 桥脑胶质瘤的常见表现不包括(2.0分) | A.T1加权像低信号 B.T2加权像高信号 C.桥脑肿胀 D.可包绕基底动脉 E.增强明显 |
E.增强明显 |
2 | 2 | 2011 | 24. Guillain-Barre综合征起病一周内最常见的表现是(2.0分) | A.四肢弛缓性瘫 B.肌萎缩 C.尿潴留 D.腱反射亢进 E.以上都不是 |
E.以上都不是 |
2 | 2 | 2011 | 25. Guillain-Barre综合征神经病变的病理特征(2.0分) | A.节段性脱髓鞘 B.轴索损伤 C.坏死 D.空泡样变 E.以上都不是 |
A.节段性脱髓鞘 |
2 | 2 | 2011 | 26. Guillain-Barre综合征免疫疗法包括(2.0分) | A.抗生素 B.静脉注射免疫球蛋白 C.机械通气 D.输血 E.维生素B12 |
B.静脉注射免疫球蛋白 |
2 | 2 | 2011 | 27. Guillain-Barre综合征的常见病因? (2.0分) | A.外伤 B.病毒感染 C.中毒 D.酗酒 E.滥用药物 |
B.病毒感染 |
2 | 2 | 2011 | 28. 前角损害可出现(2.0分) | A.上运动神经元瘫痪 B.下运动神经元瘫痪 C.共济失调 D.Babinski征阳性 E.节段性感觉障碍 |
B.下运动神经元瘫痪 |
2 | 2 | 2011 | 29. 颈膨大以上颈髓损伤出现何种瘫痪(2.0分) | A.双上肢周围性瘫,双下肢中枢性瘫 B.四肢中枢性瘫 C.双下肢周围性瘫 D.双下肢中枢性瘫 E.四肢周围性瘫 |
B.四肢中枢性瘫 |
2 | 2 | 2011 | 30. 急性脊髓炎急性期可出现(2.0分) | A.脊髓休克 B.尿便障碍 C.截瘫或四肢瘫 D.损伤平面以下深、浅感觉障碍 E.以上都是 |
E.以上都是 |
2 | 2 | 2011 | 31. Brown-Sequard综合征中,下列所述最恰当的是(2.0分) | A.同侧受损平面以下上运动神经元瘫痪 B.同侧受损平面以下震动感觉缺失 C.见于脊髓半切损伤 D.对侧受损平面以下痛温觉障碍 E.以上都是 |
E.以上都是 |
2 | 2 | 2011 | 32. 病例对照研究中,选择新病例的优点是(2.0分) | A.需要的样本较小 B.保密性问题较少 C.可以减少回忆偏倚 D.费用较低 E.便于发现病例 |
C.可以减少回忆偏倚 |
2 | 2 | 2011 | 33. 对大多数病例对照研究来说,它们不具有下列哪种特征(2.0分) | A.简单易行,耗资较少 B.可以估计相对危险度 C.可以直接计算发病率 D.暴露因素是回顾所得,故可能出现偏倚 E.在较短时间内就可获得研究结果 |
C.可以直接计算发病率 |
2 | 2 | 2011 | 34. 病例对照研究的特点是(2.0分) | A.设立比较组 B. 研究方向是由果及因 C.适用于罕见病 D. 节省时间、经费 E. 以上都是 |
E. 以上都是 |
2 | 2 | 2011 | 35. 一项雌激素与子宫内膜癌关系的配对病例对照研究,共63对。病例组与对照组两组均有雌激素暴露史者27对,两组均无暴露史者4对,病例组有暴露而对照组无暴露史者29对,其余为对照组有暴露而病例组无暴露者。OR为是(2.0分) | A.10.67 B.9.67 C.2.24 D.1.24 E. 4.47 |
B.9.67 |
2 | 2 | 2011 | 36. 在分析性研究中,通过分层分析和多因素分析可以控制(2.0分) | A.入院率偏倚 B.现患病例-新发病例偏倚 C.暴露怀疑偏倚 D.回忆偏倚 E.混杂偏倚 |
E.混杂偏倚 |
2 | 2 | 2011 | 37. 认知心理学主张研究(2.0分) | A.为研究人的行为及影响因素 B.研究影响及决定人行为的潜意识 C.研究区别于动物的动机、欲望、生命意义、情感等复杂体验 D.研究人的感知、记忆、思维等信息加工过程及影响因素 |
D.研究人的感知、记忆、思维等信息加工过程及影响因素 |
2 | 2 | 2011 | 38. 有人指出在弗洛伊德理论中,人的性腺与人脑更大的理由是弗洛伊德过分强调了(2.0分) | A.力比多 B.客体 C.主体 D.利他 |
A.力比多 |
2 | 2 | 2011 | 39. 经典精神分析理论认为由于意识到自己的思想行为不符合道德规范而产生的良心不安、羞耻感和有罪感而产生的焦虑是(2.0分) | A.现实的焦虑 B.神经症性焦虑 C.道德焦虑 D.以上都不是 |
C.道德焦虑 |
2 | 2 | 2011 | 40. 精神分析师可以通过释梦了解病人的潜意识,更好地为下列疾病提供治疗依据(2.0分) | A.焦虑症 (或旧称神经症) B.精神分裂症 C.皮肤冬季瘙痒症 D.痴呆症 |
A.焦虑症 (或旧称神经症) |
2 | 2 | 2011 | 41. 来访者中心治疗的根本原则就是人为地创造一种尊重气氛,且这种气氛满足下列条件 (2.0分) | A.绝对的 B.无条件的 C.积极的 D.以上都是 |
D.以上都是 |
2 | 2 | 2011 | 42. 公元一世纪时,古希腊哲学家Epictetus就写到,打扰人们的不是事情本身,而是(2.0分) | A.人们对事情所持有的观点 B.人们的经历 C.邻居的情绪 D.骗子所设的骗局 |
A.人们对事情所持有的观点 |
2 | 2 | 2011 | 43. 研究心身疾病时,学者们愿意借用(2.0分) | A.微积分理论 B.星象学说 C.经济发展学说 D.心理生物学理论 |
D.心理生物学理论 |
2 | 2 | 2011 | 44. 创伤性颅脑急症,常规和首选检查方法(2.0分) | A.X线 B.MRI C.超声 D.CT |
D.CT |
2 | 2 | 2011 | 45. 尾状核头部占位首先引起下列什么部位受压改变(2.0分) | A.侧脑室枕角 B.侧脑室三角部 C.颞叶 D.侧脑室额角 |
D.侧脑室额角 |
2 | 2 | 2011 | 46. 临床上最多见的脑血管畸形是(2.0分) | A.动脉瘤 B.海绵状血管瘤 C.动静脉畸形 D.毛细血管扩张症 |
C.动静脉畸形 |
2 | 2 | 2011 | 47. 下列关于脑膜瘤的描述,错位的是(2.0分) | A.多见于中年女性 B.多位于脑外,少数位于脑室内 C.MRI增强检查可出现脑膜尾征 D.增强检查一般不强化 |
D.增强检查一般不强化 |
2 | 2 | 2011 | 48. 弥漫性轴索损伤常见于__损伤 (2.0分) | A.加速性颅脑损伤 B.减速性颅脑损伤 C.挤压性颅脑损伤 D.挥鞭样损伤、剪切伤 E.枪弹开放性损伤 |
D.挥鞭样损伤、剪切伤 |
2 | 2 | 2011 | 49. 弥漫性轴索损伤常见发生的部位是(2.0分) | A.白质和灰质交界处 B.两大脑半球之间的胼胝体, C.脑干头端以及小脑 D.内囊和基底节 E.以上均正确。 |
E.以上均正确。 |
2 | 2 | 2011 | 50. 下列陈述不正确的是(2.0分) | A.正常人脑脊液T1WI呈高信号,T2WI呈低信号 B.高血压性脑出血常发生于基底节区、丘脑、小脑等。 C.脑血肿急性期CT上呈高密度,以后逐渐吸收变为等低密度。 D.脑血管造影是脑血管病诊断的金标准,但目前面临着CTA和MRA的挑战。 E.急性脑梗塞特别是发病6小时内者,CT不如MRI敏感,可能无发现。 |
A.正常人脑脊液T1WI呈高信号,T2WI呈低信号 |
2 | 2 | 2012 | 1. A husband and wife have 3 children, two boys and one girl. The husband was diagnosed with Huntington disease in his mid-fifties, as was his father. The wife has no symptoms and no family history of Huntington disease. Assuming 100% penetrance(外显率), what is the probability that all 3 of his children will eventually develop Huntington disease? (2.0分) | A.42377 B.42373 C.42371 D.42433 E.Virtually 0 |
A.42377 |
2 | 2 | 2012 | 2. Which of the following trinucleotide repeat(三核苷酸重复) disorders is inherited in an autosomal recessive fashion?(2.0分) | A.fragile X syndrome B.Friedreich ataxia(共济失调) C.Huntington disease D.myotonic dystrophy E.spinocerebellar ataxia(脊髓小脑性共济失调) |
B.Friedreich ataxia(共济失调) |
2 | 2 | 2012 | 3. Which of the following trinucleotide repeat(三核苷酸重复) disorders is likely to be more severe when inherited from the father? (2.0分) | A.fragile X syndrome B.Friedreich ataxia(共济失调) C.Huntington disease D.myotonic dystrophy(强直性肌营养不良) E.none of the above |
C.Huntington disease |
2 | 2 | 2012 | 4. Currently, a novel significant modifier of Alzheimer disease risk was identified by whole-exome and whole-genome sequencing in families with multiple individuals affected with Alzheimer disease. It was (2.0分) | A.APP B.PSEN1 C.PSEN2 D.APOE E.TREM2 |
E.TREM2 |
2 | 2 | 2012 | 5. 引导神经干动作电位,多道生理信号采集处理系统的参数应为(2.0分) | A.采用频率100Hz、时间常数0.2s、滤波频率3Hz、灵敏度5mV B.采用频率100Hz、时间常数0.02s、滤波频率3kHz、灵敏度5mV C.采用频率100KHz、时间常数0.02s、滤波频率3kHz、灵敏度5mV D.采用频率100KHz、时间常数0.02s、滤波频率3Hz、灵敏度5mV |
C.采用频率100KHz、时间常数0.02s、滤波频率3kHz、灵敏度5mV |
2 | 2 | 2012 | 6. 两引导电极间距10mm,蟾蜍坐骨神经干BAP正相振幅大于负相振幅的主因是(2.0分) | A.负引导电极处的神经纤维数量多于正引导电极 B.BAP传导至正引导电极的距离远于负引导电极,致CAP减小 C.引导电极间距离小于AP波长,正、负相AP叠加 D.AP随传导距离的增加而衰减 |
C.引导电极间距离小于AP波长,正、负相AP叠加 |
2 | 2 | 2012 | 7. 用方形电脉冲刺激蟾蜍坐骨神经干,下列叙述正确的是(2.0分) | A.刺激电极正极下神经干去极化,刺激电极负极下神经干超极化 B.刺激电极正极下神经干兴奋性增加 C.刺激电极负极下神经干兴奋性降低 D.刺激电极负极下神经干兴奋性增加 |
D.刺激电极负极下神经干兴奋性增加 |
2 | 2 | 2012 | 8. 下面关于姿势的描述错误的是(2.0分) | A.人体的姿势与身体的健康状态以及个人习惯有着密切关系 B.姿势举止的状态,不受某些疾病的影响 C.观察姿势可提供重要的疾病线索 D.观察姿势对诊断有参考意义 |
B.姿势举止的状态,不受某些疾病的影响 |
2 | 2 | 2012 | 9. 下面关于步态的描述正确的是:(2.0分) | A.步态是一种简单的运动过程 B.步态不涉及脊髓反射和大、小脑的调节 C.步态要求神经和肌肉系统的高度协调 D.步态不涉及大、小脑的调节 |
C.步态要求神经和肌肉系统的高度协调 |
2 | 2 | 2012 | 10. 下面关于醉酒步态的描述正确的是(2.0分) | A.步行时两腿间距正常 B.可以走直线 C.重心不易控制 D.可以稳定站立 |
C.重心不易控制 |
2 | 2 | 2012 | 11. 下面关于感觉性共济失调步态的描述正确的是(2.0分) | A.由浅感觉障碍引起 B.行走时步幅较大 C.两腿间距正常 D.以大脑病变的可能性较大 |
B.行走时步幅较大 |
2 | 2 | 2012 | 12. 下面关于痉挛性偏瘫步态的描述错误的是(2.0分) | A.患侧下肢屈曲困难 B.又称划圈样步态 C.多见于小脑肿瘤, D.是由一侧锥体束损害引起 |
C.多见于小脑肿瘤, |
2 | 2 | 2012 | 13. 关于慌张步态的描述错误的是(2.0分) | A.全身肌张力增高 B.行走时重心后移 C.走路时步伐细小 D.多见于震颤麻痹 |
B.行走时重心后移 |
2 | 2 | 2012 | 14. 关于摇摆步态的描述正确的是(2.0分) | A.骨盆带肌及腰肌肌力增高 B.站立时脊柱前凸以维持身体重心平衡 C.行走臀部前后摇摆 D.多见于小脑肿瘤 |
B.站立时脊柱前凸以维持身体重心平衡 |
2 | 2 | 2012 | 15. 关于舞蹈步态的描述错误的是(2.0分) | A.步行时肢体有大幅度的、不规则的、不自主运动 B.行路不稳,呈跳跃式或舞蹈样 C.下肢突然外甩,上肢扭曲 D.多见于小脑肿瘤 |
D.多见于小脑肿瘤 |
2 | 2 | 2012 | 16. 关于脊髓性间歇跛行的描述错误的是(2.0分) | A.休息后症状不会好转 B.可见于脊髓动脉内膜炎 C.也可见于脊髓发育异常 D.也可见于椎管狭窄等 |
A.休息后症状不会好转 |
2 | 2 | 2012 | 17. 关于臀中肌麻痹步态的描述错误的是(2.0分) | A.一侧臀中肌病变 B.行走时躯干向患侧弯曲 C.多见于小脑肿瘤 D.也可见于多发性肌炎,进行性营养不良症等 |
C.多见于小脑肿瘤 |
2 | 2 | 2012 | 18. 有关血管源性脑水肿,下列说法错误的是(2.0分) | A.病理生理基础是血脑屏障破坏,血浆进入细胞和间隔。 B.主要发生在脑皮质。 C.可见于肿瘤、出血、炎症等。 D.在T1加权像显示不明显。 E.Gd-DTPA增强后无异常对比增强。 |
B.主要发生在脑皮质。 |
2 | 2 | 2012 | 19. 脑脓肿壁短T2低信号最可能是(2.0分) | A.含铁血黄素沉着。 B.胶原结构。 C.血管影。 D.钙化。 E.铁沉积 |
B.胶原结构。 |
2 | 2 | 2012 | 20. MRI可提供多种信息,其中描述错误的是 (2.0分) | A.组织T1值 B.组织T2值 C.质子密度 D.组织密度值 E.组织代谢信息 |
D.组织密度值 |
2 | 2 | 2012 | 21. 表皮样囊肿MR信号特征不包括(2.0分) | A.T1WI低信号 B.T2WI高信号 C.增强扫描明显强化 D.FLAIR高信号 E.DWI高信号 |
C.增强扫描明显强化 |
2 | 2 | 2012 | 22. SWI可以有效应用于如下疾病检查不包括(2.0分) | A.鉴别脑梗塞中出血灶 B.发现静脉畸形 C.Parkinson病 D.Alzheimer 病 E.脑炎 |
E.脑炎 |
2 | 2 | 2012 | 23. 细胞毒性水肿的主要受累部位为 (2.0分) | A.脑灰质 B.脑灰白质 C.脑白质 D.脑室旁 E.脉络丛 |
B.脑灰白质 |
2 | 2 | 2012 | 24. 不属于神经元移行异常的畸形是 (2.0分) | A.灰质异位 B.平滑脑 C.脑裂畸形 D.Dandy-Walker畸形 E.厚脑回畸形 |
D.Dandy-Walker畸形 |
2 | 2 | 2012 | 25. 对蛛网膜下腔出血首选的检查方法应为(2.0分) | A.MRI。 B.PET。 C.EEG。 D.CT。 E.ECT |
D.CT。 |
2 | 2 | 2012 | 26. 衡量组织兴奋性高低的指标是 (2.0分) | A.静息电位水平 B.阈电位 C.阈强度 D.动作电位幅度 E.兴奋扩布速度 |
C.阈强度 |
2 | 2 | 2012 | 27. 临床上普鲁卡因局部麻醉药的应用是由于阻断了局部神经冲动的传导,其神经细胞发生的变化是(2.0分) | A.细胞变性 B.结构完整性破坏 C.功能完整性破坏 D.细胞膜电压门控钾通道破坏 E.细胞膜化学门控钠通道破坏 |
C.功能完整性破坏 |
2 | 2 | 2012 | 28. 在神经‐骨骼肌接头中消除乙酰胆碱的酶是 (2.0分) | A.ATP 酶 B.胆碱酯酶 C.腺苷酸环化酶 D.磷酸二酯酶 E.单胺氧化酶 |
B.胆碱酯酶 |
2 | 2 | 2012 | 29. 细胞膜上不衰减形式传播的电活动是(2.0分) | A.动作电位 B.静息电位 C.终板电位 D.感受器电位 E.突触后电位 |
A.动作电位 |
2 | 2 | 2012 | 30. 实验中,刺激神经纤维,其动作电位传导的特点是 (2.0分) | A.呈衰减性传导 B.呈双向传导 C.连续的多个动作电位可融合 D.电位幅度越大传导越慢 E.刺激越强,传导越快 |
B.呈双向传导 |
2 | 2 | 2012 | 31. 如欲调查某地高血压流行现状,可采用哪一种方法(2.0分) | A.病例对照研究 B.前瞻性调查 C.抽样调查 D.爆发调查 E.相关性研究 |
C.抽样调查 |
2 | 2 | 2012 | 32. 在检验某因素与某疾病的因果联系时,下列哪种观察法提供的证据最强? (2.0分) | A.现患调查 B.生态趋势研究 C.病例报告 D.前瞻性队列研究 E.现场试验 |
E.现场试验 |
2 | 2 | 2012 | 33. 一项子宫内膜癌与雌激素应用之间关系的匹配病例对照研究,共计73对,病例和对照两组均有雌激素暴露史者27对,两组均无暴露史者4对,病例组暴露而对照组未暴露者29对,其余为对照组暴露而病例组未暴露者。据此计算雌激素暴露与子宫内膜癌的OR值为:(2.0分) | A.6.75 B.9.67 C.2.23 D.2.72 E.3.49 |
D.2.72 |
2 | 2 | 2012 | 34. 心理防御机制来自以下理论:(2.0分) | A.行为学习理论 B.精神分析理论 C.认知理论 D.人本主义理论 |
B.精神分析理论 |
2 | 2 | 2012 | 35. 下列不属于精神分析人格理论的内容是(2.0分) | A.自我 B.本我 C.潜意识 D.超我 |
C.潜意识 |
2 | 2 | 2012 | 36. 经典条件反射把环境刺激对个体行为的促进作用称为(2.0分) | A.强化 B.消退 C.有效 D.泛化 |
A.强化 |
2 | 2 | 2012 | 37. 行为发生后积极的刺激增加,导致了该行为增强的操作条件反射称为(2.0分) | A.泛化 B.正强化 C.积极刺激 D.有效刺激 |
B.正强化 |
2 | 2 | 2012 | 38. 饮酒后感到"烦闷解除",逐渐形成了饮酒的嗜好,这属于(2.0分) | A.惩罚 B.消退 C.正强化 D.负强化 |
D.负强化 |
2 | 2 | 2012 | 39. 下列哪项不属于班杜拉示范作用的内容(2.0分) | A.注意 B.潜抑 C.行动 D.泛化 |
D.泛化 |
2 | 2 | 2012 | 40. 关于吉兰-巴雷综合症,下列哪种说法是错误的(2.0分) | A.男女发病率相近 B.各个年龄段都可发病 C.多数有呼吸道或消化道感染的前驱症状 D.由病原体直接攻击周围神经所致 |
D.由病原体直接攻击周围神经所致 |
2 | 2 | 2012 | 41. 脊髓病变的临床表现不包括(2.0分) | A.肢体瘫痪 B.面部感觉异常 C.膀胱直肠功能障碍 D.感觉平面 |
B.面部感觉异常 |
2 | 2 | 2012 | 42. 脊髓半侧损害的临床表现有(2.0分) | A.同侧肢体瘫痪 B.对侧深感觉障碍 C.同侧痛温觉障碍 D.对侧血管舒缩功能障碍 |
A.同侧肢体瘫痪 |
2 | 2 | 2012 | 43. 关于脊髓休克,正确的是 (2.0分) | A.肌张力低下 B.腱反射活跃 C.病理征阳性 D.克氏征阳性 |
A.肌张力低下 |
2 | 2 | 2012 | 44. 图像清晰而且无骨伪影干扰,是后颅窝区神经系统疾病最理想的检查方法(2.0分) | A.X线 B.MRI C.超声 D.CT |
B.MRI |
2 | 2 | 2012 | 45. 脑转移瘤原发灶多来自(2.0分) | A.乳癌 B.肾癌 C.肺癌 D.前列腺癌 |
C.肺癌 |
2 | 2 | 2012 | 46. 颅内最常见的原发肿瘤是(2.0分) | A.脑膜瘤 B.胶质瘤 C.转移瘤 D.听神经瘤 |
B.胶质瘤 |
2 | 2 | 2012 | 47. 目前临床能够检测活体组织内水分子扩散运动的无创方法的MRI检查序列是 (2.0分) | A.PWI B.DWI C.MR波谱 D.MR动态增强 E.MR T2WI |
B.DWI |
2 | 2 | 2012 | 48. 弥漫性轴索损伤指头部受到外伤作用后发生的,主要弥漫分布于脑白质、以轴索损伤为主要改变的一种原发性脑实质的损伤。其特点为 (2.0分) | A.广泛性白质变性,小灶性出血, B.神经轴索回缩球,小胶质细胞簇出现, C.常与其他颅脑损伤合并, D.死亡率高。 E.以上均正确 |
E.以上均正确 |
2 | 2 | 2012 | 49. 腔隙性脑梗死下列陈述不正确的是(2.0分) | A.腔隙性脑梗死是指大脑半球或脑干深部的小穿通动脉,血管壁发生病变,导致管腔闭塞,形成小的梗死灶。 B.腔隙性脑梗死常见的发病部位包括壳核、尾状核、内囊、丘脑及脑桥等。 C.CT诊断腔隙性脑梗死优于MRI检查。 D.小脑较少发生腔隙性脑梗死。 |
C.CT诊断腔隙性脑梗死优于MRI检查。 |
2 | 2 | 2012 | 50. 关于硬膜外血肿,错误的陈述(2.0分) | A.硬膜外血肿是位于颅骨内板与硬脑膜之间的血肿,好发于幕上半球凸面; B.硬膜外血肿大部分属于急性血肿,慢性较少。 C.硬膜外血肿的形成与颅骨损伤有密切关系,绝大多数的硬脑膜外血肿桥静脉撕断或是脑 D.挫裂伤皮质血管破裂引起出血有关。 E.典型的硬膜外血肿的患者存在中间清醒期. 即昏迷-清醒-再昏迷 |
C.硬膜外血肿的形成与颅骨损伤有密切关系,绝大多数的硬脑膜外血肿桥静脉撕断或是脑 |
2 | 2 | 2013 | 1. 1. A husband and wife have 3 children, two boys and one girl. The husband was diagnosed with Huntington disease in his mid-fifties, as was his father. The wife has no symptoms and no family history of Huntington disease. Assuming 100% penetrance(外显率), what is the probability that all 3 of his children will eventually develop Huntington disease? (2.0分) | A.42743 B.42739 C.42737 D.42798 E.Virtually 0 |
A.42743 |
2 | 2 | 2013 | 2. Which of the following trinucleotide repeat(三核苷酸重复) disorders is inherited in an autosomal recessive fashion? (2.0分) | A.fragile X syndrome B.Friedreich ataxia(共济失调) C.Huntington disease D.myotonic dystrophy E.spinocerebellar ataxia(脊髓小脑性共济失调) |
B.Friedreich ataxia(共济失调) |
2 | 2 | 2013 | 3. Which of the following trinucleotide repeat(三核苷酸重复) disorders is likely to be more severe when inherited from the father? (2.0分) | A.fragile X syndrome B.Friedreich ataxia(共济失调) C.Huntington disease D.myotonic dystrophy(强直性肌营养不良) E.none of the above |
C.Huntington disease |
2 | 2 | 2013 | 4. Currently, a novel significant modifier of Alzheimer disease risk was identified by whole-exome and whole-genome sequencing in families with multiple individuals affected with Alzheimer disease. It was (2.0分) | A.APP B.PSEN1 C.PSEN2 D.APOE E.TREM2 |
E.TREM2 |
2 | 2 | 2013 | 5. 病例对照研究的病例组最好选择(2.0分) | A.死亡病例 B.旧病例 C.可疑病例 D.新发病例 E.有待确诊的病例 |
D.新发病例 |
2 | 2 | 2013 | 6. 一项子宫内膜癌与雌激素应用之间关系的匹配病例对照研究,共计73对,病例和对照两组均有雌激素暴露史者27对,两组均无暴露史者4对,病例组暴露而对照组未暴露者29对,其余为对照组暴露而病例组未暴露者。据此计算OR值为(2.0分) | A.6.75 B.9.67 C.2.23 D.2.72 E.3.49 |
C.2.23 |
2 | 2 | 2013 | 7. 下列哪项是病例对照研究的优点? (2.0分) | A.估计对危险因素的暴露情况时,很少或没有偏倚 B.在选择出暴露因素后,可研究多种疾病的结局 C.可减少研究对象对回忆的依赖性 D.有可能确立该病的真实发病率 E.可用于研究少见病的病因 |
E.可用于研究少见病的病因 |
2 | 2 | 2013 | 8. 下面关于姿势的描述错误的是(2.0分) | A.姿势是体检的项目之一 B.姿势是身体呈现的样子 C.人体的姿势与身体的健康状态有着密切关系 D.人体的姿势与个人习惯无关 |
D.人体的姿势与个人习惯无关 |
2 | 2 | 2013 | 9. 下面关于步态的描述错误的是(2.0分) | A.步态是指患者步行的姿势 B.步态是一种复杂的运动过程 C.步态不涉及脊髓反射 D.步态涉及大、小脑的调节 |
C.步态不涉及脊髓反射 |
2 | 2 | 2013 | 10. 下面关于步态的描述错误的是(2.0分) | A.检查步态时不需要分析骨骼的畸形以及骨、关节的异常 B.不同的疾病可有不同的特殊步态 C.观察步态可提供重要的疾病线索 D.观察步态对诊断有参考意义 |
A.检查步态时不需要分析骨骼的畸形以及骨、关节的异常 |
2 | 2 | 2013 | 11. 下面关于感觉性共济失调步态的描述错误的是(2.0分) | A.由浅感觉障碍引起 B.行走时步幅较大 C.两腿间距较宽 D.提足较高 |
A.由浅感觉障碍引起 |
2 | 2 | 2013 | 12. 下面关于痉挛性偏瘫步态的描述正确的是(2.0分) | A.患侧下肢显得较短 B.患侧下肢屈曲困难 C.由双侧锥体束损害引起, D.多见于小脑肿瘤 |
B.患侧下肢屈曲困难 |
2 | 2 | 2013 | 13. 下面关于痉挛性截瘫步态的描述错误的是(2.0分) | A.又称剪刀步态 B.下肢内收肌群张力降低 C.可见于横贯性脊髓损害 D.也可见于脑性瘫痪等 |
B.下肢内收肌群张力降低 |
2 | 2 | 2013 | 14. 关于慌张步态的描述正确的是(2.0分) | A.起步时缓慢 B.全身肌张力降低 C.走路时步伐比正常大 D.躯干前倾,重心后移 |
A.起步时缓慢 |
2 | 2 | 2013 | 15. 关于摇摆步态的描述错误的是(2.0分) | A.骨盆带肌及腰肌无力 B.行走臀部前后摇摆 C.站立时脊柱前凸以维持身体重心平衡 D.见于进行性肌营养不良症 |
B.行走臀部前后摇摆 |
2 | 2 | 2013 | 16. 关于星迹步态的描述错误的是(2.0分) | A.患者闭眼前进时向患侧偏斜 B.患者闭眼后退时向反方向偏斜 C.步行时肢体有大幅度的、不规则的、不自主运动 D.见于前庭迷路病变 |
C.步行时肢体有大幅度的、不规则的、不自主运动 |
2 | 2 | 2013 | 17. 关于臀中肌麻痹步态的描述正确的是(2.0分) | A.一侧臀中肌病变 B.行走时躯干向健侧弯曲 C.多见于小脑肿瘤 D.也可见于大脑病变等 |
A.一侧臀中肌病变 |
2 | 2 | 2013 | 18. 以下关于神经纤维动作电位引导的叙述,正确的是(2.0分) | A.细胞内引导获得的AP是膜内对地的电位差 B.细胞外引导获得的AP是膜内与膜外的电位差 C.细胞外引导获得的AP是两引导电极间的膜外的电位差 D.细胞内引导获得的AP是两引导电极间的膜内的电位差 |
C.细胞外引导获得的AP是两引导电极间的膜外的电位差 |
2 | 2 | 2013 | 19. 两引导电极间距10mm,蟾蜍坐骨神经干BAP正相振幅大于负相振幅的主因是(2.0分) | A.负引导电极处的神经纤维数量多于正引导电极 B.BAP传导至正引导电极的距离远于负引导电极,致CAP减小 C.引导电极间距离小于AP波长,正、负相AP叠加 D.AP随传导距离的增加而衰减 |
C.引导电极间距离小于AP波长,正、负相AP叠加 |
2 | 2 | 2013 | 20. 关于神经干BAP正负相振幅和时程描述正确的是(2.0分) | A.正相振幅大于负相振幅、正相时程大于负相时程 B.正相振幅大于负相振幅、正相时程小于负相时程 C.正相振幅小于负相振幅、正相时程大于负相时程 D.正相振幅小于负相振幅、正相时程小于负相时程 |
B.正相振幅大于负相振幅、正相时程小于负相时程 |
2 | 2 | 2013 | 21. 吉兰-巴雷综合征的常见表现不包含(2.0分) | A.四肢无力 B.腓肠肌酸痛 C.一侧面神经瘫痪 D.呼吸肌瘫痪 |
C.一侧面神经瘫痪 |
2 | 2 | 2013 | 22. 关于吉兰-巴雷综合征的辅助检查,哪项正确(2.0分) | A.蛋白-细胞分离在发病后第二周最明显 B.脑脊液的葡萄糖常常降低 C.远端潜伏期正常 D.F波和H反射的异常 |
D.F波和H反射的异常 |
2 | 2 | 2013 | 23. 乳头水平相当于哪个脊髓节段?(2.0分) | A.T4 B.T5 C.T6 D.T7 |
A.T4 |
2 | 2 | 2013 | 24. 髓内病变的表现不包括 (2.0分) | A.痛温觉自上向下发展 B.根痛多见 C.括约肌功能障碍出现早 D.脑脊液蛋白增高不明显 |
B.根痛多见 |
2 | 2 | 2013 | 25. 癌症病人的"三阶止痛阶梯治疗",中度疼痛的病人适合选用 (2.0分) | A.阿司匹林 B.吗啡 C.曲马多 D.芬太尼 E.哌替啶 |
C.曲马多 |
2 | 2 | 2013 | 26. 与脑内阿片受体作用无关镇痛药是(2.0分) | A.罗通定 B.喷他佐辛 C.曲马朵 D.美沙酮 E.纳洛酮 |
A.罗通定 |
2 | 2 | 2013 | 27. 下列哪种情况不宜使用哌替啶镇痛 (2.0分) | A.创伤性疼痛 B.手术后疼痛 C.慢性疼痛 D.内脏绞痛 E.晚期癌症痛 |
C.慢性疼痛 |
2 | 2 | 2013 | 28. 人本心理学主张(2.0分) | A.为研究人的行为及影响因素 B.研究影响及决定人行为的潜意识 C.研究区别于动物的动机、欲望、生命意义、情感等复杂体验 D.研究人的感知、记忆、思维等信息加工过程及影响因素 |
C.研究区别于动物的动机、欲望、生命意义、情感等复杂体验 |
2 | 2 | 2013 | 29. 弗洛伊德有关人格结构模式较为独特,其中主导我们现实抉择的是(2.0分) | A.本我 B.自我 C.超我 D.忘我 |
B.自我 |
2 | 2 | 2013 | 30. 弗洛伊德认为婴幼儿期的人格停留在(2.0分) | A.口腔期 B.肛门期 C.性器期 D.潜伏期 |
A.口腔期 |
2 | 2 | 2013 | 31. 自我防御机制的形成是为了(2.0分) | A.减轻或消除人格内部的冲突 B.降低或避免焦虑 C.保持人格的完整和统一 D.以上都是 |
D.以上都是 |
2 | 2 | 2013 | 32. 依据马斯洛需要层次理论,中国知识分子的需要多停留在(2.0分) | A.生理需要 B.社交需要 C.安全需要 D.尊重的需要 |
D.尊重的需要 |
2 | 2 | 2013 | 33. 个体接受一个刺激之后,它行为的结果是消极刺激增加,从而使行为反应逐渐减弱,这种行为反射模式是(2.0分) | A.正强化 B.负强化 C.消退 D.惩罚 |
D.惩罚 |
2 | 2 | 2013 | 34. 人体许多机能失调和机体痛苦都根源于思维问题。所以,认知在成为人们成功的源泉的同时,也是人们(2.0分) | A.失败的源泉 B.苦恼的源泉 C.躯体诸多不适的源泉 D.以上均是 |
D.以上均是 |
2 | 2 | 2013 | 35. 衡量组织兴奋性高低的指标是(2.0分) | A.静息电位水平 B.阈电位 C.阈强度 D.动作电位幅度 E.兴奋扩布速度 |
C.阈强度 |
2 | 2 | 2013 | 36. 动作电位沿运动神经纤维传导抵达神经‐肌接头部位时,轴突末梢释放Ach,使终板膜产生终板电位,然后在什么部位引发动作电位(2.0分) | A.肌细胞膜 B.接头后膜 C.终板膜 D.横管膜 E.三联管膜 |
A.肌细胞膜 |
2 | 2 | 2013 | 37. 神经‐骨骼肌接头处的兴奋传递物质是(2.0分) | A.5‐羟色胺 B.乙酰胆碱 C.去甲肾上腺素 D.肾上腺素 E.多巴胺 |
B.乙酰胆碱 |
2 | 2 | 2013 | 38. 人工减小细胞浸浴液中的Na+浓度,所记录的动作电位出现(2.0分) | A.幅度变小 B.幅度变大 C.时程缩短 D.时程延长 E.复极相延长 |
A.幅度变小 |
2 | 2 | 2013 | 39. 对于脑外伤后怀疑蛛网膜下腔出血的病人,首选的检查方法应为(2.0分) | A.MRI弥散加权成像 B.PET 18F-FDG成像 C.CT或MR脑灌注成像 D.CT平扫 E.MR波谱分析 |
D.CT平扫 |
2 | 2 | 2013 | 40. T2-FLAIR序列可以改善脑室旁病变的显示,其主要原因是(2.0分) | A.抑制了水分子弥散 B.很好的T1对比 C.很好的T2对比 D.脑脊液的信号得到有效抑制 E.扫描速度快 |
D.脑脊液的信号得到有效抑制 |
2 | 2 | 2013 | 41. 关于磁共振血管造影,下列说法正确的是(2.0分) | A.可以使用TOF成像 B.只能二维显示,不能进行三维重建 C.血管和周围组织的对比比较差 D.血流速度和方向不影响成像效果 E.必须使用对比剂 |
A.可以使用TOF成像 |
2 | 2 | 2013 | 42. 以下关于水分子弥散的说法不正确的是(2.0分) | A.DWI是目前能够在活体上进行水分子弥散测量和成像的唯一方法 B.通过对水分子弥散方向的测定,可以了解神经纤维的走行方向 C.血管源性水肿导致局部水分子弥散受限 D.细胞内水肿导致局部水分子弥散受限 E.组织细胞密集导致水分子弥散受限 |
C.血管源性水肿导致局部水分子弥散受限 |
2 | 2 | 2013 | 43. 创伤性颅脑急症,常规和首选检查方法(2.0分) | A.X线 B.MRI C.超声 D.CT |
D.CT |
2 | 2 | 2013 | 44. 尾状核头部占位首先引起下列什么部位受压改变(2.0分) | A.侧脑室枕角 B.侧脑室三角部 C.颞叶 D.侧脑室额角 |
D.侧脑室额角 |
2 | 2 | 2013 | 45. 临床上最多见的脑血管畸形是(2.0分) | A.动脉瘤 B.海绵状血管瘤 C.动静脉畸形 D.毛细血管扩张症 |
C.动静脉畸形 |
2 | 2 | 2013 | 46. 下列关于脑膜瘤的描述,错位的是(2.0分) | A.多见于中年女性 B.多位于脑外,少数位于脑室内 C.MRI增强检查可出现脑膜尾征 D.增强检查一般不强化 |
D.增强检查一般不强化 |
2 | 2 | 2013 | 47. 弥漫性轴索损伤常见于_ __损伤(2.0分) | A.加速性颅脑损伤 B.减速性颅脑损伤 C.挤压性颅脑损伤 D.挥鞭样损伤、剪切伤 E.枪弹开放性损伤 |
D.挥鞭样损伤、剪切伤 |
2 | 2 | 2013 | 48. 弥漫性轴索损伤常见发生的部位是(2.0分) | A.白质和灰质交界处; B.两大脑半球之间的胼胝体, C.脑干头端以及小脑; D.内囊和基底节; E.以上均正确。 |
E.以上均正确。 |
2 | 2 | 2013 | 49. 下列陈述不正确的是(2.0分) | A.正常人脑脊液T1WI呈高信号,T2WI呈低信号 B.高血压性脑出血常发生于基底节区、丘脑、小脑等。 C.脑血肿急性期CT上呈高密度,以后逐渐吸收变为等低密度。 D.脑血管造影是脑血管病诊断的金标准,但目前面临着CTA和MRA的挑战。 E.急性脑梗塞特别是发病6小时内者,CT不如MRI敏感,可能无发现。 |
A.正常人脑脊液T1WI呈高信号,T2WI呈低信号 |
2 | 2 | 2013 | 50. __是医学影像学近年来发展的新的检查手段,作为一种无创伤性研究活体器官组织代谢、生化变化及化合物定量分析的方法(2.0分) | A.SWI(Susceptibility weighted imaging) B.PWI(perfusion imaging) C.MRS (MR Spectroscopy) D.T1WI E.T2WI |
C.MRS (MR Spectroscopy) |
2 | 2 | 2014 | 1. 以下关于蟾蜍坐骨神经干复合动作电位引导的叙述,正确的是(2.0分) | A.细胞内引导获得的AP是膜内对地的电位差 B.细胞外引导获得的AP是膜内与膜外的电位差 C.细胞外引导获得的AP是两引导电极间的膜外的电位差 D.细胞内引导获得的AP是两引导电极间的膜内的电位差 |
C.细胞外引导获得的AP是两引导电极间的膜外的电位差 |
2 | 2 | 2014 | 2. 两引导电极间距10mm,蟾蜍坐骨神经干复合动作单位正相振幅大于负相振幅的主要原因是 (2.0分) | A.负引导电极处的神经纤维数量多于正引导电极 B.BAP传导至正引导电极的距离远于负引导电极,致CAP减小 C.引导电极间距离小于AP波长,正、负相AP叠加 D.AP随传导距离的增加而衰减 |
C.引导电极间距离小于AP波长,正、负相AP叠加 |
2 | 2 | 2014 | 3. 关于神经干BAP正负相振幅和时程描述正确的是(2.0分) | A.正相振幅大于负相振幅、正相时程大于负相时程 B.正相振幅大于负相振幅、正相时程小于负相时程 C.正相振幅小于负相振幅、正相时程大于负相时程 D.正相振幅小于负相振幅、正相时程小于负相时程 |
B.正相振幅大于负相振幅、正相时程小于负相时程 |
2 | 2 | 2014 | 4. 创伤性颅脑急症,常规和首选检查方法(2.0分) | A.X线 B.MRI C.超声 D.CT |
D.CT |
2 | 2 | 2014 | 5. 尾状核头部占位首先引起下列什么部位受压改变: (2.0分) | A.侧脑室枕角 B.侧脑室三角部 C.颞叶 D.侧脑室额角 |
D.侧脑室额角 |
2 | 2 | 2014 | 6. 颅内最常见的原发肿瘤是:(2.0分) | A.脑膜瘤 B.胶质瘤 C.转移瘤 D.听神经瘤 |
B.胶质瘤 |
2 | 2 | 2014 | 7. 目前临床能够检测活体组织内水分子扩散运动的无创方法的MRI检查序列是(2.0分) | A.PWI B.DWI C.MR波谱 D.MR动态增强 E.MR T2WI |
B.DWI |
2 | 2 | 2014 | 8. 弥漫性轴索损伤常见发生的部位是 (2.0分) | A.白质和灰质交界处; B.两大脑半球之间的胼胝体, C.脑干头端以及小脑; D.内囊和基底节; E.以上均正确。 |
E.以上均正确。 |
2 | 2 | 2014 | 9. 对颅内微出血非常敏感的序列是(2.0分) | A.T1WI B.T2WI C.FLAIR(fluid attenuated inversion recovery) D.SWI(Susceptibility weighted imaging) E.PDWI(proton density weighted image) |
D.SWI(Susceptibility weighted imaging) |
2 | 2 | 2014 | 10. 以下疾病,DWI扫描不表现为弥散受限改变的是(2.0分) | A.脑内淋巴瘤 B.急性脑梗死 C.胶质母细胞瘤 D.蛛网膜囊肿 |
D.蛛网膜囊肿 |
2 | 2 | 2014 | 11. 临床疑有椎管内占位性病变时,首选影像学检查方法是 (2.0分) | A.CT检查 B.X线平片 C.椎管造影 D.MRI检查 |
D.MRI检查 |
2 | 2 | 2014 | 12. 颅脑外伤病人CT检查发现右额叶有斑片状高密度影及周围不规则低密度区,首先考虑(2.0分) | A.脑出血 B.脑挫裂伤 C.出血性脑梗塞 D.脑炎 |
B.脑挫裂伤 |
2 | 2 | 2014 | 13. 急性硬膜外血肿CT表现为(2.0分) | A.颅板下三角形高密度影 B.颅板下类圆形高密度影, C.颅板下新月形高密度影。 D.颅板下梭形高密度影 |
D.颅板下梭形高密度影 |
2 | 2 | 2014 | 14. 下面关于姿势的描述错误的是 (2.0分) | A.人体的姿势与身体的健康状态以及个人习惯有着密切关系 B.姿势举止的状态,不受某些疾病的影响 C.观察姿势可提供重要的疾病线索 D.观察姿势对诊断有参考意义 |
B.姿势举止的状态,不受某些疾病的影响 |
2 | 2 | 2014 | 15. 下面关于步态的描述正确的是(2.0分) | A.步态是一种简单的运动过程 B.步态不涉及脊髓反射和大、小脑的调节 C.步态要求神经和肌肉系统的高度协调 D.步态不涉及大、小脑的调节 |
C.步态要求神经和肌肉系统的高度协调 |
2 | 2 | 2014 | 16. 下面关于醉酒步态的描述正确的是: (2.0分) | A.步行时两腿间距正常 B.可以走直线 C.重心不易控制 D.可以稳定站立 |
C.重心不易控制 |
2 | 2 | 2014 | 17. 下面关于感觉性共济失调步态的描述正确的是(2.0分) | A.由浅感觉障碍引起 B.行走时步幅较大 C.两腿间距正常 D.以大脑病变的可能性较大 |
B.行走时步幅较大 |
2 | 2 | 2014 | 18. 下面关于痉挛性偏瘫步态的描述错误的是(2.0分) | A.患侧下肢屈曲困难 B.又称划圈样步态 C.多见于小脑肿瘤, D.是由一侧锥体束损害引起 |
C.多见于小脑肿瘤, |
2 | 2 | 2014 | 19. 关于慌张步态的描述错误的是(2.0分) | A.全身肌张力增高 B.行走时重心后移 C.走路时步伐细小 D.多见于震颤麻痹 |
B.行走时重心后移 |
2 | 2 | 2014 | 20. 关于摇摆步态的描述正确的是(2.0分) | A.骨盆带肌及腰肌肌力增高 B.站立时脊柱前凸以维持身体重心平衡 C.行走臀部前后摇摆 D.多见于小脑肿瘤 |
B.站立时脊柱前凸以维持身体重心平衡 |
2 | 2 | 2014 | 21. 关于舞蹈步态的描述错误的是(2.0分) | A.步行时肢体有大幅度的、不规则的、不自主运动 B.行路不稳,呈跳跃式或舞蹈样 C.下肢突然外甩,上肢扭曲 D.多见于小脑肿瘤 |
D.多见于小脑肿瘤 |
2 | 2 | 2014 | 22. 关于臀中肌麻痹步态的描述错误的是(2.0分) | A.一侧臀中肌病变 B.行走时躯干向患侧弯曲 C.多见于小脑肿瘤 D.也可见于多发性肌炎,进行性营养不良症等 |
C.多见于小脑肿瘤 |
2 | 2 | 2014 | 23. 关于脊髓性间歇跛行的描述错误的是(2.0分) | A.休息后症状不会好转 B.可见于脊髓动脉内膜炎 C.也可见于脊髓发育异常 D.也可见于椎管狭窄等 |
A.休息后症状不会好转 |
2 | 2 | 2014 | 24. 正常发育儿童,脑MRI上灰白质信号与成人相近的时间为(2.0分) | A.0.5岁 B.1岁 C.2岁 D.3岁 E.4岁 |
C.2岁 |
2 | 2 | 2014 | 25. MRI可提供多种信息,其中描述错误的是 (2.0分) | A.组织T1值 B.组织T2值 C.质子密度 D.组织密度值 E.组织代谢信息 |
D.组织密度值 |
2 | 2 | 2014 | 26. 脑内胶质瘤术后放疗后复查增强扫描明显强化,有效鉴别复发和放射性坏死方法为(2.0分) | A.DWI B.PWI C.MRS D.SWI E.MRI |
B.PWI |
2 | 2 | 2014 | 27. 关于脊髓多发性硬化的描述,不正确的是(2.0分) | A.女性患者多于男性 B.病变部位脊髓明显增粗 C.病变位于白质 D.颈、胸髓发生多见 E.T1WI低信号,T2WI高信号 |
B.病变部位脊髓明显增粗 |
2 | 2 | 2014 | 28. 对蛛网膜下腔出血首选的检查方法应为:(2.0分) | A.MRI。 B.PET。 C.EEG。 D.CT。 E.ECT |
D.CT。 |
2 | 2 | 2014 | 29. Gd-DTPA增强可用于:(2.0分) | A.鉴别水肿与病变组织 B.碘过敏不能行CT增强者 C.在一定过程上区分肿瘤性病变与非肿瘤性病变 D.发现软脊膜病变 E.以上均对 |
E.以上均对 |
2 | 2 | 2014 | 30. 腰椎神经受压常见原因: (2.0分) | A.椎间盘膨出 B.黄韧带增厚 C.椎小关节骨质增生 D.腰椎滑移 E.以上均是 |
E.以上均是 |
2 | 2 | 2014 | 31. 驯兽员在驯兽过程中经常对动物的正确行为用食物进行奖励,这是运用了以下何种机制: (2.0分) | A.示范 B.消退 C.正强化 D.负强化 |
C.正强化 |
2 | 2 | 2014 | 32. 罗杰斯认为儿童自我不协调的原因是: (2.0分) | A.条件性积极关注所致 B.理想的我与现实的我距离接近 C.无条件性积极关注所致 D.潜能发展的阻碍 |
A.条件性积极关注所致 |
2 | 2 | 2014 | 33. 上网后感到"烦闷解除",逐渐形成了网络成瘾,这属于 (2.0分) | A.惩罚 B.消退 C.正强化 D.负强化 |
D.负强化 |
2 | 2 | 2014 | 34. 精神分析理论认为,遵循至善原则的人格部分是 (2.0分) | A.超我 B.本我 C.它我 D.自我 |
A.超我 |
2 | 2 | 2014 | 35. 所谓"退化"是指(2.0分) | A.个体采用远离应激源的方式应付环境 B.个体采用攻击的方式应付环境 C.个体使用幼儿时期的方式应付环境 D.是一种无能为力、无所适从的行为反应 |
C.个体使用幼儿时期的方式应付环境 |
2 | 2 | 2014 | 36. 关于Miller-Fisher综合征,下列哪种说法是错误的(2.0分) | A.属于吉兰-巴雷综合征的亚型 B.可有小脑性共济失调 C.可有眼外肌麻痹 D.常导致呼吸肌瘫痪 |
D.常导致呼吸肌瘫痪 |
2 | 2 | 2014 | 37. 脊髓病变的临床表现不包括(2.0分) | A.肢体瘫痪 B.面瘫 C.膀胱直肠功能障碍 D.感觉平面 |
B.面瘫 |
2 | 2 | 2014 | 38. Brown-Sequard综合征的临床表现有(2.0分) | A.同侧肢体瘫痪 B.对侧深感觉障碍 C.同侧痛温觉障碍 D.对侧血管舒缩功能障碍 |
A.同侧肢体瘫痪 |
2 | 2 | 2014 | 39. 关于急性脊髓炎的治疗,错误的是(2.0分) | A.丙种球蛋白 B.急性期大剂量甲泼尼龙短期冲击治疗 C.人血白蛋白 D.B族维生素 |
C.人血白蛋白 |
2 | 2 | 2014 | 40. 哌替啶的不良反应不包括(2.0分) | A.抑制呼吸 B.便秘 C.心动过速 D.免疫抑制 E.惊厥 |
B.便秘 |
2 | 2 | 2014 | 41. 哌替啶比吗啡应用多的原因是(2.0分) | A.无便秘作用 B.呼吸抑制作用轻 C.作用较慢,维持时间短 D.成瘾性较吗啡轻 E.对支气管平滑肌无影响 |
D.成瘾性较吗啡轻 |
2 | 2 | 2014 | 42. 与脑内阿片受体作用无关镇痛药是(2.0分) | A.罗通定 B.喷他佐辛 C.曲马朵 D.美沙酮 E.纳洛酮 |
A.罗通定 |
2 | 2 | 2014 | 43. A husband and wife have 3 children, two boys and one girl. The husband was diagnosed with Huntington disease in his mid-fifties, as was his father. The wife has no symptoms and no family history of Huntington disease. Assuming 100% penetrance(假设外显率为100%), what is the probability(概率) that all 3 of his children will eventually(最终) develop Huntington disease? (2.0分) | A.1/8 B.1/4 C.1/2 D.3/4 E.Virtually 0 |
A.1/8 |
2 | 2 | 2014 | 44. Which of the following trinucleotide repeat(三核苷酸重复) disorders is inherited in an autosomal recessive fashion(染色体隐性遗传方式)? (2.0分) | A.Fragile X syndrome(脆性X综合征) B.Friedreich ataxia(共济失调) C.Huntington disease D.Myotonic dystrophy(强直性肌营养不良) E.spinocerebellar ataxia(脊髓小脑性共济失调) |
B.Friedreich ataxia(共济失调) |
2 | 2 | 2014 | 45. Which of the following trinucleotide repeat(三核苷酸重复) disorders is likely to be more severe when inherited from the father? (2.0分) | A.Fragile X syndrome(脆性X综合征) B.Friedreich ataxia(共济失调) C.Huntington disease D.Myotonic dystrophy(强直性肌营养不良) E.None of the above |
C.Huntington disease |
2 | 2 | 2014 | 46. Currently, a novel significant modifier(修饰基因) of Alzheimer disease risk was identified by whole-exome and whole-genome sequencing(全外显子组测序和全基因组测序) in families with multiple individuals affected with Alzheimer disease. It was(2.0分) | A.APP B.PSEN1 C.PSEN2 D.APOE E.TREM2 |
E.TREM2 |
2 | 2 | 2014 | 47. 直接导致神经末梢释放递质的因素是(2.0分) | A.末梢处的Na+内流 B.末梢处的K+外流 C.末梢处的Cl‐内流 D.末梢处的Na+‐Ca2+交换 E.末梢处的Ca2+内流 |
E.末梢处的Ca2+内流 |
2 | 2 | 2014 | 48. 神经‐肌肉接头处的兴奋传递物质是(2.0分) | A.5‐羟色胺 B.乙酰胆碱 C.去甲肾上腺素 D.肾上腺素 E.多巴胺 |
B.乙酰胆碱 |
2 | 2 | 2014 | 49. 在神经‐骨骼肌接头中消除乙酰胆碱的酶是(2.0分) | A.ATP 酶 B.胆碱酯酶 C.腺苷酸环化酶 D.磷酸二酯酶 E.单胺氧化酶 |
C.腺苷酸环化酶 |
2 | 2 | 2014 | 50. 关于重症肌无力,以下说法错误的是 (2.0分) | A.重症肌无力是一种主要累及神经肌肉接头突触后膜上乙酰胆碱受体的自身免疫性疾病 B.患者突触后膜的AchR被大量破坏,不能产生足够的终板电位,导致突触后膜传递功能障碍而发生肌无力 C.临床主要表现为部分或全身骨骼肌无力和易疲劳,休息后症状减轻,经胆碱酯酶抑制剂处理后症状加重 D.低频(3Hz)重复刺激(6-10次)评价突触后膜病变时,一般正常人EMG波幅递减不超过15%,重症肌无力病人第4、5 个波的波幅明显下降 E.重复电刺激测定时,电极摆放如运动神经检查,连续刺激神经干,观察EMG波幅增减 |
C.临床主要表现为部分或全身骨骼肌无力和易疲劳,休息后症状减轻,经胆碱酯酶抑制剂处理后症状加重 |
2 | 2 | 2015 | 1. A husband and wife have 3 children, two boys and one girl. The husband was diagnosed with Huntington disease in his mid-fifties, as was his father. The wife has no symptoms and no family history of Huntington disease. Assuming 100% penetrance(外显率), what is the probability that all 3 of his children will eventually develop Huntington disease?(2.0分) | A.43473 B.43469 C.43467 D.43528 E.Virtually 0 |
A.43473 |
2 | 2 | 2015 | 2. Which of the following trinucleotide repeat(三核苷酸重复) disorders is inherited in an autosomal recessive fashion? (2.0分) | A.fragile X syndrome B.Friedreich ataxia(共济失调) C.Huntington disease D.myotonic dystrophy E.spinocerebellar ataxia(脊髓小脑性共济失调) |
B.Friedreich ataxia(共济失调) |
2 | 2 | 2015 | 3. Which of the following trinucleotide repeat(三核苷酸重复) disorders is likely to be more severe when inherited from the father? (2.0分) | A.fragile X syndrome B.Friedreich ataxia(共济失调) C.Huntington disease D.myotonic dystrophy(强直性肌营养不良) E.none of the above |
C.Huntington disease |
2 | 2 | 2015 | 4. Currently, a novel significant modifier of Alzheimer disease risk was identified by whole-exome and whole-genome sequencing in families with multiple individuals affected with Alzheimer disease. It was (2.0分) | A.APP B.PSEN1 C.PSEN2 D.APOE E.TREM2 |
E.TREM2 |
2 | 2 | 2015 | 5. 病例对照研究的病例组最好选择:(2.0分) | A.死亡病例 B.旧病例 C.可疑病例 D.新发病例 E.有待确诊的病例 |
D.新发病例 |
2 | 2 | 2015 | 6. 一项子宫内膜癌与雌激素应用之间关系的匹配病例对照研究,共计73对,病例和对照两组均有雌激素暴露史者27对,两组均无暴露史者4对,病例组暴露而对照组未暴露者29对,其余为对照组暴露而病例组未暴露者。据此计算OR值为: (2.0分) | A.6.75 B.9.67 C.2.23 D.2.72 E.3.49 |
C.2.23 |
2 | 2 | 2015 | 7. 下列哪项是病例对照研究的优点? (2.0分) | A.估计对危险因素的暴露情况时,很少或没有偏倚 B.在选择出暴露因素后,可研究多种疾病的结局 C.可减少研究对象对回忆的依赖性 D.有可能确立该病的真实发病率 E.可用于研究少见病的病因 |
E.可用于研究少见病的病因 |
2 | 2 | 2015 | 8. 下面关于姿势的描述错误的是: (2.0分) | A.姿势是体检的项目之一 B.姿势是身体呈现的样子 C.人体的姿势与身体的健康状态有着密切关系 D.人体的姿势与个人习惯无关 |
D.人体的姿势与个人习惯无关 |
2 | 2 | 2015 | 9. 下面关于步态的描述错误的是: (2.0分) | A.步态是指患者步行的姿势 B.步态是一种复杂的运动过程 C.步态不涉及脊髓反射 D.步态涉及大、小脑的调节 |
C.步态不涉及脊髓反射 |
2 | 2 | 2015 | 10. 下面关于步态的描述错误的是: (2.0分) | A.检查步态时不需要分析骨骼的畸形以及骨、关节的异常 B.不同的疾病可有不同的特殊步态 C.观察步态可提供重要的疾病线索 D.观察步态对诊断有参考意义 |
A.检查步态时不需要分析骨骼的畸形以及骨、关节的异常 |
2 | 2 | 2015 | 11. 下面关于感觉性共济失调步态的描述错误的是(2.0分) | A.由浅感觉障碍引起 B.行走时步幅较大 C.两腿间距较宽 D.提足较高 |
A.由浅感觉障碍引起 |
2 | 2 | 2015 | 12. 下面关于痉挛性偏瘫步态的描述正确的是(2.0分) | A.患侧下肢显得较短 B.患侧下肢屈曲困难 C.由双侧锥体束损害引起, D.多见于小脑肿瘤 |
B.患侧下肢屈曲困难 |
2 | 2 | 2015 | 13. 下面关于痉挛性截瘫步态的描述错误的是: (2.0分) | A.又称剪刀步态 B.下肢内收肌群张力降低 C.可见于横贯性脊髓损害 D.也可见于脑性瘫痪等 |
B.下肢内收肌群张力降低 |
2 | 2 | 2015 | 14. 关于慌张步态的描述正确的是(2.0分) | A.起步时缓慢 B.全身肌张力降低 C.走路时步伐比正常大 D.躯干前倾,重心后移 |
A.起步时缓慢 |
2 | 2 | 2015 | 15. 关于摇摆步态的描述错误的是(2.0分) | A.骨盆带肌及腰肌无力 B.行走臀部前后摇摆 C.站立时脊柱前凸以维持身体重心平衡 D.见于进行性肌营养不良症 |
B.行走臀部前后摇摆 |
2 | 2 | 2015 | 16. 关于星迹步态的描述错误的是(2.0分) | A.患者闭眼前进时向患侧偏斜 B.患者闭眼后退时向反方向偏斜 C.步行时肢体有大幅度的、不规则的、不自主运动 D.见于前庭迷路病变 |
C.步行时肢体有大幅度的、不规则的、不自主运动 |
2 | 2 | 2015 | 17. 关于臀中肌麻痹步态的描述正确的是(2.0分) | A.一侧臀中肌病变 B.行走时躯干向健侧弯曲 C.多见于小脑肿瘤 D.也可见于大脑病变等 |
A.一侧臀中肌病变 |
2 | 2 | 2015 | 18. 以下关于蟾蜍坐骨神经干复合动作电位引导的叙述,正确的是(2.0分) | A.细胞内引导获得的AP是膜内对地的电位差 B.细胞外引导获得的AP是膜内与膜外的电位差 C.细胞外引导获得的AP是两引导电极间的膜外的电位差 D.细胞内引导获得的AP是两引导电极间的膜内的电位差 |
C.细胞外引导获得的AP是两引导电极间的膜外的电位差 |
2 | 2 | 2015 | 19. 两引导电极间距10mm,蟾蜍坐骨神经干BAP正相振幅大于负相振幅的主因是 (2.0分) | A.负引导电极处的神经纤维数量多于正引导电极 B.BAP传导至正引导电极的距离远于负引导电极,致CAP减小 C.引导电极间距离小于AP波长,正、负相AP叠加 D.AP随传导距离的增加而衰减 |
C.引导电极间距离小于AP波长,正、负相AP叠加 |
2 | 2 | 2015 | 20. 关于神经干BAP正负相振幅和时程描述正确的是(2.0分) | A.正相振幅大于负相振幅、正相时程大于负相时程 B.正相振幅大于负相振幅、正相时程小于负相时程 C.正相振幅小于负相振幅、正相时程大于负相时程 D.正相振幅小于负相振幅、正相时程小于负相时程 |
B.正相振幅大于负相振幅、正相时程小于负相时程 |
2 | 2 | 2015 | 21. 吉兰-巴雷综合征的常见表现不包含(2.0分) | A.四肢无力 B.腓肠肌酸痛 C.一侧面神经瘫痪 D.呼吸肌瘫痪 |
C.一侧面神经瘫痪 |
2 | 2 | 2015 | 22. 关于吉兰-巴雷综合征的辅助检查,哪项正确(2.0分) | A.蛋白-细胞分离在发病后第二周最明显 B.脑脊液的葡萄糖常常降低 C.远端潜伏期正常 D.F波和H反射的异常 |
D.F波和H反射的异常 |
2 | 2 | 2015 | 23. 乳头水平相当于哪个脊髓节段?(2.0分) | A.T4 B.T5 C.T6 D.T7 |
A.T4 |
2 | 2 | 2015 | 24. 创伤性颅脑急症,常规和首选检查方法(2.0分) | A.X线 B.MRI C.超声 D.CT |
D.CT |
2 | 2 | 2015 | 25. 尾状核头部占位首先引起下列什么部位受压改变(2.0分) | A.侧脑室枕角 B.侧脑室三角部 C.颞叶 D.侧脑室额角 |
D.侧脑室额角 |
2 | 2 | 2015 | 26. 颅内最常见的原发肿瘤是(2.0分) | A.脑膜瘤 B.胶质瘤 C.转移瘤 D.听神经瘤 |
B.胶质瘤 |
2 | 2 | 2015 | 27. 弥漫性轴索损伤指头部受到外伤作用后发生的,主要弥漫分布于脑白质、以轴索损伤为主要改变的一种原发性脑实质的损伤。其特点为(2.0分) | A.广泛性白质变性,小灶性出血, B.神经轴索回缩球,小胶质细胞簇出现, C.常与其他颅脑损伤合并, D.死亡率高。 E.以上均正确 |
E.以上均正确 |
2 | 2 | 2015 | 28. 以下心理防御机制中,积极成熟的是 (2.0分) | A.升华 B.转移 C.幻想 D.否认 |
A.升华 |
2 | 2 | 2015 | 29. 关于内脏操作条件反射,以下不正确的描述是(2.0分) | A.是生物反馈技术的理论基础之一 B.是对"内脏反应有意识控制"的挑战 C.可以解释临床上某些心身症状的产生 D.米勒根据其实验研究结果提出了内脏操作条件反射理论 |
B.是对"内脏反应有意识控制"的挑战 |
2 | 2 | 2015 | 30. 一个内心极度自卑的人,会过分炫耀自己的优点。其可能的心理防御机制是(2.0分) | A.否认机制 B.压抑机制 C.反向机制 D.转移机制 |
C.反向机制 |
2 | 2 | 2015 | 31. "一朝被蛇咬,十年怕井绳"的行为学机制是(2.0分) | A.泛化 B.联想 C.强化 D.恐惧 |
A.泛化 |
2 | 2 | 2015 | 32. 所谓"退化"是指(2.0分) | A.个体采用攻击的方式应付环境 B.个体使用幼儿时期的方式应付环境 C.个体采用远离应激源的方式应付环境 D.是一种无能为力、无所适从的行为反应 |
B.个体使用幼儿时期的方式应付环境 |
2 | 2 | 2015 | 33. 点上一根烟,感到心中的烦闷随烟圈吐出,逐渐形成了吸烟成瘾,这属于 (2.0分) | A.正强化 B.负强化 C.消退 D.惩罚 |
B.负强化 |
2 | 2 | 2015 | 34. 关于ABC理论,以下不正确的是(2.0分) | A.由布根塔尔提出 B.强调个体的认知评价和信念对于情绪和行为后果的关键作用 C.A、B、C之间可以相互影响 D.A代表诱发事件和当事人对这一事件的推论和解释 |
A.由布根塔尔提出 |
2 | 2 | 2015 | 35. 衡量组织兴奋性高低的指标是(2.0分) | A.静息电位水平 B.阈电位 C.阈强度 D.动作电位幅度 E.兴奋扩布速度 |
C.阈强度 |
2 | 2 | 2015 | 36. 动作电位沿运动神经纤维传导抵达神经‐肌接头部位时,轴突末梢释放Ach,使终板膜产生终板电位,然后在什么部位引发动作电位 (2.0分) | A.肌细胞膜 B.接头后膜 C.终板膜 D.横管膜 E.三联管膜 |
A.肌细胞膜 |
2 | 2 | 2015 | 37. 神经‐骨骼肌接头处的兴奋传递物质是(2.0分) | A.5‐羟色胺 B.乙酰胆碱 C.去甲肾上腺素 D.肾上腺素 E.多巴胺 |
B.乙酰胆碱 |
2 | 2 | 2015 | 38. 人工减小细胞浸浴液中的Na+浓度,所记录的动作电位出现(2.0分) | A.幅度变小 B.幅度变大 C.时程缩短 D.时程延长 E.复极相延长 |
A.幅度变小 |
2 | 2 | 2015 | 39. 有关细胞毒性水肿,下列说法正确的是 (2.0分) | A.病理生理基础是血脑屏障破坏,血浆进入细胞和间隔。 B.主要发生在急性脑缺血。 C.可见于脑出血、炎症等。 D.在SWI像显示明显。 E.Gd-DTPA增强后明显对比增强。 |
B.主要发生在急性脑缺血。 |
2 | 2 | 2015 | 40. 目前能够进行活体组织内化学物质无创性检测的方法是(2.0分) | A.PWI B.DWI C.MRS D.MR动态增强 E.MRV |
C.MRS |
2 | 2 | 2015 | 41. 脑内环形强化占位,DWI弥散受限高信号,邻近脑实质水肿,需考虑诊断(2.0分) | A.转移瘤 B.星形细胞瘤 C.脑脓肿 D.脑囊虫病 E.蛛网膜囊肿 |
C.脑脓肿 |
2 | 2 | 2015 | 42. SWI可以有效应用于如下疾病检查不包括(2.0分) | A.鉴别脑梗塞中出血灶 B.发现静脉畸形 C.Parkinson病 D.Alzheimer 病 E.脑炎 |
E.脑炎 |
2 | 2 | 2015 | 43. FLAIR序列常用于脑室旁病变显示,其主要特点是(2.0分) | A.扫描速度快。 B.很好的T1对比。 C.很好的T2对比。 D.脑脊液的信号得到有效抑制。 E.采用短的TI。 |
D.脑脊液的信号得到有效抑制。 |
2 | 2 | 2015 | 44. MRI显示陈旧出血优于CT,主要是因为: (2.0分) | A.MRI容易显示其中的去氧血红蛋白 B.含铁血黄素在T2*WI/SWI表现为低信号 C.MRI有更高的空间分辨率 D.MRI可以进行任意断面直接成像 E.MRI可长期显示脱氧血红蛋白 |
B.含铁血黄素在T2*WI/SWI表现为低信号 |
2 | 2 | 2015 | 45. 脊髓梗死好发部位:(2.0分) | A.脊髓腹侧 B.脊髓背侧 C.脊髓中央灰质 D.脊髓侧索 E.以上均好发 |
A.脊髓腹侧 |
2 | 2 | 2015 | 46. 以下脊髓病变何种为脱髓鞘病变(2.0分) | A.脊髓梗死 B.星形细胞瘤 C.ADEM D.脊髓炎 E.神经结节病 |
C.ADEM |
2 | 2 | 2015 | 47. On transverse images lesions have a round or triangular shape and are located posteriorly or laterally,the most impossible diagnosis is: (2.0分) | A.Multiple Sclerosis B.Neuromyelitis Optica C.Acute Disseminating Encephalomyelitis D.Transverse myelitis E.Astrocytoma |
A.Multiple Sclerosis |
2 | 2 | 2015 | 48. follow up A 66 year old patient who presented with progressive sensory complaints.Which diagnosis is right: (2.0分) | A.Multiple Sclerosis B.Neuromyelitis Optica C.Acute Disseminating Encephalomyelitis D.Transverse myelitis. E.Astrocytoma. |
E.Astrocytoma. |
2 | 2 | 2015 | 49. 临床疑有椎管内占位性病变时,首选影像学检查方法是 (2.0分) | A.CT检查 B.X线平片 C.椎管造影 D.MRI检查 |
D.MRI检查 |
2 | 2 | 2015 | 50. 颅脑外伤病人CT检查发现右额叶有斑片状高密度影及周围不规则低密度区,首先考虑(2.0分) | A.脑出血 B.脑挫裂伤 C.出血性脑梗塞 D.脑炎 |
B.脑挫裂伤 |
2 | 3 | 2007 | 1. 音叉放置前额,主要用于测定哪个耳朵 (2.0分) | A.咽鼓管更长 B.外听道更大 C.对频率的感觉范围更广 D.传导性或感音性耳聋 |
D.传导性或感音性耳聋 |
2 | 3 | 2007 | 2. 头不能向左侧偏以对抗检查者的阻力是: (2.0分) | A.左侧斜方肌瘫痪 B.右侧胸锁乳突肌瘫痪 C.左侧胸锁乳突肌瘫痪 D.右侧斜方肌瘫痪 |
B.右侧胸锁乳突肌瘫痪 |
2 | 3 | 2007 | 3. 左侧视束病变引起 (2.0分) | A.右侧同向性偏盲 B.左侧同向性偏盲 C.左侧单眼盲 D.左眼鼻侧盲 |
A.右侧同向性偏盲 |
2 | 3 | 2007 | 4. 右侧胸4至胸12痛觉消失,触觉存在是由于 (2.0分) | A.右侧T4-T12后角损害 B.右侧T4-T12后根损害 C.中央T4-T12灰质损害 D.右侧T2-T10后角损害 |
D.右侧T2-T10后角损害 |
2 | 3 | 2007 | 5. 脊髓中央灰质病变引起 (2.0分) | A.双侧节段性分布的触觉及深感觉下降而痛温觉保留 B.双侧节段性分布的痛温觉下降而触觉及深感觉保留 C.双侧痛温觉、触觉及深感觉均下降 D.感觉障碍不影响 |
B.双侧节段性分布的痛温觉下降而触觉及深感觉保留 |
2 | 3 | 2007 | 6. 常出现后索损害的以下哪项除外(2.0分) | A.后侧索联合变性 B.糖尿病 C.神经梅毒 D.带状疱疹 |
D.带状疱疹 |
2 | 3 | 2007 | 7. 丘脑引起的感觉障碍很少出现: (2.0分) | A.深感觉障碍重于浅感觉 B.自发性疼痛 C.感觉过度 D.复合感觉障碍 |
D.复合感觉障碍 |
2 | 3 | 2007 | 8. 鉴别原发性与继发性三叉神经痛的主要依据是(2.0分) | A.疼痛的性质 B.是否为反复发作 C.疼痛的区域 D.有无面部痛温觉减退或消失,角膜反射减退或消失 E.疼痛时伴随的症状 |
D.有无面部痛温觉减退或消失,角膜反射减退或消失 |
2 | 3 | 2007 | 9. Hunter 综合症的临床表现除外 (2.0分) | A.听觉过敏 B.面神经麻痹 C.舌前2/3味觉障碍 D.疱疹 E.颜面部痛觉下降 |
E.颜面部痛觉下降 |
2 | 3 | 2007 | 10. 左眼不能闭合,示齿时口角向左歪是 (2.0分) | A.左侧周围性面瘫 B.右侧中枢性面瘫 C.左侧中枢性面瘫 D.右侧周围性面瘫 |
A.左侧周围性面瘫 |
2 | 3 | 2007 | 11. 为了延长局麻药的局麻作用和减少不良反应,可加用(2.0分) | A.去甲肾上腺素 B.异丙肾上腺素 C.间羟胺 D.肾上腺素 E.多巴胺 |
D.肾上腺素 |
2 | 3 | 2007 | 12. 多巴胺临床上主要用于(2.0分) | A.急性肾衰竭,抗休克 B.Ⅱ、Ⅲ度房室传导阻滞 C.支气管哮喘,抗休克 D.心脏骤停,急性肾功能衰竭 E.支气管哮喘,急性心功能不全 |
A.急性肾衰竭,抗休克 |
2 | 3 | 2007 | 13. 可用于Ⅱ、Ⅲ度房室传导阻滞的药物是(2.0分) | A.多巴胺 B.异丙肾上腺素 C.去甲肾上腺素 D.肾上腺素 E.麻黄碱 |
B.异丙肾上腺素 |
2 | 3 | 2007 | 14. 下列能引起血压双向反应的药物是(2.0分) | A.间羟胺 B.麻黄碱 C.肾上腺素 D.去甲肾上腺素 E.异丙肾上腺素 |
C.肾上腺素 |
2 | 3 | 2007 | 15. 临床上酚妥拉明用于治疗顽固性充血性心力衰竭的主要原因是(2.0分) | A.抑制心脏,使其得到休息 B.扩张肺动脉,减轻右心后负荷 C.扩张外周小动脉,减轻心脏后负荷 D.兴奋心脏,加强心肌收缩力,使心率加快,心输出量增加 E.扩张外周小静脉,减轻心脏前负荷 |
C.扩张外周小动脉,减轻心脏后负荷 |
2 | 3 | 2007 | 16. 下面哪种情况,应禁用β受体阻断药? (2.0分) | A.高血压 B.快速型心律失常 C.心绞痛 D.重度A-V传导阻滞 E.甲状腺机能亢进 |
D.重度A-V传导阻滞 |
2 | 3 | 2007 | 17. 心绞痛患者长期应用普萘洛尔突然停药可发生(2.0分) | A.过敏反应 B.腹泻 C.恶心,呕吐 D.心功能不全 E.心绞痛发作 |
E.心绞痛发作 |
2 | 3 | 2007 | 18. 下列不属于β受体阻断药适应证的是(2.0分) | A.高血压 B.心绞痛 C.心律失常 D.支气管哮喘 E.青光眼 |
D.支气管哮喘 |
2 | 3 | 2007 | 19. 下列具有内在拟交感活性的β受体阻断药是 (2.0分) | A.普萘洛尔 B.噻吗洛尔 C.美托洛尔 D.吲哚洛尔 E.阿替洛尔 |
D.吲哚洛尔 |
2 | 3 | 2007 | 20. 周围神经损伤后,会发生除以下哪种外的病理生理过程? (2.0分) | A.Schwann's 细胞增生 B.Wallerians' 变性 C.近端存活轴突芽生由近向远长入Schwann's 细胞管 D.轴突、髓鞘收缩、碎裂,被巨核细胞清除 E.远存活轴突芽生由远向近长入Schwann's 细胞管 |
E.远存活轴突芽生由远向近长入Schwann's 细胞管 |
2 | 3 | 2007 | 21. 桡骨中段骨折,患肢腕不能背伸,各指掌指关节不能主动伸直,其原因 (2.0分) | A.由于骨折使患肢运动支架断裂 B.由于伸腕肌损伤 C.桡神经损伤 D.正中神经损伤 E.尺神经损伤 |
C.桡神经损伤 |
2 | 3 | 2007 | 22. 以下关于臂丛神经损伤的治疗,哪一项不正确? (2.0分) | A.开放伤应早期探查修复 B.根性撕脱伤应早期手术 C.干性损伤如果是闭合性损伤应观察3个月无明显恢复时,手术探查 D.闭合性损伤应观察3个月无明显恢复时,手术探查 E.以上说法均不正确 |
C.干性损伤如果是闭合性损伤应观察3个月无明显恢复时,手术探查 |
2 | 3 | 2007 | 23. 开放性骨折现场止血宜首选 (2.0分) | A.上止血带 B.指压出血血管 C.局部压迫包扎 D.结扎出血血管 E.抬高伤肢 |
C.局部压迫包扎 |
2 | 3 | 2007 | 24. 下列哪一项不是肱骨干骨折合并桡神经损伤的表现? (2.0分) | A.拇指不能背伸 B.各手指掌指关节不能背伸 C.爪形手 D.腕关节下垂 E.手背桡侧皮肤感觉减退 |
C.爪形手 |
2 | 3 | 2007 | 25. 以下骨折易损伤其邻近的重要血管和神经的是? (2.0分) | A.跟骨骨折 B.外踝骨折 C.胫骨干中段骨折 D.股骨干骨折 E.肱骨干中段骨折 |
E.肱骨干中段骨折 |
2 | 3 | 2007 | 26. 关于骨折治疗以下哪一项是不妥的? (2.0分) | A.开放骨折的创口在条件许可时应争取早期闭合覆盖。 B.骨折急救处理时重要的措施是转运患者到就近医院 C.肢体开放骨折现场止血的首选是压迫止血。 D.夹板或石膏外固定后应注意血液循环。 E.治疗原则是复位、固定、功能锻炼 |
B.骨折急救处理时重要的措施是转运患者到就近医院 |
2 | 3 | 2007 | 27. 缺血性肌挛缩常继发于下列哪一种损伤(2.0分) | A.肱骨髁上骨折 B.肱骨干骨折 C.胫腓骨骨折 D.髋关节脱位 E.肩关节脱位 |
C.胫腓骨骨折 |
2 | 3 | 2007 | 28. 关于骨折临床愈合的标准,下列哪一项描述是不准确的?(2.0分) | A.X线示骨折线消失 B.局部无反常活动 C.局部轻压痛以及纵向叩击痛 D.伤肢已具有规定的初步功能 E.外固定解除后能满足:上肢向前平举1公斤重物达1分钟。下肢能不扶拐 |
C.局部轻压痛以及纵向叩击痛 |
2 | 3 | 2007 | 29. 引起骨筋膜室综合征的主要发病机理(2.0分) | A.主要神经损伤 B.细菌繁殖过盛 C.肌肉痉挛 D.骨筋膜室内压高 E.血管内膜损伤 |
D.骨筋膜室内压高 |
2 | 3 | 2007 | 30. 两引导电极间距15mm,蟾蜍坐骨神经干BAP正相振幅大于负相振幅的主因是(2.0分) | A.负引导电极处的神经纤维数量多于正引导电极 B.引导电极间距离小于AP波长,正、负相AP叠加 C.BAP传导至正引导电极的距离远于负引导电极,致CAP减小 D.AP随传导距离的增加而衰减 |
B.引导电极间距离小于AP波长,正、负相AP叠加 |
2 | 3 | 2007 | 31. 对蟾蜍坐骨神经干动作电位引导实验,下列叙述正确的是(2.0分) | A.引导神经干动作电位采用双端输入方式 B.神经干动作电位信号采用直流耦合方式输入放大器 C.神经干动作电位是直流信号 D.神经干动作电位采用细胞内引导方式 |
A.引导神经干动作电位采用双端输入方式 |
2 | 3 | 2007 | 32. 下列叙述正确的是(2.0分) | A.神经干的神经冲动传导速度与神经干的兴奋性有关 B.神经干的神经冲动传导速度与刺激强度有关 C.神经干的神经冲动传导速度与实验环境温度无关 D.神经干的神经冲动传导速度与神经干受刺激和引导的方向有关 |
C.神经干的神经冲动传导速度与实验环境温度无关 |
2 | 3 | 2007 | 33. 谈论梅子时引起唾液分泌是 (2.0分) | A.第一信号系统的活动 B.副交感神经兴奋所致 C.交感神经兴奋所致 D.第二信号系统的活动 E.非条件反射 |
D.第二信号系统的活动 |
2 | 3 | 2007 | 34. 下丘脑是皮层下(2.0分) | A.重要的运动中枢 B.重要的感觉中枢 C.较高级的交感中枢 D.较高级的副交感中枢 E.较高级的内脏活动调节中枢 |
E.较高级的内脏活动调节中枢 |
2 | 3 | 2007 | 35. 边缘系统不包括(2.0分) | A.隔区 B.苍白球 C.大脑皮质边缘叶 D.杏仁核 E.下丘脑 |
B.苍白球 |
2 | 3 | 2007 | 36. 下列关于感觉皮层代表区的叙述,错误的是(2.0分) | A.内脏感觉的投射区在额叶 B.本体感觉在中央前回 C.体表感觉区在中央后回 D.听觉代表区在颞叶 E.视觉代表区在枕叶 |
A.内脏感觉的投射区在额叶 |
2 | 3 | 2007 | 37. 下列不属于交感神经兴奋作用的是(2.0分) | A.心跳加快,瞳孔开大 B.胃肠平滑肌收缩 C.支气管平滑肌舒张 D.腹腔内脏血管收缩 E.肾上腺髓质分泌肾上腺素 |
B.胃肠平滑肌收缩 |
2 | 3 | 2007 | 38. 下列属于副交感神经的作用的是(2.0分) | A.骨骼肌血管舒张 B.糖原分解增加 C.逼尿肌收缩 D.瞳孔扩大 E.消化道括约肌收缩 |
C.逼尿肌收缩 |
2 | 3 | 2007 | 39. 交感神经系统不具有的特点是 (2.0分) | A.节前纤维短,节后纤维长 B.紧张性活动 C.支配几乎所有的脏器 D.刺激节前纤维时反应比较局限 E.在应急反应中活动明显加强 |
D.刺激节前纤维时反应比较局限 |
2 | 3 | 2007 | 40. 第一感觉区位于(2.0分) | A.颞叶皮层 B.中央后回 C.中央前回 D.中央前回与岛叶之间 E.颞下回 |
B.中央后回 |
2 | 3 | 2007 | 41. 视觉皮层代表区位于(2.0分) | A.颞叶的颞上回 B.颞叶的颞横回 C.中央后回 D.扣带回 E.枕叶距状裂 |
E.枕叶距状裂 |
2 | 3 | 2007 | 42. 内侧膝状体接受下列哪种感觉纤维投射?(2.0分) | A.三叉丘系 B.视觉传导束 C.听觉传导束 D.内侧丘系 E.脊髓丘脑前束 |
C.听觉传导束 |
2 | 3 | 2007 | 43. 以下属于丘脑非特异投射系统的主要细胞群是 (2.0分) | A.感觉接替核 B.联络核和髓板内核群 C.联络核 D.感觉接替核和联络核 E.髓板内核群 |
E.髓板内核群 |
2 | 3 | 2007 | 44. 丘脑中发出特异投射纤维的最主要核团是 (2.0分) | A.丘脑前核 B.外侧膝状体 C.后腹核 D.内侧膝状体 E.外侧腹核 |
C.后腹核 |
2 | 3 | 2007 | 45. 脊髓丘脑侧束的主要功能是传导(2.0分) | A.同侧的触觉 B.对侧的触觉 C.同侧的痛、温觉 D.对侧的痛、温觉 E.同侧的深压觉 |
D.对侧的痛、温觉 |
2 | 3 | 2007 | 46. 有关感受器电位的叙述,错误的是(2.0分) | A.具有总和现象,不具有"全或无"性质 B.以电紧张形式扩布 C.是一种过渡性慢电位,在一定范围内其大小与刺激强度成正比 D.具有"全或无"性质,不具有总和现象 |
D.具有"全或无"性质,不具有总和现象 |
2 | 3 | 2007 | 47. 特殊感觉器官中不包括 (2.0分) | A.前庭 B.视网膜 C.嗅上皮 D.肌梭 E.耳蜗 |
D.肌梭 |
2 | 3 | 2007 | 48. 下列关于丘脑的感觉接替核的论述,错误的是(2.0分) | A.内侧膝状体是听觉传导的换元站 B.后外侧腹核为脊髓丘脑束与内侧丘系的换元站,同躯体四肢感觉有关 C.是一般经典感觉的第三级神经元 D.外侧膝状体是视觉传导的换元站 E.后内侧腹核为三叉丘系的换元站,与传导内脏感觉有关 |
E.后内侧腹核为三叉丘系的换元站,与传导内脏感觉有关 |
2 | 3 | 2007 | 49. 有关感受器生理特性的说明,错误的是(2.0分) | A.有换能作用 B.只要有刺激,就可以产生兴奋 C.有适应现象 D.一种感受器只对一种形式的刺激最敏感 E.各种感受器适应快慢不一 |
B.只要有刺激,就可以产生兴奋 |
2 | 3 | 2007 | 50. 反射时的长短主要取决于(2.0分) | A.感受器的敏感性 B.刺激的强弱 C.传入与传出纤维的传导速度 D.中枢突触的多少 E.效应器的敏感性 |
D.中枢突触的多少 |
2 | 3 | 2008 | 1. Adrenaline不适用于以下哪一种情况?(2.0分) | A.心脏骤停 B.局部止血 C.充血性心力衰竭 D.过敏性休克 E.支气管哮喘 |
C.充血性心力衰竭 |
2 | 3 | 2008 | 2. ropranolol可用于治疗(2.0分) | A.心律失常 B.心绞痛 C.高血压 D.心肌梗死 E.以上都是 |
E.以上都是 |
2 | 3 | 2008 | 3. 关于骨折治疗以下哪一项是不妥的?(2.0分) | A.肢体开放骨折现场止血的首选是止血带结扎止血。 B.开放骨折的创口在条件许可时应争取早期闭合覆盖。 C.骨折急救处理时重要的措施是妥善固定 D.治疗原则是复位、固定、功能锻炼 E.夹板或石膏外固定后应注意血液循环。 |
A.肢体开放骨折现场止血的首选是止血带结扎止血。 |
2 | 3 | 2008 | 4. 右侧胸4至胸12痛觉消失,触觉存在是由于(2.0分) | A.右侧T4-T12后角损害 B.右侧T4-T12后根损害 C.中央T4-T12灰质损害 D.右侧T2-T10后角损害 |
D.右侧T2-T10后角损害 |
2 | 3 | 2008 | 5. 脊髓中央灰质病变引起(2.0分) | A.双侧痛温觉、触觉及深感觉均下降 B.双侧节段性分布的触觉及深感觉下降而痛温觉保留 C.双侧节段性分布的痛温觉下降而触觉及深感觉保留 D.感觉障碍不影响 |
C.双侧节段性分布的痛温觉下降而触觉及深感觉保留 |
2 | 3 | 2008 | 6. 交叉性感觉障碍因累及(2.0分) | A.同侧的三叉神经感觉核和对侧的脊髓丘脑束 B.对侧的三叉神经脊束核和同侧的脊髓丘脑束 C.同侧的三叉神经脊束核和对侧的脊髓丘脑束 D.同侧的三叉神经运动核和对侧的脊髓丘脑束 |
C.同侧的三叉神经脊束核和对侧的脊髓丘脑束 |
2 | 3 | 2008 | 7. 感觉的产生过程中,不包括的是(2.0分) | A.神经元 B.感受器 C.中枢 D.肌肉 |
D.肌肉 |
2 | 3 | 2008 | 8. 视觉皮层代表区位于 (2.0分) | A.扣带回 B.颞叶的颞横回 C.颞叶的颞上回 D.中央后回 E.枕叶距状裂 |
E.枕叶距状裂 |
2 | 3 | 2008 | 9. 脊髓横贯性损害引起感觉障碍的特点是:(2.0分) | A.受损节段平面以下双侧深浅感觉缺失 B.受损节段平面以下双侧感觉异常和感觉过敏 C.形状不规则的条块状感觉障碍 D.受损节段平面以下双侧痛温觉缺失并伴自发性疼痛 E.受损节段平面以下痛温觉缺失而触觉及深感觉保留 |
A.受损节段平面以下双侧深浅感觉缺失 |
2 | 3 | 2008 | 10. 左眼不能闭合,示齿时口角向左歪是(2.0分) | A.左侧中枢性面瘫 B.左侧周围性面瘫 C.右侧中枢性面瘫 D.右侧周围性面瘫 |
B.左侧周围性面瘫 |
2 | 3 | 2008 | 11. 下列属于副交感神经的作用的是 (2.0分) | A.逼尿肌收缩 B.瞳孔扩大 C.骨骼肌血管舒张 D.糖原分解增加 E.消化道括约肌收缩 |
A.逼尿肌收缩 |
2 | 3 | 2008 | 12. 脊髓的何种损伤可导致痛温觉与轻触觉障碍分离? (2.0分) | A.完全横断 B.后角受损 C.脊髓离断 D.脊髓空洞症 E.前根病变 |
D.脊髓空洞症 |
2 | 3 | 2008 | 13. 外周血管痉挛性疾病可用哪一药物治疗?(2.0分) | A.Adrenaline B.Anisodamine C.Atropine D.Phentolamine E.Propranolol |
D.Phentolamine |
2 | 3 | 2008 | 14. 以下属于丘脑特异性投射系统的核团是 (2.0分) | A.网状核 B.感觉接替核 C.髓板内核群 D.联络核 E.感觉接替核和联络核 |
E.感觉接替核和联络核 |
2 | 3 | 2008 | 15. 以下属于丘脑非特异投射系统的主要细胞群是 (2.0分) | A.联络核和髓板内核群 B.感觉接替核 C.感觉接替核和联络核 D.联络核 E.髓板内核群 |
E.髓板内核群 |
2 | 3 | 2008 | 16. 头不能向左侧偏以对抗检查者的阻力是(2.0分) | A.右侧胸锁乳突肌瘫痪 B.左侧胸锁乳突肌瘫痪 C.左侧斜方肌瘫痪 D.右侧斜方肌瘫痪 |
A.右侧胸锁乳突肌瘫痪 |
2 | 3 | 2008 | 17. 下列属于骨筋膜室综合征早期临床表现,但其中有一项在诊断时要慎重考虑?(2.0分) | A.患侧指(趾)被动牵拉痛 B.患侧指(趾)主动活动障碍 C.无脉征 D.患肢剧烈疼痛 E.筋膜室内组织压力升高 |
C.无脉征 |
2 | 3 | 2008 | 18. 第一感觉区位于 (2.0分) | A.中央后回 B.中央前回 C.颞叶皮层 D.中央前回与岛叶之间 E.颞下回 |
A.中央后回 |
2 | 3 | 2008 | 19. 下列哪一项不是肱骨干骨折合并桡神经损伤的表现?(2.0分) | A.各手指掌指关节不能背伸 B.腕关节下垂 C.爪形手 D.拇指不能背伸 E.手背桡侧皮肤感觉减退 |
C.爪形手 |
2 | 3 | 2008 | 20. 交感神经系统不具有的特点是 (2.0分) | A.紧张性活动 B.支配几乎所有的脏器 C.节前纤维长,节后纤维短 D.在应急反应中活动明显加强 |
C.节前纤维长,节后纤维短 |
2 | 3 | 2008 | 21. 用方形电脉冲刺激组织,方形电脉冲的刺激"三要素"是(2.0分) | A.方形电脉冲的幅度、波宽、方波前沿的强度变化率 B.方形电脉冲的幅度、频率、方波前沿的强度变化率 C.方形电脉冲的幅度、波宽、方波后沿的强度变化率 D.方形电脉冲的幅度、波宽、频率 |
A.方形电脉冲的幅度、波宽、方波前沿的强度变化率 |
2 | 3 | 2008 | 22. 用方形电脉冲刺激组织,在一定的刺激波宽下,刚能引起组织发生兴奋的刺激称为(2.0分) | A.阈上刺激 B.阈刺激 C.阈下刺激 D.最大刺激 |
B.阈刺激 |
2 | 3 | 2008 | 23. 下列并发症在骨折早期就应注意防治?(2.0分) | A.骨筋膜室综合征 B.压褥 C.下肢静脉栓塞形成 D.感染 E.以上都是 |
E.以上都是 |
2 | 3 | 2008 | 24. 两引导电极间距10mm,蟾蜍坐骨神经干BAP正相振幅大于负相振幅的主因是(2.0分) | A.引导电极间距离小于AP波长,正、负相AP叠加 B.BAP传导至正引导电极的距离远于负引导电极,致CAP减小 C.负引导电极处的神经纤维数量多于正引导电极 D.AP随传导距离的增加而衰减 |
A.引导电极间距离小于AP波长,正、负相AP叠加 |
2 | 3 | 2008 | 25. 影响骨折愈合的局部因素有以下哪一项?(2.0分) | A.是否为开放骨折 B.软组织损伤程度 C.骨折的部位和类型 D.是否存在感染 E.以上都是 |
E.以上都是 |
2 | 3 | 2008 | 26. 引导神经干动作电位,多道生理信号采集处理系统的参数应为(2.0分) | A.采用频率100Hz、时间常数0.02s、滤波频率3kHz、灵敏度5mV B.采用频率100Hz、时间常数0.2s、滤波频率3Hz、灵敏度5mV C.采用频率100KHz、时间常数0.02s、滤波频率3kHz、灵敏度5mV D.采用频率100KHz、时间常数0.02s、滤波频率3Hz、灵敏度5mV |
C.采用频率100KHz、时间常数0.02s、滤波频率3kHz、灵敏度5mV |
2 | 3 | 2008 | 27. 为了改善周围神经损伤的疗效,下列哪项研究是有益的?(2.0分) | A.促进神经芽生的速度 B.妥善合理的功能重建 C.延缓神经功能效应器的退变 D.改善桥接通道的通过能力 E.以上都是 |
E.以上都是 |
2 | 3 | 2008 | 28. 患者瞬目时发生上唇不自主颤动,主要因为(2.0分) | A.再生的面神经长入邻近神经纤维 B.三叉神经痛 C.小脑损伤 D.习惯动作 E.局灶性癫痫 |
A.再生的面神经长入邻近神经纤维 |
2 | 3 | 2008 | 29. 内侧膝状体接受下列哪种感觉纤维投射? (2.0分) | A.三叉丘系 B.内侧丘系 C.视觉传导束 D.听觉传导束 E.脊髓丘脑前束 |
D.听觉传导束 |
2 | 3 | 2008 | 30. 感受器能把刺激所包含的环境变化信息,转移到动作电位的序列之中,感受器的这种生理特性称为 (2.0分) | A.感受器的适宜刺激 B..感受器的编码作用 C.感受器的换能作用 D.感受器的适应现象 E.感受器的启动电位 |
B..感受器的编码作用 |
2 | 3 | 2008 | 31. 鉴别原发性与继发性三叉神经痛的主要依据是:(2.0分) | A.疼痛的区域 B.疼痛的性质 C.是否为反复发作 D.有无面部痛温觉减退或消失,角膜反射减退或消失 E.疼痛时伴随的症状 |
D.有无面部痛温觉减退或消失,角膜反射减退或消失 |
2 | 3 | 2008 | 32. wallerian 变性指的是:(2.0分) | A.神经断裂后,其支配的效应器失功能化 B.神经断裂后,其近侧段轴突和髓鞘碎裂溶解。 C.神经断裂后,其远侧段轴突和髓鞘碎裂溶解。 D.神经断裂后,其母神经元发生凋亡。 E.以上都不是 |
C.神经断裂后,其远侧段轴突和髓鞘碎裂溶解。 |
2 | 3 | 2008 | 33. 关于骨折的临床表现,以下哪项是正确的?(2.0分) | A.骨折后可有吸收热,一般在38-39C?。 B.骨折都有较明显的外伤史 C.骨折有时可缺乏骨折的专有体征 D.为明确骨折的诊断,应着重检查肢体有无异常活动、骨擦音或骨擦感等骨折的专有体征 E.以上都不是 |
C.骨折有时可缺乏骨折的专有体征 |
2 | 3 | 2008 | 34. 用方形电脉冲刺激蟾蜍坐骨神经干,下列叙述正确的是(2.0分) | A.B.刺激电极正极下神经干兴奋性增加 B.A.刺激电极正极下神经干去极化,刺激电极负极下神经干超极化 C.C.刺激电极负极下神经干兴奋性降低 D.D.刺激电极负极下神经干兴奋性增加 |
D.D.刺激电极负极下神经干兴奋性增加 |
2 | 3 | 2008 | 35. 在周围神经损伤的治疗中,下列哪一项是不正确的?(2.0分) | A.开放性神经损伤中,神经修复时间应根据损伤程度、伤后时间、有无污染及有无复合损伤等因素决定。 B.闭合性神经损伤,若暴力小症状轻,可先观察和非手术治疗3个月。 C.闭合性神经损伤,若暴力大症状重,应早期手术探查。 D.周围神经损伤的治疗原则是尽早的恢复神经的连续性。 E.若神经损伤后时间过长,效应器退变,即使神经吻合后也无法恢复功能。因此,神经伤后两年不考虑再行神经修复术。 |
E.若神经损伤后时间过长,效应器退变,即使神经吻合后也无法恢复功能。因此,神经伤后两年不考虑再行神经修复术。 |
2 | 3 | 2008 | 36. 老年人髋部骨折的治疗理念是(2.0分) | A.坚强内固定以争取尽早下床活动,减少并发症 B.坚强内固定以获得良好关节活动度 C.力争解剖复位,减少股骨头坏死 D.力争解剖复位,减少跛行 E.以上都不是 |
A.坚强内固定以争取尽早下床活动,减少并发症 |
2 | 3 | 2008 | 37. 以下骨折易损伤其邻近的重要血管和神经,其中哪一部位骨折除外?(2.0分) | A.胫骨干中段骨折 B.股骨髁上骨折 C.腓骨颈骨折 D.肱骨髁上骨折 E.肱骨干中段骨折 |
A.胫骨干中段骨折 |
2 | 3 | 2008 | 38. 下列哪一药不是儿茶酚胺类药物(2.0分) | A.Ephedrine B.Adrenaline C.Dopamine D.Noradrenaline E.Isoprenaline |
A.Ephedrine |
2 | 3 | 2008 | 39. Dopamine舒张肾与肠系膜血管主要由于(2.0分) | A.兴奋β受体 B.兴奋H1受体 C.阻断α受体 D.释放组胺 E.兴奋多巴胺受体 |
E.兴奋多巴胺受体 |
2 | 3 | 2008 | 40. 周围神经损伤后,以下和何种类型损伤在病理上可发生wallerian 变性?(2.0分) | A.神经震荡 B.神经断裂 C.A+B D.轴索中断 E.B+C |
C.A+B |
2 | 3 | 2008 | 41. 在周围神经损伤的诊断中,下列哪一项检查客观性较好?(2.0分) | A.肌力检查。 B.茚三酮指印试验 C.两点辨别试验 D.痛觉检查 E.肌电图 |
B.茚三酮指印试验 |
2 | 3 | 2008 | 42. 左侧视束病变引起(2.0分) | A.左侧单眼盲 B.右侧同向性偏盲 C.左侧同向性偏盲 D.左眼鼻侧盲 |
B.右侧同向性偏盲 |
2 | 3 | 2008 | 43. 治疗过敏性休克的首选药物是 (2.0分) | A.抗组胺药 B.糖皮质激素 C.Noradrenaline D.Adrenaline E.Atropine |
D.Adrenaline |
2 | 3 | 2008 | 44. 对蟾蜍坐骨神经干动作电位引导实验,下列叙述正确的是(2.0分) | A.电刺激神经干,处于负刺激电极下的神经纤维发生去极化 B.神经干动作电位的振幅与刺激强度成正比 C.3 mol/L KCl处理神经干,KCl使神经纤维产生超极化抑制 D.4g/L普如卡因处理神经干,普如卡因使神经纤维产生超极化抑制 |
A.电刺激神经干,处于负刺激电极下的神经纤维发生去极化 |
2 | 3 | 2008 | 45. 骨折的全身表现正确的是_______。(2.0分) | A.一般体温正常,严重骨折时体温可升高 B.若出现呼吸困难而难以用其他脏器损伤解释时,应考虑有无脂肪栓塞综合征或急性肺栓塞。 C.可因严重创伤、大出血、剧痛等而发生休克 D.开放骨折体温升高应考虑感染 E.以上都是 |
E.以上都是 |
2 | 3 | 2008 | 46. Isoprenaline的特点是(2.0分) | A.可减少组织耗氧量 B.是一种β2受体选择性激动药 C.是一种β1、β2受体激动药 D.是一种β1受体选择性激动药 E.可收缩骨骼肌血管 |
C.是一种β1、β2受体激动药 |
2 | 3 | 2008 | 47. 音叉放置前额,主要用于测定哪个耳朵(2.0分) | A.咽鼓管更长 B.外听道更大 C.对频率的感觉范围更广 D.传导性或感音性耳聋 |
D.传导性或感音性耳聋 |
2 | 3 | 2008 | 48. Propranolol不具有下列哪一项作用?(2.0分) | A.减慢窦性心率 B.延缓用胰岛素后血糖的恢复 C.降低心肌耗氧量 D.扩张支气管平滑肌 E.抑制肾素释放 |
D.扩张支气管平滑肌 |
2 | 3 | 2008 | 49. 属于稳定性骨折的是 (2.0分) | A.粉碎性骨折 B.嵌插骨折 C.螺旋形骨折 D.斜形骨折 E.以上都不是 |
B.嵌插骨折 |
2 | 3 | 2008 | 50. 既可阻断α受体又可阻断β受体的药物是(2.0分) | A.Labetolol B.Phentolamine C.Atenolol D.Propranolol E.Prazosin |
A.Labetolol |
2 | 3 | 2009 | 1. 音叉放置前额,主要用于测定哪个耳朵(2.0分) | A.对频率的感觉范围更广 B.外听道更大 C.咽鼓管更长 D.传导性或感音性耳聋 |
NaN |
2 | 3 | 2009 | 2. 头不能向左侧偏以对抗检查者的阻力是: (2.0分) | A.左侧胸锁乳突肌瘫痪 B.右侧胸锁乳突肌瘫痪 C.左侧斜方肌瘫痪 D.右侧斜方肌瘫痪 |
NaN |
2 | 3 | 2009 | 3. 左侧视束病变引起 (2.0分) | A.左侧同向性偏盲 B.右侧同向性偏盲 C.左侧单眼盲 D.左眼鼻侧盲 |
NaN |
2 | 3 | 2009 | 4. 右侧胸4至胸12痛觉消失,触觉存在是由于(2.0分) | A.中央T4-T12灰质损害 B.右侧T4-T12后根损害 C.右侧T4-T12后角损害 D.右侧T2-T10后角损害 |
NaN |
2 | 3 | 2009 | 5. 脊髓中央灰质病变引起(2.0分) | A.双侧节段性分布的痛温觉下降而触觉及深感觉保留 B.双侧节段性分布的触觉及深感觉下降而痛温觉保留 C.双侧痛温觉、触觉及深感觉均下降 D.感觉障碍不影响 |
NaN |
2 | 3 | 2009 | 6. 交叉性感觉障碍因累及(2.0分) | A.同侧的三叉神经脊束核和对侧的脊髓丘脑束 B.对侧的三叉神经脊束核和同侧的脊髓丘脑束 C.同侧的三叉神经感觉核和对侧的脊髓丘脑束 D.同侧的三叉神经运动核和对侧的脊髓丘脑束 |
NaN |
2 | 3 | 2009 | 7. 丘脑引起的感觉障碍很少出现:(2.0分) | A.感觉过度 B.自发性疼痛 C.深感觉障碍重于浅感觉 D.复合感觉障碍 |
NaN |
2 | 3 | 2009 | 8. Hunter 综合症的临床表现除外(2.0分) | A.面神经麻痹 B.听觉过敏 C.舌前2/3味觉障碍 D.疱疹 E.颜面部痛觉下降 |
NaN |
2 | 3 | 2009 | 9. 听觉过敏主要是损伤同侧的哪个颅神经(2.0分) | A.Ⅴ B.Ⅶ C.Ⅷ D.Ⅸ |
NaN |
2 | 3 | 2009 | 10. 鉴别原发性与继发性三叉神经痛的主要依据是:(2.0分) | A.有无面部痛温觉减退或消失,角膜反射减退或消失 B.疼痛的性质 C.是否为反复发作 D.疼痛的区域 E.疼痛时伴随的 |
NaN |
2 | 3 | 2009 | 11. 用方形电脉冲刺激组织,方形电脉冲的刺激"三要素"是 (2.0分) | A.方形电脉冲的幅度、波宽、方波前沿的强度变化率 B.方形电脉冲的幅度、波宽、方波后沿的强度变化率 C.方形电脉冲的幅度、频率、方波前沿的强度变化率 D.方形电脉冲的幅度、波宽、频率 |
NaN |
2 | 3 | 2009 | 12. 用方形电脉冲刺激组织,在一定的刺激波宽下,刚能引起组织发生兴奋的刺激称为(2.0分) | A.阈下刺激 B.阈刺激 C.阈上刺激 D.最大刺激 |
NaN |
2 | 3 | 2009 | 13. 对蟾蜍坐骨神经干动作电位引导实验,下列叙述正确的是 (2.0分) | A.神经干动作电位的振幅与刺激强度成正比 B.电刺激神经干,处于负刺激电极下的神经纤维发生去极化 C.3 mol/L KCl处理神经干,KCl使神经纤维产生超极化抑制 D.4g/L普如卡因处理神经干,普如卡因使神经纤维产生超极化抑制 |
NaN |
2 | 3 | 2009 | 14. 下列叙述正确的是(2.0分) | A.神经干的神经冲动传导速度与刺激强度有关 B.神经干的神经冲动传导速度与神经干的兴奋性有关 C.神经干的神经冲动传导速度与实验环境温度无关 D.神经干的神经冲动传导速度与神经干受刺激和引导的方向有关 |
NaN |
2 | 3 | 2009 | 15. 肾上腺素(adrenaline)不适用于以下哪一种情况?(2.0分) | A.心脏骤停 B.充血性心力衰竭 C.局部止血 D.过敏性休克 E.支气管哮喘 |
NaN |
2 | 3 | 2009 | 16. 为了延长局麻药的作用时间和减少不良反应,可配伍应用的药物是 (2.0分) | A.肾上腺素(adrenaline) B.异丙肾上腺素(isoproterenol) C.多巴胺(dopamine) D.去甲肾上腺素(noradrenaline) E.麻黄碱(ephedrine) |
NaN |
2 | 3 | 2009 | 17. 急、慢性鼻炎、鼻窦炎引起鼻充血时,可用于滴鼻的药物是 (2.0分) | A.去甲肾上腺素(noradrenaline) B.麻黄碱(ephedrine) C.异丙肾上腺素(isoproterenol) D.肾上腺素(adrenaline) E.以上都不是 |
NaN |
2 | 3 | 2009 | 18. 无尿休克病人禁用的药物是 (2.0分) | A.去甲肾上腺素(noradrenaline) B.阿托品(atropine) C.多巴胺(dopamine) D.间羟胺(metaraminol) E.肾上腺素(adrenaline) |
NaN |
2 | 3 | 2009 | 19. 去氧肾上腺素(phenylephrine)的扩瞳作用机制是 (2.0分) | A.阻断虹膜括约肌的M受体 B.激动虹膜括约肌的M受体 C.阻断虹膜开大肌的 受体 D.兴奋虹膜开大肌的 受体 E.激动睫状肌的M受体 |
NaN |
2 | 3 | 2009 | 20. 肾上腺素(adrenaline)升压作用可被下列哪一类药物所翻转? (2.0分) | A.M受体阻断药 B.N受体阻断药 C.β受体阻断药 D.α受体阻断药 E.H1受体阻断药 |
NaN |
2 | 3 | 2009 | 21. 酚妥拉明(phentolamine)的不良反应有 (2.0分) | A.增高血压 B.诱发溃疡病 C.抑制呼吸中枢 D.静脉给药可引起心动过缓 E.静脉给药发生外漏时引起皮肤缺血坏死 |
NaN |
2 | 3 | 2009 | 22. 普萘洛尔(propranolol)不具有下列哪一项作用? (2.0分) | A.扩张支气管平滑肌 B.降低心肌耗氧量 C.减慢窦性心率 D.延缓用胰岛素后血糖的恢复 E.抑制肾素释放 |
NaN |
2 | 3 | 2009 | 23. β受体阻断药禁用于支气管哮喘是由于 (2.0分) | A.阻断支气管β2受体 B.阻断支气管β1受体 C.阻断支气管α受体 D.阻断支气管M受体 E.激动支气管M受体 |
NaN |
2 | 3 | 2009 | 24. 既可阻断α受体又可阻断β受体的药物是 (2.0分) | A.拉贝洛尔(labetolol) B.普萘洛尔(propranolol) C.阿替洛尔(atenolol) D.酚妥拉明(phentolamine) E.哌唑嗪(prazosin) |
NaN |
2 | 3 | 2009 | 25. 感觉的产生过程中,不包括的是(2.0分) | A.感受器 B.神经元 C.中枢 D.肌肉 |
NaN |
2 | 3 | 2009 | 26. 感受器能把刺激所包含的环境变化信息,转移到动作电位的序列之中,感受器的这种生理特性称为 (2.0分) | A.感受器的适宜刺激 B.感受器的换能作用 C.感受器的编码作用 D.感受器的适应现象 E.感受器的启动电位 |
NaN |
2 | 3 | 2009 | 27. 脊髓的何种损伤可导致痛温觉与轻触觉障碍分离? (2.0分) | A.完全横断 B.脊髓空洞症 C.脊髓离断 D.后角受损 E.前根病变 |
NaN |
2 | 3 | 2009 | 28. 以下属于丘脑特异性投射系统的核团是 (2.0分) | A.联络核 B.网状核 C.感觉接替核 D.髓板内核群 E.感觉接替核和联络核 |
NaN |
2 | 3 | 2009 | 29. 以下属于丘脑非特异投射系统的主要细胞群是 (2.0分) | A.联络核和髓板内核群 B.感觉接替核和联络核 C.感觉接替核 D.联络核 E.髓板内核群 |
NaN |
2 | 3 | 2009 | 30. 内侧膝状体接受下列哪种感觉纤维投射? (2.0分) | A.内侧丘系 B.三叉丘系 C.视觉传导束 D.听觉传导束 E.脊髓丘脑前束 |
NaN |
2 | 3 | 2009 | 31. 视觉皮层代表区位于 (2.0分) | A.颞叶的颞横回 B.颞叶的颞上回 C.中央后回 D.扣带回 E.枕叶距状裂 |
NaN |
2 | 3 | 2009 | 32. 第一感觉区位于 (2.0分) | A.中央前回 B.中央后回 C.颞叶皮层 D.中央前回与岛叶之间 E.颞下回 |
NaN |
2 | 3 | 2009 | 33. 交感神经系统不具有的特点是 (2.0分) | A.节前纤维长,节后纤维短 B.支配几乎所有的脏器 C.紧张性活动 D.在应急反应中活动明显加强 |
NaN |
2 | 3 | 2009 | 34. 下列属于副交感神经的作用的是 (2.0分) | A.瞳孔扩大 B.糖原分解增加 C.逼尿肌收缩 D.骨骼肌血管舒张 E.消化道括约肌收缩 |
NaN |
2 | 3 | 2009 | 35. 下列不属于交感神经兴奋作用的是 (2.0分) | A.心跳加快 B.瞳孔开大 C.支气管平滑肌舒张 D.胃肠平滑肌收缩 E.肾上腺髓质分泌肾上腺素 |
NaN |
2 | 3 | 2009 | 36. 下列关于感觉皮层代表区的叙述,错误的是 (2.0分) | A.本体感觉在中央前回 B.内脏感觉的投射区在额叶 C.体表感觉区在中央后回 D.听觉代表区在颞叶 E.视觉代表区在枕叶 |
NaN |
2 | 3 | 2009 | 37. 非条件反射的特点不包括 (2.0分) | A.生来具有 B.无需皮层参与 C.反射弧固定 D.不受大脑调控 E.数量有限 |
NaN |
2 | 3 | 2009 | 38. 下丘脑是皮层下 (2.0分) | A.重要的感觉中枢 B.重要的运动中枢 C.较高级的交感中枢 D.较高级的副交感中枢 E.较高级的内脏活动调节中枢 |
NaN |
2 | 3 | 2009 | 39. 谈论梅子时引起唾液分泌是 (2.0分) | A.交感神经兴奋所致 B.副交感神经兴奋所致 C.第一信号系统的活动 D.第二信号系统的活动 E.非条件反射 |
NaN |
2 | 3 | 2009 | 40. 周围神经损伤后,以下和何种类型损伤在病理上可发生wallerian 变性? (2.0分) | A.神经断裂 B.轴索中断 C.神经震荡 D.A+B E.B+C |
NaN |
2 | 3 | 2009 | 41. 在周围神经损伤的诊断中,下列哪一项检查客观性较好(2.0分) | A.两点辨别试验 B.肌力检查。 C.痛觉检查 D.茚三酮指印试验 E.肌电图 |
NaN |
2 | 3 | 2009 | 42. 为了改善周围神经损伤的疗效,下列哪项研究是有益的? (2.0分) | A.促进神经芽生的速度 B.改善桥接通道的通过能力 C.延缓神经功能效应器的退变 D.妥善合理的功能重建 E.以上都是 |
NaN |
2 | 3 | 2009 | 43. 关于骨折的临床表现,以下哪项是正确的? (2.0分) | A.骨折都有较明显的外伤史 B.骨折有时可缺乏骨折的专有体征 C.为明确骨折的诊断,应着重检查肢体有无异常活动、骨擦音或骨擦感等骨折的专有体征 D.骨折后可有吸收热,一般在38-39C?。 E.以上都不是 |
NaN |
2 | 3 | 2009 | 44. 下列哪一项不是肱骨干骨折合并桡神经损伤的表现? (2.0分) | A.腕关节下垂 B.各手指掌指关节不能背伸 C.爪形手 D.拇指不能背伸 E.手背桡侧皮肤感觉减退 |
NaN |
2 | 3 | 2009 | 45. 以下骨折易损伤其邻近的重要血管和神经,其中哪一部位骨折除外? (2.0分) | A.肱骨髁上骨折 B.股骨髁上骨折 C.腓骨颈骨折 D.胫骨干中段骨折 E.肱骨干中段骨折 |
NaN |
2 | 3 | 2009 | 46. 下列并发症在骨折早期就应注意防治? (2.0分) | A.骨筋膜室综合征 B.下肢静脉栓塞形成 C.压褥 D.感染 E.以上都是 |
NaN |
2 | 3 | 2009 | 47. 骨盆骨折可能出现哪些并发症_______。(2.0分) | A.失血性休克 B.直肠、膀胱、尿道损伤 C.神经损伤 D.机体系统炎症反应综合征(SIRS) E.以上都是 |
NaN |
2 | 3 | 2009 | 48. 骨筋膜室综合症最主要的治疗原则是 (2.0分) | A.给予血管舒张剂,消除血管痉挛 B.抬高患肢,以利消肿 C.被动按摩,以利消肿 D.臂从麻醉,解除血管痉挛 E.解除包扎及固定物,经观察无好转,切开筋膜减压 |
NaN |
2 | 3 | 2009 | 49. 关于DCO,以下哪一项不合适? (2.0分) | A.对ISS评分>18分的多发骨折患者,应遵循DCO的原则 B.对ISS评分>20分的多发骨折患者,尽量对骨折进行确定性固定 C.外固定架固定是 DCO的一个常用方法 D.股骨、骨盆等大骨折的尽早稳定有助于抗休克 E.以上都准确 |
NaN |
2 | 3 | 2009 | 50. 关于骨折治疗以下哪一项是不妥的?(2.0分) | A.治疗原则是复位、固定、功能锻炼 B.骨折急救处理时重要的措施是妥善固定 C.肢体开放骨折现场止血的首选是止血带结扎止血。 D.开放骨折的创口在条件许可时应争取早期闭合覆盖。 E.夹板或石膏外固定后应注意血液循环。 |
NaN |
2 | 3 | 2011 | 1. 衡量组织兴奋性高低的指标是(2.0分) | A.静息电位水平 B.阈电位 C.阈强度 D.动作电位幅度 E.兴奋扩布速度 |
C.阈强度 |
2 | 3 | 2011 | 2. 临床上普鲁卡因局部麻醉药的应用是由于阻断了局部神经冲动的传导,其神经细胞发生的变化是(2.0分) | A.细胞变性 B.结构完整性破坏 C.功能完整性破坏 D.细胞膜电压门控钾通道破坏 E.细胞膜化学门控钠通道破坏 |
C.功能完整性破坏 |
2 | 3 | 2011 | 3. 在神经‐骨骼肌接头中消除乙酰胆碱的酶是(2.0分) | A.ATP 酶 B.胆碱酯酶 C.腺苷酸环化酶 D.磷酸二酯酶 E.单胺氧化酶 |
B.胆碱酯酶 |
2 | 3 | 2011 | 4. 细胞膜上不衰减形式传播的电活动是(2.0分) | A.动作电位 B.静息电位 C.终板电位 D.感受器电位 E.突触后电位 |
A.动作电位 |
2 | 3 | 2011 | 5. 实验中,如果同时刺激神经纤维的两端,产生的两个动作电位(2.0分) | A.将各自通过中点后传导到另一端 B.将在中点相遇,然后传回到起始点 C.只有较强的动作电位通过中点而到达另一端 D.将在中点相遇后停止传导 E.到达中点后将复合成一个更大的动作电位 |
D.将在中点相遇后停止传导 |
2 | 3 | 2011 | 6. Adrenaline不适用于以下哪一种情况?(2.0分) | A.心脏骤停 B.充血性心力衰竭 C.局部止血 D.过敏性休克 E.支气管哮喘 |
B.充血性心力衰竭 |
2 | 3 | 2011 | 7. 为了延长局麻药的作用时间和减少不良反应,可配伍应用的药物是 (2.0分) | A.Adrenaline B.Isoproterenol C.dopamine D.noradrenaline E.ephedrine |
A.Adrenaline |
2 | 3 | 2011 | 8. 急、慢性鼻炎、鼻窦炎引起鼻充血时,可用于滴鼻的药物是 (2.0分) | A.Noradrenaline B.Ephedrine C.isoproterenol D.adrenaline E.以上都不是 |
B.Ephedrine |
2 | 3 | 2011 | 9. 无尿休克病人禁用的药物是 (2.0分) | A.去甲肾上腺素 B.阿托品 C.多巴胺 D.间羟胺 E.肾上腺素 |
A.去甲肾上腺素 |
2 | 3 | 2011 | 10. 去氧肾上腺素(phenylephrine)的扩瞳作用机制是(2.0分) | A.阻断虹膜括约肌的M受体 B.激动虹膜括约肌的M受体 C.阻断虹膜开大肌的 受体 D.兴奋虹膜开大肌的 受体 E.激动睫状肌的M受体 |
D.兴奋虹膜开大肌的 受体 |
2 | 3 | 2011 | 11. Epinephrine升压作用可被下列哪一类药物所翻转? (2.0分) | A.M受体阻断药 B.N受体阻断药 C.β受体阻断药 D.α受体阻断药 E.H1受体阻断药 |
D.α受体阻断药 |
2 | 3 | 2011 | 12. Phentolamine的不良反应有 (2.0分) | A.增高血压 B.诱发溃疡病 C.抑制呼吸中枢 D.静脉给药可引起心动过缓 E.静脉给药发生外漏时引起皮肤缺血坏死 |
B.诱发溃疡病 |
2 | 3 | 2011 | 13. P ropranolol不具有下列哪一项作用? (2.0分) | A.扩张支气管平滑肌 B.降低心肌耗氧量 C.减慢窦性心率 D.延缓用胰岛素后血糖的恢复 E.抑制肾素释放 |
A.扩张支气管平滑肌 |
2 | 3 | 2011 | 14. β受体阻断药禁用于支气管哮喘是由于 (2.0分) | A.阻断支气管β2受体 B.阻断支气管β1受体 C.阻断支气管α受体 D.阻断支气管M受 E.激动支气管M受体 |
A.阻断支气管β2受体 |
2 | 3 | 2011 | 15. 既可阻断α受体又可阻断β受体的药物是 (2.0分) | A.拉贝洛尔 B.普萘洛尔 C.阿替洛尔 D.酚妥拉明 E.哌唑嗪 |
A.拉贝洛尔 |
2 | 3 | 2011 | 16. 感觉的产生过程中,不包括的是(2.0分) | A.感受器 B.神经元 C.中枢 D.肌肉 |
D.肌肉 |
2 | 3 | 2011 | 17. 感受器能把刺激所包含的环境变化信息,转移到动作电位的序列之中,感受器的这种生理特性称为 (2.0分) | A.感受器的适宜刺激 B.感受器的换能作用 C.感受器的编码作用 D.感受器的适应现象 E.感受器的启动电位 |
C.感受器的编码作用 |
2 | 3 | 2011 | 18. 脊髓的何种损伤可导致痛温觉与轻触觉障碍分离? (2.0分) | A.完全横断 B.脊髓空洞症 C.脊髓离断 D.后角受损 E.前根病变 |
B.脊髓空洞症 |
2 | 3 | 2011 | 19. 以下属于丘脑特异性投射系统的核团是 (2.0分) | A.联络核 B.网状核 C.感觉接替核 D.髓板内核群 E.感觉接替核和联络核 |
E.感觉接替核和联络核 |
2 | 3 | 2011 | 20. 以下属于丘脑非特异投射系统的主要细胞群是 (2.0分) | A.联络核和髓板内核群 B.感觉接替核和联络核 C.感觉接替核 D.联络核 E.髓板内核群 |
E.髓板内核群 |
2 | 3 | 2011 | 21. 内侧膝状体接受下列哪种感觉纤维投射? (2.0分) | A.内侧丘系 B.三叉丘系 C.视觉传导束 D.听觉传导束 E.脊髓丘脑前束 |
D.听觉传导束 |
2 | 3 | 2011 | 22. 视觉皮层代表区位于 (2.0分) | A.颞叶的颞横回 B.颞叶的颞上回 C.中央后回 D.扣带回 E.枕叶距状裂 |
E.枕叶距状裂 |
2 | 3 | 2011 | 23. 第一感觉区位于 (2.0分) | A.中央前回 B.中央后回 C.颞叶皮层 D.中央前回与岛叶之间 E.颞下回 |
B.中央后回 |
2 | 3 | 2011 | 24. 交感神经系统不具有的特点是 (2.0分) | A.节前纤维长,节后纤维短 B.支配几乎所有的脏器 C.紧张性活动 D.在应急反应中活动明显加强 |
A.节前纤维长,节后纤维短 |
2 | 3 | 2011 | 25. 下列属于副交感神经的作用的是 (2.0分) | A.瞳孔扩大 B.糖原分解增加 C.逼尿肌收缩 D.骨骼肌血管舒张 E.消化道括约肌收缩 |
C.逼尿肌收缩 |
2 | 3 | 2011 | 26. 下列不属于交感神经兴奋作用的是 (2.0分) | A.心跳加快 B.瞳孔开大 C.支气管平滑肌舒张 D.胃肠平滑肌收缩 E.肾上腺髓质分泌肾上腺素 |
D.胃肠平滑肌收缩 |
2 | 3 | 2011 | 27. 下列关于感觉皮层代表区的叙述,错误的是 (2.0分) | A.本体感觉在中央前回 B.内脏感觉的投射区在额叶 C.体表感觉区在中央后回 D.听觉代表区在颞叶 E.视觉代表区在枕叶 |
B.内脏感觉的投射区在额叶 |
2 | 3 | 2011 | 28. 非条件反射的特点不包括 (2.0分) | A.生来具有 B.无需皮层参与 C.反射弧固定 D.反射形式多变 E.数量有限 |
D.反射形式多变 |
2 | 3 | 2011 | 29. 下丘脑是皮层下 (2.0分) | A.重要的感觉中枢 B.重要的运动中枢 C.较高级的交感中枢 D.较高级的副交感中枢 E.较高级的内脏活动调节中枢 |
E.较高级的内脏活动调节中枢 |
2 | 3 | 2011 | 30. 不引起生理性瞳孔缩小的是(2.0分) | A.光反射 B.近反射 C.调节反射 D.辐辏反射 E.惊吓反应 |
E.惊吓反应 |
2 | 3 | 2011 | 31. 有关视力、视角的论述,正确的是(2.0分) | A.视力大小与视角大小成正比 B.视角一分角能看清物体时,视力为 1.0 C.视锥细胞直径越大,视力越好 D.双眼视物时可以使视力提高 E.同一距离物体越大形成的视角就越小 |
B.视角一分角能看清物体时,视力为 1.0 |
2 | 3 | 2011 | 32. 房水的主要功能是(2.0分) | A.折光成像 B.对玻璃体有保护作用 C.对角膜和晶状体有营养作用 D.对视网膜有保护作用 E.促进睫状肌收缩 |
C.对角膜和晶状体有营养作用 |
2 | 3 | 2011 | 33. 瞳孔在弱光下散大,而在强光下缩小,称为(2.0分) | A.互感性对光反射 B.辐辏反射 C.瞳孔对光反射 D.瞳孔调节反射 E.瞳孔近反射 |
C.瞳孔对光反射 |
2 | 3 | 2011 | 34. 人眼的调节功能主要是由以下哪种变化来实现的? (2.0分) | A.角膜的曲度变化 B.角膜和晶状体的曲度变化 C.晶状体的曲度变化 D.玻璃体的曲度变化 E.晶状体和玻璃体的曲度变化 |
C.晶状体的曲度变化 |
2 | 3 | 2011 | 35. 眼尽最大能力调节时所能看清物体的最近距离,称为(2.0分) | A.节点 B.前主焦点 C.远点 D.近点 E.后主焦点 |
D.近点 |
2 | 3 | 2011 | 36. 下列哪种非正视眼矫正用凹透镜?(2.0分) | A.近视眼 B.远视眼 C.散光眼 D.老视眼 E.老花眼 |
A.近视眼 |
2 | 3 | 2011 | 37. 关于瞳孔反射的说明,正确的是 (2.0分) | A.视近物时瞳孔扩大 B.瞳孔近反射与对光反射的通路相同 C.动眼神经损伤使瞳孔缩小 D.交感神经兴奋使瞳孔扩大 E.副交感神经兴奋使瞳孔扩大 |
D.交感神经兴奋使瞳孔扩大 |
2 | 3 | 2011 | 38. 瞳孔近反射的作用是(2.0分) | A.增加眼的折光能力 B.可增加球面像差 C.增加入眼的光量 D.增加视觉的清晰度 E.增加色像差 |
D.增加视觉的清晰度 |
2 | 3 | 2011 | 39. 人眼近点的远近主要决定于(2.0分) | A.瞳孔的直径 B.晶状体的弹性 C.角膜的曲度 D.空气与角膜接触的界面 E.眼球的前后径 |
B.晶状体的弹性 |
2 | 3 | 2011 | 40. 关于视紫红质的特性,错的是(2.0分) | A.只存在于视杆细胞中 B.是一种结合蛋白质 C.在暗处只有合成 D.对光的敏感度与其含量呈正比 E.亮处分解,暗处合成是一个可逆反应 |
C.在暗处只有合成 |
2 | 3 | 2011 | 41. 关于视杆系统的描述,错误的是 (2.0分) | A.由视杆细胞、双极细胞、节细胞等组成 B.对光敏感度较高 C.能分辨颜色、司昼光觉 D.分辨力较低 E.感受弱光刺激 |
C.能分辨颜色、司昼光觉 |
2 | 3 | 2011 | 42. 为了改善周围神经损伤的疗效,下列哪项研究是有益的? (2.0分) | A.促进神经芽生的速度 B.改善桥接通道的通过能力 C.延缓神经功能效应器的退变 D.妥善合理的功能重建 E.以上都是 |
E.以上都是 |
2 | 3 | 2011 | 43. 关于骨折的临床表现,以下哪项是正确的? (2.0分) | A.骨折都有较明显的外伤史 B.骨折有时可缺乏骨折的专有体征 C.为明确骨折的诊断,应着重检查肢体有无异常活动、骨擦音或骨擦感等骨折的专有体征 D.骨折后可有吸收热,一般在38-39C?。 E.以上都不是 |
B.骨折有时可缺乏骨折的专有体征 |
2 | 3 | 2011 | 44. 下列哪一项不是肱骨干骨折合并桡神经损伤的表现? (2.0分) | A.腕关节下垂 B.各手指掌指关节不能背伸 C.爪形手 D.拇指不能背伸 E.手背桡侧皮肤感觉减退 |
C.爪形手 |
2 | 3 | 2011 | 45. 以下骨折易损伤其邻近的重要血管和神经,其中哪一部位骨折除外? (2.0分) | A.肱骨髁上骨折 B.股骨髁上骨折 C.腓骨颈骨折 D.胫骨干中段骨折 E.肱骨干中段骨折 |
D.胫骨干中段骨折 |
2 | 3 | 2011 | 46. 下列并发症在骨折早期就应注意防治? (2.0分) | A.骨筋膜室综合征 B.下肢静脉栓塞形成 C.压褥 D.感染 E.以上都是 |
E.以上都是 |
2 | 3 | 2011 | 47. 骨盆骨折可能出现哪些并发症(2.0分) | A.失血性休克 B.直肠、膀胱、尿道损伤 C.神经损伤 D.机体系统炎症反应综合征(SIRS) E.以上都是 |
E.以上都是 |
2 | 3 | 2011 | 48. 骨筋膜室综合症最主要的治疗原则是(2.0分) | A.给予血管舒张剂,消除血管痉挛 B.抬高患肢,以利消肿 C.被动按摩,以利消肿 D.臂从麻醉,解除血管痉挛 E.解除包扎及固定物,经观察无好转,切开筋膜减压 |
E.解除包扎及固定物,经观察无好转,切开筋膜减压 |
2 | 3 | 2011 | 49. 关于DCO,以下哪一项不合适? (2.0分) | A.对ISS评分>18分的多发骨折患者,应遵循DCO的原则 B.对ISS评分>20分的多发骨折患者,尽量对骨折进行确定性固定 C.外固定架固定是 DCO的一个常用方法 D.股骨、骨盆等大骨折的尽早稳定有助于抗休克 E.以上都准确 |
C.外固定架固定是 DCO的一个常用方法 |
2 | 3 | 2011 | 50. 关于骨折治疗以下哪一项是不妥的? (2.0分) | A.治疗原则是复位、固定、功能锻炼 B.骨折急救处理时重要的措施是妥善固定 C.肢体开放骨折现场止血的首选是止血带结扎止血。 D.开放骨折的创口在条件许可时应争取早期闭合覆盖。 E.夹板或石膏外固定后应注意血液循环。 |
C.肢体开放骨折现场止血的首选是止血带结扎止血。 |
2 | 3 | 2012 | 1. Epinephrine不适用于以下哪一种情况? (2.0分) | A.心脏骤停 B.充血性心力衰竭 C.局部止血 D.过敏性休克 E.支气管哮喘 |
B.充血性心力衰竭 |
2 | 3 | 2012 | 2. 为了延长局麻药的作用时间和减少不良反应,可配伍应用的药物是 (2.0分) | A.adrenaline B.isoproterenol C.dopamine D.noradrenaline E.ephedrine |
A.adrenaline |
2 | 3 | 2012 | 3. 急、慢性鼻炎、鼻窦炎引起鼻充血时,可用于滴鼻的药物是 (2.0分) | A.noradrenaline B.ephedrine C.isoproterenol D.adrenaline E.以上都不是 |
B.ephedrine |
2 | 3 | 2012 | 4. 治疗氯丙嗪(chlorpromazine)过量引起血压下降时,应选用的药物是 (2.0分) | A.肾上腺素 B.去甲肾上腺素 C.阿托品 D.多巴胺 E.异丙肾上腺素 |
B.去甲肾上腺素 |
2 | 3 | 2012 | 5. Epinephrine升压作用可被下列哪一类药物所翻转? (2.0分) | A.M受体阻断药 B.N受体阻断药 C.β受体阻断药 D.α受体阻断药 E.H1受体阻断药 |
D.α受体阻断药 |
2 | 3 | 2012 | 6. Phentolamine的不良反应有 (2.0分) | A.增高血压 B.诱发溃疡病 C.抑制呼吸中枢 D.静脉给药可引起心动过缓 E.静脉给药发生外漏时引起皮肤缺血坏死 |
B.诱发溃疡病 |
2 | 3 | 2012 | 7. Propranolol不具有下列哪一项作用? (2.0分) | A.扩张支气管平滑肌 B.降低心肌耗氧量 C.减慢窦性心率 D.延缓用胰岛素后血糖的恢复 E.抑制肾素释放 |
A.扩张支气管平滑肌 |
2 | 3 | 2012 | 8. β受体阻断药禁用于支气管哮喘是由于 (2.0分) | A.阻断支气管β2受体 B.阻断支气管β1受体 C.阻断支气管α受体 D.阻断支气管M受体 E.激动支气管M受体 |
A.阻断支气管β2受体 |
2 | 3 | 2012 | 9. 既可阻断α受体又可阻断β受体的药物是 (2.0分) | A.拉贝洛尔 B.普萘洛尔 C.阿替洛尔 D.酚妥拉明 E.哌唑嗪 |
A.拉贝洛尔 |
2 | 3 | 2012 | 10. 感觉的产生过程中,不包括的是(2.0分) | A.感受器 B.神经元 C.中枢 D.肌肉 |
D.肌肉 |
2 | 3 | 2012 | 11. 感受器能把刺激所包含的环境变化信息,转移到动作电位的序列之中,感受器的 这种生理特性称为 (2.0分) | A.感受器的适宜刺激 B.感受器的换能作用 C.感受器的编码作用 D.感受器的适应现象 E.感受器的启动电位 |
C.感受器的编码作用 |
2 | 3 | 2012 | 12. 以下属于丘脑特异性投射系统的核团是 (2.0分) | A.联络核 B.网状核 C.感觉接替核 D.髓板内核群 E.感觉接替核和联络核 |
E.感觉接替核和联络核 |
2 | 3 | 2012 | 13. 内侧膝状体接受下列哪种感觉纤维投射? (2.0分) | A.内侧丘系 B.三叉丘系 C.视觉传导束 D.听觉传导束 E.脊髓丘脑前束 |
D.听觉传导束 |
2 | 3 | 2012 | 14. 视觉皮层代表区位于(2.0分) | A.颞叶的颞横回 B.颞叶的颞上回 C.中央后回 D.扣带回 E.枕叶距状裂 |
E.枕叶距状裂 |
2 | 3 | 2012 | 15. 以下属于丘脑非特异投射系统的主要细胞群是 (2.0分) | A.联络核和髓板内核群 B.感觉接替核和联络核 C.感觉接替核 D.联络核 E.髓板内核群 |
E.髓板内核群 |
2 | 3 | 2012 | 16. 脊髓的何种损伤可导致痛温觉与轻触觉障碍分离? (2.0分) | A.完全横断 B.脊髓空洞症 C.脊髓离断 D.后角受损 E.前根病变 |
B.脊髓空洞症 |
2 | 3 | 2012 | 17. 第一感觉区位于 (2.0分) | A.中央前回 B.中央后回 C.颞叶皮层 D.中央前回与岛叶之间 E.颞下回 |
B.中央后回 |
2 | 3 | 2012 | 18. 下列属于副交感神经的作用的是 (2.0分) | A.瞳孔扩大 B.糖原分解增加 C.逼尿肌收缩 D.骨骼肌血管舒张 E.消化道括约肌收缩 |
C.逼尿肌收缩 |
2 | 3 | 2012 | 19. 交感神经系统不具有的特点是 (2.0分) | A.节前纤维长,节后纤维短 B.支配几乎所有的脏器 C.紧张性活动 D.在应急反应中活动明显加强 |
A.节前纤维长,节后纤维短 |
2 | 3 | 2012 | 20. 下列不属于交感神经兴奋作用的是 (2.0分) | A.心跳加快 B.瞳孔开大 C.支气管平滑肌舒张 D.胃肠平滑肌收缩 E.肾上腺髓质分泌肾上腺素 |
D.胃肠平滑肌收缩 |
2 | 3 | 2012 | 21. 下列关于感觉皮层代表区的叙述,错误的是 (2.0分) | A.本体感觉在中央前回 B.内脏感觉的投射区在额叶 C.体表感觉区在中央后回 D.听觉代表区在颞叶 E.视觉代表区在枕叶 |
B.内脏感觉的投射区在额叶 |
2 | 3 | 2012 | 22. 下丘脑是皮层下 (2.0分) | A.重要的感觉中枢 B.重要的运动中枢 C.较高级的交感中枢 D.较高级的副交感中枢 E.较高级的内脏活动调节中枢 |
E.较高级的内脏活动调节中枢 |
2 | 3 | 2012 | 23. 非条件反射的特点不包括 (2.0分) | A.生来具有 B.无需皮层参与 C.反射弧固定 D.反射形式多变 E.数量有限 |
D.反射形式多变 |
2 | 3 | 2012 | 24. 房水的主要功能是 (2.0分) | A.折光成像 B.对玻璃体有保护作用 C.对角膜和晶状体有营养作用 D.对视网膜有保护作用 E.促进睫状肌收缩 |
C.对角膜和晶状体有营养作用 |
2 | 3 | 2012 | 25. 不引起生理性瞳孔缩小的是 (2.0分) | A.光反射 B.近反射 C.调节反射 D.辐辏反射 E.惊吓反应 |
E.惊吓反应 |
2 | 3 | 2012 | 26. 有关视力、视角的论述,正确的是 (2.0分) | A.视力大小与视角大小成正比 B.视角一分角能看清物体时,视力为 1.0 C.视锥细胞直径越大,视力越好 D.双眼视物时可以使视力提高 E.同一距离物体越大形成的视角就越小 |
B.视角一分角能看清物体时,视力为 1.0 |
2 | 3 | 2012 | 27. 瞳孔在弱光下散大,而在强光下缩小,称为(2.0分) | A.互感性对光反射 B.辐辏反射 C.瞳孔对光反射 D.瞳孔调节反射 E.瞳孔近反射 |
C.瞳孔对光反射 |
2 | 3 | 2012 | 28. 人眼的调节功能主要是由以下哪种变化来实现的? (2.0分) | A.角膜的曲度变化 B.角膜和晶状体的曲度变化 C.晶状体的曲度变化 D.玻璃体的曲度变化 E.晶状体和玻璃体的曲度变化 |
C.晶状体的曲度变化 |
2 | 3 | 2012 | 29. 下列哪种非正视眼矫正用凹透镜? (2.0分) | A.近视眼 B.远视眼 C.散光眼 D.老视眼 E.老花眼 |
A.近视眼 |
2 | 3 | 2012 | 30. 关于瞳孔反射的说明,正确的是 (2.0分) | A.视近物时瞳孔扩大 B.瞳孔近反射与对光反射的通路相同 C.动眼神经损伤使瞳孔缩小 D.交感神经兴奋使瞳孔扩大 E.副交感神经兴奋使瞳孔扩大 |
D.交感神经兴奋使瞳孔扩大 |
2 | 3 | 2012 | 31. 眼尽最大能力调节时所能看清物体的最近距离,称为 (2.0分) | A.节点 B.前主焦点 C.远点 D.近点 E.后主焦点 |
D.近点 |
2 | 3 | 2012 | 32. 瞳孔近反射的作用是 (2.0分) | A.增加眼的折光能力 B.可增加球面像差 C.增加入眼的光量 D.增加视觉的清晰度 E.增加色像差 |
D.增加视觉的清晰度 |
2 | 3 | 2012 | 33. 关于视杆系统的描述,错误的是(2.0分) | A.由视杆细胞、双极细胞、节细胞等组成 B.对光敏感度较高 C.能分辨颜色、司昼光觉 D.分辨力较低 E.感受弱光刺激 |
C.能分辨颜色、司昼光觉 |
2 | 3 | 2012 | 34. 人眼近点的远近主要决定于(2.0分) | A.瞳孔的直径 B.晶状体的弹性 C.角膜的曲度 D.空气与角膜接触的界面 E.眼球的前后径 |
B.晶状体的弹性 |
2 | 3 | 2012 | 35. 关于视紫红质的特性,错的是 (2.0分) | A.只存在于视杆细胞中 B.是一种结合蛋白质 C.在暗处只有合成 D.对光的敏感度与其含量呈正比 E.亮处分解,暗处合成是一个可逆反应 |
C.在暗处只有合成 |
2 | 3 | 2012 | 36. 为了改善周围神经损伤的疗效,下列哪项研究是有益的? (2.0分) | A.促进神经芽生的速度 B.改善桥接通道的通过能力 C.延缓神经功能效应器的退变 D.妥善合理的功能重建 E.以上都是 |
E.以上都是 |
2 | 3 | 2012 | 37. 关于骨折的临床表现,以下哪项是正确的? (2.0分) | A.骨折都有较明显的外伤史 B.骨折有时可缺乏骨折的专有体征 C.为明确骨折的诊断,应着重检查肢体有无异常活动、骨擦音或骨擦感等骨折的专有体征 D.骨折后可有吸收热,一般在38-39C?。 E.以上都不是 |
B.骨折有时可缺乏骨折的专有体征 |
2 | 3 | 2012 | 38. 以下骨折易损伤其邻近的重要血管和神经,其中哪一部位骨折除外? (2.0分) | A.肱骨髁上骨折 B.股骨髁上骨折 C.腓骨颈骨折 D.胫骨干中段骨折 E.肱骨干中段骨折 |
D.胫骨干中段骨折 |
2 | 3 | 2012 | 39. 下列并发症在骨折早期就应注意防治? (2.0分) | A.骨筋膜室综合征 B.下肢静脉栓塞形成 C.压褥 D.感染 E.以上都是 |
E.以上都是 |
2 | 3 | 2012 | 40. 下列哪一项不是肱骨干骨折合并桡神经损伤的表现? (2.0分) | A.腕关节下垂 B.各手指掌指关节不能背伸 C.爪形手 D.拇指不能背伸 E.手背桡侧皮肤感觉减退 |
C.爪形手 |
2 | 3 | 2012 | 41. 骨筋膜室综合症最主要的治疗原则是(2.0分) | A.给予血管舒张剂,消除血管痉挛 B.抬高患肢,以利消肿 C.被动按摩,以利消肿 D.臂从麻醉,解除血管痉挛 E.解除包扎及固定物,经观察无好转,切开筋膜减压 |
E.解除包扎及固定物,经观察无好转,切开筋膜减压 |
2 | 3 | 2012 | 42. 骨盆骨折可能出现哪些并发症_______。 (2.0分) | A.失血性休克 B.直肠、膀胱、尿道损伤 C.神经损伤 D.机体系统炎症反应综合征(SIRS) E.以上都是 |
E.以上都是 |
2 | 3 | 2012 | 43. 关于DCO,以下哪一项不合适? (2.0分) | A.对ISS评分>18分的多发骨折患者,应遵循DCO的原则 B.对ISS评分>20分的多发骨折患者,尽量对骨折进行确定性固定 C.外固定架固定是 DCO的一个常用方法 D.股骨、骨盆等大骨折的尽早稳定有助于抗休克 E.以上都准确 |
C.外固定架固定是 DCO的一个常用方法 |
2 | 3 | 2012 | 44. 关于骨折治疗以下哪一项是不妥的? (2.0分) | A.治疗原则是复位、固定、功能锻炼 B.骨折急救处理时重要的措施是妥善固定 C.肢体开放骨折现场止血的首选是止血带结扎止血。 D.开放骨折的创口在条件许可时应争取早期闭合覆盖。 E.夹板或石膏外固定后应注意血液循环。 |
C.肢体开放骨折现场止血的首选是止血带结扎止血。 |
2 | 3 | 2012 | 45. 衡量组织兴奋性高低的指标是 (2.0分) | A.静息电位水平 B.阈电位 C.阈强度 D.动作电位幅度 E.兴奋扩布速度 |
C.阈强度 |
2 | 3 | 2012 | 46. 临床上普鲁卡因局部麻醉药的应用是由于阻断了局部神经冲动的传导,其神经细胞发生的变化是(2.0分) | A.细胞变性 B.结构完整性破坏 C.功能完整性破坏 D.细胞膜电压门控钾通道破坏 E.细胞膜化学门控钠通道破坏 |
C.功能完整性破坏 |
2 | 3 | 2012 | 47. 在神经‐骨骼肌接头中消除乙酰胆碱的酶是(2.0分) | A.ATP 酶 B.胆碱酯酶 C.腺苷酸环化酶 D.磷酸二酯酶 E.单胺氧化酶 |
B.胆碱酯酶 |
2 | 3 | 2012 | 48. 细胞膜上不衰减形式传播的电活动是(2.0分) | A.动作电位 B.静息电位 C.终板电位 D.感受器电位 E.突触后电位 |
A.动作电位 |
2 | 3 | 2012 | 49. 实验中,如果同时刺激神经纤维的两端,产生的两个动作电位 (2.0分) | A.将各自通过中点后传导到另一端 B.将在中点相遇,然后传回到起始点 C.只有较强的动作电位通过中点而到达另一端 D.将在中点相遇后停止传导 E.到达中点后将复合成一个更大的动作电位 |
D.将在中点相遇后停止传导 |
2 | 3 | 2012 | 50. 与propranolol相比,metoprolol的特点是 (2.0分) | A.抑制糖原分解 B.对哮喘病人慎用 C.用于治疗高血压病 D.对β1受体有选择性 E.没有内在拟交感活性 |
D.对β1受体有选择性 |
2 | 3 | 2013 | 1. Epinephrine不适用于以下哪一种情况?(2.0分) | A.心脏骤停 B.充血性心力衰竭 C.局部止血 D.过敏性休克 E.支气管哮喘 |
B.充血性心力衰竭 |
2 | 3 | 2013 | 2. 为了延长局麻药的作用时间和减少不良反应,可配伍应用的药物是 (2.0分) | A.adrenaline B.isoproterenol C.dopamine D.noradrenaline E.ephedrine |
A.adrenaline |
2 | 3 | 2013 | 3. 急、慢性鼻炎、鼻窦炎引起鼻充血时,可用于滴鼻的药物是 (2.0分) | A.noradrenaline B.ephedrine C.isoproterenol D.adrenaline E.以上都不是 |
B.ephedrine |
2 | 3 | 2013 | 4. 治疗氯丙嗪(chlorpromazine)过量引起血压下降时,应选用的药物是(2.0分) | A.肾上腺素 B.去甲肾上腺素 C.阿托品 D.多巴胺 E.异丙肾上腺素 |
B.去甲肾上腺素 |
2 | 3 | 2013 | 5. Epinephrine升压作用可被下列哪一类药物所翻转? (2.0分) | A.M受体阻断药 B.N受体阻断药 C.β受体阻断药 D.α受体阻断药 E.H1受体阻断药 |
D.α受体阻断药 |
2 | 3 | 2013 | 6. Phentolamine的不良反应有 (2.0分) | A.增高血压 B.诱发溃疡病 C.抑制呼吸中枢 D.静脉给药可引起心动过缓 E.静脉给药发生外漏时引起皮肤缺血坏死 |
B.诱发溃疡病 |
2 | 3 | 2013 | 7. Propranolol不具有下列哪一项作用? (2.0分) | A.扩张支气管平滑肌 B.降低心肌耗氧量 C.减慢窦性心率 D.延缓用胰岛素后血糖的恢复 E.抑制肾素释放 |
A.扩张支气管平滑肌 |
2 | 3 | 2013 | 8. β受体阻断药禁用于支气管哮喘是由于 (2.0分) | A.阻断支气管β2受体 B.阻断支气管β1受体 C.阻断支气管α受体 D.阻断支气管M受体 E.激动支气管M受体 |
A.阻断支气管β2受体 |
2 | 3 | 2013 | 9. 既可阻断α受体又可阻断β受体的药物是 (2.0分) | A.拉贝洛尔 B.普萘洛尔 C.阿替洛尔 D.酚妥拉明 E.哌唑嗪 |
A.拉贝洛尔 |
2 | 3 | 2013 | 10. 感觉的产生过程中,不包括的是(2.0分) | A.感受器 B.神经元 C.中枢 D.肌肉 |
D.肌肉 |
2 | 3 | 2013 | 11. 感受器能把刺激所包含的环境变化信息,转移到动作电位的序列之中,感受器的这种生理特性称为 (2.0分) | A.感受器的适宜刺激 B.感受器的换能作用 C.感受器的编码作用 D.感受器的适应现象 E.感受器的启动电位 |
C.感受器的编码作用 |
2 | 3 | 2013 | 12. 以下属于丘脑特异性投射系统的核团是 (2.0分) | A.联络核 B.网状核 C.感觉接替核 D.髓板内核群 E.感觉接替核和联络核 |
E.感觉接替核和联络核 |
2 | 3 | 2013 | 13. 内侧膝状体接受下列哪种感觉纤维投射? (2.0分) | A.内侧丘系 B.三叉丘系 C.视觉传导束 D.听觉传导束 E.脊髓丘脑前束 |
D.听觉传导束 |
2 | 3 | 2013 | 14. 视觉皮层代表区位于 (2.0分) | A.颞叶的颞横回 B.颞叶的颞上回 C.中央后回 D.扣带回 E.枕叶距状裂 |
E.枕叶距状裂 |
2 | 3 | 2013 | 15. 以下属于丘脑非特异投射系统的主要细胞群是 (2.0分) | A.联络核和髓板内核群 B.感觉接替核和联络核 C.感觉接替核 D.联络核 E.髓板内核群 |
E.髓板内核群 |
2 | 3 | 2013 | 16. 脊髓的何种损伤可导致痛温觉与轻触觉障碍分离? (2.0分) | A.完全横断 B.脊髓空洞症 C.脊髓离断 D.后角受损 E.前根病变 |
B.脊髓空洞症 |
2 | 3 | 2013 | 17. 第一感觉区位于(2.0分) | A.中央前回 B.中央后回 C.颞叶皮层 D.中央前回与岛叶之间 E.颞下回 |
B.中央后回 |
2 | 3 | 2013 | 18. 下列属于副交感神经的作用的是 (2.0分) | A.瞳孔扩大 B.糖原分解增加 C.逼尿肌收缩 D.骨骼肌血管舒张 E.消化道括约肌收缩 |
C.逼尿肌收缩 |
2 | 3 | 2013 | 19. 交感神经系统不具有的特点是 (2.0分) | A.节前纤维长,节后纤维短 B.支配几乎所有的脏器 C.紧张性活动 D.在应急反应中活动明显加强 |
A.节前纤维长,节后纤维短 |
2 | 3 | 2013 | 20. 下列不属于交感神经兴奋作用的是 (2.0分) | A.心跳加快 B.瞳孔开大 C.支气管平滑肌舒张 D.胃肠平滑肌收缩 E.肾上腺髓质分泌肾上腺素 |
D.胃肠平滑肌收缩 |
2 | 3 | 2013 | 21. 下列关于感觉皮层代表区的叙述,错误的是 (2.0分) | A.本体感觉在中央前回 B.内脏感觉的投射区在额叶 C.体表感觉区在中央后回 D.听觉代表区在颞叶 E.视觉代表区在枕叶 |
B.内脏感觉的投射区在额叶 |
2 | 3 | 2013 | 22. 下丘脑是皮层下 (2.0分) | A.重要的感觉中枢 B.重要的运动中枢 C.较高级的交感中枢 D.较高级的副交感中枢 E.较高级的内脏活动调节中枢 |
E.较高级的内脏活动调节中枢 |
2 | 3 | 2013 | 23. 非条件反射的特点不包括 (2.0分) | A.生来具有 B.无需皮层参与 C.反射弧固定 D.反射形式多变 E.数量有限 |
D.反射形式多变 |
2 | 3 | 2013 | 24. 上斜肌由哪条神经支配,上斜肌使眼球往哪个方向转动? (2.0分) | A.动眼神经,司眼球向下和稍向外转动 B.动眼神经,司眼球向上和稍向外转动 C.滑车神经,司眼球向上和稍向内转动 D.动眼神经,司眼球向下和稍向内转动 E.滑车神经,司眼球向下和稍向外转动 |
E.滑车神经,司眼球向下和稍向外转动 |
2 | 3 | 2013 | 25. 只受单侧(对侧)皮质脑干束支配的核团,除支配眼裂以上的面神经核团,还有: (2.0分) | A.三叉神经运动核 B.迷走神经背侧核 C.迷走神经疑核 D.舌下神经核 E.动眼神经核 |
D.舌下神经核 |
2 | 3 | 2013 | 26. 伸舌时,舌尖偏向病灶对侧,这是哪个部位损害(2.0分) | A.舌下神经核病变 B.舌下神经核上性病变 C.面神经核病变 D.面神经病变 E.舌下神经病变 |
B.舌下神经核上性病变 |
2 | 3 | 2013 | 27. 患者出现左侧偏盲,但存在黄斑回避现象,请问患者视通路的哪一部分发生损害? (2.0分) | A.左侧视束 B.右侧视束 C.右侧视放射 D.左侧枕叶中枢 E.右侧枕叶中枢 |
E.右侧枕叶中枢 |
2 | 3 | 2013 | 28. 男,62岁,因左侧面颊部呈发作性烧灼痛三个月就诊。每次发作持续10余分钟。体格检查:左侧眼眶下面颊部痛觉减退,咀嚼肌无力,角膜反射似较右侧迟钝,下列哪项处理是正确的: (2.0分) | A.积极查找病因 B.首选卡马西平口服 C.对症处理 D.手术切断三叉神经周围支 E.神经营养治疗 |
A.积极查找病因 |
2 | 3 | 2013 | 29. 患者出现双颞侧视野偏盲,颅脑MRI显示:垂体占位性病变,则该患者视通路的哪一部分受到压迫?(2.0分) | A.左侧视神经 B.整个视交叉 C.左侧视束 D.左侧视辐射 E.视交叉正中部 |
E.视交叉正中部 |
2 | 3 | 2013 | 30. 关于核上性眼肌麻痹,以下说法错误的是(2.0分) | A.双眼同时受累 B.无复视 C.反射性运动仍保存 D.刺激性病灶引起双侧眼球向对侧凝视 E.破坏性病灶引起双侧眼球向对侧凝视 |
E.破坏性病灶引起双侧眼球向对侧凝视 |
2 | 3 | 2013 | 31. 下列说法,哪些是错误的?(2.0分) | A.脑桥侧视中枢与动眼神经核的上行纤维损害,会出现两眼向病变对侧注视时,患侧眼球不能内收,对侧眼球外展时伴有眼震;辐辏反射正常。 B.一侧动眼神经完全性麻痹时,表现为上睑下垂,眼球向外上,瞳孔散大,光反射及调节反射均消失,并有复视。 C.两眼向病变同侧注视时,患侧眼球不能外展,对侧眼球内收正常,为后核间性眼肌麻痹。 D.一个半综合征,是一侧脑桥被盖部病变,引起患者眼球水平注视时,既不能内收又不能外展;对侧眼球水平注视时不能内收,可以外展,但有水平眼震。 E.上丘是眼球垂直同向运动的皮质下中枢,上丘的上半司眼球的向上运动,下半司眼球的向下运动。 |
B.一侧动眼神经完全性麻痹时,表现为上睑下垂,眼球向外上,瞳孔散大,光反射及调节反射均消失,并有复视。 |
2 | 3 | 2013 | 32. 以下不是前庭周围性眩晕特点的是(2.0分) | A.前庭周围性眩晕的病变部位是内耳前庭感受器及前庭神经病变 B.呈发作性,症状较重,持续时间较短 C.常伴耳鸣、听力减退、恶心、呕吐等 D.前庭功能试验无反应或反应减弱 E.眼震幅度粗大、眼震形式多变 |
E.眼震幅度粗大、眼震形式多变 |
2 | 3 | 2013 | 33. 舌咽神经司下列哪个腺体的分泌?(2.0分) | A.泪腺 B.舌下腺 C.颌下腺 D.腮腺 E.唾液腺 |
D.腮腺 |
2 | 3 | 2013 | 34. 发生脊髓损害后,肢体痛、温觉消失,是由于病变侵犯(2.0分) | A.皮质脊髓侧束 B.脊髓丘脑侧束 C.薄束与楔束 D.红核脊髓束 E.脊髓小脑束 |
B.脊髓丘脑侧束 |
2 | 3 | 2013 | 35. 阿罗瞳孔的病变部位在:(2.0分) | A.睫状神经节 B.外侧膝状体 C.中脑顶盖前区 D.动眼神经 E.视神经 |
C.中脑顶盖前区 |
2 | 3 | 2013 | 36. 为了改善周围神经损伤的疗效,下列哪项研究是有益的? (2.0分) | A.促进神经芽生的速度 B.改善桥接通道的通过能力 C.延缓神经功能效应器的退变 D.妥善合理的功能重建 E.以上都是 |
E.以上都是 |
2 | 3 | 2013 | 37. 关于骨折的临床表现,以下哪项是正确的? (2.0分) | A.骨折都有较明显的外伤史 B.骨折有时可缺乏骨折的专有体征 C.为明确骨折的诊断,应着重检查肢体有无异常活动、骨擦音或骨擦感等骨折的专有体征 D.骨折后可有吸收热,一般在38-39C?。 E.以上都不是 |
B.骨折有时可缺乏骨折的专有体征 |
2 | 3 | 2013 | 38. 以下骨折易损伤其邻近的重要血管和神经,其中哪一部位骨折除外? (2.0分) | A.肱骨髁上骨折 B.股骨髁上骨折 C.腓骨颈骨折 D.胫骨干中段骨折 E.肱骨干中段骨折 |
D.胫骨干中段骨折 |
2 | 3 | 2013 | 39. 下列并发症在骨折早期就应注意防治? (2.0分) | A.骨筋膜室综合征 B.下肢静脉栓塞形成 C.压褥 D.感染 E.以上都是 |
E.以上都是 |
2 | 3 | 2013 | 40. 下列哪一项不是肱骨干骨折合并桡神经损伤的表现? (2.0分) | A.腕关节下垂 B.各手指掌指关节不能背伸 C.爪形手 D.拇指不能背伸 E.手背桡侧皮肤感觉减退 |
C.爪形手 |
2 | 3 | 2013 | 41. 骨筋膜室综合症最主要的治疗原则是(2.0分) | A.给予血管舒张剂,消除血管痉挛 B.抬高患肢,以利消肿 C.被动按摩,以利消肿 D.臂从麻醉,解除血管痉挛 E.解除包扎及固定物,经观察无好转,切开筋膜减压 |
E.解除包扎及固定物,经观察无好转,切开筋膜减压 |
2 | 3 | 2013 | 42. 骨盆骨折可能出现哪些并发症_______。(2.0分) | A.失血性休克 B.直肠、膀胱、尿道损伤 C.神经损伤 D.机体系统炎症反应综合征(SIRS) E.以上都是 |
E.以上都是 |
2 | 3 | 2013 | 43. 关于DCO,以下哪一项不合适? (2.0分) | A.对ISS评分>18分的多发骨折患者,应遵循DCO的原则 B.对ISS评分>20分的多发骨折患者,尽量对骨折进行确定性固定 C.外固定架固定是 DCO的一个常用方法 D.股骨、骨盆等大骨折的尽早稳定有助于抗休克 E.以上都准确 |
C.外固定架固定是 DCO的一个常用方法 |
2 | 3 | 2013 | 44. 关于骨折治疗以下哪一项是不妥的? (2.0分) | A.治疗原则是复位、固定、功能锻炼 B.骨折急救处理时重要的措施是妥善固定 C.肢体开放骨折现场止血的首选是止血带结扎止血。 D.开放骨折的创口在条件许可时应争取早期闭合覆盖。 E.夹板或石膏外固定后应注意血液循环。 |
C.肢体开放骨折现场止血的首选是止血带结扎止血。 |
2 | 3 | 2013 | 45. 实验中,如果同时刺激神经纤维的两端,产生的两个动作电位(2.0分) | A.将各自通过中点后传导到另一端 B.将在中点相遇,然后传回到起始点 C.只有较强的动作电位通过中点而到达另一端 D.将在中点相遇后停止传导 E.到达中点后将复合成一个更大的动作电位 |
D.将在中点相遇后停止传导 |
2 | 3 | 2013 | 46. 与propranolol相比,metoprolol的特点是 (2.0分) | A.抑制糖原分解 B.对哮喘病人慎用 C.用于治疗高血压病 D.对β1受体有选择性 E.没有内在拟交感活性 |
D.对β1受体有选择性 |
2 | 3 | 2013 | 47. 感觉平面有助于判断病损部位,可据体表判断受损节段,下腹壁反射对应的脊髓反射中枢位于哪个节段?(2.0分) | A.T5-6 B.T7-8 C.T6-7 D.T9-10 E.T11-12 |
E.T11-12 |
2 | 3 | 2013 | 48. 脊髓空洞症的感觉障碍是(2.0分) | A.节段型 B.传导束型 C.分离性节段型 D.混合型 E.以上均不是 |
C.分离性节段型 |
2 | 3 | 2013 | 49. 周围神经受累感觉障碍呈(2.0分) | A.节段型感觉障碍 B.节段分离性感觉障碍 C.偏身感觉障碍 D.末梢型感觉障碍 E.运动功能障碍 |
D.末梢型感觉障碍 |
2 | 3 | 2013 | 50. 下列关于Bell麻痹不正确的是(2.0分) | A.Bell麻痹具有自限性。 B.急性起病,病情多在3d左右达到高峰。 C.发病常为双侧。 D.闭目时瘫痪侧眼球转向内上方,露出角膜下的白色巩膜,称Bell现象。 E.在发病1-2周后,面神经传导测定复合肌肉动作电位波幅不足健侧10%,恢复较差。 |
C.发病常为双侧。 |
2 | 3 | 2014 | 1. Epinephrine不适用于以下哪一种情况?(2.0分) | A.心脏骤停 B.充血性心力衰竭 C.局部止血 D.过敏性休克 E.支气管哮喘 |
B.充血性心力衰竭 |
2 | 3 | 2014 | 2. 为了延长局麻药的作用时间和减少不良反应,可配伍应用的药物是 (2.0分) | A.adrenaline B.isoproterenol C.dopamine D.noradrenaline E.ephedrine |
A.adrenaline |
2 | 3 | 2014 | 3. 急、慢性鼻炎、鼻窦炎引起鼻充血时,可用于滴鼻的药物是 (2.0分) | A.noradrenaline B.ephedrine C.isoproterenol D.adrenaline E.以上都不是 |
B.ephedrine |
2 | 3 | 2014 | 4. 治疗氯丙嗪(chlorpromazine)过量引起血压下降时,应选用的药物是 (2.0分) | A.肾上腺素 B.去甲肾上腺素 C.阿托品 D.多巴胺 E.异丙肾上腺素 |
B.去甲肾上腺素 |
2 | 3 | 2014 | 5. Dopamine可用于治疗(2.0分) | A.伴有心输出量和尿量减少的休克病人 B.Penicillin G引起的过敏性休克 C.心源性哮喘 D.支气管哮喘 E.缓慢型心律失常 |
A.伴有心输出量和尿量减少的休克病人 |
2 | 3 | 2014 | 6. 多巴胺使肾和肠系膜血管扩张的原因是(2.0分) | A.兴奋β受体 B.兴奋?受体 C.兴奋M受体 D.兴奋DA受体 E.兴奋N受体 |
D.兴奋DA受体 |
2 | 3 | 2014 | 7. Epinephrine升压作用可被下列哪一类药物所翻转? (2.0分) | A.M受体阻断药 B.N受体阻断药 C.β受体阻断药 D.α受体阻断药 E.H1受体阻断药 |
D.α受体阻断药 |
2 | 3 | 2014 | 8. Phentolamine的不良反应有 (2.0分) | A.增高血压 B.诱发溃疡病 C.抑制呼吸中枢 D.静脉给药可引起心动过缓 E.静脉给药发生外漏时引起皮肤缺血坏死 |
B.诱发溃疡病 |
2 | 3 | 2014 | 9. Propranolol不具有下列哪一项作用? (2.0分) | A.扩张支气管平滑肌 B.降低心肌耗氧量 C.减慢窦性心率 D.延缓用胰岛素后血糖的恢复 E.抑制肾素释放 |
A.扩张支气管平滑肌 |
2 | 3 | 2014 | 10. β受体阻断药禁用于支气管哮喘是由于 (2.0分) | A.阻断支气管β2受体 B.阻断支气管β1受体 C.阻断支气管α受体 D.阻断支气管M受体 E.激动支气管M受体 |
A.阻断支气管β2受体 |
2 | 3 | 2014 | 11. 既可阻断α受体又可阻断β受体的药物是 (2.0分) | A.拉贝洛尔 B.普萘洛尔 C.阿替洛尔 D.酚妥拉明 E.哌唑嗪 |
A.拉贝洛尔 |
2 | 3 | 2014 | 12. 感觉的产生过程中,不包括的是(2.0分) | A.感受器 B.神经元 C.中枢 D.肌肉 |
D.肌肉 |
2 | 3 | 2014 | 13. 感受器能把刺激所包含的环境变化信息,转移到动作电位的序列之中,感受器的这种生理特性称为 (2.0分) | A.感受器的适宜刺激 B.感受器的换能作用 C.感受器的编码作用 D.感受器的适应现象 E.感受器的启动电位 |
C.感受器的编码作用 |
2 | 3 | 2014 | 14. 脊髓的何种损伤可导致痛温觉与轻触觉障碍分离? (2.0分) | A.完全横断 B.脊髓空洞症 C.脊髓离断 D.后角受损 E.前根病变 |
B.脊髓空洞症 |
2 | 3 | 2014 | 15. 以下属于丘脑特异性投射系统的核团是 (2.0分) | A.联络核 B.网状核 C.感觉接替核 D.髓板内核群 E.感觉接替核和联络核 |
E.感觉接替核和联络核 |
2 | 3 | 2014 | 16. 以下属于丘脑非特异投射系统的主要细胞群是(2.0分) | A.联络核和髓板内核群 B.感觉接替核和联络核 C.感觉接替核 D.联络核 E.髓板内核群 |
E.髓板内核群 |
2 | 3 | 2014 | 17. 内侧膝状体接受下列哪种感觉纤维投射? (2.0分) | A.内侧丘系 B.三叉丘系 C.C.视觉传导束 D.D.听觉传导束 E.E.脊髓丘脑前束 |
D.D.听觉传导束 |
2 | 3 | 2014 | 18. 以下哪种离子通道介导冷觉感知 (2.0分) | A.TRPM2 B.TRPV1 C.TRPV4 D.TRPM8 E.TRPC1 |
D.TRPM8 |
2 | 3 | 2014 | 19. 第一感觉区位于 (2.0分) | A.中央前回 B.中央后回 C.颞叶皮层 D.中央前回与岛叶之间 E.颞下回 |
B.中央后回 |
2 | 3 | 2014 | 20. 交感神经系统不具有的特点是 (2.0分) | A.节前纤维长,节后纤维短 B.支配几乎所有的脏器 C.紧张性活动 D.在应急反应中活动明显加强 |
A.节前纤维长,节后纤维短 |
2 | 3 | 2014 | 21. 下列属于副交感神经的作用的是 (2.0分) | A.瞳孔扩大 B.糖原分解增加 C.逼尿肌收缩 D.骨骼肌血管舒张 E.消化道括约肌收缩 |
C.逼尿肌收缩 |
2 | 3 | 2014 | 22. 下列不属于交感神经兴奋作用的是 (2.0分) | A.心跳加快 B.瞳孔开大 C.支气管平滑肌舒张 D.胃肠平滑肌收缩 E.肾上腺髓质分泌肾上腺素 |
D.胃肠平滑肌收缩 |
2 | 3 | 2014 | 23. 下列关于感觉皮层代表区的叙述,错误的是 (2.0分) | A.本体感觉在中央前回 B.内脏感觉的投射区在额叶 C.体表感觉区在中央后回 D.听觉代表区在颞叶 E.视觉代表区在枕叶 |
B.内脏感觉的投射区在额叶 |
2 | 3 | 2014 | 24. 非条件反射的特点不包括 (2.0分) | A.生来具有 B.无需皮层参与 C.反射弧固定 D.反射形式多变 E.数量有限 |
D.反射形式多变 |
2 | 3 | 2014 | 25. 感觉的最高级中枢是 (2.0分) | A.下丘脑 B.大脑皮层 C.海马 D.丘脑 E.脑干 |
D.丘脑 |
2 | 3 | 2014 | 26. 谈论梅子时引起唾液分泌是 (2.0分) | A.交感神经兴奋所致 B.副交感神经兴奋所致 C.第一信号系统的活动 D.第二信号系统的活动 E.非条件反射 |
D.第二信号系统的活动 |
2 | 3 | 2014 | 27. wallerian 变性指的是(2.0分) | A.神经断裂后,其母神经元发生凋亡。 B.神经断裂后,其近侧段轴突和髓鞘碎裂溶解。 C.神经断裂后,其远侧段轴突和髓鞘碎裂溶解。 D.神经断裂后,其支配的效应器失功能化 E.以上都不是 |
C.神经断裂后,其远侧段轴突和髓鞘碎裂溶解。 |
2 | 3 | 2014 | 28. 在周围神经损伤的治疗中,下列哪一项是正确的?(2.0分) | A.周围神经损伤的治疗原则是尽早的恢复神经的连续性。 B.闭合性神经损伤,若暴力小症状轻,可先观察和非手术治疗3个月。 C.闭合性神经损伤,若暴力大、症状重,应早期手术探查。 D.开放性神经损伤中,神经修复时间应根据损伤程度、伤后时间、有无污染及有无复合损伤等因素决定。 E.以上均正确。 |
E.以上均正确。 |
2 | 3 | 2014 | 29. 属于稳定性骨折的是(2.0分) | A.斜形骨折 B.嵌插骨折 C.螺旋形骨折 D.粉碎性骨折 E.以上都不是 |
B.嵌插骨折 |
2 | 3 | 2014 | 30. 影响骨折愈合的局部因素有以下哪一项?(2.0分) | A.骨折的部位和类型 B.软组织损伤程度 C.是否为开放骨折 D.是否存在感染 E.以上都是 |
E.以上都是 |
2 | 3 | 2014 | 31. 下列哪一项不是肱骨干骨折合并桡神经损伤的表现?(2.0分) | A.腕关节下垂 B.各手指掌指关节不能背伸 C.爪形手 D.拇指不能背伸 E.手背桡侧皮肤感觉减退 |
C.爪形手 |
2 | 3 | 2014 | 32. 以下骨折易损伤其邻近的重要血管和神经,其中哪一部位骨折除外?(2.0分) | A.肱骨髁上骨折 B.股骨髁上骨折 C.腓骨颈骨折 D.胫骨干中段骨折 E.肱骨干中段骨折 |
D.胫骨干中段骨折 |
2 | 3 | 2014 | 33. 关于骨折治疗以下哪一项是不妥的?(2.0分) | A.治疗原则是复位、固定、功能锻炼 B.骨折急救处理时重要的措施是妥善固定 C.肢体开放骨折现场止血的首选是止血带捆扎止血。 D.开放骨折的创口在条件许可时应争取早期闭合覆盖。 E.夹板或石膏外固定后应注意血液循环。 |
C.肢体开放骨折现场止血的首选是止血带捆扎止血。 |
2 | 3 | 2014 | 34. 骨盆骨折可能出现哪些并发症_______。(2.0分) | A.失血性休克 B.直肠、膀胱、尿道损伤 C.神经损伤 D.机体系统炎症反应综合征(SIRS) E.以上都是 |
E.以上都是 |
2 | 3 | 2014 | 35. 骨筋膜室综合症最主要的治疗原则是(2.0分) | A.给予血管舒张剂,消除血管痉挛 B.抬高患肢,以利消肿 C.被动按摩,以利消肿 D.臂从麻醉,解除血管痉挛 E.解除包扎及固定物,经观察无好转,切开筋膜减压 |
E.解除包扎及固定物,经观察无好转,切开筋膜减压 |
2 | 3 | 2014 | 36. 关于DCO,以下哪一项不合适?(2.0分) | A.对ISS评分>18分的多发骨折患者,应遵循DCO的原则 B.对ISS评分>20分的多发骨折患者,尽量对骨折进行确定性固定 C.外固定架固定是 DCO的一个常用方法 D.股骨、骨盆等大骨折的尽早稳定有助于抗休克 E.以上都准确 |
B.对ISS评分>20分的多发骨折患者,尽量对骨折进行确定性固定 |
2 | 3 | 2014 | 37. 患者右侧额纹存在,右侧眼睑闭合正常,右侧鼻唇沟变浅,露齿时口角偏向左侧,可能是: (2.0分) | A.右侧中枢性面神经麻痹 B.左侧中枢性面神经麻痹 C.右侧周围性面神经麻痹 D.左侧周围性面神经麻痹 E.双侧周围性面神经麻痹 |
B.左侧中枢性面神经麻痹 |
2 | 3 | 2014 | 38. 患者出现左侧偏盲,但存在黄斑回避现象,请问患者视通路的哪一部分发生损害? (2.0分) | A.左侧视束 B.右侧视束 C.右侧视放射 D.左侧枕叶中枢 E.右侧枕叶中枢 |
E.右侧枕叶中枢 |
2 | 3 | 2014 | 39. 继发性三叉神经痛与原发性三叉神经痛主要鉴别特征是(2.0分) | A.触发点的存在 B.伴有神经系统损伤的体征 C.伴有角膜炎 D.伴有口腔疾患 E.疼痛范围小 |
B.伴有神经系统损伤的体征 |
2 | 3 | 2014 | 40. 男,62岁,因左侧面颊部呈发作性烧灼痛三个月就诊。每次发作持续10余分钟。体格检查:左侧眼眶下面颊部痛觉减退,咀嚼肌无力,角膜反射似较右侧迟钝,下列哪项处理是正确的: (2.0分) | A.积极查找病因 B.首选卡马西平口服 C.对症处理 D.手术切断三叉神经周围支 E.神经营养治疗 |
A.积极查找病因 |
2 | 3 | 2014 | 41. 下列哪些属于浅感觉?(2.0分) | A.位置觉 B.两点辨别觉 C.触觉 D.实体觉 E.运动觉 |
C.触觉 |
2 | 3 | 2014 | 42. 上斜肌由哪条神经支配,上斜肌使眼球往哪个方向转动?(2.0分) | A.动眼神经,司眼球向下和稍向外转动 B.动眼神经,司眼球向上和稍向外转动 C.滑车神经,司眼球向上和稍向内转动 D.动眼神经,司眼球向下和稍向内转动 E.滑车神经,司眼球向下和稍向外转动 |
E.滑车神经,司眼球向下和稍向外转动 |
2 | 3 | 2014 | 43. 左眼的直接对光反射存在,但间接对光反射消失,右眼直接对光反射消失,间接对光反射存在,是由于损害以下哪个部位?(2.0分) | A.左侧视神经 B.左侧动眼神经 C.右侧视神经 D.右侧动眼神经 E.视交叉 |
C.右侧视神经 |
2 | 3 | 2014 | 44. 只受单侧(对侧)皮质脑干束支配的核团,除支配眼裂以上的面神经核团,还有: (2.0分) | A.三叉神经运动核 B.迷走神经背侧核 C.迷走神经疑核 D.舌下神经核 E.动眼神经核 |
D.舌下神经核 |
2 | 3 | 2014 | 45. L2平面腰髓半侧(左边)损害会引起以下症状,除了(2.0分) | A.左足Babinski征阳性 B.左下肢深感觉减退 C.左下肢浅感觉减退 D.左下肢肌张力增高 E.左下肢肌力减退 |
C.左下肢浅感觉减退 |
2 | 3 | 2014 | 46. 男性,45岁,双脚有踩棉花样感1年,睁眼尚能站稳,闭目时出现明显的站立不稳,其病变是:(2.0分) | A.脊髓侧索 B.脊髓后索 C.小脑蚓部 D.延髓外侧部 E.桥脑 |
B.脊髓后索 |
2 | 3 | 2014 | 47. 患者出现双颞侧视野偏盲,颅脑MRI显示:垂体占位性病变,则该患者视通路的哪一部分受到压迫?(2.0分) | A.左侧视神经 B.整个视交叉 C.左侧视束 D.左侧视辐射 E.视交叉正中部 |
E.视交叉正中部 |
2 | 3 | 2014 | 48. 右侧图示情况,如何定位?(2.0分) | A.动眼神经轻瘫 B.动眼神经全瘫 C.外展神经轻瘫 D.滑车神经轻瘫 E.三叉神经轻瘫 |
D.滑车神经轻瘫 |
2 | 3 | 2014 | 49. 关于核上性眼肌麻痹,以下说法错误的是(2.0分) | A.双眼同时受累 B.无复视 C.反射性运动仍保存 D.刺激性病灶引起双侧眼球向对侧凝视 E.破坏性病灶引起双侧眼球向对侧凝视 |
E.破坏性病灶引起双侧眼球向对侧凝视 |
2 | 3 | 2014 | 50. 右侧图示情况,如何定位?(2.0分) | A.左侧周围性面瘫 B.右侧周围性面瘫 C.左侧中枢性面瘫 D.右侧中枢性面瘫 E.左侧三叉神经麻痹 |
B.右侧周围性面瘫 |
2 | 3 | 2015 | 1. Epinephrine不适用于以下哪一种情况?(2.0分) | A.心脏骤停 B.充血性心力衰竭 C.局部止血 D.过敏性休克 E.支气管哮喘 |
B.充血性心力衰竭 |
2 | 3 | 2015 | 2. 为了延长局麻药的作用时间和减少不良反应,可配伍应用的药物是 (2.0分) | A.adrenaline B.isoproterenol C.dopamine D.noradrenaline E.ephedrine |
A.adrenaline |
2 | 3 | 2015 | 3. 急、慢性鼻炎、鼻窦炎引起鼻充血时,可用于滴鼻的药物是 (2.0分) | A.noradrenaline B.ephedrine C.isoproterenol D.adrenaline E.以上都不是 |
B.ephedrine |
2 | 3 | 2015 | 4. 治疗氯丙嗪(chlorpromazine)过量引起血压下降时,应选用的药物是 (2.0分) | A.肾上腺素 B.去甲肾上腺素 C.阿托品 D.多巴胺 E.异丙肾上腺素 |
B.去甲肾上腺素 |
2 | 3 | 2015 | 5. Epinephrine升压作用可被下列哪一类药物所翻转? (2.0分) | A.M受体阻断药 B.N受体阻断药 C.β受体阻断药 D.α受体阻断药 E.H1受体阻断药 |
D.α受体阻断药 |
2 | 3 | 2015 | 6. Phentolamine的不良反应有 (2.0分) | A.增高血压 B.诱发溃疡病 C.抑制呼吸中枢 D.静脉给药可引起心动过缓 E.静脉给药发生外漏时引起皮肤缺血坏死 |
B.诱发溃疡病 |
2 | 3 | 2015 | 7. Propranolol不具有下列哪一项作用? (2.0分) | A.扩张支气管平滑肌 B.降低心肌耗氧量 C.减慢窦性心率 D.延缓用胰岛素后血糖的恢复 E.抑制肾素释放 |
A.扩张支气管平滑肌 |
2 | 3 | 2015 | 8. β受体阻断药禁用于支气管哮喘是由于 (2.0分) | A.阻断支气管β2受体 B.阻断支气管β1受体 C.阻断支气管α受体 D.阻断支气管M受体 E.激动支气管M受体 |
A.阻断支气管β2受体 |
2 | 3 | 2015 | 9. 既可阻断α受体又可阻断β受体的药物是 (2.0分) | A.拉贝洛尔 B.普萘洛尔 C.阿替洛尔 D.酚妥拉明 E.哌唑嗪 |
A.拉贝洛尔 |
2 | 3 | 2015 | 10. 感觉的产生过程中,不包括的是(2.0分) | A.感受器 B.神经元 C.中枢 D.肌肉 |
D.肌肉 |
2 | 3 | 2015 | 11. 感受器能把刺激所包含的环境变化信息,转移到动作电位的序列之中,感受器的这种生理特性称为 (2.0分) | A.感受器的适宜刺激 B.感受器的换能作用 C.感受器的编码作用 D.感受器的适应现象 E.感受器的启动电位 |
C.感受器的编码作用 |
2 | 3 | 2015 | 12. 以下属于丘脑特异性投射系统的核团是 (2.0分) | A.联络核 B.网状核 C.感觉接替核 D.髓板内核群 E.感觉接替核和联络核 |
E.感觉接替核和联络核 |
2 | 3 | 2015 | 13. 内侧膝状体接受下列哪种感觉纤维投射? (2.0分) | A.内侧丘系 B.三叉丘系 C.视觉传导束 D.听觉传导束 E.脊髓丘脑前束 |
D.听觉传导束 |
2 | 3 | 2015 | 14. 视觉皮层代表区位于 (2.0分) | A.颞叶的颞横回 B.颞叶的颞上回 C.中央后回 D.扣带回 E.枕叶距状裂 |
E.枕叶距状裂 |
2 | 3 | 2015 | 15. 以下属于丘脑非特异投射系统的主要细胞群是 (2.0分) | A.联络核和髓板内核群 B.感觉接替核和联络核 C.感觉接替核 D.联络核 E.髓板内核群 |
E.髓板内核群 |
2 | 3 | 2015 | 16. 脊髓的何种损伤可导致痛温觉与轻触觉障碍分离? (2.0分) | A.完全横断 B.脊髓空洞症 C.脊髓离断 D.后角受损 E.前根病变 |
B.脊髓空洞症 |
2 | 3 | 2015 | 17. 第一感觉区位于 (2.0分) | A.中央前回 B.中央后回 C.颞叶皮层 D.中央前回与岛叶之间 E.颞下回 |
B.中央后回 |
2 | 3 | 2015 | 18. 下列属于副交感神经的作用的是 (2.0分) | A.瞳孔扩大 B.糖原分解增加 C.逼尿肌收缩 D.骨骼肌血管舒张 E.消化道括约肌收缩 |
C.逼尿肌收缩 |
2 | 3 | 2015 | 19. 交感神经系统不具有的特点是 (2.0分) | A.节前纤维长,节后纤维短 B.支配几乎所有的脏器 C.紧张性活动 D.在应急反应中活动明显加强 |
A.节前纤维长,节后纤维短 |
2 | 3 | 2015 | 20. 下列不属于交感神经兴奋作用的是 (2.0分) | A.心跳加快 B.瞳孔开大 C.支气管平滑肌舒张 D.胃肠平滑肌收缩 E.肾上腺髓质分泌肾上腺素 |
D.胃肠平滑肌收缩 |
2 | 3 | 2015 | 21. 下列关于感觉皮层代表区的叙述,错误的是 (2.0分) | A.本体感觉在中央前回 B.内脏感觉的投射区在额叶 C.体表感觉区在中央后回 D.听觉代表区在颞叶 E.视觉代表区在枕叶 |
B.内脏感觉的投射区在额叶 |
2 | 3 | 2015 | 22. 下丘脑是皮层下 (2.0分) | A.重要的感觉中枢 B.重要的运动中枢 C.较高级的交感中枢 D.较高级的副交感中枢 E.较高级的内脏活动调节中枢 |
E.较高级的内脏活动调节中枢 |
2 | 3 | 2015 | 23. 非条件反射的特点不包括 (2.0分) | A.生来具有 B.无需皮层参与 C.反射弧固定 D.反射形式多变 E.数量有限 |
D.反射形式多变 |
2 | 3 | 2015 | 24. 上斜肌由哪条神经支配,上斜肌使眼球往哪个方向转动? (2.0分) | A.动眼神经,司眼球向下和稍向外转动 B.动眼神经,司眼球向上和稍向外转动 C.滑车神经,司眼球向上和稍向内转动 D.动眼神经,司眼球向下和稍向内转动 E.滑车神经,司眼球向下和稍向外转动 |
E.滑车神经,司眼球向下和稍向外转动 |
2 | 3 | 2015 | 25. 只受单侧(对侧)皮质脑干束支配的核团,除支配眼裂以上的面神经核团,还有: (2.0分) | A.三叉神经运动核 B.迷走神经背侧核 C.迷走神经疑核 D.舌下神经核 E.动眼神经核 |
D.舌下神经核 |
2 | 3 | 2015 | 26. 伸舌时,舌尖偏向病灶对侧,这是哪个部位损害(2.0分) | A.舌下神经核病变 B.舌下神经核上性病变 C.面神经核病变 D.面神经病变 E.舌下神经病变 |
B.舌下神经核上性病变 |
2 | 3 | 2015 | 27. 患者出现左侧偏盲,但存在黄斑回避现象,请问患者视通路的哪一部分发生损害? (2.0分) | A.左侧视束 B.右侧视束 C.右侧视放射 D.左侧枕叶中枢 E.右侧枕叶中枢 |
E.右侧枕叶中枢 |
2 | 3 | 2015 | 28. 男,62岁,因左侧面颊部呈发作性烧灼痛三个月就诊。每次发作持续10余分钟。体格检查:左侧眼眶下面颊部痛觉减退,咀嚼肌无力,角膜反射似较右侧迟钝,下列哪项处理是正确的: (2.0分) | A.积极查找病因 B.首选卡马西平口服 C.对症处理 D.手术切断三叉神经周围支 E.神经营养治疗 |
A.积极查找病因 |
2 | 3 | 2015 | 29. 患者出现双颞侧视野偏盲,颅脑MRI显示:垂体占位性病变,则该患者视通路的哪一部分受到压迫?(2.0分) | A.左侧视神经 B.整个视交叉 C.左侧视束 D.左侧视辐射 E.视交叉正中部 |
E.视交叉正中部 |
2 | 3 | 2015 | 30. 关于核上性眼肌麻痹,以下说法错误的是:(2.0分) | A.双眼同时受累 B.无复视 C.反射性运动仍保存 D.刺激性病灶引起双侧眼球向对侧凝视 E.破坏性病灶引起双侧眼球向对侧凝视 |
E.破坏性病灶引起双侧眼球向对侧凝视 |
2 | 3 | 2015 | 31. 下列说法,哪些是错误的?(2.0分) | A.脑桥侧视中枢与动眼神经核的上行纤维损害,会出现两眼向病变对侧注视时,患侧眼球不能内收,对侧眼球外展时伴有眼震;辐辏反射正常。 B.一侧动眼神经完全性麻痹时,表现为上睑下垂,眼球向外上,瞳孔散大,光反射及调节反射均消失,并有复视。 C.两眼向病变同侧注视时,患侧眼球不能外展,对侧眼球内收正常,为后核间性眼肌麻痹。 D.一个半综合征,是一侧脑桥被盖部病变,引起患者眼球水平注视时,既不能内收又不能外展;对侧眼球水平注视时不能内收,可以外展,但有水平眼震。 E.上丘是眼球垂直同向运动的皮质下中枢,上丘的上半司眼球的向上运动,下半司眼球的向下运动。 |
B.一侧动眼神经完全性麻痹时,表现为上睑下垂,眼球向外上,瞳孔散大,光反射及调节反射均消失,并有复视。 |
2 | 3 | 2015 | 32. 以下不是前庭周围性眩晕特点的是(2.0分) | A.前庭周围性眩晕的病变部位是内耳前庭感受器及前庭神经病变 B.呈发作性,症状较重,持续时间较短 C.常伴耳鸣、听力减退、恶心、呕吐等 D.前庭功能试验无反应或反应减弱 E.眼震幅度粗大、眼震形式多变 |
E.眼震幅度粗大、眼震形式多变 |
2 | 3 | 2015 | 33. 舌咽神经司下列哪个腺体的分泌?(2.0分) | A.泪腺 B.舌下腺 C.颌下腺 D.腮腺 E.唾液腺 |
D.腮腺 |
2 | 3 | 2015 | 34. 发生脊髓损害后,肢体痛、温觉消失,是由于病变侵犯(2.0分) | A.皮质脊髓侧束 B.脊髓丘脑侧束 C.薄束与楔束 D.红核脊髓束 E.脊髓小脑束 |
B.脊髓丘脑侧束 |
2 | 3 | 2015 | 35. 阿罗瞳孔的病变部位在:(2.0分) | A.睫状神经节 B.外侧膝状体 C.中脑顶盖前区 D.动眼神经 E.视神经 |
C.中脑顶盖前区 |
2 | 3 | 2015 | 36. 为了改善周围神经损伤的疗效,下列哪项研究是有益的? (2.0分) | A.促进神经芽生的速度 B.改善桥接通道的通过能力 C.延缓神经功能效应器的退变 D.妥善合理的功能重建 E.以上都是 |
E.以上都是 |
2 | 3 | 2015 | 37. 关于骨折的临床表现,以下哪项是正确的? (2.0分) | A.骨折都有较明显的外伤史 B.骨折有时可缺乏骨折的专有体征 C.为明确骨折的诊断,应着重检查肢体有无异常活动、骨擦音或骨擦感等骨折的专有体征 D.骨折后可有吸收热,一般在38-39C?。 E.以上都不是 |
B.骨折有时可缺乏骨折的专有体征 |
2 | 3 | 2015 | 38. 以下骨折易损伤其邻近的重要血管和神经,其中哪一部位骨折除外? (2.0分) | A.肱骨髁上骨折 B.股骨髁上骨折 C.腓骨颈骨折 D.胫骨干中段骨折 E.肱骨干中段骨折 |
D.胫骨干中段骨折 |
2 | 3 | 2015 | 39. 下列并发症在骨折早期就应注意防治? (2.0分) | A.骨筋膜室综合征 B.下肢静脉栓塞形成 C.压褥 D.感染 E.以上都是 |
E.以上都是 |
2 | 3 | 2015 | 40. 下列哪一项不是肱骨干骨折合并桡神经损伤的表现? (2.0分) | A.腕关节下垂 B.各手指掌指关节不能背伸 C.爪形手 D.拇指不能背伸 E.手背桡侧皮肤感觉减退 |
C.爪形手 |
2 | 3 | 2015 | 41. 骨筋膜室综合症最主要的治疗原则是 (2.0分) | A.给予血管舒张剂,消除血管痉挛 B.抬高患肢,以利消肿 C.被动按摩,以利消肿 D.臂从麻醉,解除血管痉挛 E.解除包扎及固定物,经观察无好转,切开筋膜减压 |
E.解除包扎及固定物,经观察无好转,切开筋膜减压 |
2 | 3 | 2015 | 42. 骨盆骨折可能出现哪些并发症_______。(2.0分) | A.失血性休克 B.直肠、膀胱、尿道损伤 C.神经损伤 D.机体系统炎症反应综合征(SIRS) E.以上都是 |
E.以上都是 |
2 | 3 | 2015 | 43. 关于DCO,以下哪一项不合适? (2.0分) | A.对ISS评分>18分的多发骨折患者,应遵循DCO的原则 B.对ISS评分>20分的多发骨折患者,尽量对骨折进行确定性固定 C.外固定架固定是 DCO的一个常用方法 D.股骨、骨盆等大骨折的尽早稳定有助于抗休克 E.以上都准确 |
C.外固定架固定是 DCO的一个常用方法 |
2 | 3 | 2015 | 44. 关于骨折治疗以下哪一项是不妥的? (2.0分) | A.治疗原则是复位、固定、功能锻炼 B.骨折急救处理时重要的措施是妥善固定 C.肢体开放骨折现场止血的首选是止血带结扎止血。 D.开放骨折的创口在条件许可时应争取早期闭合覆盖。 E.夹板或石膏外固定后应注意血液循环。 |
C.肢体开放骨折现场止血的首选是止血带结扎止血。 |
2 | 3 | 2015 | 45. 实验中,如果同时刺激神经纤维的两端,产生的两个动作电位 (2.0分) | A.将各自通过中点后传导到另一端 B.将在中点相遇,然后传回到起始点 C.只有较强的动作电位通过中点而到达另一端 D.将在中点相遇后停止传导 E.到达中点后将复合成一个更大的动作电位 |
D.将在中点相遇后停止传导 |
2 | 3 | 2015 | 46. 与propranolol相比,metoprolol的特点是 (2.0分) | A.抑制糖原分解 B.对哮喘病人慎用 C.用于治疗高血压病 D.对β1受体有选择性 E.没有内在拟交感活性 |
D.对β1受体有选择性 |
2 | 3 | 2015 | 47. 感觉平面有助于判断病损部位,可据体表判断受损节段,下腹壁反射对应的脊髓反射中枢位于哪个节段?(2.0分) | A.T5-6 B.T7-8 C.T6-7 D.T9-10 E.T11-12 |
E.T11-12 |
2 | 3 | 2015 | 48. 脊髓空洞症的感觉障碍是(2.0分) | A.节段型 B.传导束型 C.分离性节段型 D.混合型 E.以上均不是 |
C.分离性节段型 |
2 | 3 | 2015 | 49. 周围神经受累感觉障碍呈:(2.0分) | A.节段型感觉障碍 B.节段分离性感觉障碍 C.偏身感觉障碍 D.末梢型感觉障碍 E.运动功能障碍 |
D.末梢型感觉障碍 |
2 | 3 | 2015 | 50. 下列关于Bell麻痹不正确的是(2.0分) | A.Bell麻痹具有自限性。 B.急性起病,病情多在3d左右达到高峰。 C.发病常为双侧。 D.闭目时瘫痪侧眼球转向内上方,露出角膜下的白色巩膜,称Bell现象。 E.在发病1-2周后,面神经传导测定复合肌肉动作电位波幅不足健侧10%,恢复较差。 |
C.发病常为双侧。 |
2 | 4 | 2007 | 1. 瞳孔对光反射中枢在(2.0分) | A.中脑 B.下丘脑 C.大脑皮质 D.脑桥 E.延脑 |
A.中脑 |
2 | 4 | 2007 | 2. 某地结肠癌的患病率是12/1,000,在该地进行一项结肠癌的筛检,若该检查的灵敏度是70%,特异度是75%,则阳性预测值是(2.0分) | A.66% B.3.3% C.96.7% D. 33% E.45% |
B.3.3% |
2 | 4 | 2007 | 3. 以下哪一项不属于屈光不正: (2.0分) | A.散光 B.单纯近视 C.远视 D.近视散光 E.老视 |
E.老视 |
2 | 4 | 2007 | 4. 成人时期双侧眼球突出常见原因: (2.0分) | A.海绵状血管瘤 B.颈动脉海绵窦瘘 C.甲状腺相关眼病 D.淋巴瘤 E.蝶骨嵴脑膜瘤 |
C.甲状腺相关眼病 |
2 | 4 | 2007 | 5. 黄斑的中央凹处视敏度最高,是因为(2.0分) | A.视锥细胞多而集中,单线联系 B.视杆细胞多而集中,单线联系 C.视杆细胞多而集中,聚合联系 D.视杆细胞,视锥细胞都多,单线联系 E.视杆细胞多而集中,辐散式联系 |
A.视锥细胞多而集中,单线联系 |
2 | 4 | 2007 | 6. 当用光照射左眼时(2.0分) | A.两侧瞳孔都不变 B.右瞳孔缩小 C.两侧瞳孔都缩小 D.左瞳孔缩小 E.左瞳孔缩小,右瞳孔不变 |
C.两侧瞳孔都缩小 |
2 | 4 | 2007 | 7. 关于视锥细胞的叙述,错误的是(2.0分) | A.主要分布在视网膜的中央部分 B.对光的敏感度较差 C.能分辨颜色 D.其感光色素为视紫红质 E.对物体有高分辨能力 |
D.其感光色素为视紫红质 |
2 | 4 | 2007 | 8. 对暗光敏感的区域是 (2.0分) | A.整个视网膜 B.视盘处 C.视网膜周缘部分 D.视网膜的中央凹处 E.视乳头 |
C.视网膜周缘部分 |
2 | 4 | 2007 | 9. 为提高诊断试验的灵敏度,对两个独立试验可:(2.0分) | A.并联使用 B.串联使用 C.先串联后并联使用 D.要求每个试验假阳性率低 E.要求每个试验特异度低 |
A.并联使用 |
2 | 4 | 2007 | 10. 葡萄膜炎治疗的基本原则是 (2.0分) | A.降低眼压、消除炎症、提高视力 B.控制感染、消炎止痛、对症处理 C.消除病因、拮抗炎症、预防并发症 D.散大瞳孔、拮抗炎症、消除病因 E.预防感染、保护视功能、降低眼压 |
D.散大瞳孔、拮抗炎症、消除病因 |
2 | 4 | 2007 | 11. 在病例对照研究研究中,用于估计评价某因素与疾病之间联系强度的指标是:(2.0分) | A.发病率 B.比值比 C.暴露率 D.相对危险度 E.特异危险度 |
B.比值比 |
2 | 4 | 2007 | 12. 下列关于晶状体的叙述,错误的是(2.0分) | A.白内障是指晶状体透明度降低或颜色改变导致光学质量下降的退行性改变 B.晶状体具有屈光力 C.晶状体前表面较凸,后表面较平坦 D.晶状体具有调节能力,视近物时晶状体变凸 E.晶状体一生都不在不断生长 |
C.晶状体前表面较凸,后表面较平坦 |
2 | 4 | 2007 | 13. 在进行子宫内膜癌病因的病例对照研究时,下列哪一组病例最为理想:(2.0分) | A.一个地区肿瘤死亡监测系统登记的所有子宫内膜癌病人 B.一个地区多所医院诊断的所有子宫内膜癌病人 C.一个地区肿瘤发病监测系统登记的所有子宫内膜癌病人 D.一所肿瘤专科医院收治的所有子宫内膜癌病人 E.一个地区多所医院新诊断的子宫内膜癌病人 |
C.一个地区肿瘤发病监测系统登记的所有子宫内膜癌病人 |
2 | 4 | 2007 | 14. 按色觉三原色学说,三种视锥细胞分别敏感的颜色是(2.0分) | A.红、黄、黑 B.红、蓝、紫 C.绿、蓝、白 D.红、绿、蓝 E.红、白、青 |
D.红、绿、蓝 |
2 | 4 | 2007 | 15. 禁用于治疗原发性闭角型青光眼的药物是(2.0分) | A.高渗脱水剂 B.缩瞳剂 C.碳酸酐酶抑制剂 D.扩瞳剂 E.β受体阻滞剂 |
D.扩瞳剂 |
2 | 4 | 2007 | 16. 男性患者,75 岁,双眼视力下降3 年,既往史:糖尿病20 年。查体:视力右眼0.3,左眼0.02,不能矫正;右眼底后极部可见多数小出血点和棉绒斑和微血管瘤。左眼虹膜新生血管(+),玻璃体内积血。.右眼诊断首先考虑 (2.0分) | A.年龄相关性黄斑变性 B.视网膜中央静脉阻塞 C.高血压性视网膜病变 D.糖尿病视网膜病变 E.肾病性视网膜病变 |
D.糖尿病视网膜病变 |
2 | 4 | 2007 | 17. 房水产生的部位是(2.0分) | A.睫状突 B.视网膜 C.晶状体 D.虹膜 E.玻璃体 |
A.睫状突 |
2 | 4 | 2007 | 18. 基底动脉型偏头痛与典型偏头痛的区别在于: (2.0分) | A.发作间期神经系统阳性体征 B.先兆持续时间 C.男性多见 D.无恶心、呕吐 E.先兆的临床特征 |
E.先兆的临床特征 |
2 | 4 | 2007 | 19. 检测单纯疱疹病毒特异性抗体IgM、IgG时,对于该病毒感染具有确定诊断价值的是:(2.0分) | A.病程中1次抗体滴度呈6倍以上增加 B.病程中2次或2次以上抗体滴度呈4倍以上增高 C.病程中4次抗体滴度呈2倍以下增加 D.病程中4次抗体滴度呈3倍增加 E.病程中6次以上抗体滴度呈3倍以下增加 |
B.病程中2次或2次以上抗体滴度呈4倍以上增高 |
2 | 4 | 2007 | 20. 中枢神经系统结核病最常见的是: (2.0分) | A.结核性脑膜脑炎 B.结核性脑膜炎 C.结核性脑炎 D.结核性脑脊髓炎 E.结核性脊髓炎 |
B.结核性脑膜炎 |
2 | 4 | 2007 | 21. 偏头痛的诊断依据是:(2.0分) | A.可有家族史 B.头痛呈周期性发作,每次发作的过程相似 C.应用麦角胺制剂多有显效 D.多在青春期前后发病 E.以上都是 |
E.以上都是 |
2 | 4 | 2007 | 22. 对听神经有损害的抗痨药是:(2.0分) | A.乙胺丁醇 B.利福平 C.异烟肼 D.吡嗪酰氨 |
NaN |
2 | 4 | 2007 | 23. 下列哪种疾病属于感染性葡萄膜炎(2.0分) | A.Vogt-小柳-原田综合征 B.急性视网膜坏死综合征 C.交感性眼炎 D.Behcet病 E.以上都是 |
B.急性视网膜坏死综合征 |
2 | 4 | 2007 | 24. 化脓性脑膜炎的脑脊液检查特点: (2.0分) | A.糖明显下降,氯化物下降,蛋白明显升高,细胞数升高,以淋巴增高为主 B.糖明显下降,氯化物下降,蛋白明显升高,细胞数升高,以中性粒细胞为主 C.糖正常,氯化物升高,蛋白明显下降,细胞数升高,以中性粒细胞为主 D.细胞数增高,淋巴为主,糖正常,氯化物正常,蛋白升高 E.糖明显升高,氯化物正常,蛋白正常,细胞数正常 |
B.糖明显下降,氯化物下降,蛋白明显升高,细胞数升高,以中性粒细胞为主 |
2 | 4 | 2007 | 25. 关于脑膜炎双球菌不正确的: (2.0分) | A.革兰阳性菌 B.仅存在于人体的细菌 C.严格的需氧菌 D.奈瑟菌属 E.在普通培养基上不能生长 |
A.革兰阳性菌 |
2 | 4 | 2007 | 26. "向日葵样白内障"是以下哪种病变的表现 (2.0分) | A.眼内铁质异物 B.眼内铜质异物 C.青光眼 D.糖尿病 E.眼内炎 |
B.眼内铜质异物 |
2 | 4 | 2007 | 27. 以下哪项不是老年性白内障膨胀期的表现 (2.0分) | A.可有虹膜投影 B.可引起青光眼 C.可有复视或多视 D.前房变浅 E.患者视力可突然提高 |
E.患者视力可突然提高 |
2 | 4 | 2007 | 28. 下列哪项不是白内障囊外摘除术的适应证(2.0分) | A.成熟期的白内障 B.严重后囊膜混浊增厚 C.外伤性白内障 D.有视网膜脱离病史的患者 E.虹膜广泛后粘连的并发性白内障 |
B.严重后囊膜混浊增厚 |
2 | 4 | 2007 | 29. 病例对照研究的病例组最好选择:(2.0分) | A.旧病例 B.死亡病例 C.现患病例 D.新发病例 |
D.新发病例 |
2 | 4 | 2007 | 30. 眼内压是眼球内容物作用于眼球壁的压力。正常人的眼压值是( )mmHg. (2.0分) | A.10-18 B.10-21 C.18-20 D.10-21 E.15-21 |
D.10-21 |
2 | 4 | 2007 | 31. 房水的主要功能是(2.0分) | A.对角膜和晶状体有营养作用 B.对玻璃体有保护作用 C.折光成像 D.对视网膜有保护作用 E.促进睫状肌收缩 |
A.对角膜和晶状体有营养作用 |
2 | 4 | 2007 | 32. 诊断结核性脑膜炎的金标准是: (2.0分) | A.脑脊液中结核抗体阳性 B.脑脊液中细胞数增多,蛋白质增高,糖和氯化物降低 C.脑脊液中结核菌培养阳性 D.血中抗结核抗体阳性 E.PPD皮试阳性 |
C.脑脊液中结核菌培养阳性 |
2 | 4 | 2007 | 33. 典型偏头痛与普通型偏头痛的区别在于: (2.0分) | A.畏光 B.阳性家族史 C.视觉先兆 D.偏侧头痛 E.恶心、呕吐 |
C.视觉先兆 |
2 | 4 | 2007 | 34. 急性虹膜睫状体炎的主要体征包括(2.0分) | A.睫状充血、KP、玻璃体混浊、视网膜水肿 B.混合性充血、角膜水肿、玻璃体混浊 C.睫状充血、KP、前房浑浊、瞳孔缩小 D.睫状充血、前房混浊、瞳孔散大、眼压升高 E.睫状充血、KP、虹膜后粘连、梅花型瞳孔 |
C.睫状充血、KP、前房浑浊、瞳孔缩小 |
2 | 4 | 2007 | 35. 在诊断标准确定之后,诊断结果的阳性预测值取决于(2.0分) | A.似然比 B.敏感度 C.特异度 D.患病率 |
D.患病率 |
2 | 4 | 2007 | 36. 关于葡萄膜的解剖、生理和病理特点,下列哪项有误(2.0分) | A.葡萄膜是眼免疫性疾病的好发部位 B.葡萄膜又称色素膜、血管膜 C.葡萄膜炎是最常见的葡萄膜病变 D.葡萄膜由虹膜、睫状体和脉络膜组成 E.葡萄膜炎继发青光眼是由于房水分泌增加 |
E.葡萄膜炎继发青光眼是由于房水分泌增加 |
2 | 4 | 2007 | 37. 关于基质型单疱病毒性角膜炎,错误的是: (2.0分) | A.发作时可有明显的视力下降 B.禁用激素 C.多次反复发作可遗留角膜疤痕 D.反复发作可出现角膜内皮失代偿 E.可有角膜感觉减退 |
B.禁用激素 |
2 | 4 | 2007 | 38. 女性,32岁,双眼睁不开1个月,半月前出现复视,以上症状晨起较轻,劳累或傍晚加重,眼睑检查发现双眼上睑不能上举,睑裂5mm,神经系统检查未发现其他异常。诊断时应考虑以下哪种疾病可能: (2.0分) | A.交感神经麻痹 B.动眼神经麻痹 C.重症肌无力 D.内眦赘皮 E.先天性上睑下垂 |
C.重症肌无力 |
2 | 4 | 2007 | 39. 对结核杆菌有很强的抑制和杀灭作用但因增加了维生素B6的排泄而产生周围神经炎副作用的抗痨(2.0分) | A.利福平 B.异烟肼 C.乙胺丁醇 D.吡嗪酰氨 |
B.异烟肼 |
2 | 4 | 2007 | 40. 糖尿病的眼部常见并发症不包括(2.0分) | A.屈光不正 B.卡氏肉瘤 C.白内障 D.干眼症 E.视网膜病变 |
B.卡氏肉瘤 |
2 | 4 | 2007 | 41. 有关视力、视角的论述,正确的是(2.0分) | A.视角一分角能看清物体时,视力为1.0 B.视锥细胞直径越大,视力越好 C.双眼视物时可以使视力提高 D.视力大小与视角大小成正比 E.同一距离物体越大形成的视角就越小 |
A.视角一分角能看清物体时,视力为1.0 |
2 | 4 | 2007 | 42. 眼部化学伤救治最重要的是(2.0分) | A.现场急救,冲洗眼部 B.促进眼表修复 C.抗感染 D.急诊切开球结膜 E.1%阿托品散瞳 |
A.现场急救,冲洗眼部 |
2 | 4 | 2007 | 43. 角膜占眼球屈光力的 (2.0分) | A.60% B.90% C.70% D.80% E.50% |
C.70% |
2 | 4 | 2007 | 44. 关于眼球穿通伤的处理,下列哪项不正确 (2.0分) | A.复杂的眼球穿通伤,可以分期手术 B.眼球穿通伤患者应注射破伤风血清,并应用抗生素预防感染 C.严重的穿通伤,应尽早摘除眼球,以防止交感性眼炎发生 D.单纯性角膜伤口,如果前房存在,可以不缝合 E.角膜伤口有虹膜崁顿时,可视具体情况回纳或剪除 |
C.严重的穿通伤,应尽早摘除眼球,以防止交感性眼炎发生 |
2 | 4 | 2007 | 45. 根据国际眼外伤学会理事会推荐的命名法,以下哪种外伤属于闭合性眼外伤(2.0分) | A.角膜板层裂伤 B.球内异物 C.贯通伤 D.巩膜裂伤 E.穿通伤 |
A.角膜板层裂伤 |
2 | 4 | 2007 | 46. 关于角膜实质层的描述,错误的是: (2.0分) | A.由致密胶原纤维束组成的板层有规律的相互重叠而成 B.实质层损伤后不能再生 C.实质层来源于中胚层 D.占角膜全厚度的90% E.实质层内没有细胞 |
E.实质层内没有细胞 |
2 | 4 | 2007 | 47. 人眼看近物时(2.0分) | A.晶状体变凸,折光力增加 B.晶状体变凸,折光力下降 C.晶状体扁平,折光力下降 D.晶状体变扁平,折光力增加 E.晶状体曲率减小,折光力增大 |
A.晶状体变凸,折光力增加 |
2 | 4 | 2007 | 48. 有关晶状体的叙述,错误的是(2.0分) | A.近点距离的远近受其调节能力所决定 B.调节能力与弹性无关 C.随着年龄的增大,弹性逐渐减退 D.所视物体近移时,曲率增大 E.近点越近,调节能力越强 |
B.调节能力与弹性无关 |
2 | 4 | 2007 | 49. 患者,男,35岁,因"右眼红痛、视物模糊3天"就诊,查体后初步诊断为细菌性角膜炎,以下处理哪一项是不恰当的: (2.0分) | A.使用抗生素治疗前先行角膜刮片培养 B.应当在培养结果和药物敏感性试验的结果指导下启动治疗 C.疾病治愈后如遗留角膜疤痕混浊,可考虑行角膜移植 D.急性期用广谱抗生素眼药水频繁滴眼,重症病人同时全身用药 E.根据病情需要可滴用阿托品眼水 |
B.应当在培养结果和药物敏感性试验的结果指导下启动治疗 |
2 | 4 | 2007 | 50. 调节性共同性内斜视早期的治疗是(2.0分) | A.用功能训练来矫正 B.手术矫正 C.戴远视镜矫正 D.功能训练加手术矫正 E.以上均不是 |
C.戴远视镜矫正 |
2 | 4 | 2008 | 1. 中枢神经系统结核病最常见的是: (2.0分) | A.结核性脑脊髓炎 B.结核性脑膜炎 C.结核性脑膜脑炎 D.结核性脑炎 E.结核性脊髓炎 |
B.结核性脑膜炎 |
2 | 4 | 2008 | 2. 入射光线的折射主要发生在(2.0分) | A.晶状体前表面 B.角膜后表面 C.角膜前表面 D.晶状体后表面 E.玻璃体前表面 |
C.角膜前表面 |
2 | 4 | 2008 | 3. 下列有关视杆细胞外段膜电位变化的描述,错误的是(2.0分) | A.未经光照时有钠离子进入细胞 B.未经光照时细胞存在暗电流 C.未经光照射膜电位仅-30~-40mV D.光照后表现为一种去极化型慢电流 E.光照后的电位变化即为感受器电位 |
D.光照后表现为一种去极化型慢电流 |
2 | 4 | 2008 | 4. 视杆系统(2.0分) | A.对光敏感度低,有色觉,分辨力弱 B.对光敏感度高,有色觉,分辨力弱 C.对光敏感度高,无色觉,分辨力弱 D.对光敏感度低,无色觉,分辨力高 E.对光敏感度低,有色觉,分辨力高 |
C.对光敏感度高,无色觉,分辨力弱 |
2 | 4 | 2008 | 5. 角膜混浊较厚,略呈白色,但仍可透见虹膜称(2.0分) | A.角膜白斑 B.角膜斑翳 C.角膜云翳 D.角膜瘘 |
B.角膜斑翳 |
2 | 4 | 2008 | 6. 哪一类偏倚可以在研究设计阶段和资料分析阶段加以控制? (2.0分) | A.混杂偏倚 B.入院率偏倚 C.选择偏倚 D.信息偏倚 |
A.混杂偏倚 |
2 | 4 | 2008 | 7. 女性,32岁,双眼睁不开1个月,半月前出现复视,以上症状晨起较轻,劳累或傍晚加重,眼科检查发现双眼上睑不能上举,睑裂5mm,神经系统检查未发现其他异常。诊断时应考虑以下哪种疾病可能是(2.0分) | A.交感神经麻痹 B.重症肌无力 C.内眦赘皮 D.动眼神经麻痹 E.先天性上睑下垂 |
B.重症肌无力 |
2 | 4 | 2008 | 8. 眼的刺激症状包括(2.0分) | A.疼痛、畏光、流泪、眼睑痉挛 B.疼痛、结膜充血、分泌物增加 C.疼痛、畏光、泪溢、上睑下垂 D.眼痒、流泪、异物感 E.眼胀、眼红、视力下降 |
A.疼痛、畏光、流泪、眼睑痉挛 |
2 | 4 | 2008 | 9. 检测单纯疱疹病毒特异性抗体IgM、IgG时,对于该病毒感染具有确定诊断价值的是: (2.0分) | A.病程中2次或2次以上抗体滴度呈4倍以上增高 B.病程中1次抗体滴度呈6倍以上增加 C.病程中4次抗体滴度呈3倍增加 D.病程中4次抗体滴度呈2倍以下增加 E.病程中6次以上抗体滴度呈3倍以下增加 |
A.病程中2次或2次以上抗体滴度呈4倍以上增高 |
2 | 4 | 2008 | 10. 下列关于眼调节的叙述,错误的是(2.0分) | A.主要靠改变晶状体的折光能力来实现 B.晶状体弹性减弱时,眼的调节能力降低 C.正常人眼视近物时需要调节才能清晰成像于视网膜 D.瞳孔缩小时增大球面像差和色像差 E.双眼会聚也起到重要作用 |
D.瞳孔缩小时增大球面像差和色像差 |
2 | 4 | 2008 | 11. 对听神经有损害的抗痨药是: (2.0分) | A.利福平 B.异烟肼 C.乙胺丁醇 D.吡嗪酰氨 E.链霉素 |
E.链霉素 |
2 | 4 | 2008 | 12. Mill 准则中不包括以下哪种方法?(2.0分) | A.求异法 B.求同法 C.演绎法 D.共变法 E.剩余法 |
C.演绎法 |
2 | 4 | 2008 | 13. 对于可疑的结核性脑膜炎何时上用抗结核药的叙述正确的是: (2.0分) | A.脑脊液中必需抗酸染色阳性才能上用抗结核药 B.除脑脊液中蛋白糖和氯化物的典型改变外还必需有既往结核感染史 C.只要患者的临床症状和体征及实验室检查高度提示该病就应及早抗痨治疗 D.只有脑脊液中结核杆菌培养阳性才能上用抗结核药 E.只要有结核接触史就应抗痨治疗 |
C.只要患者的临床症状和体征及实验室检查高度提示该病就应及早抗痨治疗 |
2 | 4 | 2008 | 14. 以下不属于环境病因的是: (2.0分) | A.年龄 B.婚姻状态 C.饮酒 D.病原体 E.不良嗜好 |
A.年龄 |
2 | 4 | 2008 | 15. 典型偏头痛与普通型偏头痛的区别在于: (2.0分) | A.视觉先兆 B.阳性家族史 C.偏侧头痛 D.畏光 E.恶心、呕吐 |
A.视觉先兆 |
2 | 4 | 2008 | 16. 以下哪一类偏倚不属于信息偏倚? (2.0分) | A.回忆偏倚 B.无差异错分偏倚 C.现患病例--新病例偏倚 D.说谎偏倚 |
C.现患病例--新病例偏倚 |
2 | 4 | 2008 | 17. 如果晶状体核的颜色为琥珀色的硬度应为: (2.0分) | A.2度 B.l度 C.3度 D.4度 E.5度 |
C.3度 |
2 | 4 | 2008 | 18. 基底动脉型偏头痛与典型偏头痛的区别在于: (2.0分) | A.发作间期神经系统阳性体征 B.无恶心、呕吐 C.先兆持续时间 D. 男性多见 E.先兆的临床特征 |
E.先兆的临床特征 |
2 | 4 | 2008 | 19. 疾病因素模型中的卫生保健因素对疾病的哪个方面意义较大? (2.0分) | A.治疗 B.诊断 C.预防 D.预后 |
C.预防 |
2 | 4 | 2008 | 20. 关于脑膜炎双球菌不正确的: (2.0分) | A.奈瑟菌属 B.仅存在于人体的细菌 C.革兰阳性菌 D.严格的需氧菌 E.在普通培养基上不能生长 |
C.革兰阳性菌 |
2 | 4 | 2008 | 21. 树枝状角膜炎的病原体是(2.0分) | A.单纯疱疹病毒 B.微小RNA病毒 C.腺病毒8型 D.肺炎双球菌 |
A.单纯疱疹病毒 |
2 | 4 | 2008 | 22. 葡萄膜炎继发青光眼的处理方法可选择(2.0分) | A.控制炎症、散大瞳孔 B.激光虹膜周边切开术 C.降眼压药物治疗 D.抗青光眼手术 E.以上都是 |
E.以上都是 |
2 | 4 | 2008 | 23. 后葡萄膜炎的临床表现有(2.0分) | A.视网膜水肿、浸润、出血、渗出 B.眼前黑影飘动 C.视力下降 D.玻璃体混浊 E.以上均可有 |
E.以上均可有 |
2 | 4 | 2008 | 24. 男性患者,70 岁,双眼视力下降3 年,既往史:糖尿病20 年。查体:视力右眼0.3,左眼0.02,不能矫正;右眼底后极部可见多数小出血点和棉绒斑和微血管瘤。左眼虹膜新生血管(+),玻璃体内积血。.右眼诊断首先考虑(2.0分) | A.年龄相关性黄斑变性 B.高血压性视网膜病变 C.糖尿病视网膜病变 D.视网膜中央静脉阻塞 E.肾病性视网膜病变 |
C.糖尿病视网膜病变 |
2 | 4 | 2008 | 25. 眼的折光系统不包括(2.0分) | A.视网膜 B.晶状体 C.角膜 D.房水 E.玻璃体 |
A.视网膜 |
2 | 4 | 2008 | 26. 成人时期双侧眼球突出常见原因是(2.0分) | A.海绵状血管瘤 B.淋巴瘤 C.颈动脉海绵窦瘘 D.甲状腺相关眼病 E.蝶骨嵴脑膜瘤 |
D.甲状腺相关眼病 |
2 | 4 | 2008 | 27. 关于葡萄膜的解剖、生理和病理特点,下列哪项有误(2.0分) | A.葡萄膜由虹膜、睫状体和脉络膜组成 B.葡萄膜炎是最常见的葡萄膜病变 C.葡萄膜又称色素膜、血管膜 D.葡萄膜是眼免疫性疾病的好发部位 E.葡萄膜炎继发青光眼是由于房水分泌增加 |
E.葡萄膜炎继发青光眼是由于房水分泌增加 |
2 | 4 | 2008 | 28. 急性闭角型青光眼,其解剖变异是 (2.0分) | A.角膜大 B.眼球大 C.前房浅、房角窄 D.晶体小 E.瞳孔小 |
C.前房浅、房角窄 |
2 | 4 | 2008 | 29. "D"字形瞳孔可见于(2.0分) | A.眼球穿孔伤 B.角膜溃疡穿孔 C.虹膜根部离断 D.睫状体脱离 E.房角后退 |
C.虹膜根部离断 |
2 | 4 | 2008 | 30. 患者,男性,农民,右眼被谷粒弹伤,视物模糊半月就诊,检查:右眼视力0.1,眼睑痉挛极轻,球结膜轻度混合性充血,角膜有黄白色牙膏样病灶,病灶旁可见伪足,角膜轻微水肿,前房积脓1mm,最可能的诊断是: (2.0分) | A.匐行性角膜溃疡 B.真菌性角膜溃疡炎 C.绿脓杆菌性角膜炎 D.病毒性角膜炎 |
B.真菌性角膜溃疡炎 |
2 | 4 | 2008 | 31. 眼部碱性化学伤的特点是(2.0分) | A.创面较浅 B.损伤区边界清楚 C.眼组织凝固、变性和坏死 D.修复较快 E.眼表及眼内组织均可受累 |
E.眼表及眼内组织均可受累 |
2 | 4 | 2008 | 32. 角膜是主要的屈光介质,其屈光度其相当于: (2.0分) | A.20D凹透镜 B.19D凸透镜 C.42D凸透镜 D.43D凸透镜 |
D.43D凸透镜 |
2 | 4 | 2008 | 33. 治疗闭角型青光眼最首要的用药是(2.0分) | A.受体阻滞剂 B.缩瞳剂 C.扩瞳剂 D.碳酸酐酶抑制剂 |
B.缩瞳剂 |
2 | 4 | 2008 | 34. 关于远视眼的临床表现,下列哪项叙述不正确 (2.0分) | A.可引起调节性内斜视; B.近距离工作视疲劳更明显; C.高度远视,远视力下降而近视力正常; D.眼球小、前房浅。 |
C.高度远视,远视力下降而近视力正常; |
2 | 4 | 2008 | 35. 以下哪组临床表现最能提示球内异物可能(2.0分) | A.角膜穿孔、前房消失、虹膜脱出、晶状体混浊 B.外伤史、角膜线状伤口、虹膜穿孔、晶状体局限性混浊 C.角膜溃疡、前房积脓、虹膜后粘联、晶状体混浊 D.外伤史、角膜板层裂伤、虹膜根部离断、晶状体混浊 E.以上均有可能 |
B.外伤史、角膜线状伤口、虹膜穿孔、晶状体局限性混浊 |
2 | 4 | 2008 | 36. 在无调节情况下,平行光线进入眼内聚焦于视网膜之后,称为(2.0分) | A.近视 B.弱视 C.远视 D.以上都不是 |
C.远视 |
2 | 4 | 2008 | 37. 以下各条中哪一个不属于因果推断的标准? (2.0分) | A.关联的可重复性 B.关联的合理性 C.关联来自大样本研究 D.关联的时间顺序 E.关联的强度 |
C.关联来自大样本研究 |
2 | 4 | 2008 | 38. 下面哪种方法可以明确诊断白内障: (2.0分) | A.裂隙灯显微镜 B.视觉诱发电位 C.干涉光断层扫描 D.验光 E.视野检查 |
A.裂隙灯显微镜 |
2 | 4 | 2008 | 39. 以下恶性高血压的描述不正确的为(2.0分) | A.尿中可见管型 B.视网膜出血、水肿、渗出,视乳头水肿 C.多见于中青年 D.常合并孔源性视网膜脱离 E.可伴左心衰竭 |
D.常合并孔源性视网膜脱离 |
2 | 4 | 2008 | 40. 白内障的主要症状是什么: (2.0分) | A.视力障碍 B.眼充血 C.眼痛 D.压痛 E.眼分泌物 |
A.视力障碍 |
2 | 4 | 2008 | 41. 每眼眼外肌共有(2.0分) | A.8 条 B.6条 C.4条 D.9条 |
B.6条 |
2 | 4 | 2008 | 42. 诊断结核性脑膜炎的金标准是: (2.0分) | A.脑脊液中细胞数增多,蛋白质增高,糖和氯化物降低 B.血中抗结核抗体阳性 C.脑脊液中结核菌培养阳性 D.脑脊液中结核抗体阳性 E.PPD皮试阳性 |
C.脑脊液中结核菌培养阳性 |
2 | 4 | 2008 | 43. 年龄相关性白内障的最好治疗方法是: (2.0分) | A.放射治疗 B.药物治疗 C.手术治疗 D.验光戴镜 E.补充营养 |
C.手术治疗 |
2 | 4 | 2008 | 44. 高度近视引起的并发性白内障混浊部位多为: (2.0分) | A.前囊 B.后囊 C.核 D.前囊下皮质 E.后囊下皮质 |
C.核 |
2 | 4 | 2008 | 45. 研究实施过程中,由于测量仪器存在偏差,是测量所得的数据与真实情况存在差异,这属于哪一类偏倚? (2.0分) | A.选择偏倚 B.混杂偏倚 C.入院率偏倚 D.信息偏倚 |
D.信息偏倚 |
2 | 4 | 2008 | 46. 眼内压是眼球内容物作用于眼球壁的压力。正常人的眼压值是( )mmHg。(2.0分) | A.10~20 B.18~20 C.10~18 D.10~21 |
D.10~21 |
2 | 4 | 2008 | 47. 一眼原发性急性闭角型青光眼急性发作,对侧没发作眼有青光眼解剖学基础,对侧眼为(2.0分) | A.临床前期 B.间歇期 C.先兆期 D.不能确定 |
A.临床前期 |
2 | 4 | 2008 | 48. 屈光参差患者度数相差超过( ),可因融像困难而出现症状。(2.0分) | A.2.00D; B.1.50D; C.2.50D; D.3.00D。 |
D.3.00D。 |
2 | 4 | 2008 | 49. 皮质性白内障膨胀期和初发期的临床表现不同之处是: (2.0分) | A.视力急剧下降 B.混浊形态不同 C.水裂 D.空泡 E.板层分离 |
A.视力急剧下降 |
2 | 4 | 2008 | 50. 下列哪项措施,不能降低青光眼急性期的眼压(2.0分) | A.0.5%噻吗心安滴眼 B.缩瞳剂滴眼 C.大量抗生素滴眼 D.口服乙酰唑胺 E.高渗剂静脉快滴 |
C.大量抗生素滴眼 |
2 | 4 | 2009 | 1. 眼的折光系统不包括 (2.0分) | A.房水 B.角膜 C.视网膜 D.晶状体 E.玻璃体 |
C.视网膜 |
2 | 4 | 2009 | 2. 根据简化眼的参数,眼前5 m远处,宽3 mm的"E"字笔画在视网膜上像的大小约为? (2.0分) | A.1 μm B.2 μm C.5 μm D.9 μm E.10 μm |
D.9 μm |
2 | 4 | 2009 | 3. 视锥系统 (2.0分) | A.对光敏感度高,有色觉,分辨力弱 B.对光敏感度低,有色觉,分辨力高 C.对光敏感度低,有色觉,分辨力弱 D.对光敏感度高,无色觉,分辨力弱 E.对光敏感度低,无色觉,分辨力高 |
B.对光敏感度低,有色觉,分辨力高 |
2 | 4 | 2009 | 4. 下列关于眼调节的叙述,正确的是(2.0分) | A.正常人眼视近物时需要调节才能清晰成像于视网膜 B.晶状体弹性增强时,眼的调节能力降低 C.主要靠改变玻璃体的折光能力来实现 D.瞳孔缩小时增大球面像差和色像差 E.交感神经兴奋引起瞳孔缩小 |
A.正常人眼视近物时需要调节才能清晰成像于视网膜 |
2 | 4 | 2009 | 5. 瞳孔对光反射 (2.0分) | A.中枢位于中脑 B.感受器是角膜 C.效应器是睫状肌 D.传出神经是动眼神经中的交感纤维 E.传入神经是三叉神经中的眼支 |
A.中枢位于中脑 |
2 | 4 | 2009 | 6. 关于葡萄膜的解剖、生理和病理特点,下列哪项有误(2.0分) | A.葡萄膜由虹膜、睫状体和脉络膜组成 B.葡萄膜又称色素膜、血管膜 C.葡萄膜炎是最常见的葡萄膜病变 D.葡萄膜是眼免疫性疾病的好发部位 E.葡萄膜炎继发青光眼是由于房水分泌增加 |
E.葡萄膜炎继发青光眼是由于房水分泌增加 |
2 | 4 | 2009 | 7. 眼的刺激症状包括 (2.0分) | A.疼痛、畏光、流泪、眼睑痉挛 B.疼痛、畏光、泪溢、上睑下垂 C.疼痛、结膜充血、分泌物增加 D.眼痒、流泪、异物感 E.眼胀、眼红、视力下降 |
A.疼痛、畏光、流泪、眼睑痉挛 |
2 | 4 | 2009 | 8. 后葡萄膜炎的临床表现有 (2.0分) | A.眼前黑影飘动 B.视力下降 C.玻璃体混浊 D.视网膜水肿、浸润、出血、渗出 E.以上均可有 |
E.以上均可有 |
2 | 4 | 2009 | 9. 葡萄膜炎继发青光眼的处理方法可选择(2.0分) | A.降眼压药物治疗 B.激光虹膜周边切开术 C.控制炎症、散大瞳孔 D.抗青光眼手术 E.以上都是 |
E.以上都是 |
2 | 4 | 2009 | 10. 男性患者,75 岁,双眼视力下降3 年,既往史:糖尿病20 年。查体:视力右眼0.3,左眼0.02,不能矫正;右眼底后极部可见多数小出血点和棉绒斑和微血管瘤。左眼虹膜新生血管(+),玻璃体内积血。.右眼诊断首先考虑(2.0分) | A.年龄相关性黄斑变性 B.糖尿病视网膜病变 C.高血压性视网膜病变 D.视网膜中央静脉阻塞 E.肾病性视网膜病变 |
B.糖尿病视网膜病变 |
2 | 4 | 2009 | 11. 成人时期双侧眼球突出常见原因(2.0分) | A.淋巴瘤 B.海绵状血管瘤 C.颈动脉海绵窦瘘 D.甲状腺相关眼病 E.蝶骨嵴脑膜瘤 |
D.甲状腺相关眼病 |
2 | 4 | 2009 | 12. 以下恶性高血压的描述不正确的为 (2.0分) | A.合并原发性视网膜脱离 B.视网膜出血、水肿、渗出,视乳头水肿 C.多见于中青年 D.尿中可见管型 E.可伴左心衰竭 |
A.合并原发性视网膜脱离 |
2 | 4 | 2009 | 13. 高血压性视网膜病变Keith-Waganer II期眼底表现为 (2.0分) | A.视网膜动脉痉挛或合并轻度硬化 B.视网膜动脉硬化合并视网膜水肿、出血、渗出 C.视网膜动脉硬化合并视盘水肿 D.视网膜动脉硬化,动静脉交叉处改变,动脉反光增强 E.视网膜动脉硬化,继发视网膜脱离 |
D.视网膜动脉硬化,动静脉交叉处改变,动脉反光增强 |
2 | 4 | 2009 | 14. 根据国际眼外伤学会理事会推荐的命名法,以下哪种外伤属于闭合性眼外伤(2.0分) | A.角膜板层裂伤 B.球内异物 C.巩膜裂伤 D.贯通伤 E.穿通伤 |
A.角膜板层裂伤 |
2 | 4 | 2009 | 15. 关于眼球穿通伤的处理,下列哪项不正确 (2.0分) | A.单纯性角膜伤口,如果前房存在,可以不缝合 B.复杂的眼球穿通伤,可以分期手术 C.眼球穿通伤患者应注射破伤风血清,并应用抗生素预防感染 D.严重的穿通伤,应尽早摘除眼球,以防止交感性眼炎发生 E.角膜伤口有虹膜崁顿时,可视具体情况回纳或剪除 |
D.严重的穿通伤,应尽早摘除眼球,以防止交感性眼炎发生 |
2 | 4 | 2009 | 16. "向日葵样白内障"是以下哪种病变的表现(2.0分) | A.糖尿病 B.青光眼 C.眼内铁质异物 D.眼内铜质异物 E.眼内炎 |
D.眼内铜质异物 |
2 | 4 | 2009 | 17. 眼部碱性化学伤得特点是(2.0分) | A.眼组织凝固、变性和坏死 B.损伤区边界清楚 C.创面较浅 D.修复较快 E.眼表及眼内组织均可受累 |
E.眼表及眼内组织均可受累 |
2 | 4 | 2009 | 18. 典型偏头痛与普通型偏头痛的区别在于: (2.0分) | A.畏光 B.阳性家族史 C.偏侧头痛 D.视觉先兆 E.恶心、呕吐 |
D.视觉先兆 |
2 | 4 | 2009 | 19. 基底动脉型偏头痛与典型偏头痛的区别在于(2.0分) | A.男性多见 B.发作间期神经系统阳性体征 C.无恶心、呕吐 D.先兆持续时间 E.先兆的临床特征 |
E.先兆的临床特征 |
2 | 4 | 2009 | 20. 检测单纯疱疹病毒特异性抗体IgM、IgG时,对于该病毒感染具有确定诊断价值的是: (2.0分) | A.病程中2次或2次以上抗体滴度呈4倍以上增高 B.病程中1次抗体滴度呈6倍以上增加 C.病程中4次抗体滴度呈2倍以下增加 D.病程中4次抗体滴度呈3倍增加 E.病程中6次以上抗体滴度呈3倍以下增加 |
A.病程中2次或2次以上抗体滴度呈4倍以上增高 |
2 | 4 | 2009 | 21. 对老年人结核性脑膜炎的临床特点描述不正确的是: (2.0分) | A.呕吐较少见 B.头痛较少见 C.颅高压发生率低 D.多数患者脑脊液改变典型 E.脑梗死发生率相对较多 |
D.多数患者脑脊液改变典型 |
2 | 4 | 2009 | 22. 对结核杆菌有很强的抑制和杀灭作用但因增加了维生素B6的排泄而产生周围神经炎副作用的抗痨(2.0分) | A.异烟肼 B.利福平 C.乙胺丁醇 D.吡嗪酰氨 E.链霉素 |
A.异烟肼 |
2 | 4 | 2009 | 23. 化脓性脑膜炎的脑脊液检查特点: (2.0分) | A.糖正常,氯化物升高,蛋白明显下降,细胞数升高,以中性粒细胞为主 B.糖明显下降,氯化物下降,蛋白明显升高,细胞数升高,以中性粒细胞为主 C.糖明显下降,氯化物下降,蛋白明显升高,细胞数升高,以淋巴增高为主 D.细胞数增高,淋巴为主,糖正常,氯化物正常,蛋白升高 E.糖明显升高,氯化物正常,蛋白正常,细胞数正常 |
B.糖明显下降,氯化物下降,蛋白明显升高,细胞数升高,以中性粒细胞为主 |
2 | 4 | 2009 | 24. 偏头痛的诊断依据是: (2.0分) | A.多在青春期前后发病 B.头痛呈周期性发作,每次发作的过程相似 C.应用麦角胺制剂多有显效 D.可有家族史 E.以上都是 |
E.以上都是 |
2 | 4 | 2009 | 25. 角膜是主要的屈光介质,其屈光度其相当于: (2.0分) | A.19D凸透镜 B.20D凹透镜 C.42D凸透镜 D.43D凸透镜 |
D.43D凸透镜 |
2 | 4 | 2009 | 26. 树枝状角膜炎的病原体是(2.0分) | A.腺病毒8型 B.微小RNA病毒 C.单纯疱疹病毒 D.肺炎双球菌 |
C.单纯疱疹病毒 |
2 | 4 | 2009 | 27. 角膜混浊较厚,略呈白色,但仍可透见虹膜称(2.0分) | A.角膜白斑 B.角膜云翳 C.角膜斑翳 D.角膜瘘 |
C.角膜斑翳 |
2 | 4 | 2009 | 28. 每眼眼外肌共有(2.0分) | A.4条 B.6条 C.8 条 D.9条 |
B.6条 |
2 | 4 | 2009 | 29. 在无调节情况下,平行光线进入眼内聚焦于视网膜之后,称为(2.0分) | A.弱视 B.近视 C.远视 D.以上都不是 |
C.远视 |
2 | 4 | 2009 | 30. 由于调节过强和调节痉挛引起远视力低下者称为 (2.0分) | A.假性近视; B.老视; C.轴性近视; D.变性近视。 |
A.假性近视; |
2 | 4 | 2009 | 31. 关于远视眼的临床表现,下列哪项叙述不正确(2.0分) | A.高度远视,远视力下降而近视力正常; B.近距离工作视疲劳更明显; C.可引起调节性内斜视; D.眼球小、前房浅。 |
A.高度远视,远视力下降而近视力正常; |
2 | 4 | 2009 | 32. 屈光参差患者度数相差超过( ),可因融像困难而出现症状。(2.0分) | A.1.50D; B.2.00D; C.2.50D; D.3.00D。 |
D.3.00D。 |
2 | 4 | 2009 | 33. 关于麻痹性斜视(非共同性斜视)的临床表现,下列哪项不对。(2.0分) | A.眼球运动受限 B.第一斜视较大于第二斜视角 C.眼球向不同方向注视时斜视角不等; D.常有代偿头位; |
B.第一斜视较大于第二斜视角 |
2 | 4 | 2009 | 34. 白内障的主要症状是什么: (2.0分) | A.视力障碍 B.眼痛 C.眼充血 D.压痛 E.眼分泌物 |
A.视力障碍 |
2 | 4 | 2009 | 35. 皮质性白内障膨胀期和初发期的临床表现不同之处是: (2.0分) | A.水裂 B.混浊形态不同 C.视力急剧下降 D.空泡 E.板层分离 |
C.视力急剧下降 |
2 | 4 | 2009 | 36. 下面哪种方法可以明确诊断白内障: (2.0分) | A.验光 B.视觉诱发电位 C.干涉光断层扫描 D.裂隙灯显微镜 E.视野检查 |
D.裂隙灯显微镜 |
2 | 4 | 2009 | 37. 年龄相关性白内障的最好治疗方法是: (2.0分) | A.手术治疗 B.药物治疗 C.放射治疗 D.验光戴镜 E.补充营养 |
A.手术治疗 |
2 | 4 | 2009 | 38. 高度近视引起的并发性白内障混浊部位多为: (2.0分) | A.前囊 B.前囊下皮质 C.核 D.后囊 E.后囊下皮质 |
C.核 |
2 | 4 | 2009 | 39. 如果晶状体核的颜色为琥珀色的硬度应为: (2.0分) | A.l度 B.2度 C.3度 D.4度 E.5度 |
C.3度 |
2 | 4 | 2009 | 40. 眼内压是眼球内容物作用于眼球壁的压力。正常人的眼压值是 mmHg。(2.0分) | A.10~20 B.10~18 C.18~20 D.10~21 |
D.10~21 |
2 | 4 | 2009 | 41. 一眼原发性急性闭角型青光眼急性发作,对侧没发作眼有青光眼解剖学基础,对侧眼为(2.0分) | A.临床前期 B.先兆期 C.间歇期 D.不能确定 |
A.临床前期 |
2 | 4 | 2009 | 42. 治疗闭角型青光眼最首要的用药是 (2.0分) | A.扩瞳剂 B.缩瞳剂 C.受体阻滞剂 D.碳酸酐酶抑制剂 |
B.缩瞳剂 |
2 | 4 | 2009 | 43. 急性闭角型青光眼,其解剖变异是(2.0分) | A.眼球大 B.角膜大 C.前房浅、房角窄 D.晶体小 E.瞳孔小 |
C.前房浅、房角窄 |
2 | 4 | 2009 | 44. 下列哪项措施,不能降低青光眼急性期的眼压 (2.0分) | A.缩瞳剂滴眼 B.大量抗生素滴眼 C.口服乙酰唑胺 D.0.5%噻吗心安滴眼 E.高渗剂静脉快滴 |
B.大量抗生素滴眼 |
2 | 4 | 2009 | 45. 开角型青光眼晚期课件 (2.0分) | A.青光眼斑 B.眼科持续升高 C.眼胀痛 D.虹视 E.眼底青光眼杯 |
E.眼底青光眼杯 |
2 | 4 | 2009 | 46. 医学科学研究中,进行统计学假设检验的目的是排除以下哪一类关联的影响 ? (2.0分) | A.虚假关联 B.间接关联 C.统计学关联 D.偶然关联 |
A.虚假关联 |
2 | 4 | 2009 | 47. 以下各项中哪一项是进行因果推断必须满足的 ? (2.0分) | A.关联的时间顺序 B.关联的合理性 C.关联的特异性 D.关联的可重复性 |
D.关联的可重复性 |
2 | 4 | 2009 | 48. 以下不属于宿主因素的是(2.0分) | A.性别 B.年龄 C.宗教信仰 D.心理状况 |
C.宗教信仰 |
2 | 4 | 2009 | 49. 至少有30次病例对照研究,7次队列研究,不同地点、时间、人群研究证明肺癌与吸烟的关系,这可以为哪一项病因推断标准提供证据? (2.0分) | A.特异性 B.可重复性 C.一致性 D.合理性 |
D.合理性 |
2 | 4 | 2009 | 50. Mill 准则中不包括以下哪种方法? (2.0分) | A.求同法 B.求异法 C.剩余法 D.归纳法 |
B.求异法 |
2 | 4 | 2010 | 1. 下面哪种表现属于偏头痛并发症 (2.0分) | A.视网膜性偏头痛 B.先偏瘫,麻木、失语数十分钟后发生头痛 C.慢性偏头痛 D.眼肌麻痹性偏头痛 |
C.慢性偏头痛 |
2 | 4 | 2010 | 2. 典型偏头痛的先兆症状可能是由于(2.0分) | A.颅外动脉收缩 B.颅外动脉扩张 C.颅内动脉收缩 D.颅内动脉扩张 E.颅内和颅外动脉扩张 |
C.颅内动脉收缩 |
2 | 4 | 2010 | 3. 对发作频繁(每月发作2次或以上)的偏头痛患者应选用用(2.0分) | A.非类固醇消炎痛药物 B.发作早期给麦角胺咖啡因 C.5-羟色胺受体激动剂 D.心得安、苯噻啶等药物 E.皮质类固醇 |
D.心得安、苯噻啶等药物 |
2 | 4 | 2010 | 4. 关于单纯疱疹性脑炎的叙述,下列哪一项正确:(2.0分) | A.主要损害白质 B.主要特征为出血性坏死 C.主要特征为脱髓鞘病变 D.主要特征为变性病变 E.以顶、枕叶受损最严重 |
B.主要特征为出血性坏死 |
2 | 4 | 2010 | 5. 可以导致球后视神经炎,引起弱视、视野缩小等不良反应的抗痨药是(2.0分) | A.异烟肼 B.利福平 C.乙胺丁醇 D.吡嗪酰氨 E.链霉素 |
C.乙胺丁醇 |
2 | 4 | 2010 | 6. 引起新生儿化脓性脑膜炎最常见的致病菌是(2.0分) | A.肺炎链球菌 B.流感嗜血杆菌 C.大肠埃希菌 D.金黄色葡萄球菌 E.隐球菌 |
C.大肠埃希菌 |
2 | 4 | 2010 | 7. 化脓性脑膜炎的脑脊液检查特点:(2.0分) | A.糖正常,氯化物升高,蛋白明显下降,细胞数升高,以中性粒细胞为主 B.糖明显下降,氯化物下降,蛋白明显升高,细胞数升高,以中性粒细胞为主 C.糖明显下降,氯化物下降,蛋白明显升高,细胞数升高,以淋巴增高为主 D.细胞数增高,淋巴为主,糖正常,氯化物正常,蛋白升高 E.糖明显升高,氯化物正常,蛋白正常,细胞数正常 |
B.糖明显下降,氯化物下降,蛋白明显升高,细胞数升高,以中性粒细胞为主 |
2 | 4 | 2010 | 8. 眼的折光系统不包括 (2.0分) | A.晶状体 B.视网膜 C.角膜 D.房水 E.玻璃体 |
B.视网膜 |
2 | 4 | 2010 | 9. 入射光线的折射主要发生在(2.0分) | A.角膜前表面 B.角膜后表面 C.晶状体前表面 D.晶状体后表面 E.玻璃体前表面 |
A.角膜前表面 |
2 | 4 | 2010 | 10. 下列关于眼调节的叙述,错误的是(2.0分) | A.正常人眼视近物时需要调节才能清晰成像于视网膜 B.晶状体弹性减弱时,眼的调节能力降低 C.主要靠改变晶状体的折光能力来实现 D.瞳孔缩小时增大球面像差和色像差 E.双眼会聚也起到重要作用 |
D.瞳孔缩小时增大球面像差和色像差 |
2 | 4 | 2010 | 11. 视杆系统(2.0分) | A.对光敏感度高,有色觉,分辨力弱 B.对光敏感度低,有色觉,分辨力弱 C.对光敏感度高,无色觉,分辨力弱 D.对光敏感度低,无色觉,分辨力高 E.对光敏感度低,有色觉,分辨力高 |
C.对光敏感度高,无色觉,分辨力弱 |
2 | 4 | 2010 | 12. 下列有关视杆细胞外段膜电位变化的描述,错误的是(2.0分) | A.未经光照射膜电位仅-30~-40mV B.未经光照时细胞存在暗电流 C.未经光照时有钠离子进入细胞 D.光照后表现为一种去极化型慢电流 E.光照后的电位变化即为感受器电位 |
D.光照后表现为一种去极化型慢电流 |
2 | 4 | 2010 | 13. 关于葡萄膜的解剖、生理和病理特点,下列哪项有误 (2.0分) | A.葡萄膜由虹膜、睫状体和脉络膜组成 B.葡萄膜又称色素膜、血管膜 C.葡萄膜炎是最常见的葡萄膜病变 D.葡萄膜是眼免疫性疾病的好发部位 E.葡萄膜炎继发青光眼时可滴匹罗卡品眼药水控制眼压 |
E.葡萄膜炎继发青光眼时可滴匹罗卡品眼药水控制眼压 |
2 | 4 | 2010 | 14. 眼的刺激症状包括 (2.0分) | A.疼痛、畏光、流泪、眼睑痉挛 B.疼痛、畏光、泪溢、上睑下垂 C.疼痛、结膜充血、分泌物增加 D.眼痒、流泪、异物感 E.眼胀、眼红、视力下降 |
A.疼痛、畏光、流泪、眼睑痉挛 |
2 | 4 | 2010 | 15. 后葡萄膜炎的临床表现有(2.0分) | A.眼前黑影飘动 B.视力下降 C.玻璃体混浊 D.视网膜水肿、浸润、出血、渗出 E.以上均可有 |
E.以上均可有 |
2 | 4 | 2010 | 16. 葡萄膜炎继发青光眼的处理方法可选择 (2.0分) | A.降眼压药物治疗 B.激光虹膜周边切开术 C.控制炎症、散大瞳孔 D.抗青光眼手术 E.以上都是 |
E.以上都是 |
2 | 4 | 2010 | 17. 男性患者,75 岁,双眼视力下降3 年,既往史:糖尿病20 年。查体:视力右眼0.3,左眼0.02,不能矫正;右眼底后极部可见多数小出血点和棉绒斑和微血管瘤。左眼虹膜新生血管(+),玻璃体内积血。.右眼诊断首先考虑(2.0分) | A.年龄相关性黄斑变性 B.糖尿病视网膜病变 C.高血压性视网膜病变 D.视网膜中央静脉阻塞 E.肾病性视网膜病变 |
B.糖尿病视网膜病变 |
2 | 4 | 2010 | 18. 成人时期双侧眼球突出常见原因(2.0分) | A.淋巴瘤 B.海绵状血管瘤 C.颈动脉海绵窦瘘 D.甲状腺相关眼病 E.蝶骨嵴脑膜瘤 |
D.甲状腺相关眼病 |
2 | 4 | 2010 | 19. 以下恶性高血压的描述不正确的为 (2.0分) | A.合并原发性视网膜脱离 B.视网膜出血、水肿、渗出,视乳头水肿 C.多见于中青年 D.尿中可见管型 E.可伴左心衰竭 |
A.合并原发性视网膜脱离 |
2 | 4 | 2010 | 20. 高血压性视网膜病变Keith-Waganer II期眼底表现为 (2.0分) | A.视网膜动脉痉挛或合并轻度硬化 B.视网膜动脉硬化合并视网膜水肿、出血、渗出 C.视网膜动脉硬化合并视盘水肿 D.视网膜动脉硬化,动静脉交叉处改变,动脉反光增强 E.视网膜动脉硬化,继发视网膜脱离 |
D.视网膜动脉硬化,动静脉交叉处改变,动脉反光增强 |
2 | 4 | 2010 | 21. 对老年人结核性脑膜炎的临床特点描述不正确的是:(2.0分) | A.呕吐较少见 B.头痛较少见 C.颅高压发生率低 D.多数患者脑脊液改变典型 E.脑梗死发生率相对较多 |
D.多数患者脑脊液改变典型 |
2 | 4 | 2010 | 22. 根据国际眼外伤学会理事会推荐的命名法,以下哪种外伤属于闭合性眼外伤(2.0分) | A.角膜板层裂伤 B.球内异物 C.巩膜裂伤 D.贯通伤 E.穿通伤 |
A.角膜板层裂伤 |
2 | 4 | 2010 | 23. 关于眼球穿通伤的处理,下列哪项不正确(2.0分) | A.单纯性角膜伤口,如果前房存在,可以不缝合 B.复杂的眼球穿通伤,可以分期手术 C.眼球穿通伤患者应注射破伤风血清,并应用抗生素预防感染 D.严重的穿通伤,应尽早摘除眼球,以防止交感性眼炎发生 E.角膜伤口有虹膜崁顿时,可视具体情况回纳或剪除 |
D.严重的穿通伤,应尽早摘除眼球,以防止交感性眼炎发生 |
2 | 4 | 2010 | 24. "向日葵样白内障"是以下哪种病变的表现(2.0分) | A.糖尿病 B.青光眼 C.眼内铁质异物 D.眼内铜质异物 E.眼内炎 |
D.眼内铜质异物 |
2 | 4 | 2010 | 25. 眼部碱性化学伤得特点是 (2.0分) | A.眼组织凝固、变性和坏死 B.损伤区边界清楚 C.创面较浅 D.修复较快 E.眼表及眼内组织均可受累 |
E.眼表及眼内组织均可受累 |
2 | 4 | 2010 | 26. 下列哪项不是因果关联的判断标准(2.0分) | A.样本大小 B.关联的时序性 C.关联强度 D.剂量-反应关系 |
A.样本大小 |
2 | 4 | 2010 | 27. 不同国家的学者在不同时间对本国人群作研究,均得出高盐饮食引起高血压,由此支持高盐饮食是影响高血压发生的危险因素这一推论运用了判断标准中的 (2.0分) | A.关联强度 B.剂量-反应关系 C.关联的可重复性 D.关联的时序性 |
C.关联的可重复性 |
2 | 4 | 2010 | 28. 因果关联是指 (2.0分) | A.暴露-疾病有较强的统计学关联 B.暴露-疾病分类资料存在相关 C.暴露-疾病排除偏倚后的关联 D.暴露-疾病有时间先后的无偏关联 |
D.暴露-疾病有时间先后的无偏关联 |
2 | 4 | 2010 | 29. 病例对照研究中,用于研究的对象一般是现患病例,由此可能产生的偏倚是(2.0分) | A.回忆偏倚 B.测量偏倚 C.易感性偏倚 D.奈曼偏倚 |
D.奈曼偏倚 |
2 | 4 | 2010 | 30. 病例对照研究中,如果研究者对病例组调查和询问的态度认真,而对对照组漫不经心地调查询问,从而导致错误的结论,其中可能发生的偏倚是(2.0分) | A.入院率偏倚 B.暴露怀疑偏倚 C.奈曼偏倚 D.易感性偏倚 |
B.暴露怀疑偏倚 |
2 | 4 | 2010 | 31. 某因素在病因学上与某疾病本无关联,但可引起所研究疾病的症状或体征,从而促使患者及早就诊,提高了早期病例检出率,从而过高的估计了暴露程度,这种偏倚是 (2.0分) | A.诊断怀疑偏倚 B.检出症候偏倚 C.信息偏倚 D.混杂偏倚 |
B.检出症候偏倚 |
2 | 4 | 2010 | 32. 角膜是主要的屈光介质,其屈光度其相当于: (2.0分) | A.19D凸透镜 B.20D凹透镜 C.42D凸透镜 D.43D凸透镜 |
D.43D凸透镜 |
2 | 4 | 2010 | 33. 树枝状角膜炎的病原体是(2.0分) | A.腺病毒8型 B.微小RNA病毒 C.单纯疱疹病毒 D.肺炎双球菌 |
C.单纯疱疹病毒 |
2 | 4 | 2010 | 34. 角膜混浊较厚,略呈白色,但仍可透见虹膜称(2.0分) | A.角膜白斑 B.角膜云翳 C.角膜斑翳 D.角膜瘘 |
C.角膜斑翳 |
2 | 4 | 2010 | 35. 每眼眼外肌共有(2.0分) | A.4条 B.6条 C.8 条 D.9条 |
B.6条 |
2 | 4 | 2010 | 36. 在无调节情况下,平行光线进入眼内聚焦于视网膜之后,称为 (2.0分) | A.弱视 B.近视 C.远视 D.以上都不是 |
C.远视 |
2 | 4 | 2010 | 37. 由于调节过强和调节痉挛引起远视力低下者称为 (2.0分) | A.假性近视; B.老视; C.轴性近视; D.变性近视。 |
A.假性近视; |
2 | 4 | 2010 | 38. 关于远视眼的临床表现,下列哪项叙述不正确 (2.0分) | A.高度远视,远视力下降而近视力正常; B.近距离工作视疲劳更明显; C.可引起调节性内斜视; D.眼球小、前房浅。 |
A.高度远视,远视力下降而近视力正常; |
2 | 4 | 2010 | 39. 屈光参差患者度数相差超过( ),可因融像困难而出现症状。(2.0分) | A.1.50D; B.2.00D; C.2.50D; D.3.00D。 |
D.3.00D。 |
2 | 4 | 2010 | 40. 关于麻痹性斜视(非共同性斜视)的临床表现,下列哪项不对。 (2.0分) | A.眼球运动受限 B.第一斜视较大于第二斜视角 C.眼球向不同方向注视时斜视角不等; D.常有代偿头位; |
B.第一斜视较大于第二斜视角 |
2 | 4 | 2010 | 41. 皮质性白内障膨胀期和初发期的临床表现不同之处是: (2.0分) | A.水裂 B.混浊形态不同 C.视力急剧下降 D.空泡 E.板层分离 |
C.视力急剧下降 |
2 | 4 | 2010 | 42. 白内障的主要症状是什么:(2.0分) | A.视力障碍 B.眼痛 C.眼充血 D.压痛 E.眼分泌物 |
A.视力障碍 |
2 | 4 | 2010 | 43. 下面哪种方法可以明确诊断白内障:(2.0分) | A.验光 B.视觉诱发电位 C.干涉光断层扫描 D.裂隙灯显微镜 E.视野检查 |
D.裂隙灯显微镜 |
2 | 4 | 2010 | 44. 年龄相关性白内障的最好治疗方法是: (2.0分) | A.手术治疗 B.药物治疗 C.放射治疗 D.验光戴镜 E.补充营养 |
A.手术治疗 |
2 | 4 | 2010 | 45. 高度近视引起的并发性白内障混浊部位多为: (2.0分) | A.前囊 B.前囊下皮质 C.核 D.后囊 E.后囊下皮质 |
C.核 |
2 | 4 | 2010 | 46. 如果晶状体核的颜色为琥珀色的硬度应为: (2.0分) | A.l度 B.2度 C.3度 D.4度 E.5度 |
C.3度 |
2 | 4 | 2010 | 47. 眼内压是眼球内容物作用于眼球壁的压力。正常人的眼压值是()mmHg。 (2.0分) | A.10~20 B.10~18 C.18~20 D.10~21 |
D.10~21 |
2 | 4 | 2010 | 48. 一眼原发性急性闭角型青光眼急性发作,对侧没发作眼有青光眼解剖学基础,对侧眼为(2.0分) | A.临床前期 B.先兆期 C.间歇期 D.不能确定 |
A.临床前期 |
2 | 4 | 2010 | 49. 治疗闭角型青光眼最首要的用药是(2.0分) | A.扩瞳剂 B.缩瞳剂 C.受体阻滞剂 D.碳酸酐酶抑制剂 |
B.缩瞳剂 |
2 | 4 | 2010 | 50. 急性闭角型青光眼,其解剖变异是(2.0分) | A.眼球大 B.角膜大 C.前房浅、房角窄 D.晶体小 E.瞳孔小 |
C.前房浅、房角窄 |
2 | 4 | 2011 | 1. 下列症状中不属于前葡萄膜炎症状的是(3.0分) | A.眼痛 B.眼红 C.分泌物增多 D.视物模糊 E.畏光、流泪 |
C.分泌物增多 |
2 | 4 | 2011 | 2. 关于葡萄膜的解剖、生理和病理特点,下列哪项有误(3.0分) | A.葡萄膜由虹膜、睫状体和脉络膜组成 B.葡萄膜又称色素膜、血管膜 C.葡萄膜炎是最常见的葡萄膜病变 D.葡萄膜是眼免疫性疾病的好发部位 E.葡萄膜炎继发青光眼时可滴匹罗卡品眼药水控制眼压 |
E.葡萄膜炎继发青光眼时可滴匹罗卡品眼药水控制眼压 |
2 | 4 | 2011 | 3. 以下哪个是急性前葡萄膜炎与急性闭角型青光眼的鉴别要点(3.0分) | A.房水闪光 B.前房角狭窄 C.虹膜后粘连 D.晶状体混浊 E.玻璃体混浊 |
B.前房角狭窄 |
2 | 4 | 2011 | 4. 下列哪种疾病属于感染性葡萄膜炎 (3.0分) | A.Behcet病 B.Vogt-小柳-原田综合征 C.急性视网膜坏死综合征 D.交感性眼炎 E.Fuchs虹膜异色性葡萄膜炎 |
C.急性视网膜坏死综合征 |
2 | 4 | 2011 | 5. 男性患者,55 岁,双眼视力下降1 年,既往史:糖尿病10年。查体:矫正视力右眼0.3,左眼指数/眼前;右眼后极部视网膜可见多数小出血点和棉绒斑和微血管瘤。左眼玻璃体内积血。.右眼诊断首先考虑(3.0分) | A.年龄相关性黄斑变性 B.糖尿病视网膜病变 C.高血压性视网膜病变 D.视网膜中央静脉阻塞 E.肾病性视网膜病变 |
B.糖尿病视网膜病变 |
2 | 4 | 2011 | 6. 成人时期双侧眼球突出常见原因(3.0分) | A.淋巴瘤 B.海绵状血管瘤 C.颈动脉海绵窦瘘 D.甲状腺相关眼病 E.蝶骨嵴脑膜瘤 |
D.甲状腺相关眼病 |
2 | 4 | 2011 | 7. 急进型恶性高血压视网膜病变的临床表现为(3.0分) | A.合并原发性视网膜脱离 B.视乳头水肿、视网膜出血、水肿、渗出 C.视网膜动脉硬化,动静脉交叉处改变,动脉反光增强 D.视网膜动脉痉挛或合并硬化 E.视网膜动脉硬化,继发视网膜脱离 |
B.视乳头水肿、视网膜出血、水肿、渗出 |
2 | 4 | 2011 | 8. 高血压性视网膜病变Keith-Waganer Ⅲ期眼底表现为(3.0分) | A.视网膜动脉痉挛或合并轻度硬化 B.视网膜动脉硬化合并视网膜水肿、出血、渗出 C.视网膜动脉硬化合并视盘水肿 D.视网膜动脉硬化,动静脉交叉处改变,动脉反光增强 E.视网膜动脉硬化,继发视网膜脱离 |
B.视网膜动脉硬化合并视网膜水肿、出血、渗出 |
2 | 4 | 2011 | 9. 根据国际眼外伤学会理事会推荐的命名法,以下哪种外伤属于开放性眼外伤(3.0分) | A.角膜板层裂伤 B.球内异物 C.钝挫伤 D.酸烧伤 E.辐射伤 |
B.球内异物 |
2 | 4 | 2011 | 10. "D"字形瞳孔可见于(3.0分) | A.眼球穿孔伤 B.角膜溃疡穿孔 C.睫状体脱离 D.虹膜根部离断 E.房角后退 |
D.虹膜根部离断 |
2 | 4 | 2011 | 11. 患者,男,46岁,右眼视力下降2年就诊。检查:右眼结膜无充血,角膜有2mm线状疤痕,对应虹膜可见创口,创口处后粘连于晶状体,晶状体前囊有大量棕色色素沉着,晶状体混浊,眼底窥不清。追问病史:3年前榔头敲打铁钉时眼球感觉有刺痛伴热泪涌出,2天后疼痛好转未就医。首先需要考虑的诊断是(3.0分) | A.虹膜睫状体炎 B.青光眼 C.眼内铁质异物 D.眼内铜质异物 E.眼内炎 |
C.眼内铁质异物 |
2 | 4 | 2011 | 12. 急性虹膜睫状体炎的主要体征包括(4.0分) | A.睫状充血、瞳孔缩小、KP、前房浑浊、 B.睫状充血、瞳孔散大、前房混浊、眼压升高 C.角膜水肿、瞳孔散大、前房混浊、玻璃体混浊 D.睫状充血、KP、玻璃体混浊、视网膜水肿 E.瞳孔缩小、虹膜后粘连、晶状体前色素沉着 |
A.睫状充血、瞳孔缩小、KP、前房浑浊、 |
2 | 4 | 2011 | 13. 糖尿病性视网膜病变最早期的表现是(3.0分) | A.硬性渗出 B.微血管瘤 C.棉絮斑 D.出血 E.视网膜水肿 |
B.微血管瘤 |
2 | 4 | 2011 | 14. 眼的折光系统不包括(3.0分) | A.房水 B.角膜 C.晶状体 D.视网膜 E.玻璃体 |
D.视网膜 |
2 | 4 | 2011 | 15. 入射光线的折射主要发生在(3.0分) | A.晶状体前表面 B.晶状体后表面 C.角膜前表面 D.角膜后表面 E.玻璃体前表面 |
C.角膜前表面 |
2 | 4 | 2011 | 16. 根据简化眼的参数,眼前5 m远处,宽3 mm的"E"字笔画在视网膜上像的大小约为? (3.0分) | A.1 μm B.2 μm C.5 μm D.9 μm E.10 μm |
D.9 μm |
2 | 4 | 2011 | 17. 正常人视力的限度为(3.0分) | A.视网膜中央凹处一个视杆细胞的平均直径 B.视网膜中央凹处一个视杆细胞的平均半径 C.视网膜中央凹处一个视锥细胞的平均半径 D.视网膜中央凹处一个视锥细胞的平均直径 E.人所能看清楚的最小视网膜上像的大小 |
E.人所能看清楚的最小视网膜上像的大小 |
2 | 4 | 2011 | 18. 视锥系统(3.0分) | A.对光敏感度高,有色觉,分辨力弱 B.对光敏感度低,有色觉,分辨力高 C.对光敏感度低,有色觉,分辨力弱 D.对光敏感度高,无色觉,分辨力弱 E.对光敏感度低,无色觉,分辨力高 |
B.对光敏感度低,有色觉,分辨力高 |
2 | 4 | 2011 | 19. 在同一光照条件下,视野最小的是 (3.0分) | A.白色 B.红色 C.黄色 D.蓝色 E.绿色 |
E.绿色 |
2 | 4 | 2011 | 20. 眼的折光系统不包括(3.0分) | A.房水 B.角膜 C. 晶状体 D.视网膜 E.玻璃体 |
E.玻璃体 |
2 | 4 | 2011 | 21. 入射光线的折射主要发生在(3.0分) | A.晶状体前表面 B.晶状体后表面 C.角膜前表面 D.角膜后表面 E.玻璃体前表面 |
D.角膜后表面 |
2 | 4 | 2011 | 22. 根据简化眼的参数,眼前5 m远处,宽3 mm的"E"字笔画在视网膜上像的大小约为?(3.0分) | A.1 μm B.2 μm C.5 μm D.9 μm E.10 μm |
D.9 μm |
2 | 4 | 2011 | 23. 正常人视力的限度为(3.0分) | A.视网膜中央凹处一个视杆细胞的平均直径 B.视网膜中央凹处一个视杆细胞的平均半径 C.视网膜中央凹处一个视锥细胞的平均半径 D.视网膜中央凹处一个视锥细胞的平均直径 E.人所能看清楚的最小视网膜上像的大小 |
A.视网膜中央凹处一个视杆细胞的平均直径 |
2 | 4 | 2011 | 24. 视锥系统(3.0分) | A.对光敏感度高,有色觉,分辨力弱 B.对光敏感度低,有色觉,分辨力高 C.对光敏感度低,有色觉,分辨力弱 D.对光敏感度高,无色觉,分辨力弱 E.对光敏感度低,无色觉,分辨力高 |
E.对光敏感度低,无色觉,分辨力高 |
2 | 4 | 2011 | 25. 在同一光照条件下,视野最小的是(3.0分) | A.白色 B.红色 C.黄色 D.蓝色 E.绿色 |
C.黄色 |
2 | 4 | 2011 | 26. 下列哪项不是因果关联的判断标准(3.0分) | A.样本大小 B.关联的时序性 C.关联强度 D.剂量-反应关系 |
A.样本大小 |
2 | 4 | 2011 | 27. 当怀疑的病因(暴露)减少或去除,引起疾病发生率下降,可进一步支持因果关联,这一种判断运用的标准是(3.0分) | A.生物学合理性 B.剂量-反应关系 C.实验证据(终止效应 D.关联强度 |
C.实验证据(终止效应 |
2 | 4 | 2011 | 28. 不同国家的学者在不同时间对本国人群作研究,均得出高盐饮食引起高血压,由此支持高盐饮食是影响高血压发生的危险因素这一推论运用了判断标准中的(3.0分) | A.关联强度 B.剂量-反应关系 C.关联的可重复性 D.关联的时序性 |
C.关联的可重复性 |
2 | 4 | 2011 | 29. 幽门螺杆菌结合部位在胃窦细胞,它可随胃窦细胞进入十二脂肠引起炎症、削弱粘膜,使其易于遭受酸的损伤,由此支持幽门螺杆菌是十二批肠溃疡的发病原因,这一推理运用了因果判断标准中的(3.0分) | A.关联的时序性 B.关联强度 C.关联的可重复性 D.关联的一致性 |
D.关联的一致性 |
2 | 4 | 2011 | 30. 因果关联是指(3.0分) | A.暴露-疾病有较强的统计学关联 B.暴露-疾病分类资料存在相关 C.暴露-疾病排除偏倚后的关联 D.暴露-疾病有时间先后的无偏关联 |
D.暴露-疾病有时间先后的无偏关联 |
2 | 4 | 2011 | 31. 病例对照研究利用医院就诊或住院病人作为研究对象时易导致的偏倚种类为(3.0分) | A.伯克森偏倚 B.报告偏倚 C.奈曼偏倚 D.检出症候偏倚 |
A.伯克森偏倚 |
2 | 4 | 2011 | 32. 病例对照研究中,用于研究的对象一般是现患病例,由此可能产生的偏倚是(3.0分) | A.回忆偏倚 B.测量偏倚 C.易感性偏倚 D.奈曼偏倚 |
D.奈曼偏倚 |
2 | 4 | 2011 | 33. 在研究口服避孕药与乳腺癌的关系时,因为怀疑年龄是可能的混杂因素,因此,研究人员只选择40-60岁的妇女为研究对象,这种控制混杂偏倚的方法为(3.0分) | A.随机化 B.匹配 C.限制 D.盲法 |
C.限制 |
2 | 4 | 2012 | 1. 眼的折光系统不包括 (2.5分) | A.房水 B.角膜 C.视网膜 D.晶状体 E.玻璃体 |
C.视网膜 |
2 | 4 | 2012 | 2. 瞳孔对光反射(2.5分) | A.传出神经是动眼神经中的交感纤维 B.感受器是角膜 C.效应器是睫状肌 D.中枢位于中脑 E.传入神经是三叉神经中的眼支 |
D.中枢位于中脑 |
2 | 4 | 2012 | 3. 在同一光照条件下,视野最大的是(2.5分) | A.白色 B.红色 C.黄色 D.蓝色 E.绿色 |
A.白色 |
2 | 4 | 2012 | 4. 下列关于眼调节的叙述,不正确的是(2.5分) | A.正常人眼视近物时需要调节才能清晰成像于视网膜 B.晶状体弹性增强时,眼的调节能力增强 C.主要靠改变玻璃体的折光能力来实现 D.瞳孔缩小时减少球面像差和色像差 E.副交感神经兴奋引起瞳孔缩小 |
C.主要靠改变玻璃体的折光能力来实现 |
2 | 4 | 2012 | 5. 正常人视力的限度为(2.5分) | A.视网膜中央凹处一个视杆细胞的平均直径 B.视网膜中央凹处一个视杆细胞的平均半径 C.视网膜中央凹处一个视锥细胞的平均半径 D.视网膜中央凹处一个视锥细胞的平均直径 E.人所能看清楚的最小视网膜上像的大小 |
E.人所能看清楚的最小视网膜上像的大小 |
2 | 4 | 2012 | 6. 葡葡膜炎最常见的种类为 (2.5分) | A.前葡萄膜炎 B.中间葡萄膜炎 C.后葡萄膜炎 D.交感性眼炎 E.视网膜脉络膜炎 |
A.前葡萄膜炎 |
2 | 4 | 2012 | 7. 中间葡萄膜炎典型体征为(2.5分) | A.出现飞蚊症并有加重 B.视网膜新生血管 C.视神经萎缩 D.玻璃体雪球样混浊、雪堤样改变 E.黄斑囊样水肿 |
D.玻璃体雪球样混浊、雪堤样改变 |
2 | 4 | 2012 | 8. 虹膜炎继发性青光眼是由于 (2.5分) | A.玻璃体大量炎症细胞 B.房水分泌过多 C.血-房水屏障功能破坏 D.虹膜周边前粘连、瞳孔闭锁 E.炎症反复发作使房角后退 |
D.虹膜周边前粘连、瞳孔闭锁 |
2 | 4 | 2012 | 9. 增殖性糖尿病视网膜病变的最主要标志是(2.5分) | A.微动脉瘤 B.视网膜内微血管异常 C.棉絮斑 D.新生血管形成 E.硬性渗出 |
D.新生血管形成 |
2 | 4 | 2012 | 10. 视乳头水肿出现在高血压性视网膜病变的(2.5分) | A.1级 B.2级 C.3级 D.4级 E.1级和2级 |
D.4级 |
2 | 4 | 2012 | 11. 眼球内金属异物存留时,最重要的治疗措施(2.5分) | A.取出异物 B.抗炎 C.止血 D.治疗并发症 E.随访 |
A.取出异物 |
2 | 4 | 2012 | 12. 虹膜睫状体挫伤时,三分之一虹膜根部发生离断,这时瞳孔的形状为(2.5分) | A.瞳孔膜闭 B.梅花瞳 C.D字形 D.瞳孔扩大 E.以上都不是 |
C.D字形 |
2 | 4 | 2012 | 13. 下列不属于高血压视网膜病变的有(2.5分) | A.静脉隐蔽现象 B.眼底视盘水肿 C.Roth斑 D.Salus征 E.Gunn征 |
C.Roth斑 |
2 | 4 | 2012 | 14. 下列那条肌肉不受第Ⅲ颅神经支配(2.5分) | A.上直肌 B.内直肌 C.下直肌 D.上斜肌; |
D.上斜肌; |
2 | 4 | 2012 | 15. 真菌性角膜炎的确诊可根据(2.5分) | A.植物性外伤史 B.刮片细菌学检查 C.溃疡形态及特征 D.刮片找到菌丝 |
D.刮片找到菌丝 |
2 | 4 | 2012 | 16. 在无调节情况下,平行光线进入眼内聚焦于视网膜之后,称为(2.5分) | A.弱视 B.近视 C.远视 D.以上都不是 |
C.远视 |
2 | 4 | 2012 | 17. 关于近视眼的临床表现,下列哪项不常见 (2.5分) | A.远视力减退; B.视疲劳; C.眼位偏斜; D.眼轴变短。 |
D.眼轴变短。 |
2 | 4 | 2012 | 18. 屈光参差患者度数相差超过( ),可因融像困难而出现症状。(2.5分) | A.1.50D; B.2.00D; C.2.50D; D.3.00D。 |
D.3.00D。 |
2 | 4 | 2012 | 19. 关于麻痹性斜视(非共同性斜视)的临床表现,下列哪项不对(2.5分) | A.眼球运动受限 B.第一斜视较大于第二斜视角 C.眼球向不同方向注视时斜视角不等; D.常有代偿头位; |
B.第一斜视较大于第二斜视角 |
2 | 4 | 2012 | 20. 角膜是主要的屈光介质,其屈光度其相当于(2.5分) | A.19D凸透镜 B.20D凹透镜 C.42D凸透镜 D.43D凸透镜 |
D.43D凸透镜 |
2 | 4 | 2012 | 21. 近视眼患者未矫正前成像位置在视网膜的 (2.5分) | A.前方 B.后方; C.视乳头部; D.不确定。 |
A.前方 |
2 | 4 | 2012 | 22. 下列哪项不是因果关联的判断标准(2.5分) | A.样本大小 B.关联的时序性 C.关联强度 D.剂量-反应关系 |
A.样本大小 |
2 | 4 | 2012 | 23. 不同国家的学者在不同时间对本国人群作研究,均得出高盐饮食引起高血压,由此支持高盐饮食是影响高血压发生的危险因素这一推论运用了判断标准中的(2.5分) | A.关联强度 B.剂量-反应关系 C.关联的可重复性 D.关联的时序性 |
C.关联的可重复性 |
2 | 4 | 2012 | 24. 幽门螺杆菌结合部位在胃窦细胞,它可随胃窦细胞进入十二脂肠引起炎症、削弱粘膜,使其易于遭受酸的损伤,由此支持幽门螺杆菌是十二批肠溃疡的发病原因,这一推理运用了因果判断标准中的(2.5分) | A.关联的时序性 B.关联强度 C.关联的可重复性 D.关联的一致性 |
D.关联的一致性 |
2 | 4 | 2012 | 25. 因果关联是指 (2.5分) | A.暴露-疾病有较强的统计学关联 B.暴露-疾病分类资料存在相关 C.暴露-疾病排除偏倚后的关联 D.暴露-疾病有时间先后的无偏关联 |
D.暴露-疾病有时间先后的无偏关联 |
2 | 4 | 2012 | 26. 病例对照研究中,用于研究的对象一般是现患病例,由此可能产生的偏倚是(2.5分) | A.回忆偏倚 B.测量偏倚 C.易感性偏倚 D.奈曼偏倚 |
D.奈曼偏倚 |
2 | 4 | 2012 | 27. 病例对照研究中,如果研究者对病例组调查和询问的态度认真,而对对照组漫不经心地调查询问,从而导致错误的结论,其中可能发生的偏倚是 (2.5分) | A.入院率偏倚 B.暴露怀疑偏倚 C.奈曼偏倚 D.易感性偏倚 |
B.暴露怀疑偏倚 |
2 | 4 | 2012 | 28. 某因素在病因学上与某疾病本无关联,但可引起所研究疾病的症状或体征,从而促使患者及早就诊,提高了早期病例检出率,从而过高的估计了暴露程度,这种偏倚是(2.5分) | A.诊断怀疑偏倚 B.检出症候偏倚 C.信息偏倚 D.混杂偏倚 |
B.检出症候偏倚 |
2 | 4 | 2012 | 29. An overweight 36-year-old woman presents with three to four severe, debilitating headaches per month for the last 2 years. The headaches last 1 to 2 days. They are sometimes localized bifrontally, but more often localized to the right temple, right frontal region, and behind the right eye. There is often rhinorrhea and congestion associated with her headaches. She denies any prodrome or auras. The pain is usually a deep ache, but throbbing when severe. She sometimes gets some nausea, but no vomiting. She has to wear sunglasses and go to a quit room because she "can't function." What is the most likely diagnosis? (2.5分) | A.Cluster headache B.Intermittent sinus headache C.Episodic tension headache D.Idiopathic intracranial hypertension E.Episodic migraine |
E.Episodic migraine |
2 | 4 | 2012 | 30. A 22-year-old mildly overweight woman presents to the emergency department with increasingfrequency of previously diagnosed migraines. Other medical history is unremarkable with exception of mild asthma and recurrent constipation. The attacks are occurring 4 days per week and are lasting the entire day. What would be the best preventive medication to start in this patient? (2.5分) | A.Amitriptyline B.Propranolol C.Sumatriptan D.Topiramate E.Verapamil |
D.Topiramate |
2 | 4 | 2012 | 31. What would be your first treatment of choice in this patient, assuming there are no contraindications? (2.5分) | A.Sumatriptan B.Amitriptyline C.Propranolol D.Indomethacin E.Topiramate |
D.Indomethacin |
2 | 4 | 2012 | 32. Which of the following is the least common cause of bacterial meningitis in neonates? (2.5分) | A.Neisseria meningitidis B.Escherichiacoli C.Enteric gram-negative bacilli D.Listeria monocytogenes E.Group B streptococci |
A.Neisseria meningitidis |
2 | 4 | 2012 | 33. A 79-year-old woman with diabetes, hypertension, and chronic leg ulcersdevelops headaches and fever. On examination, she has neck stiffness and mildconfusion. Brain CT scan is unremarkable. Lumbar puncture is performed, andCSF shows 112 WBCs/μL (normal up to 5 lymphocytes/μL) with 81%neutrophils, protein level of 60 mg/dL (normal up to 45 mg/dL), and glucose of80 mg/dL, with a blood glucose of 198 mg/dL. CSF is sent for culture. Whichempiric antibiotic regimen would you start? (2.5分) | A.Vancomycin and ceftriaxone B.Vancomycin, ceftriaxone, and ampicillin C.Vancomycin, ceftriaxone, and acyclovir D.Ampicillin and gentamicin E.Penicillin and gentamicin |
B.Vancomycin, ceftriaxone, and ampicillin |
2 | 4 | 2012 | 34. A 35-year-old man suffers an SAH with intraventricular extension and develops hydrocephalus. An external ventricular drain is placed. Over the next several days, he develops fever, and his CSF shows 210 WBCs/μL with 75% neutrophils (normal up to 5 lymphocytes/μL) and a protein level of 80 mg/dL (normal up to 45 mg/dL). Which of the following is the least likely pathogen causing this infection? (2.5分) | A.Staphylococcus aureus B.Staphylococcus epidermidis C.Pseudomonas aeruginosa D.Streptococcus pneumoniae E.Propionibacterium acnes |
C.Pseudomonas aeruginosa |
2 | 4 | 2012 | 35. 6个月患儿,发热咳嗽5天,近一日呕吐,喷射状,抽搐二次,静点抗生素3天,查体嗜睡,前囟饱满,两肺闻及少许中小水泡音,X线片右下肺少许斑片状阴影,血常规白细胞14.8×10的9次方/L,淋巴31%,脑脊液检查外观混浊,白细胞500×10的6次方/L,蛋白阳性,糖、氯化物均下降,作OT试验,硬结直径为0.5cm。(2.5分) | A.结核性脑膜炎 B.病毒性脑膜炎 C.肺炎双球菌脑膜炎 D.流行性乙型脑炎 E.肺炎,中毒性脑病 |
C.肺炎双球菌脑膜炎 |
2 | 4 | 2012 | 36. A 21-year-old woman with a history of recurrent sinusitis presents with fever,diplopia and facial pain. On examination, she has proptosis of the right eyewith periorbital edema, limited extraocular movements of the right eye, andreduced sensation over her right upper face and cheek. Fundoscopicexamination demonstrates papilledema and retinal hemorrhages. What is themost likely diagnosis?(2.5分) | A.Superior sagittal sinus thrombosis B.Meningitis C.Brain abscess D.Cavernous sinus thrombosis E.Transverse sinus thrombosis |
D.Cavernous sinus thrombosis |
2 | 4 | 2012 | 37. 男性,8岁。于8月19日开始发热,头痛,当时测体温38℃,在外院诊断为上感,给予布洛芬退热,头孢菌素静滴无效,8月22日出现嗜睡,体温高达40℃,8月23日因昏迷伴抽出入院。查体:神志不清,压眶有反应,体温无出血点,颈强阳性,克氏症、布氏症阴性。近期有结核患者接触史。为快速诊断应首选下列检查中的(2.5分) | A.结核菌素试验 B.血常规 C.胸片 D.脑脊液常规 E.大便常规 |
D.脑脊液常规 |
2 | 4 | 2012 | 38. 该患者入院后急查实验室结果:血常规:WBC14.5X109/L,N87%。CSF:细胞数 286X106/L,蛋白 0.54g/L,糖 3.5mmol/L,氯化物 129mmol/L。下列处理正确的是:(2.5分) | A.物理降温为主,药物降温为辅,肛温控制在38℃为宜 B.抗结核治疗 C.氨苄西林静脉点滴 D.室温维持在30℃以上 E.应用红霉素静滴 |
A.物理降温为主,药物降温为辅,肛温控制在38℃为宜 |
2 | 4 | 2012 | 39. 该患者感染过程中出现最早的有诊断意义的抗体是(2.5分) | A.补体结合抗体 B.血凝抑制抗体 C.中和抗体 D.Vi抗体 E.特异性IgM抗体 |
E.特异性IgM抗体 |
2 | 4 | 2012 | 40. Regarding CJD, which of the following is correct? (2.5分) | A.Granulovacuolar degeneration and neurofibrillary tangles are seen on histopathologic specimens B.The familial form is autosomal recessive caused by a mutation in the prion protein gene C.It is caused by scrapie prion protein (PrP), which has decreased β-sheet content as compared with cellular prion protein (PrPC) D.The EEG finding shows a repetitive periodic pattern E.It is associated with very low CD4 counts |
D.The EEG finding shows a repetitive periodic pattern |
2 | 4 | 2013 | 1. 视锥系统 (2.5分) | A.对光敏感度高,有色觉,分辨力弱 B.对光敏感度低,有色觉,分辨力高 C.对光敏感度低,有色觉,分辨力弱 D.对光敏感度高,无色觉,分辨力弱 E.对光敏感度低,无色觉,分辨力高 |
B.对光敏感度低,有色觉,分辨力高 |
2 | 4 | 2013 | 2. 根据简化眼的参数,眼前5 m远处,宽3 mm的"E"字笔画在视网膜上像的大小约为?(2.5分) | A.1 μm B.2 μm C.5 μm D.9 μm E.10 μm |
D.9 μm |
2 | 4 | 2013 | 3. 眼的折光系统不包括(2.5分) | A.房水 B.角膜 C.晶状体 D.视网膜 E.玻璃体 |
D.视网膜 |
2 | 4 | 2013 | 4. 下列关于眼调节的叙述,正确的是 (2.5分) | A.正常人眼视近物时需要调节才能清晰成像于视网膜 B.晶状体弹性增强时,眼的调节能力降低 C.主要靠改变玻璃体的折光能力来实现 D.瞳孔缩小时增大球面像差和色像差 E.交感神经兴奋引起瞳孔缩小 |
A.正常人眼视近物时需要调节才能清晰成像于视网膜 |
2 | 4 | 2013 | 5. 下列组织中,哪项是脑膜炎的疾病部位 (2.5分) | A.硬脑膜 B.软脑膜 C.蛛网膜 D.硬脑膜和软脑膜 E.软脑膜和蛛网膜 |
E.软脑膜和蛛网膜 |
2 | 4 | 2013 | 6. 病毒性脑膜炎最常见的病原体是(2.5分) | A.柯萨奇病毒 B.肠道病毒 C.单纯疱疹病毒 D.带状疱疹病毒 E.以上都不是 |
B.肠道病毒 |
2 | 4 | 2013 | 7. A 25-year-old woman with a history of recurrent sinusitis presents with fever, diplopia and facial pain. On examination, she has proptosis of the right eye with periorbital edema, limited extraocular movements of the right eye, and reduced sensation over her right upper face and cheek. Fundoscopic examination demonstrates papilledema and retinal hemorrhages and Her CSF shows openning pressure 280mmH2O with normal cell count. What is the most likely diagnosis? (2.5分) | A.Superior sagittal sinus thrombosis B.Meningitis C.Brain abscess D.Cavernous sinus thrombosis E.Transverse sinus thrombosis |
D.Cavernous sinus thrombosis |
2 | 4 | 2013 | 8. 引起脑炎的最常见病原体是 (2.5分) | A.肺炎链球菌 B.带状疱疹病毒 C.单纯疱疹病毒 D.肠道病毒 E.以上都不是 |
C.单纯疱疹病毒 |
2 | 4 | 2013 | 9. A 23-year-old woman is brought to the emergency department after having a seizure. As per her roommate, the patient had been behaving oddly over the past week. Approximately 3 days ago, she developed fever, and for the past 2 days, she has been confused, talking strangely, and very lethargic. Today she had a generalized tonic-clonic seizure. Her CSF was reported as "reddish" in color, with 59 WBCs (normal up to 5 lymphocytes/μL), 3500 RBCs, protein of 58 mg/dL (normal up to 45 mg/dL), and normal glucose levels. An MRI is obtained and is shown in figure. Which is the most likely diagnosis? (2.5分) | A.West Nile encephalitis B.Bacterial meningitis C.HSV encephalitis D.Enterovirus meningitis E.Tuberculous meningitis |
C.HSV encephalitis |
2 | 4 | 2013 | 10. 针对脑炎的治疗中一般不包括(2.5分) | A.抗病毒药物 B.激素 C.血浆置换 D.免疫球蛋白 E.抗生素 |
E.抗生素 |
2 | 4 | 2013 | 11. An overweight 32-year-old woman presents with three to four severe, debilitating headaches per month for the last 2 years. The headaches last 1 to 2 days. They are sometimes localized bifrontally, but more often localized to the right temple, right frontal region, and behind the right eye. She denies any prodrome or auras. The pain is usually a deep ache, but throbbing when severe. She sometimes gets some nausea, but no vomiting. She has to wear sunglasses and go to a quit room because she "can't function." What is the most likely diagnosis? (2.5分) | A.Cluster headache B.Intermittent sinus headache C.Episodic tension headache D.Idiopathic intracranial hypertension E.Migraine |
E.Migraine |
2 | 4 | 2013 | 12. 以下症状不会出现在偏头痛病人的是(2.5分) | A.偏瘫 B.恶心呕吐 C.意识丧失 D.眩晕 E.癫痫发作 |
E.癫痫发作 |
2 | 4 | 2013 | 13. A 55-year-old man who has a history of intravenous drug use and multiple sexual partners developed right hemiparesis and aphasia. Brain MRI showed restricted diffusion in the left hemisphere but in an unusual distribution. Rapid plasma reagin was detected as positive. CSF VDRL test was also found to be reactive. Based on the information provided, which of the following best describes this patient's disease? (2.5分) | A.Primary syphilis B.Secondary syphilis C.Meningovascular syphilis D.Tabes dorsalis E.Parenchymal syphilis |
C.Meningovascular syphilis |
2 | 4 | 2013 | 14. 葡葡膜炎最常见的种类为(2.5分) | A.前葡萄膜炎 B.中间葡萄膜炎 C.后葡萄膜炎 D.交感性眼炎 E.视网膜脉络膜炎 |
A.前葡萄膜炎 |
2 | 4 | 2013 | 15. 中间葡萄膜炎典型体征为(2.5分) | A.出现飞蚊症并有加重 B.视网膜新生血管 C.视神经萎缩 D.玻璃体雪球样混浊、雪堤样改变 E.黄斑囊样水肿 |
D.玻璃体雪球样混浊、雪堤样改变 |
2 | 4 | 2013 | 16. 下面对交感性眼炎描述正确的是 (2.5分) | A.发生在外伤后的单侧葡萄膜炎 B.发生在外伤后的单侧巩膜炎 C.发生在外伤后的双侧葡萄膜炎 D.发生在外伤后的双侧巩膜炎 E.炎症发生在受伤眼,不累及对侧眼 |
C.发生在外伤后的双侧葡萄膜炎 |
2 | 4 | 2013 | 17. 急性虹膜睫状体炎治疗关键是(2.5分) | A.缩瞳 B.全身应用皮质类固醇药物 C.维生素应用 D.扩瞳 E.抗生素使用 |
D.扩瞳 |
2 | 4 | 2013 | 18. 增殖性糖尿病视网膜病变的最主要标志是(2.5分) | A.微动脉瘤 B.视网膜内微血管异常 C.棉絮斑 D.新生血管形成 E.硬性渗出 |
D.新生血管形成 |
2 | 4 | 2013 | 19. 下列不属于高血压视网膜病变的有(2.5分) | A.静脉隐蔽现象 B.眼底视盘水肿 C.Roth斑 D.Salus征 E.Gunn征 |
C.Roth斑 |
2 | 4 | 2013 | 20. 视乳头水肿出现在高血压性视网膜病变的(2.5分) | A.1级 B.2级 C.3级 D.4级 E.1级和2级 |
D.4级 |
2 | 4 | 2013 | 21. 眼部化学烧伤的最重要处置措施(2.5分) | A.急诊手术治疗 B.抗菌素联合激素眼药水滴眼 C.立即就近取水,充分冲洗 D.请烧伤科会诊处理 E.请烧伤科会诊处理 |
A.急诊手术治疗 |
2 | 4 | 2013 | 22. 电光性眼炎是哪种辐射引起(2.5分) | A.紫外线辐射 B.红外线辐射 C.激光 D.微波 E.以上都不是 |
A.紫外线辐射 |
2 | 4 | 2013 | 23. 下面哪项是视神经挫伤的临床表现(2.5分) | A.视力下降 B.患眼瞳孔直接光反应正常 C.患眼瞳孔间接光反应迟钝 D.眼球破裂 E.以上都不是 |
A.视力下降 |
2 | 4 | 2013 | 24. 虹膜睫状体挫伤时,三分之一虹膜根部发生离断,这时瞳孔的形状为 (2.5分) | A.瞳孔膜闭 B.梅花瞳 C.D字形 D.瞳孔扩大 E.以上都不是 |
C.D字形 |
2 | 4 | 2013 | 25. 流行病学研究资料的分层分析,用于解决(2.5分) | A.混杂偏倚 B.选择偏倚 C.信息偏倚 D.失访偏倚 |
A.混杂偏倚 |
2 | 4 | 2013 | 26. 职业性疾病研究中的健康工人效应是属于(2.5分) | A.诊断怀疑偏倚 B.测量偏倚 C.入院率偏倚 D.易感性偏倚 |
D.易感性偏倚 |
2 | 4 | 2013 | 27. 当怀疑的病因(暴露)减少或去除,引起疾病发生率下降,可进一步支持因果关联,这一种判断运用的标准是(2.5分) | A.生物学合理性 B.剂量-反应关系 C.实验证据(终止效应) D.关联强度 |
C.实验证据(终止效应) |
2 | 4 | 2013 | 28. 幽门螺杆菌结合部位在胃窦细胞,它可随胃窦细胞进入十二脂肠引起炎症、削弱粘膜,使其易于遭受酸的损伤,由此支持幽门螺杆菌是十二批肠溃疡的发病原因,这一推理运用了因果判断标准中的 (2.5分) | A.关联的时序性 B.关联强度 C.关联的可重复性 D.关联的一致性 |
D.关联的一致性 |
2 | 4 | 2013 | 29. 病例对照研究利用医院就诊或住院病人作为研究对象时易导致的偏倚种类为(2.5分) | A.伯克森偏倚 B.报告偏倚 C.奈曼偏倚 D.检出症候偏倚 |
A.伯克森偏倚 |
2 | 4 | 2013 | 30. 在研究口服避孕药与乳腺癌的关系时,因为怀疑年龄是可能的混杂因素,因此,研究人员只选择40-60岁的妇女为研究对象,这种控制混杂偏倚的方法为(2.5分) | A.随机化 B.匹配 C.限制 D.盲法 |
C.限制 |
2 | 4 | 2013 | 31. 角膜是主要的屈光介质,其屈光度其相当于: (2.5分) | A.19D凸透镜 B.20D凹透镜 C.42D凸透镜 D.43D凸透镜 |
D.43D凸透镜 |
2 | 4 | 2013 | 32. 下列那条肌肉不受第Ⅲ颅神经支配: (2.5分) | A.上直肌 B.内直肌 C.下直肌 D.上斜肌; |
D.上斜肌; |
2 | 4 | 2013 | 33. 真菌性角膜炎的确诊可根据(2.5分) | A.植物性外伤史 B.刮片细菌学检查 C.溃疡形态及特征 D.刮片找到菌丝 |
D.刮片找到菌丝 |
2 | 4 | 2013 | 34. 占角膜病致盲原因首位的是(2.5分) | A.细菌性角膜炎 B.单纯疱疹病毒性角膜炎 C.真菌性角膜炎 D.衣原体性角膜炎 |
B.单纯疱疹病毒性角膜炎 |
2 | 4 | 2013 | 35. 用角膜映光点法检查眼位,当角膜映光点位于瞳孔缘时,其偏斜度大约是(2.5分) | A.25° B.20° C.5° D.5° |
C.5° |
2 | 4 | 2013 | 36. 患者,男性,农民,右眼被谷粒弹伤,视物模糊半月就诊,检查:右眼视力0.1,眼睑痉挛极轻,球结膜轻度混合性充血,角膜有黄白色牙膏样病灶,病灶旁可见伪足,角膜轻微水肿,前房积脓1mm,最可能的诊断是: (2.5分) | A.真菌性角膜溃疡炎 B.匐行性角膜溃疡 C.绿脓杆菌性角膜炎 D.病毒性角膜炎 |
A.真菌性角膜溃疡炎 |
2 | 4 | 2013 | 37. 关于近视眼的临床表现,下列哪项不常见 (2.5分) | A.远视力减退; B.视疲劳; C.眼位偏斜; D.眼轴变短。 |
D.眼轴变短。 |
2 | 4 | 2013 | 38. 下列哪项与轴性近视有关。(2.5分) | A.角膜曲率过大; B.球形晶状体 C.过度使用调节 D.眼轴>24mm |
D.眼轴>24mm |
2 | 4 | 2013 | 39. 近视眼患者未矫正前成像位置在视网膜的(2.5分) | A.前方 B.后方 C.视乳头部 D.不确定 |
A.前方 |
2 | 4 | 2013 | 40. 弱视治疗最简单.有效和常用的治疗方法是(2.5分) | A.遮盖疗法 B.红光刺激 C.压抑疗法 D.理疗 |
A.遮盖疗法 |
2 | 4 | 2014 | 1. 下列哪种细胞是传导发生器电位的? (2.0分) | A.色素上皮细胞 B.双极细胞 C.视锥细胞 D.视杆细胞 E.水平细胞 |
B.双极细胞 |
2 | 4 | 2014 | 2. 最小视网膜像的大小等于(2.0分) | A.视网膜中央凹处一个视杆细胞的平均直径 B.视网膜中央凹处一个视杆细胞的平均半径 C.视网膜中央凹处一个视锥细胞的平均半径 D.视网膜中央凹处一个视锥细胞的平均直径 |
D.视网膜中央凹处一个视锥细胞的平均直径 |
2 | 4 | 2014 | 3. 在同一光照条件下,视野最大的是(2.0分) | A.白色 B.红色 C.黄色 D.蓝色 E.绿色 |
A.白色 |
2 | 4 | 2014 | 4. 下列关于视锥细胞的哪个是正确的?(2.0分) | A.密集于视网膜中央凹 B.会聚现象普遍存在 C.能识别三种色素(红、绿、蓝) D.对视紫红质敏感 |
A.密集于视网膜中央凹 |
2 | 4 | 2014 | 5. 瞳孔对光反射 (2.0分) | A.中枢位于中脑 B.感受器是角膜 C.效应器是睫状肌 D.传出神经是动眼神经中的交感纤维 E.传入神经是三叉神经中的眼支 |
A.中枢位于中脑 |
2 | 4 | 2014 | 6. 意识障碍伴瞳孔缩小的最有可能的病变(2.0分) | A.阿托品中毒 B.癫痫发作 C.酒精中毒 D.巴比妥类中毒 E.低血糖状态 |
D.巴比妥类中毒 |
2 | 4 | 2014 | 7. 下列哪项不是头颅的痛敏结构(2.0分) | A.脑组织 B.头皮 C.颅骨骨膜 D.颅底硬脑膜 E.迷走神经 |
A.脑组织 |
2 | 4 | 2014 | 8. 慢性头痛伴有颅高压症状的最有可能的病变(2.0分) | A.蛛网膜下腔出血 B.偏头痛 C.颅内肿瘤 D.椎基底动脉供血不足 E.脑外伤后遗症 |
C.颅内肿瘤 |
2 | 4 | 2014 | 9. 各类脑膜炎当中,常出现颅神经损害的是(2.0分) | A.化脓性脑膜炎 B.病毒性脑膜炎 C.结核性脑膜炎 D.隐球菌性脑膜炎 E.C+D |
E.C+D |
2 | 4 | 2014 | 10. 化脓性脑膜炎最常见的病原体是(2.0分) | A.流血嗜血杆菌 B.肺炎链球菌 C.金黄色葡萄球菌 D.脑膜炎奈瑟菌 E.以上都不是 |
B.肺炎链球菌 |
2 | 4 | 2014 | 11. A 26-year-old woman with a history of recurrent sinusitis presents with fever, diplopia and facial pain. On examination, she has proptosis of the right eye with periorbital edema, limited extraocular movements of the right eye, and reduced sensation over her right upper face and cheek. Fundoscopic examination demonstrates papilledema and retinal hemorrhages and Her CSF shows openning pressure 280mmH2O with normal cell count. What is the most likely diagnosis? (2.0分) | A.Superior sagittal sinus thrombosis B.Meningitis C.Cavernous sinus thrombosis D.Brain abscess E.Transverse sinus thrombosis |
C.Cavernous sinus thrombosis |
2 | 4 | 2014 | 12. 单纯疱疹病毒性脑炎最常累及部位不包括(2.0分) | A.颞叶 B.额叶 C.边缘叶 D.丘脑 E.大脑皮层 |
D.丘脑 |
2 | 4 | 2014 | 13. A 19-year-old woman is brought to the emergency department after having a seizure. As per her roommate, the patient had been behaving oddly over the past week. Approximately 3 days ago, she developed fever, and for the past 2 days, she has been confused, talking strangely, and very lethargic. Today she had a generalized tonic-clonic seizure. Her CSF was reported as "reddish" in color, with 59 WBCs (normal up to 5 lymphocytes/μL), 3500 RBCs, protein of 58 mg/dL (normal up to 45 mg/dL), and normal glucose levels. An cranial CT is obtained and negative. Which is the most likely diagnosis? (2.0分) | A.West Nile encephalitis B.HSV encephalitis C.Bacterial meningitis D.Enterovirus meningitis E.Tuberculous meningitis |
B.HSV encephalitis |
2 | 4 | 2014 | 14. 引起脑炎的最常见病原体是(2.0分) | A.肺炎链球菌 B.带状疱疹病毒 C.单纯疱疹病毒 D.肠道病毒 E.以上都不是 |
C.单纯疱疹病毒 |
2 | 4 | 2014 | 15. 偏头痛的发病机制中不包括 (2.0分) | A.遗传机制 B.皮质扩散性抑制 C.炎性脱髓鞘学说 D.三叉神经血管学说 E.以上均不是 |
C.炎性脱髓鞘学说 |
2 | 4 | 2014 | 16. 偏头痛急性期治疗时首选的药物不包括 (2.0分) | A.NSAIDs B.曲坦类药物 C.麦角胺类 D.β-blocker E.地西泮 |
D.β-blocker |
2 | 4 | 2014 | 17. An overweight 28-year-old woman presents with three to four severe, debilitating headaches per month for the last 2 years. The headaches last 1 to 2 days. They are sometimes localized bifrontally, but more often localized to the right temple, right frontal region, and behind the right eye. She denies any prodrome or auras. The pain is usually a deep ache, but throbbing when severe. She sometimes gets some nausea, but no vomiting. She has to wear sunglasses and go to a quit room because she "can't function." What is the most likely diagnosis?(2.0分) | A.Cluster headache B.Intermittent sinus headache C.Episodic tension headache D.Migraine E.Idiopathic intracranial hypertension |
D.Migraine |
2 | 4 | 2014 | 18. 最常见的葡萄膜病变是(2.0分) | A.葡萄膜炎 B.葡萄膜原发性肿瘤 C.葡萄膜继发性肿瘤 D.葡萄膜退行性变 E.葡萄膜先天异常 |
A.葡萄膜炎 |
2 | 4 | 2014 | 19. 根据炎症的发病部位分类,以下哪一项不属于葡萄膜炎此分类(2.0分) | A.前葡萄膜炎 B.中间葡萄膜炎 C.后葡萄膜炎 D.全葡萄膜炎 E.眼内炎 |
E.眼内炎 |
2 | 4 | 2014 | 20. 下列哪种疾病属于葡萄膜炎的并发症 (2.0分) | A.白内障 B.青光眼 C.低眼压 D.黄斑水肿 E.以上都是 |
E.以上都是 |
2 | 4 | 2014 | 21. 下列哪种疾病属于感染性葡萄膜炎(2.0分) | A.Behcet病 B.Vogt-小柳-原田综合征 C.急性视网膜坏死综合征 D.交感性眼炎 E.Fuchs虹膜异色性葡萄膜炎 |
C.急性视网膜坏死综合征 |
2 | 4 | 2014 | 22. 糖尿病性视网膜病变最早期的表现是(2.0分) | A.硬性渗出 B.微血管瘤 C.棉絮斑 D.出血 E.视网膜水肿 |
B.微血管瘤 |
2 | 4 | 2014 | 23. 增殖性与非增殖性糖尿病视网膜病变的主要区别标志为(2.0分) | A.硬性渗出 B.新生血管 C.微动脉瘤 D.黄斑水肿 E.软性渗出 |
B.新生血管 |
2 | 4 | 2014 | 24. 女性,32岁,双眼睁不开1个月,半月前出现复视,以上症状晨起较轻,劳累或傍晚加重,眼睑检查发现双眼上睑不能上举,睑裂5mm,双眼视力1.0。诊断时应考虑以下哪种疾病可能(2.0分) | A.交感神经麻痹 B.动眼神经麻痹 C.内眦赘皮 D.重症肌无力 E.先天性上睑下垂 |
D.重症肌无力 |
2 | 4 | 2014 | 25. 成人时期双侧眼球突出常见原因(2.0分) | A.淋巴瘤 B.海绵状血管瘤 C.颈动脉海绵窦瘘 D.甲状腺相关眼病 E.蝶骨嵴脑膜瘤 |
D.甲状腺相关眼病 |
2 | 4 | 2014 | 26. 根据国际眼外伤学会理事会推荐的命名法,以下哪种外伤属于开放性眼外伤(2.0分) | A.眼睑裂伤 B.球结膜裂伤 C.角膜板层裂伤 D.前房出血 E.球内异物 |
E.球内异物 |
2 | 4 | 2014 | 27. 以下哪个不是虹膜挫伤瞳孔括约肌断裂的表现(2.0分) | A.瞳孔扩大。 B.近视力出现障碍。 C.瞳孔缘不规则裂口 D.瞳孔光反应迟钝 E.单眼复视 |
E.单眼复视 |
2 | 4 | 2014 | 28. 关于晶状体脱位的处理哪一项是错误的(2.0分) | A.晶状体脱位进入前房,需急诊手术摘除 B.晶状体嵌顿于瞳孔,需急诊手术摘除 C.晶状体半脱位时,需急诊手术摘除 D.晶状体脱位引发急性继发性青光眼,需急诊手术摘除 E.晶状体脱位于玻璃体腔,可行玻璃体手术切除 |
C.晶状体半脱位时,需急诊手术摘除 |
2 | 4 | 2014 | 29. 化学性眼烧伤的急救原则是 (2.0分) | A.到医院急救处理 B.中和注射 C.到医院用无菌生理盐水冲洗 D.包扎患眼 E.就地清水冲洗 |
E.就地清水冲洗 |
2 | 4 | 2014 | 30. 眼的屈光系统中,屈光率最大的是(2.0分) | A.角膜 B.晶状体 C.玻璃体 D.房水 E.视网膜 |
A.角膜 |
2 | 4 | 2014 | 31. 眼球中起暗房作用的是(2.0分) | A.视网膜 B.角膜和巩膜 C.角膜 D.巩膜 E.葡萄膜 |
E.葡萄膜 |
2 | 4 | 2014 | 32. 屈光系统不包括下列哪一项(2.0分) | A.晶状体 B.角膜 C.房水 D.虹膜 E.玻璃体 |
D.虹膜 |
2 | 4 | 2014 | 33. 以下哪一项属于屈光不正(2.0分) | A.近视 B.远视 C.散光 D.A和B E.以上都是 |
E.以上都是 |
2 | 4 | 2014 | 34. 在无调节情况下,平行光线进入眼内聚焦于视网膜之后,称为(2.0分) | A.弱视 B.近视 C.远视 D.散光 E.以上都不是 |
C.远视 |
2 | 4 | 2014 | 35. 由动眼神经支配的肌肉是(2.0分) | A.外直肌 B.上斜肌 C.额肌 D.眼轮匝肌 E.提上睑肌 |
E.提上睑肌 |
2 | 4 | 2014 | 36. 支配眼睑闭合的神经是(2.0分) | A.动眼神经 B.交感神经 C.面神经 D.三叉神经 E.滑车神经 |
C.面神经 |
2 | 4 | 2014 | 37. 用角膜映光点法检查眼位,当角膜映光点位于角膜缘时,其偏斜度大约是(2.0分) | A.45° B.25° C.15° D.10° E.5° |
A.45° |
2 | 4 | 2014 | 38. 眼外肌中最长的肌肉是(2.0分) | A.上斜肌 B.下直肌 C.上直肌 D.内直肌 E.外直肌 |
A.上斜肌 |
2 | 4 | 2014 | 39. 后天性麻痹性斜视常见的病因除了(2.0分) | A.炎症 B.外伤 C.血管性疾病 D.肿瘤 E.中枢神经控制失调 |
E.中枢神经控制失调 |
2 | 4 | 2014 | 40. 角膜由以下结构组成,除了(2.0分) | A.上皮层 B.基质层 C.前弹力膜 D.后弹力膜 E.内界膜 |
E.内界膜 |
2 | 4 | 2014 | 41. 角膜的营养来源于(2.0分) | A.角膜缘血管网 B.空气 C.泪膜 D.房水 E.以上皆是 |
E.以上皆是 |
2 | 4 | 2014 | 42. 治疗细菌性角膜炎,抗生素滴眼液的起始使用方法,正确的是(2.0分) | A.第一小时每5分钟一次,第二小时每15分钟一次,此后每30分钟一次维持 B.第一小时每5分钟一次,第二小时每10分钟一次,第三小时每20分钟一次,此后每30分钟一次维持 C.第一小时每10分钟一次,第二小时每20分钟一次,此后每30分钟一次维持 D.第一小时每10分钟一次,第二小时每20分钟一次,第三小时每30分钟一次,此后每一小时一次维持 E.每30分钟一次持续频点 |
A.第一小时每5分钟一次,第二小时每15分钟一次,此后每30分钟一次维持 |
2 | 4 | 2014 | 43. 下列抗真菌药物,属于多烯类的是(2.0分) | A.两性霉素B B.伏利康唑 C.伊曲康唑 D.5氟胞嘧啶 E.克拉霉素 |
A.两性霉素B |
2 | 4 | 2014 | 44. 对于单纯疱疹病毒不正确的描述是(2.0分) | A.初次感染后处于潜伏状态 B.主要有HSV-I和HSV-II两种血清型 C.感染角膜后可有上皮型、基质炎型、坏死型等多种表现 D.原发感染常见于老年人 E.可使用阿昔洛韦进行治疗 |
D.原发感染常见于老年人 |
2 | 4 | 2014 | 45. 当怀疑的病因(暴露)减少或去除,引起疾病发生率下降,可进一步支持因果关联,这一种判断运用的标准是(2.0分) | A.生物学合理性 B.剂量-反应关系 C.实验证据(终止效应) D.关联强度 |
C.实验证据(终止效应) |
2 | 4 | 2014 | 46. 幽门螺杆菌结合部位在胃窦细胞,它可随胃窦细胞进入十二脂肠引起炎症、削弱粘膜,使其易于遭受酸的损伤,由此支持幽门螺杆菌是十二批肠溃疡的发病原因,这一推理运用了因果判断标准中的(2.0分) | A.关联的时序性 B.关联强度 C.关联的可重复性 D.关联的一致性 |
D.关联的一致性 |
2 | 4 | 2014 | 47. 病例对照研究利用医院就诊或住院病人作为研究对象时易导致的偏倚种类为(2.0分) | A.伯克森偏倚 B.报告偏倚 C.奈曼偏倚 D.检出症候偏倚 |
A.伯克森偏倚 |
2 | 4 | 2014 | 48. 在研究口服避孕药与乳腺癌的关系时,因为怀疑年龄是可能的混杂因素,因此,研究人员只选择40-60岁的妇女为研究对象,这种控制混杂偏倚的方法为(2.0分) | A.随机化 B.匹配 C.限制 D.盲法 |
C.限制 |
2 | 4 | 2014 | 49. 流行病学研究资料的分层分析,用于解决(2.0分) | A.混杂偏倚 B.选择偏倚 C.信息偏倚 D.失访偏倚 |
A.混杂偏倚 |
2 | 4 | 2014 | 50. 职业性疾病研究中的健康工人效应是属于(2.0分) | A.诊断怀疑偏倚 B.测量偏倚 C.入院率偏倚 D.易感性偏倚 |
D.易感性偏倚 |
2 | 4 | 2015 | 1. 下列哪项不是因果关联的判断标准(2.0分) | A.样本大小 B.关联的时序性 C.关联强度 D.剂量-反应关系 |
A.样本大小 |
2 | 4 | 2015 | 2. 不同国家的学者在不同时间对本国人群作研究,均得出高盐饮食引起高血压,由此支持高盐饮食是影响高血压发生的危险因素这一推论运用了判断标准中的 (2.0分) | A.关联强度 B.剂量-反应关系 C.关联的可重复性 D.关联的时序性 |
C.关联的可重复性 |
2 | 4 | 2015 | 3. 因果关联是指 (2.0分) | A.暴露-疾病有较强的统计学关联 B.暴露-疾病分类资料存在相关 C.暴露-疾病排除偏倚后的关联 D.暴露-疾病有时间先后的无偏关联 |
D.暴露-疾病有时间先后的无偏关联 |
2 | 4 | 2015 | 4. 病例对照研究中,用于研究的对象一般是现患病例,由此可能产生的偏倚是(2.0分) | A.回忆偏倚 B.测量偏倚 C.易感性偏倚 D.奈曼偏倚 |
D.奈曼偏倚 |
2 | 4 | 2015 | 5. 病例对照研究中,如果研究者对病例组调查和询问的态度认真,而对对照组漫不经心地调查询问,从而导致错误的结论,其中可能发生的偏倚是(2.0分) | A.入院率偏倚 B.暴露怀疑偏倚 C.奈曼偏倚 D.易感性偏倚 |
B.暴露怀疑偏倚 |
2 | 4 | 2015 | 6. 某因素在病因学上与某疾病本无关联,但可引起所研究疾病的症状或体征,从而促使患者及早就诊,提高了早期病例检出率,从而过高的估计了暴露程度,这种偏倚是(2.0分) | A.诊断怀疑偏倚 B.检出症候偏倚 C.信息偏倚 D.混杂偏倚 |
B.检出症候偏倚 |
2 | 4 | 2015 | 7. 眼的折光系统不包括(2.0分) | A.晶状体 B.视网膜 C.角膜 D.房水 E.玻璃体 |
B.视网膜 |
2 | 4 | 2015 | 8. 视锥系统(2.0分) | A.对光敏感度高,有色觉,分辨力弱 B.对光敏感度低,有色觉,分辨力高 C.对光敏感度低,有色觉,分辨力弱 D.对光敏感度高,无色觉,分辨力弱 E.对光敏感度低,无色觉,分辨力高 |
B.对光敏感度低,有色觉,分辨力高 |
2 | 4 | 2015 | 9. 下列哪种因素可引起青光眼(2.0分) | A.眼内压降低 B.眼球变形 C.房水流失 D.房水排出受阻 E.角膜曲度改变 |
D.房水排出受阻 |
2 | 4 | 2015 | 10. 下列哪项关于偏头痛的描述是不正确的(2.0分) | A.反复头痛 B.单侧、搏动性头痛 C.日常体力活动可加剧头痛 D.不伴有呕吐、畏光畏声 |
D.不伴有呕吐、畏光畏声 |
2 | 4 | 2015 | 11. 下列关于视锥细胞的哪个是正确的?(2.0分) | A.密集于视网膜中央凹 B.会聚现象普遍存在 C.能识别三种色素(红、绿、蓝) D.对视紫红质敏感 |
A.密集于视网膜中央凹 |
2 | 4 | 2015 | 12. 意识障碍伴瞳孔缩小的最有可能的病变:(2.0分) | A.巴比妥类中毒 B.低血糖状态 C.酒精中毒 D.阿托品中毒 E.癫痫发作 |
A.巴比妥类中毒 |
2 | 4 | 2015 | 13. 6岁患儿,头痛反复发作并伴有肢体偏瘫,头痛消失后偏瘫持续数小时可缓解。家族中有类似患者。头部MRI检查无明显异常。该患儿最可能的诊断是: (2.0分) | A.短暂性脑缺血发作 B.家族型偏瘫型偏头痛 C.基底动脉型偏头痛? D.脑肿瘤 E.遗传性头痛? |
B.家族型偏瘫型偏头痛 |
2 | 4 | 2015 | 14. 下列哪项关于轻度昏迷的描述是不正确的(2.0分) | A.对疼痛刺激尚可出现痛苦表情或肢体退缩等防御反应 B.有自主活动 C.意识大部分丧失 D.对声、光刺激无反应 E.角膜反射、眼球运动、吞咽反射存在 |
B.有自主活动 |
2 | 4 | 2015 | 15. 以下哪项不属于脑膜炎病因(2.0分) | A.梅毒螺旋体 B.病毒 C.肿瘤脑膜转移 D.脑梗死 E.化疗 |
D.脑梗死 |
2 | 4 | 2015 | 16. 化脓性脑膜炎最常见的病原体是(2.0分) | A.流血嗜血杆菌 B.肺炎链球菌 C.金黄色葡萄球菌 D.脑膜炎奈瑟菌 E.以上都不是 |
B.肺炎链球菌 |
2 | 4 | 2015 | 17. A 26-year-old woman with a history of recurrent sinusitis presents with fever, diplopia and facial pain. On examination, she has proptosis of the right eye with periorbital edema, limited extraocular movements of the right eye, and reduced sensation over her right upper face and cheek. Fundoscopic examination demonstrates papilledema and retinal hemorrhages and Her CSF shows openning pressure 280mmH2O with normal cell count. What is the most likely diagnosis? (2.0分) | A.Superior sagittal sinus thrombosis B.Meningitis C.Cavernous sinus thrombosis D.Brain abscess E.Transverse sinus thrombosis |
C.Cavernous sinus thrombosis |
2 | 4 | 2015 | 18. 单纯疱疹病毒性脑炎最常累及部位不包括:(2.0分) | A.颞叶 B.额叶 C.边缘叶 D.丘脑 E.大脑皮层 |
D.丘脑 |
2 | 4 | 2015 | 19. A 19-year-old woman is brought to the emergency department after having a seizure. As per her roommate, the patient had been behaving oddly over the past week. Approximately 3 days ago, she developed fever, and for the past 2 days, she has been confused, talking strangely, and very lethargic. Today she had a generalized tonic-clonic seizure. Her CSF was reported as "reddish" in color, with 59 WBCs (normal up to 5 lymphocytes/μL), 3500 RBCs, protein of 58 mg/dL (normal up to 45 mg/dL), and normal glucose levels. An cranial CT is obtained and negative. Which is the most likely diagnosis? (2.0分) | A.West Nile encephalitis B.HSV encephalitis C.Bacterial meningitis D.Enterovirus meningitis E.Tuberculous meningitis |
B.HSV encephalitis |
2 | 4 | 2015 | 20. 引起脑炎的最常见病原体是 (2.0分) | A.肺炎链球菌 B.带状疱疹病毒 C.单纯疱疹病毒 D.肠道病毒 E.以上都不是 |
C.单纯疱疹病毒 |
2 | 4 | 2015 | 21. 自身免疫性脑炎最常见类型(2.0分) | A.抗LGI-1脑炎 B.抗NMDA受体脑炎 C.抗Caspr2脑炎 D.West Nile 脑炎 E.抗GABA-A受体脑炎 |
B.抗NMDA受体脑炎 |
2 | 4 | 2015 | 22. 偏头痛的发病机制中不包括(2.0分) | A.遗传机制 B.皮质扩散性抑制 C.炎性脱髓鞘学说 D.三叉神经血管学说 E.以上均不是 |
C.炎性脱髓鞘学说 |
2 | 4 | 2015 | 23. 偏头痛急性期治疗时首选的药物不包括(2.0分) | A.NSAIDs B.曲坦类药物 C.麦角胺类 D.β-blocker E.地西泮 |
D.β-blocker |
2 | 4 | 2015 | 24. An overweight 28-year-old woman presents with three to four severe, debilitating headaches per month for the last 2 years. The headaches last 1 to 2 days. They are sometimes localized bifrontally, but more often localized to the right temple, right frontal region, and behind the right eye. She denies any prodrome or auras. The pain is usually a deep ache, but throbbing when severe. She sometimes gets some nausea, but no vomiting. She has to wear sunglasses and go to a quit room because she "can't function." What is the most likely diagnosis? (2.0分) | A.Cluster headache B.Intermittent sinus headache C.Episodic tension headache D.Migraine E.Idiopathic intracranial hypertension |
D.Migraine |
2 | 4 | 2015 | 25. 睑腺炎最常见的致病菌是:(2.0分) | A.链球菌 B.金黄色葡萄球菌 C.莫-阿双杆菌 D.真菌 E.绿脓杆菌 |
B.金黄色葡萄球菌 |
2 | 4 | 2015 | 26. 外睑腺炎手术时,其手术切口应(2.0分) | A.与皮肤面垂直 B.与睑板面垂直 C.与睑缘垂直 D.与睑缘平行 E.与眶缘垂直 |
D.与睑缘平行 |
2 | 4 | 2015 | 27. 老年人睑板腺囊肿(霰粒肿)术后复发,首先应排除(2.0分) | A.手术未切除干净 B.瘢痕组织增生 C.睑板腺癌可能 D.继发感染 E.局部红肿 |
C.睑板腺癌可能 |
2 | 4 | 2015 | 28. 眼眶病分类中哪项不正确: (2.0分) | A.肿瘤病变 B.炎症性病变 C.先天性异常 D.血管性病变 E.囊性病变 |
E.囊性病变 |
2 | 4 | 2015 | 29. 眶蜂窝织炎处理最重要的是: (2.0分) | A.治疗原发灶 B.积极手术 C.适当的皮质激素治疗 D.足量的广谱抗生素 E.局部加强用眼液、眼膏抗炎 |
D.足量的广谱抗生素 |
2 | 4 | 2015 | 30. 接诊眼外伤病人是,首先应注意(2.0分) | A.视力情况 B.眼球有无破裂 C.有无感染 D.有无休克及重要脏器损伤 E.有无异物 |
D.有无休克及重要脏器损伤 |
2 | 4 | 2015 | 31. 眼外伤检查前要全面询问病史,包括: (2.0分) | A.何时、怎样受伤以及致伤力大小 B.有否有眼内异物的可能 C.受伤前眼部的状态 D.经何急诊处理(TAT注射、抗生素等) E.以上各项均是 |
E.以上各项均是 |
2 | 4 | 2015 | 32. 碱性烧伤的特点为: (2.0分) | A.使组织蛋白凝固坏死 B.使组织坏死 C.能溶解脂肪和蛋白质,使细胞分解坏死 D.使角膜上皮点状脱落 E.破坏内皮细胞的脂肪外膜 |
C.能溶解脂肪和蛋白质,使细胞分解坏死 |
2 | 4 | 2015 | 33. 最容易引起感染的角膜异物是(2.0分) | A.塑料 B.铁屑 C.植物性异物 D.灰尘 E.煤屑 |
C.植物性异物 |
2 | 4 | 2015 | 34. 交感性眼炎的潜伏期一般为(2.0分) | A.3-4个月 B.2-3个月 C.2周-2个月 D.4-6个月 E.1-2周 |
C.2周-2个月 |
2 | 4 | 2015 | 35. 临床上最早出现的、比较确切的糖尿病性视网膜病变的体征是: (2.0分) | A.微动脉瘤 B.硬性渗出 C.视网膜内出血 D.视网膜水肿 E.新生血管形成 |
A.微动脉瘤 |
2 | 4 | 2015 | 36. 糖尿病性视网膜病变引起视力丧失的主要原因是(2.0分) | A.慢性黄斑水肿 B.玻璃体积血 C.牵拉性视网膜脱离 D.新生血管性青光眼 E.以上均是 |
E.以上均是 |
2 | 4 | 2015 | 37. 白塞病的主要体征不包括(2.0分) | A.复发性口腔溃疡 B.关节红肿疼痛 C.皮肤结节性红斑 D.生殖器溃疡 E.复发性前房积脓性虹膜睫状体炎 |
B.关节红肿疼痛 |
2 | 4 | 2015 | 38. 关于甲状腺相关眼病的病程,下列不正确的是(2.0分) | A.与自身免疫相关 B.有明显个体差异 C.伴有甲亢的患者甲亢治愈后眼病多可自行治愈 D.甲亢治疗后少数患者会引起眼病的加重 E.甲状腺相关眼病的病程通常经历活动期和静止期两个过程,活动期对眼病的影响较大,也是药物或放射治疗最佳时期 |
C.伴有甲亢的患者甲亢治愈后眼病多可自行治愈 |
2 | 4 | 2015 | 39. 眼型重症肌无力可出现的临床表现包括: (2.0分) | A.复视 B.眼球活动障碍 C.上睑下垂 D.斜视 E.以上均是 |
E.以上均是 |
2 | 4 | 2015 | 40. 角膜各层组织中,最厚的一层是(2.0分) | A.上皮层 B.基质层 C.前弹力膜 D.后弹力膜 E.内皮层 |
B.基质层 |
2 | 4 | 2015 | 41. 角膜的营养来源于(2.0分) | A.角膜缘血管网 B.空气 C.泪膜 D.房水 E. 以上皆是 |
E. 以上皆是 |
2 | 4 | 2015 | 42. 关于角膜的描述,不正确的是 (2.0分) | A.组织学分为5层 B.屈光系统中,屈光力最强 C.中央薄,周边厚 D.前弹力层和后弹力层有再生能力 E.内皮细胞层无再生能力 |
D.前弹力层和后弹力层有再生能力 |
2 | 4 | 2015 | 43. 真菌性角膜炎治疗时,不正确的是 (2.0分) | A.局部应用抗真菌药物 B.必要时全身应用抗真菌药物 C.病原未明确时,使用光谱抗真菌药物 D.需根据病原培养及药敏结果调整用药 E.眼部炎症反应重时,可加用局部激素抗炎治疗 |
E.眼部炎症反应重时,可加用局部激素抗炎治疗 |
2 | 4 | 2015 | 44. 正常眼压值范围 (2.0分) | A.11~20 mmHg B.10~20 mmHg C.10~21 mmHg D.11~22 mmHg E.11~21 mmHg |
C.10~21 mmHg |
2 | 4 | 2015 | 45. 急性闭角型青光眼的发病因素不包括 (2.0分) | A.眼轴短 B.大角膜 C.前房浅 D.房角窄 E.晶状体膨胀 |
B.大角膜 |
2 | 4 | 2015 | 46. 哪项不是开角型青光眼的特点(2.0分) | A.房角开放 B.患者多无自觉症状 C.前房极浅,虹膜膨隆 D.双眼发病 E.眼底有青光眼性视神经损害 |
C.前房极浅,虹膜膨隆 |
2 | 4 | 2015 | 47. 葡萄膜的组成,正确的是(2.0分) | A.角膜、虹膜、脉络膜 B.虹膜、睫状体、视网膜 C.虹膜、视网膜、脉络膜 D.角膜、虹膜、视网膜 E.虹膜、睫状体、脉络膜 |
E.虹膜、睫状体、脉络膜 |
2 | 4 | 2015 | 48. 葡萄膜炎的治疗药物不包括(2.0分) | A.毛果芸香碱 B.阿托品 C.糖皮质激素 D.非甾体抗炎药 E.抗病毒药物 |
A.毛果芸香碱 |
2 | 4 | 2015 | 49. KP是指(2.0分) | A.角膜内皮细胞 B.角膜混浊 C.角膜后沉淀物 D.前房渗出物 E.防水闪灰 |
C.角膜后沉淀物 |
2 | 4 | 2015 | 50. 睫状充血与结膜充血的鉴别,说法错误的是(2.0分) | A.睫状充血以角膜缘血管为主 B.结膜充血以穹窿部血管为主 C.眼表点用肾上腺素后,睫状充血减轻,结膜充血改变不明显 D.睫状充血颜色多暗红,结膜充血颜色多鲜红 E.睫状充血多无分泌物,结膜充血常有分泌物 |
B.结膜充血以穹窿部血管为主 |
2 | 5 | 2007 | 1. 在脑电图检查中,根据国际10-20系统电极放置法,参考电极放置于何处:(2.0分) | A.额部 B.乳突 C.枕部 D.顶部 |
B.乳突 |
2 | 5 | 2007 | 2. Methods for controlling confounding include all of the following Except-(2.0分) | A.Restriction during sample selection B.Stratification during analysis C.Randomization during sample selection D.Using cohort study design E.Matching on potential confounders |
D.Using cohort study design |
2 | 5 | 2007 | 3. Information bias include all of the following biases, EXCEPT-(2.0分) | A.Recalling bias B.Interviewer bias C.Volunteer bias D.Exposure Suspicion bias |
C.Volunteer bias |
2 | 5 | 2007 | 4. Parkinson病rCBF显像的特征性改变是:(2.0分) | A.大小脑交叉失联络 B.大脑皮质放射性分布增高 C.两侧基底核区放射性分布减少 D.两侧基地核放射性分布增高 E.大脑皮质放射性分布减低 |
C.两侧基底核区放射性分布减少 |
2 | 5 | 2007 | 5. 流行病学的病因定义为 (2.0分) | A.存在时必定引起疾病的因素 B.使疾病发生概率升高的因素 C.疾病发生机制中的生物因素 D.对疾病发生必不可少的因素 E.引起疾病发生的诸多因素; |
B.使疾病发生概率升高的因素 |
2 | 5 | 2007 | 6. 运动单位动作电位时限增宽,波幅增高,多相波百分比增高,神经传导速度减慢最符合哪种病变:(2.0分) | A.肌肉源性损害 B.神经-肌肉接头损害 C.神经源性损害 D.混合性 |
C.神经源性损害 |
2 | 5 | 2007 | 7. 背侧丘脑中,与躯体运动调节有关的特异性中继核是 (2.0分) | A.腹后核 B.腹前核 C.前核 D.内侧膝状体 E.外侧膝状体 |
B.腹前核 |
2 | 5 | 2007 | 8. 第1躯体运动区位于(2.0分) | A.中央后回和中央旁小叶后部 B.额中回后部 C.额下回后部 D.中央前回和中央旁小叶前部 E.中央前回和中央后回 |
D.中央前回和中央旁小叶前部 |
2 | 5 | 2007 | 9. Hungtington舞蹈病的病理生理机制(2.0分) | A.黑质多巴胺能神经元缺失 B.A型溶血链球菌感染 C.γ-氨基丁酸的合成减少 D.铜代谢障碍 E.脑动脉硬化 |
C.γ-氨基丁酸的合成减少 |
2 | 5 | 2007 | 10. 萎缩性瘫痪是由于 (2.0分) | A.中央前回运动区全部损伤 B.下运动神经元麻痹 C.黑质病变 D.纹状体受损 E.小脑后叶受损 |
B.下运动神经元麻痹 |
2 | 5 | 2007 | 11. 安坦治疗帕金森病的作用机制是 (2.0分) | A.阻断多巴胺受体,降低黑质-纹状体通路中多巴胺的作用 B.阻断中枢胆碱受体,减弱黑质-纹状体通路中乙酰胆碱的作用 C.兴奋多巴胺受体,增强黑质-纹状体通路中多巴胺的作用 D.兴奋中枢胆碱受体,增强黑质-纹状体通路中乙酰胆碱的作用 E.抑制5-HT在脑中的生成和作用 |
B.阻断中枢胆碱受体,减弱黑质-纹状体通路中乙酰胆碱的作用 |
2 | 5 | 2007 | 12. H反射检查通常在哪一区域肌肉(2.0分) | A.股四头肌 B.腓肠肌 C.肱二头肌 D.任意肢体肌肉 |
B.腓肠肌 |
2 | 5 | 2007 | 13. Hungtington舞蹈病的遗传方式(2.0分) | A.常染色体隐性遗传 B.常染色体显性遗传 C.X染色体显性遗传 D.X染色体隐性遗传 E.Y染色体显性遗传 |
B.常染色体显性遗传 |
2 | 5 | 2007 | 14. 遗忘机制的假说包括(2.0分) | A.衰减说 B.线索依赖说 C.压抑说 D.三者皆有 |
D.三者皆有 |
2 | 5 | 2007 | 15. 肝豆状核变性是由于(2.0分) | A.铅沉积 B.钙沉积 C.铜沉积 D.铁沉积 E.锡沉积 |
C.铜沉积 |
2 | 5 | 2007 | 16. 以下有关脑肿瘤的葡萄糖代谢影象正确的是:(2.0分) | A.放疗和化疗效果明显者后期典型表现是局部放射性增高 B.脑肿瘤复发常表现为放射性增高 C.瘢痕组织局部放射性明显增加 D.原发脑肿瘤典型表现是局部放射性减低 E.放疗后水肿部位常表现为局部放射性浓集 |
B.脑肿瘤复发常表现为放射性增高 |
2 | 5 | 2007 | 17. 急性蜂窝组织炎时放射性核素骨显像的表现是:(2.0分) | A.血流、血池相放射性明显增加,且消失迅速,延迟相正常 B.血流相正常,血池相放射性增高,消失缓慢,延迟相正常 C.血流相放射性灌注增加,血池相及延迟相呈放射性减低 D.血流、血池相正常,延迟相呈放射性明显增加 E.以上均不是 |
A.血流、血池相放射性明显增加,且消失迅速,延迟相正常 |
2 | 5 | 2007 | 18. 左旋多巴常与卡比多巴合用,因为卡比多巴(2.0分) | A.有多巴胺能作用 B.是一种止吐剂 C.有抗组胺作用 D.有抗胆碱能作用 E.是一种多巴脱羧酶抑制剂 |
E.是一种多巴脱羧酶抑制剂 |
2 | 5 | 2007 | 19. 舞蹈病主要是因为下列哪条通路受累引起的?(2.0分) | A.黑质‐纹状体胆碱能抑制通路 B.黑质‐纹状体多巴胺能抑制通路 C.黑质‐纹状体胆碱能易化通路 D.黑质‐纹状体多巴胺能易化通路 E.纹状体内胆碱能和γ‐氨基丁酸能神经元病变 |
E.纹状体内胆碱能和γ‐氨基丁酸能神经元病变 |
2 | 5 | 2007 | 20. 关于病因的具体所指,错误的是: (2.0分) | A.包括疾病的启动因素或病原体; B.包括外围的远因以及治病机制的近因; C.包括宿主、环境和治病因素(动因); D.包括生物、心里和社会因素; E.包括交错病因两种的直接和间接病因; |
A.包括疾病的启动因素或病原体; |
2 | 5 | 2007 | 21. 关于F-波的电生理基础(2.0分) | A.一个小的逆行传导的肌肉反应动作电位 B.周围神经接受超强刺激后,引起一个大的顺行传导的复合肌肉动作电位 C.周围神经接受超强刺激后,引起一个小的顺行传导的复合肌肉动作电位 D.一个大的逆行传导的肌肉反应动作电位 |
B.周围神经接受超强刺激后,引起一个大的顺行传导的复合肌肉动作电位 |
2 | 5 | 2007 | 22. 脊髓突然被横断后,断面以下脊髓所支配的骨骼肌的紧张度(2.0分) | A.降低,能恢复但与正常不同 B.增强,但不能恢复正常 C.增强,但能恢复正常 D.降低,但能恢复正常 |
B.增强,但不能恢复正常 |
2 | 5 | 2007 | 23. 关于帕金森病临床表现一般不出现的是(2.0分) | A.路标现象 B.齿轮样强直 C.大写征 D.搓丸样动作 E.面具脸 |
C.大写征 |
2 | 5 | 2007 | 24. 三相波可见于以下疾病,哪一项是错误的:(2.0分) | A.肝性脑病 B.CJD C.其他原因所致中毒代谢性脑病 D.脱髓鞘性脑病 E.脱髓鞘性脑病 |
D.脱髓鞘性脑病 |
2 | 5 | 2007 | 25. 帕金森病患者的常见步态为(2.0分) | A.偏瘫步态 B.剪刀步态 C.慌张步态 D.共济失调步态 E.摇摆步态 |
C.慌张步态 |
2 | 5 | 2007 | 26. 以下哪一种异常EEG不属于癫痫样放电:(2.0分) | A.棘波 B.尖波 C.3Hz棘慢波综合 D.以上都不是 |
D.以上都不是 |
2 | 5 | 2007 | 27. 与强迫特质对应的正常人格特质是(2.0分) | A.社交性 B.外向性 C.责任心 D.开放性 |
C.责任心 |
2 | 5 | 2007 | 28. H反射检查通常在哪一区域肌肉 (2.0分) | A.腓肠肌 B.肱二头肌 C.股四头肌 D.任意肢体肌肉 |
A.腓肠肌 |
2 | 5 | 2007 | 29. 下列哪一种因果联结方式是正确而完整的?(2.0分) | A.多因多果; B.单因多果; C.多因单果; D.单因单果 E.直接/间接病因链; |
A.多因多果; |
2 | 5 | 2007 | 30. 清醒状态下的正常人几乎没有下列哪种节律波:(2.0分) | A.8-13 Hz B.4-7 Hz C.15-25 Hz D.<4 Hz |
D.<4 Hz |
2 | 5 | 2007 | 31. The following regions have mainly GABAergic output neurons Except-: (2.0分) | A.The caudate nucleus and the putamen. B.The pars compacta of the substantia nigra C.The external segment of the globus pallidus (GPe) D.The internal segment of the globus pallidus (GPi). |
B.The pars compacta of the substantia nigra |
2 | 5 | 2007 | 32. Which type of the study designs can minimize the recalling bias?(2.0分) | A.Retrospective cohort study B.Case-control study C.Cross-sectional study D.Prospective cohort study |
D.Prospective cohort study |
2 | 5 | 2007 | 33. 决定人执行功能的脑区是(2.0分) | A.顶叶 B.额叶 C.颞叶 D.视上回 |
B.额叶 |
2 | 5 | 2007 | 34. 运动单位动作电位时限增宽,波幅增高,多相波百分比增高,神经传导速度减慢最符合哪种病变:(2.0分) | A.肌肉源性损害 B.神经-肌肉接头损害 C.神经源性损害 D.混合性 |
C.神经源性损害 |
2 | 5 | 2007 | 35. NEO-PI-R是根据下列哪种方法编制的(2.0分) | A.逻辑分析法 B.经验法 C.因素分析法 D.综合法 |
C.因素分析法 |
2 | 5 | 2007 | 36. 迄今为止,在家族性PD中发现多个遵循孟德尔方式遗传的突变基因,其中以下哪些不是以隐性方式遗传? (2.0分) | A.DJ-1 B.PINK1 C.PARKIN D.SNCA |
D.SNCA |
2 | 5 | 2007 | 37. 鱼滕酮是脂溶性的天然杀虫剂,它在多胺能神经元内的主要作用靶点是? (2.0分) | A.线粒体复合物I B.蛋白酶体 C.线粒体复合物II D.溶酶体 E.细胞膜上的磷脂 |
A.线粒体复合物I |
2 | 5 | 2007 | 38. 下列不能用于治疗帕金森病的药物有 (2.0分) | A.L-dopa B.Diazepam C.Selegiline(司来吉兰) D.Trihexyphenidyl(苯海索) |
B.Diazepam |
2 | 5 | 2007 | 39. L-dopa引起的不良反应不包括下列哪些项? (2.0分) | A.不随意运动 B.精神障碍 C.体位性低血压 D.牙龈增生 E.胃肠道反应 |
D.牙龈增生 |
2 | 5 | 2007 | 40. Bromocriptine(溴隐亭)治疗帕金森病,是由于 (2.0分) | A.提高脑内DA浓度 B.激动GABA受体 C.中枢抗胆碱作用 D.激动DA受体 E.使DA降解减少 |
D.激动DA受体 |
2 | 5 | 2007 | 41. 下列关于脊髓休克的论述,错误的是(2.0分) | A.动物进化程度越高,其恢复速度越慢 B.脊髓休克的产生,是由于突然失去了高位中枢的调节作用 C.断面以下的脊髓反射、感觉和随意运动可逐渐恢复 D.脊髓突然被横断后,断面以下的脊髓反射活动即暂时丧失 E.反射恢复后,第二次横断脊髓,不再导致休克 |
C.断面以下的脊髓反射、感觉和随意运动可逐渐恢复 |
2 | 5 | 2007 | 42. 人脑的左半球在下列方面占优势(2.0分) | A.概念、抽象、逻辑 B.音乐绘画 C.空间知觉 D.以上都不是 |
A.概念、抽象、逻辑 |
2 | 5 | 2007 | 43. 知觉的特性包括(2.0分) | A.理解 B.选择 C.恒常 D.三者皆有 |
D.三者皆有 |
2 | 5 | 2007 | 44. 关于F-波的电生理基础(2.0分) | A.一个小的逆行传导的肌肉反应动作电位 B.周围神经接受超强刺激后,引起一个大的顺行传导的复合肌肉动作电位 C.周围神经接受超强刺激后,引起一个小的顺行传导的复合肌肉动作电位 D.一个大的逆行传导的肌肉反应动作电位 |
B.周围神经接受超强刺激后,引起一个大的顺行传导的复合肌肉动作电位 |
2 | 5 | 2007 | 45. 注意的功能包括(2.0分) | A.对活动进行调节与监督 B.保持 C.选择 D.以上皆有 |
D.以上皆有 |
2 | 5 | 2007 | 46. 流行病中病因推断的性质是 (2.0分) | A.医学研究中因果关系的逻辑标准; B.流行病学的一般思维方式和逻辑方法; C.分析流行病学的指导框架和评价准则; D.预防医学或公共卫生学的逻辑思维框架; E.人群研究的一般思维方式和逻辑方法; |
C.分析流行病学的指导框架和评价准则; |
2 | 5 | 2007 | 47. 骨显像时超级影像是指下列哪种情况:(2.0分) | A.肾影明显,膀胱内放射性增多,骨影浓而清晰,软组织本底高 B.肾影明显,膀胱内放射性减少,骨影淡而不清晰,软组织本底高 C.肾影不明显,膀胱内放射性减少,骨影淡而不清晰,软组织本底低 D.肾影不明显,膀胱内放射性很少,骨影浓而清晰,软组织本底低 E.肾影不明显,膀胱内放射性增多,骨影浓而清晰,软组织本底低 |
D.肾影不明显,膀胱内放射性很少,骨影浓而清晰,软组织本底低 |
2 | 5 | 2007 | 48. 内囊位于(2.0分) | A.豆状核与尾状核之间 B.豆状核与屏状核之间 C.新纹状体之间 D.背侧丘脑与尾状核之间 E.尾状核、背侧丘脑与豆状核之间 |
E.尾状核、背侧丘脑与豆状核之间 |
2 | 5 | 2007 | 49. 急性骨髓炎诊断与鉴别诊断最恰当的手段是:(2.0分) | A.ECT骨局部断层显像 B.ECT局部骨静态显像 C.X-CT检查 D.ECT骨三相显像 E.X片检查 |
D.ECT骨三相显像 |
2 | 5 | 2007 | 50. 有关SPECT脑血流显像诊断TIA,以下说法错误的是:(2.0分) | A.可以灵敏的检出TIA缺血病灶 B.发病后尽早检查可提高诊断阳性率 C.影像上常表现为相应部位的放射性稀疏或缺损区 D.发病后立即检查不能提高诊断阳性率 |
D.发病后立即检查不能提高诊断阳性率 |
2 | 5 | 2008 | 1. L-dopa引起的不良反应不包括下列哪些项? (3.0分) | A.不随意运动 B.牙龈增生 C.体位性低血压 D.精神障碍 E.胃肠道反应 |
B.牙龈增生 |
2 | 5 | 2008 | 2. Bromocriptine(溴隐亭)治疗帕金森病,是由于 (3.0分) | A.中枢抗胆碱作用 B.激动DA受体 C.激动GABA受体 D.提高脑内DA浓度 E.使DA降解减少 |
B.激动DA受体 |
2 | 5 | 2008 | 3. 视觉功能主要依赖于视觉皮层,但视觉皮层单独工作并不能很好地完成视觉任务,它必须动用 (3.0分) | A.第一机能系统保证必要的皮层张力和维持一定的觉醒水平 B.第二机能系统实现对通过视神经进入大脑的视觉信息进行分析和综合 C.第三机能系统保证有目的探索,比如眼睛随注视目标的运动 D.以上皆有 |
D.以上皆有 |
2 | 5 | 2008 | 4. 感觉的功能有 (3.0分) | A.生存(感觉危险、逃避,适应) B.享受(感觉寻求,唤醒,满足) C.以上两者皆有 D.以上两者皆无 |
C.以上两者皆有 |
2 | 5 | 2008 | 5. 记忆的基本过程包括(3.0分) | A.识记 B.保持 C.再认与再现 D.三者皆有 |
D.三者皆有 |
2 | 5 | 2008 | 6. 遗忘机制的假说包括(3.0分) | A.衰减说 B.压抑说 C.线索依赖说 D.三者皆有 |
D.三者皆有 |
2 | 5 | 2008 | 7. 思维的特征除了概括性外,还有(3.0分) | A.间接性 B.情绪性 C.全或无特性 D.不应期特性 |
A.间接性 |
2 | 5 | 2008 | 8. 注意的功能包括(3.0分) | A.选择 B.保持 C.对活动进行调节与监督 D.以上皆有 |
D.以上皆有 |
2 | 5 | 2008 | 9. 根据传入纤维的不同来源,大脑小脑这一机能区是由小脑哪部分和齿状核构成的?(3.0分) | A.绒球小结叶 B.小脑半球外侧部 C.小脑蚓 D.小脑半球中间部 |
B.小脑半球外侧部 |
2 | 5 | 2008 | 10. 运动性语言中枢位于(3.0分) | A.额下回后部 B.颞上回后部 C.额中回后部 D.角回 |
A.额下回后部 |
2 | 5 | 2008 | 11. 不经过内囊后肢的投射纤维是 (4.0分) | A.皮质脊髓束 B.丘脑中央辐射 C.顶桥束 D.皮质核束 |
D.皮质核束 |
2 | 5 | 2008 | 12. 基底神经核的间接通路一定含有以下神经核参与 (3.0分) | A.纹状体和内侧苍白球 B.纹状体和黑质 C.外侧苍白球和底丘脑核 D.外侧苍白球和红核 |
B.纹状体和黑质 |
2 | 5 | 2008 | 13. 全身骨骼显像利用以下哪种机制:(3.0分) | A.化学吸附和离子交换 B.选择性排泄 C.循环通路 D.细胞吞噬 E.通透弥散 |
A.化学吸附和离子交换 |
2 | 5 | 2008 | 14. 下列那一项不是骨显像的适应症:(3.0分) | A.诊断转移性骨肿瘤 B.粉碎性骨折 C.移植骨 D.代谢性骨疾病 E.应力性骨折 |
B.粉碎性骨折 |
2 | 5 | 2008 | 15. 临床上判断早期股骨头缺血性坏死最好的方法是(3.0分) | A.X片检查 B.超声检查 C.ECT全身骨显像 D.ECT局部断层检查 E.X-CT检查 |
D.ECT局部断层检查 |
2 | 5 | 2008 | 16. 用脑血流灌注显像诊断脑死亡,最特征的影像是什么?(3.0分) | A.两大脑半球没有放射性分布 B.一侧大脑半球没有放射性分布 C.头皮不显影 D.两大脑半球放射性低下 E.上失状窦显影 |
A.两大脑半球没有放射性分布 |
2 | 5 | 2008 | 17. 脑肿瘤的葡萄糖代谢影像以下正确的是:(3.0分) | A.脑肿瘤复发常表现为放射性增高 B.放疗和化疗效果明显者后期典型表现是局部放射性增高 C.瘢痕组织局部放射性明显增加 D.原发脑肿瘤典型表现是局部放射性减低 E.放疗后水肿部位常表现为局部放射性浓集 |
A.脑肿瘤复发常表现为放射性增高 |
2 | 5 | 2008 | 18. 进行rCBF显像,以下哪一项检查前准备不正确:(3.0分) | A.口服过氯酸钾以封闭脉络丛、甲状腺和鼻黏膜 B.使OM线垂直于地面 C.保持室内光线充足 D.戴眼罩,塞耳塞 E.光照暗淡,保持安静 |
C.保持室内光线充足 |
2 | 5 | 2008 | 19. 脑灌注显像的原理基于: (3.0分) | A.显像剂在脑细胞代谢 B.由于血脑屏障受到破坏 C.显像剂在脑内浓聚数量与血流正比 D.显像剂与受体结合 E.以上都不是 |
C.显像剂在脑内浓聚数量与血流正比 |
2 | 5 | 2008 | 20. 要研究大脑功能与代谢的关系,应当选用下列何项最佳组合? (3.0分) | A.3H标记的葡萄糖,PET显像 B.18F标记的葡萄糖,SPECT显像 C.99mTc标记的葡萄糖,双探头SPECT显像 D.18F标记的脱氧葡萄糖糖,PET显像 E.99mTc标记的葡萄糖,PET显像 |
D.18F标记的脱氧葡萄糖糖,PET显像 |
2 | 5 | 2008 | 21. SPECT脑血流灌注显像和PET葡萄糖代谢显像诊断原发性癫痫:(3.0分) | A.发作期局部葡萄糖代谢率降低,rCBF增高 B.发作间期局部葡萄糖代谢率降低,rCBF减低 C.发作间期局部葡萄糖代谢率增高,rCBF减低 D.发作期局部葡萄糖代谢率增高,rCBF减低 E.发作间期局部葡萄糖代谢率增高,rCBF增高 |
B.发作间期局部葡萄糖代谢率降低,rCBF减低 |
2 | 5 | 2008 | 22. 关于帕金森病的描述错误的是: (3.0分) | A.帕金森病主要临床症状为运动障碍 B.基因突变导致的帕金森病只占所有帕金森病的很小比例 C.帕金森病主要表现为运动过度 D.年龄是影响帕金森病的重要因素 |
C.帕金森病主要表现为运动过度 |
2 | 5 | 2008 | 23. 以下哪一个结果不属于基底神经节: (3.0分) | A.尾(状)核、壳核 B.海马 C.丘脑底核 D.黑质 |
B.海马 |
2 | 5 | 2008 | 24. 鱼滕酮是脂溶性的天然杀虫剂,它在多胺能神经元内的主要作用靶点是? (3.0分) | A.线粒体复合物I B.线粒体复合物II C.蛋白酶体 D.溶酶体 E.细胞膜上的磷脂 |
A.线粒体复合物I |
2 | 5 | 2008 | 25. Parkinson's disease is caused by mainly loss of neurons that secret the neurotransmitter.(3.0分) | A.Ach. B.Norepinephrine. C.GABA. D.Dopamine. |
D.Dopamine. |
2 | 5 | 2008 | 26. In Huntington's disease, there is a loss of(3.0分) | A.Dopamine in the neostriatum B.Substance P in the substantia nigra C.ACh and GABA in intrastriatal and cortical neurons D.Serotonin in the neostriatum E.Most of the pallidal neurons |
C.ACh and GABA in intrastriatal and cortical neurons |
2 | 5 | 2008 | 27. The neurotoxin 1-methyl-4-phenyl-1,2,3,6-tetrahydropyridine (MPTP) has recently been applied experimentally with considerable success as a model for (3.0分) | A. Huntington's disease B. Hemiballism C.Parkinson's disease D. Tardive dyskinesia E. Dystonia |
C.Parkinson's disease |
2 | 5 | 2008 | 28. Which of the following motor abnormalities does NOT indicate extrapyramidal disease? (3.0分) | A.Cog wheel rigidity B.Flaccidity C.Clasp knife spasticity D.Lead pipe rigidity E.Static tremors |
C.Clasp knife spasticity |
2 | 5 | 2008 | 29. A 65 year-old man presents with "hand tremors". This may be eventually attributed to the following conditions EXCEPT: (3.0分) | A.Senility B.Thyrotoxicosis C.Cerebellar neuroblastoma D.lschemic injury to the Subthalmic nucleus E.Chronic liver disease |
C.Cerebellar neuroblastoma |
2 | 5 | 2008 | 30. A 15 year old boy presents with tremor of both hands. Over the previous months he has developed a mild dysarthria. He has a history of behavioural problems, of a depressive/psychotic nature. The most likely diagnosis is: (3.0分) | A.Alzheimer's disease B.Huntington's disease C.Neuroacanthocytosis D.variant Creutzfeldt-Jakob disease E.Wilson's disease |
B.Huntington's disease |
2 | 5 | 2008 | 31. A 60 year old man has Parkinson's disease. He is started on treatment with L- dopa and dopa decarboxylase inhibitor therapy. However he continues to have troublesome tremor. Which of the following drugs would be most likely to help? (3.0分) | A.Tolcapone. B.Benzexol. C.Propranolol D.Sinemet E.Selegiline |
D.Sinemet |
2 | 5 | 2008 | 32. A 72 year old female presents with general slowness. Examination reveals a tremor of the hands. What frequency of tremor would you suspect in Parkinson's disease? (3.0分) | A.1 Hz B.2 Hz C.5 Hz D.8 Hz E.10 Hz |
C.5 Hz |
2 | 5 | 2008 | 33. Parkinson's disease: (3.0分) | A.an association with dysphagia has been shown B.the majority of patients are above their optimal weight C.tremor is typically unilateral in the early stages D.there is an association with accelerated mental retardation |
C.tremor is typically unilateral in the early stages |
2 | 5 | 2009 | 1. γ运动神经元轴突末梢释放的递质是 (2.0分) | A.乙酰胆碱 B.去甲肾上腺素 C.甘氨酸 D.谷氨酸 E.5‐羟色胺 |
A.乙酰胆碱 |
2 | 5 | 2009 | 2. 运动调节的最后公路是指 (2.0分) | A. 运动神经元 B. 运动神经元 C. 运动神经元 D.以上都是 E.以上都不是 |
A. 运动神经元 |
2 | 5 | 2009 | 3. 腱反射具有的特点是 (2.0分) | A.是多突触反射 B.肌肉的收缩几乎是一次同步性的收缩 C.感受器为腱器官 D.可由重力作用引起 E.主要表现在屈肌上 |
B.肌肉的收缩几乎是一次同步性的收缩 |
2 | 5 | 2009 | 4. 下列关于腱器官的描述,正确的是 (2.0分) | A.与梭外肌纤维呈并联关系 B.与梭内肌纤维呈并联关系 C.是一种长度感受器 D.传入纤维是Ⅱ类纤维 E.作用意义在于避免肌肉拉伤 |
E.作用意义在于避免肌肉拉伤 |
2 | 5 | 2009 | 5. 下列哪种情况最能说明去大脑僵直是由骨骼肌牵张反射亢进引起的? (2.0分) | A.刺激网状结构易化区,引起僵直 B.横断脊髓后,断面以下僵直消失 C.刺激前角运动神经元,引起僵直 D.切断腹根传出纤维,僵直消失 E.切断背根传入纤维,僵直消失 |
E.切断背根传入纤维,僵直消失 |
2 | 5 | 2009 | 6. 下列哪一部位或神经元受损可导致舞蹈病? (2.0分) | A.大脑皮层内胆碱能神经元 B.大脑皮层‐谷氨酸能投射神经元 C.新纹状体内γ‐氨基丁酸能中间神经元 D.黑质‐纹状体多巴胺能投射神经元 E.弓状核‐正中隆起γ‐氨基丁酸能投射神经元 |
C.新纹状体内γ‐氨基丁酸能中间神经元 |
2 | 5 | 2009 | 7. 下列关于皮质运动区功能特征的叙述,错误的是 (2.0分) | A.对躯体运动的支配有交叉的性质,但对头面部的支配多数是双侧性的 B.功能定位总的配布是倒置的,头面部代表区内部的配布为正的 C.肌肉的运动越精细、越复杂,其代表区越大 D.人工刺激所引起的肌肉运动反应为协同性收缩 E.运动区的基本功能单位呈柱状结构 |
D.人工刺激所引起的肌肉运动反应为协同性收缩 |
2 | 5 | 2009 | 8. 人类基底神经节调节功能障碍,主要表现形式不包括 (2.0分) | A.肌肉强直 B.肌张力障碍 C.随意运动完全丧失 D.静止性震颤 E.不自主的舞蹈样运动 |
C.随意运动完全丧失 |
2 | 5 | 2009 | 9. 人类小脑受损后可出现一些症状,但下列哪一项是不会见到的? (2.0分) | A.运动共济失调 B.位置性眼震颤 C.平衡失调 D.安静时出现震颤,做精细运动时震颤消失 |
D.安静时出现震颤,做精细运动时震颤消失 |
2 | 5 | 2009 | 10. 在中脑上、下叠体之间切断脑干的动物,出现四肢伸直,坚硬如柱,头尾昂起,脊柱挺硬,称为去大脑僵直,其原因是脑干网状结构 (2.0分) | A.抑制区活动增强 B.易化区活动增强 C.组织受到破坏 D.组织受到刺激 E.出现抑制解除 |
B.易化区活动增强 |
2 | 5 | 2009 | 11. 某老年患者,全身肌紧张增高、随意运动减少、动作缓慢、面部表情呆板,临床诊断为震颤麻痹,其病变主要位于 (2.0分) | A.黑质 B.红核 C.小脑 D.纹状体 E.苍白球 |
A.黑质 |
2 | 5 | 2009 | 12. 决定人执行功能的脑区是(2.0分) | A.颞叶 B.额叶 C.顶叶 D.视上回 |
B.额叶 |
2 | 5 | 2009 | 13. 人脑的左半球在下列方面占优势(2.0分) | A.概念、抽象、逻辑 B.空间知觉 C.音乐绘画 D.以上都不是 |
A.概念、抽象、逻辑 |
2 | 5 | 2009 | 14. 知觉的特性包括 (2.0分) | A.选择 B.理解 C.恒常 D.三者皆有 |
D.三者皆有 |
2 | 5 | 2009 | 15. 遗忘机制的假说包括 (2.0分) | A.衰减说 B.压抑说 C.线索依赖说 D.三者皆有 |
D.三者皆有 |
2 | 5 | 2009 | 16. 注意的功能包括(2.0分) | A.选择 B.保持 C.对活动进行调节与监督 D.以上皆有 |
D.以上皆有 |
2 | 5 | 2009 | 17. 与强迫特质对应的正常人格特质是 (2.0分) | A.社交性 B.责任心 C.外向性 D.开放性 |
B.责任心 |
2 | 5 | 2009 | 18. 红核位于(2.0分) | A.间脑 B.中脑 C.脑桥 D.延髓 E.小脑 |
B.中脑 |
2 | 5 | 2009 | 19. 第1躯体运动区位于(2.0分) | A.中央前回和中央旁小叶前部 B.额中回后部 C.额下回后部 D.中央后回和中央旁小叶后部 E.中央前回和中央后回 |
A.中央前回和中央旁小叶前部 |
2 | 5 | 2009 | 20. 视区位于(2.0分) | A.颞横回 B.角回 C.距状沟两侧的皮质 D.缘上回 E.以上都不是 |
C.距状沟两侧的皮质 |
2 | 5 | 2009 | 21. 不属于小脑核的是 (2.0分) | A.顶核 B.球状核 C.栓状核 D.齿状核 E.下橄榄核 |
E.下橄榄核 |
2 | 5 | 2009 | 22. 患者有足下垂和足背皮肤感觉缺失,损伤可能涉及(2.0分) | A.腓深神经 B.胫神经和腓浅神经 C.腓总神经 D.股神经 E.闭孔神经 |
C.腓总神经 |
2 | 5 | 2009 | 23. 全身骨骼显像利用以下哪种机制(2.0分) | A.化学吸附和离子交换 B.选择性排泄 C.循环通路 D.细胞吞噬 E.通透弥散 |
A.化学吸附和离子交换 |
2 | 5 | 2009 | 24. 下列那一项不是骨显像的适应症 (2.0分) | A.诊断转移性骨肿瘤 B.粉碎性骨折 C.移植骨 D.代谢性骨疾病 E.应力性骨折 |
B.粉碎性骨折 |
2 | 5 | 2009 | 25. 临床上判断早期股骨头缺血性坏死最好的方法是(2.0分) | A.X片检查 B.超声检查 C.ECT全身骨显像 D.ECT局部断层检查 E.X-CT检查 |
D.ECT局部断层检查 |
2 | 5 | 2009 | 26. 用脑血流灌注显像诊断脑死亡,最特征的影像是什么?(2.0分) | A.两大脑半球没有放射性分布 B.一侧大脑半球没有放射性分布 C.头皮不显影 D.两大脑半球放射性低下 E.上失状窦显影 |
A.两大脑半球没有放射性分布 |
2 | 5 | 2009 | 27. 脑肿瘤的葡萄糖代谢影像以下正确的是:(2.0分) | A.脑肿瘤复发常表现为放射性增高 B.放疗和化疗效果明显者后期典型表现是局部放射性增高 C.瘢痕组织局部放射性明显增加 D. 原发脑肿瘤典型表现是局部放射性减低 E.放疗后水肿部位常表现为局部放射性浓集 |
A.脑肿瘤复发常表现为放射性增高 |
2 | 5 | 2009 | 28. 进行rCBF显像,以下哪一项检查前准备不正确(2.0分) | A.口服过氯酸钾以封闭脉络丛.甲状腺和鼻黏膜 B.使OM线垂直于地面 C.保持室内光线充足 D.戴眼罩,塞耳塞 E.光照暗淡,保持安静 |
C.保持室内光线充足 |
2 | 5 | 2009 | 29. 脑灌注显像的原理基于: (2.0分) | A.显像剂在脑细胞代谢 B.由于血脑屏障受到破坏 C.显像剂在脑内浓聚数量与血流正比 D.显像剂与受体结合 E.以上都不是 |
C.显像剂在脑内浓聚数量与血流正比 |
2 | 5 | 2009 | 30. SPECT脑血流灌注显像和PET葡萄糖代谢显像诊断原发性癫痫:(2.0分) | A.发作期局部葡萄糖代谢率降低,rCBF增高 B.发作间期局部葡萄糖代谢率降低,rCBF减低 C.发作间期局部葡萄糖代谢率增高,rCBF减低 D.发作期局部葡萄糖代谢率增高,rCBF减低 E.发作间期局部葡萄糖代谢率增高,rCBF增高 |
B.发作间期局部葡萄糖代谢率降低,rCBF减低 |
2 | 5 | 2009 | 31. 正常成人在放松、闭眼状态下,枕部和顶部的脑电活动表现为:(2.0分) | A.0-3 Hz B.4-7 Hz C.8-12 Hz D.13-25 Hz |
C.8-12 Hz |
2 | 5 | 2009 | 32. 头颅CT上表现密度最高的是:(2.0分) | A.血栓 B.白质 C.灰质 D.脑脊液 |
A.血栓 |
2 | 5 | 2009 | 33. 哪个部位产生脑电活动节律? (2.0分) | A.丘脑 B.皮层 C.脑干 D.脊髓 |
A.丘脑 |
2 | 5 | 2009 | 34. 出现持续性肌无力,进行肌电图检查的主要目的为(2.0分) | A. 确定骨骼肌病变是多发性肌炎还是肌营养性不良 B. 确定病变属于神经源性还是肌源性 C. 确定骨骼肌病变是周期性麻痹还是重症肌无力 D. 确定病变是格林巴利综合征、还是糖尿病性周围神经病变所致 E. 确定骨骼肌病变是肌纤维膜还是肌核病变 |
B. 确定病变属于神经源性还是肌源性 |
2 | 5 | 2009 | 35. 关于帕金森病的描述错误的是: (2.0分) | A.帕金森病主要临床症状为运动障碍 B.基因突变导致的帕金森病只占所有帕金森病的很小比例 C.帕金森病主要表现为运动过度 D.年龄是影响帕金森病的重要因素 |
C.帕金森病主要表现为运动过度 |
2 | 5 | 2009 | 36. 以下哪一个结果不属于基底神经节: (2.0分) | A.尾(状)核、壳核 B.海马 C.丘脑底核 D.黑质 |
B.海马 |
2 | 5 | 2009 | 37. 鱼滕酮是脂溶性的天然杀虫剂,它在多胺能神经元内的主要作用靶点是? (2.0分) | A.线粒体复合物I B.线粒体复合物II C.蛋白酶体 D.溶酶体 E.细胞膜上的磷脂 |
A.线粒体复合物I |
2 | 5 | 2009 | 38. 只有透过血脑屏障在脑内变成多巴胺才能产生抗帕金森病作用的药物是 (2.0分) | A.司来吉兰(selegiline) B.苯海索(trihexyphenidy) C.金刚烷胺(amantadine) D.左旋多巴(levodopa) E.以上都不是 |
D.左旋多巴(levodopa) |
2 | 5 | 2009 | 39. 单用时无抗帕金森病作用的药物是 (2.0分) | A.苯扎托品(benztropine) B.卡比多巴(carbidopa) C.苯海索(trihexyphenidy) D.左旋多巴(levodopa) E.溴隐亭(bromocriptine) |
B.卡比多巴(carbidopa) |
2 | 5 | 2009 | 40. 硝替卡朋(nitecapone)的作用机制是(2.0分) | A.抑制中枢乙酰胆碱酯酶(AChE) B.抑制中枢单胺氧化酶(MAO) C.抑制中枢儿茶酚氧位甲基转移酶(COMT) D.阻断中枢M胆碱受体 E.促进中枢DA的释放 |
C.抑制中枢儿茶酚氧位甲基转移酶(COMT) |
2 | 5 | 2009 | 41. 关于苯海索(trihexyphenidy)的叙述错误的是 (2.0分) | A.阻断黑质纹状体多巴胺通路 B.阻断中枢胆碱受体 C.对心脏的影响较阿托品弱 D.抗震颤疗效好 E.能致尿潴留,前列腺肥大者慎用 |
C.对心脏的影响较阿托品弱 |
2 | 5 | 2009 | 42. 具有抗病毒作用的抗帕金森病药物是 (2.0分) | A.左旋多巴(levodopa) B.卡比多巴(carbidopa) C.金刚烷胺(amantadine) D.溴隐亭(bromocriptine) E.司来吉兰(selegiline) |
B.卡比多巴(carbidopa) |
2 | 5 | 2009 | 43. 某人在意外事故中脊髓受到损伤,丧失横断面以下的一切躯体与内脏反射活动。但数周以后屈肌反射、腱反射等比较简单的反射开始逐渐恢复,这表明该患者在受伤当时出现了 (2.0分) | A.脑震荡 B.脑水肿 C.脊休克 D.脊髓水肿 E.疼痛性休克 |
C.脊休克 |
2 | 5 | 2009 | 44. Parkinson's disease is mainly caused by loss of neurons that secret_____. (2.0分) | A.Ach. B.Norepinephrine. C.GABA. D.Dopamine. |
D.Dopamine. |
2 | 5 | 2009 | 45. The neurotoxin 1-methyl-4-phenyl-1,2,3,6-tetrahydropyridine (MPTP) has recently been applied experimentally with considerable success as a model for(2.0分) | A.Huntington's disease B.Hemiballism C.Parkinson's disease D.Tardive dyskinesia E.Dystonia |
C.Parkinson's disease |
2 | 5 | 2009 | 46. A 65-year-old man presents with "hand tremors". This may be eventually attributed to the following conditions EXCEPT:(2.0分) | A.Brain stem tumor B.Thyrotoxicosis C.Senility D.lschemic injury to the Subthalmic nucleus E.Chronic liver disease |
A.Brain stem tumor |
2 | 5 | 2009 | 47. A 60-year-old man has Parkinson's disease. He is started on treatment with L- dopa and dopa decarboxylase inhibitor therapy. But he continues to have troublesome tremor. Which of the following drugs would be most likely to help?(2.0分) | A.Tolcapone. B.Benzexol. C.Propranolol D.Artane E.Selegiline |
D.Artane |
2 | 5 | 2009 | 48. Which is true concerning the clinical features of Parkinson's disease: (2.0分) | A.Most patients present with dysphagia B.The majority of patients are above their optimal weight C.Tremor is typically unilateral at onset D.Accelerated mental retardation will occur in early stages |
C.Tremor is typically unilateral at onset |
2 | 5 | 2009 | 49. The basic pathological mechanism of hepatolenticular degeneration is:(2.0分) | A.Biliary copper discharge obstacles B.The dyssynthesis of ceruloplasmin C.Abnormal proteins within the cell. D.Lysosomal defects |
B.The dyssynthesis of ceruloplasmin |
2 | 5 | 2009 | 50. A 72-year-old female present is present with general slowness. Examination reveals a tremor of the left hand. What frequency of tremor would you suspect? (2.0分) | A.1 Hz B.2 Hz C.5 Hz D.8 Hz E.10 Hz |
C.5 Hz |
2 | 5 | 2010 | 1. 知觉的特性包括(2.5分) | A.选择 B.理解 C.恒常 D.三者皆有 |
D.三者皆有 |
2 | 5 | 2010 | 2. 决定人执行功能的脑区是 (2.5分) | A.颞叶 B.额叶 C.顶叶 D.视上回 |
B.额叶 |
2 | 5 | 2010 | 3. 与强迫型人格障碍对应的正常人格特质是(2.5分) | A.社交性 B.责任心 C.外向性 D.开放性 |
B.责任心 |
2 | 5 | 2010 | 4. NEO-PI-R人格特质模式中除了神经质、宜人质,还包括(2.5分) | A.外向性 B.责任心 C.开放性 D.三者皆有 |
D.三者皆有 |
2 | 5 | 2010 | 5. 下面哪种药物不用于治疗帕金森病 (2.5分) | A.Pethidine B.L-dopa C.Selegiline D.Amantatine |
A.Pethidine |
2 | 5 | 2010 | 6. L-dopa的特点是 (2.5分) | A.不良反应少见 B.只有在脑内才能作用转变为DA C.作用迅速 D.对老年患者疗效较好 |
NaN |
2 | 5 | 2010 | 7. 下列药物组合能增强疗效的是 (2.5分) | A.L-dopa与vitamine B6 B.L-dopa与metoclopramide C.L-dopa与trihexyphenidyl D.L-dopa与chlorpromazine |
C.L-dopa与trihexyphenidyl |
2 | 5 | 2010 | 8. L-dopa引起的不良反应不包括下列哪项? (2.5分) | A.不随意运动 B.牙龈增生 C.体位性低血压 D.精神障碍 |
B.牙龈增生 |
2 | 5 | 2010 | 9. 单用时无抗帕金森病作用的药物是 (2.5分) | A.Pergolide B.Carbidopa C.Trihexyphenidyl D.L-dopa |
B.Carbidopa |
2 | 5 | 2010 | 10. 具有抗病毒作用的抗帕金森病药物是(2.5分) | A.左旋多巴 B.卡比多巴 C.金刚烷胺 D.溴隐亭 |
C.金刚烷胺 |
2 | 5 | 2010 | 11. 骨显像时超级影像最常见于下列哪种情况:(2.5分) | A.肺性肥大性骨关节病 B.甲状旁腺功能亢进 C.多发性骨髓瘤 D.骨肉瘤 |
B.甲状旁腺功能亢进 |
2 | 5 | 2010 | 12. 骨显像最有价值的临床应用是:(2.5分) | A.移植骨监测 B.骨骼原发瘤的诊断 C.骨髓炎的诊断 D.骨转移瘤的早期诊断 |
D.骨转移瘤的早期诊断 |
2 | 5 | 2010 | 13. 急性骨髓炎时放射性核素骨显像的表现是:(2.5分) | A.血流、血池相放射性明显增加,消失缓慢,延迟相放射性增加 B.血流相正常,血池相放射性增高,消失缓慢,延迟相正常 C.血流、血池相正常,延迟相呈放射性明显增加 D.血流相放射性灌注增加,血池相及延迟相呈放射性减低 |
A.血流、血池相放射性明显增加,消失缓慢,延迟相放射性增加 |
2 | 5 | 2010 | 14. 以下哪项是判断早期股骨头缺血性坏死的最好方法:(2.5分) | A.X片检查 B.骨密度测定 C.骨骼核素显像 D.MRI |
C.骨骼核素显像 |
2 | 5 | 2010 | 15. 有关骨骼核素显像的原理以下哪项说法是正确的:(2.5分) | A.化学吸附和离子交换 B.选择性排泄 C.微循环栓塞 D.细胞吞噬 |
A.化学吸附和离子交换 |
2 | 5 | 2010 | 16. 有关核素脑血流灌注显像的原理,以下说法正确的是: (2.5分) | A.显像剂为水溶性 B.由于血脑屏障受到破坏 C.显像剂在脑内浓聚数量与血流正比 D.显像剂与受体结合 |
C.显像剂在脑内浓聚数量与血流正比 |
2 | 5 | 2010 | 17. 脑代谢显像最常用的显像剂是:(2.5分) | A.99mTc-ECD B.99mTc-MDP C.18F-FDG D.99mTc-HMPAO |
C.18F-FDG |
2 | 5 | 2010 | 18. 以下有关脑肿瘤的葡萄糖代谢影像正确的是:(2.5分) | A.脑肿瘤复发常表现为放射性增高 B.放疗和化疗效果明显者后期典型表现是局部放射性增高 C.瘢痕组织局部放射性明显增加 D.原发脑肿瘤典型表现是局部放射性减低 |
A.脑肿瘤复发常表现为放射性增高 |
2 | 5 | 2010 | 19. Parkinson病rCBF显像的特征性改变是:(2.5分) | A.大脑皮质放射性分布增高 B.大小脑交叉失联络 C.两侧基底核区放射性分布减少 D.两侧基地核放射性分布增高 |
C.两侧基底核区放射性分布减少 |
2 | 5 | 2010 | 20. 下列有关脑血流灌注显像剂哪一种说法正确(2.5分) | A.脂溶性物质 B.正电荷物质 C.大分子物质 D.极性物质 |
A.脂溶性物质 |
2 | 5 | 2010 | 21. 关于帕金森病的主要临床症状描述错误的是: (2.5分) | A.运动迟缓 B.姿态失衡 C.学习记忆丧失 D.静息震颤 |
C.学习记忆丧失 |
2 | 5 | 2010 | 22. 下列哪些因素不会导致帕金森病中蛋白异常聚集:(2.5分) | A.蛋白折叠异常 B.异常蛋白质清除机制障碍 C.Huntintin基因突变 D.线粒体功能异常 |
C.Huntintin基因突变 |
2 | 5 | 2010 | 23. 百草枯是脂溶性的天然杀虫剂,它在多胺能神经元内的主要作用靶点是? (2.5分) | A.线粒体复合物I B.线粒体复合物II C.蛋白酶体 D.溶酶体 |
A.线粒体复合物I |
2 | 5 | 2010 | 24. 一个运动单元所必须包括的运动神经元是:(2.5分) | A.α运动神经元 B.β运动神经元 C C.γ运动神经元 D D.上述三种神经元 |
A.α运动神经元 |
2 | 5 | 2010 | 25. 反牵张反射的关键感受器是:(2.5分) | A.肌腱器 B.肌梭 C.小脑 D.网状结构 |
A.肌腱器 |
2 | 5 | 2010 | 26. 小脑中负责运动执行的区域是: (2.5分) | A.脊髓小脑 B.前庭小脑 C.皮层小脑 D.蚓部 |
A.脊髓小脑 |
2 | 5 | 2010 | 27. 脑干内的非脑神经核为 (2.5分) | A.丸动眼神经核 B.动眼神经副核 C.红核 D.展神经核 |
C.红核 |
2 | 5 | 2010 | 28. 与中脑红核没有联系的小脑核是 (2.5分) | A.顶核 B.球状核 C.栓状核 D.中间核 |
A.顶核 |
2 | 5 | 2010 | 29. 中脑内有(2.5分) | A.黑质 B.面神经核 C.三叉神经脊束核 D.展神经核 |
A.黑质 |
2 | 5 | 2010 | 30. 正常成人在放松、闭眼状态下,枕部和顶部的脑电活动表现为:(2.5分) | A.0-3 Hz B.4-7 Hz C.8-13Hz D.13-25 Hz |
C.8-13Hz |
2 | 5 | 2010 | 31. 头颅CT上表现密度最低的是:(2.5分) | A.骨皮质 B.白质 C.灰质 D.脑脊液 |
D.脑脊液 |
2 | 5 | 2010 | 32. 哪个部位产生脑电活动节律?(2.5分) | A.丘脑 B.皮层 C.脑干 D.脊髓 |
A.丘脑 |
2 | 5 | 2010 | 33. 哪项不是常用的脑电图诱发方式 (2.5分) | A.促惊厥药 B.睡眠剥夺 C.过度换气 D.间歇性闪光刺激 |
A.促惊厥药 |
2 | 5 | 2010 | 34. 按照频率由高到低次序排列,脑电波类型的顺序应是(2.5分) | A.β、α、θ和δ B.α、β、θ和δ C.θ、δ、β和α D.β、θ、α和δ |
A.β、α、θ和δ |
2 | 5 | 2010 | 35. Parkinson's disease is mainly caused by loss of neurons that secret__(2.5分) | A.Ach. B.Norepinephrine. C.GABA. D.Dopamine. |
D.Dopamine. |
2 | 5 | 2010 | 36. The neurotoxin 1-methyl-4-phenyl-1,2,3,6-tetrahydropyridine (MPTP) has recently been applied experimentally with considerable success as a model for(2.5分) | A.Huntington's disease B.Hemiballism C.Parkinson's disease D.Tardive dyskinesia |
C.Parkinson's disease |
2 | 5 | 2010 | 37. A 65-year-old man presents with "hand tremors". This may be eventually attributed to the following conditions EXCEPT: (2.5分) | A.Brain stem tumor B.Thyrotoxicosis C.Senility D.lschemic injury to the Subthalmic nucleus |
A.Brain stem tumor |
2 | 5 | 2010 | 38. A 60-year-old man has Parkinson's disease. He is started on treatment with L- dopa and dopa decarboxylase inhibitor therapy. But he continues to have troublesome tremor. Which of the following drugs would be most likely to help? (2.5分) | A.Tolcapone. B.Benzexol. C.Propranolol D.Artane |
D.Artane |
2 | 5 | 2010 | 39. Which is true concerning the clinical features of Parkinson's disease (2.5分) | A.Most patients present with dysphagia B.The majority of patients are above their optimal weight C.Tremor is typically unilateral at onset D.Accelerated mental retardation will occur in early stages |
C.Tremor is typically unilateral at onset |
2 | 5 | 2010 | 40. The basic pathological mechanism of hepatolenticular degeneration is (2.5分) | A.Biliary copper discharge obstacles B.The dyssynthesis of ceruloplasmin C.Abnormal proteins within the cell. D.Lysosomal defects |
B.The dyssynthesis of ceruloplasmin |
2 | 5 | 2011 | 1. 骨显像显示骨病灶与X线诊断相比较,其灵敏度和特异性是(3.0分) | A.灵敏度和特异性均高 B.灵敏度和特异性均低 C.灵敏度高而特异性低 D.灵敏度低而特异性高 E.灵敏度和特异性均相仿 |
C.灵敏度高而特异性低 |
2 | 5 | 2011 | 2. 骨显像用于原发性骨肿瘤,主要是(3.0分) | A.良恶性鉴别 B.定性诊断 C.排除炎症性骨病变 D.排除外伤性骨病变 E.了解肿瘤确切范围和其他骨组织情况 |
E.了解肿瘤确切范围和其他骨组织情况 |
2 | 5 | 2011 | 3. 骨显像对骨转移性肿瘤的诊断颇具临床价值,是由于(3.0分) | A.灵敏度高 B.可显示全身骨病灶 C.提供放射性核素治疗的依据 D.可显示骨病灶的代谢活性 E.以上均是 |
E.以上均是 |
2 | 5 | 2011 | 4. 67岁,男性患者,肺癌术后3个月,自觉全省疼痛,尤以腰部为甚,X线检查未见明星异常,建议患者首选的检查项目是(3.0分) | A.CT B.全身骨显像 C.MRI D.腰椎活检 E.PET/CT |
B.全身骨显像 |
2 | 5 | 2011 | 5. 骨显像时超级影像最常见于下列哪种情况(3.0分) | A.肺性肥大性骨关节病 B.甲状旁腺功能亢进 C.多发性骨髓瘤 D.骨肉瘤 E.骨质疏松 |
B.甲状旁腺功能亢进 |
2 | 5 | 2011 | 6. 急性骨髓炎时放射性核素骨显像的表现是(3.0分) | A.血流、血池相放射性明显增加,消失缓慢,延迟相放射性增加 B.血流相正常,血池相放射性增高,消失缓慢,延迟相正常 C.血流、血池相正常,延迟相呈放射性明显增加 D.血流相放射性灌注增加,血池相及延迟相呈放射性减低 E.以上均不是 |
A.血流、血池相放射性明显增加,消失缓慢,延迟相放射性增加 |
2 | 5 | 2011 | 7. 骨显像最常采用的显像方式是(3.0分) | A.局部显像 B.全身显像 C.断层显像 D.三相显像 E.动态显像 |
B.全身显像 |
2 | 5 | 2011 | 8. 有关骨骼核素显像的原理以下哪项说法是正确的(3.0分) | A.化学吸附和离子交换 B.选择性排泄 C.微循环栓塞 D.细胞吞噬 E.特异性结合 |
A.化学吸附和离子交换 |
2 | 5 | 2011 | 9. 下列何种脑显像剂能通过血脑屏障 (3.0分) | A.Na99mTcO4 B.99mTc-DTPA C.99mTc-葡萄糖酸盐 D.99mTc-ECD E.113mIn-DTPA |
D.99mTc-ECD |
2 | 5 | 2011 | 10. 早老性痴呆SPECT脑灌注显像可见到(3.0分) | A.一侧顶叶和颞叶呈明显的非对称性的血流减低区 B.一侧顶叶和颞叶呈明显的非对称性的血流增高区 C.双侧顶叶和颞叶呈明显的对称性的血流减低区 D.双侧顶叶和颞叶呈明显的对称性的血流增高区 E.以上都不是 |
C.双侧顶叶和颞叶呈明显的对称性的血流减低区 |
2 | 5 | 2011 | 11. 18F-FDG用于葡萄糖代谢显像,主要由于脱氧葡萄糖较葡萄糖(4.0分) | A.进入脑组织快 B.进入脑组织多 C.在脑内代谢为氟脱氧葡萄糖-6-磷酸后能进一步代谢 D.在脑内代谢产物氟脱氧葡萄糖-6-磷酸不能进一步代谢 E.在脑内清除快 |
D.在脑内代谢产物氟脱氧葡萄糖-6-磷酸不能进一步代谢 |
2 | 5 | 2011 | 12. 下列哪一项不是脑血流灌注显像剂的特点(3.0分) | A.良好的脂溶性物质 B.电中性 C.小分子物质 D.水溶性好 E.在脑内滞留时间长,清除慢 |
D.水溶性好 |
2 | 5 | 2011 | 13. 下列哪一项不是脑血流灌注显像介入试验的临床应用(3.0分) | A.短暂性脑缺血发作 B.隐匿性脑缺血病灶的探测 C.精神分裂症的诊断 D.脑血管储备能力的确定 E.失联络现象中血管反应性的判断 |
B.隐匿性脑缺血病灶的探测 |
2 | 5 | 2011 | 14. 下面哪种药物不用于治疗帕金森病 (3.0分) | A.Pethidine (哌替啶) B.L-dopa C.Selegiline D.Amantatine E.Trihexyphenidyl(苯海索) |
A.Pethidine (哌替啶) |
2 | 5 | 2011 | 15. 下列药物组合能增强疗效的是 (3.0分) | A.L-dopa与vitamin B6 B.L-dopa与甲氧氯普胺 C.L-dopa与苯海索 D.L-dopa与chlorpromazine E.Carbidopa与selegiline |
C.L-dopa与苯海索 |
2 | 5 | 2011 | 16. 单用时无抗帕金森病作用的药物是 (3.0分) | A.Pergolide B.Carbidopa C.Trihexyphenidyl D.L-dopa E.Bromocriptine |
B.Carbidopa |
2 | 5 | 2011 | 17. 具有抗病毒作用的抗帕金森病药物是 (3.0分) | A.左旋多巴 B.卡比多巴 C.金刚烷胺 D.溴隐亭 E.司来吉兰 |
C.金刚烷胺 |
2 | 5 | 2011 | 18. 关于苯海索(trihexyphenidy)的叙述错误的是 (3.0分) | A.阻断黑质纹状体多巴胺通路 B.阻断中枢胆碱受体 C.对心脏的影响较阿托品弱 D.抗震颤疗效好 E.能致尿潴留,前列腺肥大者慎用 |
A.阻断黑质纹状体多巴胺通路 |
2 | 5 | 2011 | 19. 普拉克索(pramipexole)治疗帕金森病,是由于 (3.0分) | A.中枢抗胆碱作用 B.激动DA受体 C.激动GABA受体 D.提高脑内DA浓度 E.使DA降解减少 |
B.激动DA受体 |
2 | 5 | 2011 | 20. 关于帕金森病的描述错误的是(3.0分) | A.帕金森病主要临床症状为运动障碍 B.帕金森病主要表现为运动过度 C.家族性帕金森病只占所有帕金森病的很小比例 D.路易小体是其特征性病理变化 |
B.帕金森病主要表现为运动过度 |
2 | 5 | 2011 | 21. 以下哪一个结果不属于基底神经节(3.0分) | A.尾(状)核、壳核 B.下丘脑 C.丘脑底核 D.黑质 |
B.下丘脑 |
2 | 5 | 2011 | 22. 鱼滕酮是脂溶性的天然杀虫剂,它在多胺能神经元内的主要作用靶点是? (3.0分) | A.线粒体复合物I B.线粒体复合物II C.蛋白酶体 D.溶酶体 E.细胞膜上的磷脂 |
A.线粒体复合物I |
2 | 5 | 2011 | 23. 一个运动单元所必须包括的运动神经元是(3.0分) | A.α运动神经元 B.β运动神经元 C.γ运动神经元 D.上述三种神经元 |
A.α运动神经元 |
2 | 5 | 2011 | 24. 下列关于牵张反射的叙述,错误的是(3.0分) | A.骨骼肌受到外力牵拉时能反射性地引起受牵拉的同一肌肉的收缩 B.牵张反射是维持姿势的基本反射 C.牵张反射的感受器是肌梭 D.在脊髓与高位中枢离断后,牵张反射即永远消失 |
D.在脊髓与高位中枢离断后,牵张反射即永远消失 |
2 | 5 | 2011 | 25. 反牵张反射的关键感受器是(3.0分) | A.肌腱器 B.肌梭 C.小脑 D.网状结构 |
A.肌腱器 |
2 | 5 | 2011 | 26. 新纹状体神经元使用的主要神经递质是(3.0分) | A.谷氨酸 B.乙酰胆碱 C.多巴胺 D.GABA |
D.GABA |
2 | 5 | 2011 | 27. 脊休克的特点(3.0分) | A.永久丧失反射能力 B.复杂反射能力不可恢复 C.简单反射能力不可恢复 D.随意运动不可恢复 |
D.随意运动不可恢复 |
2 | 5 | 2011 | 28. 不是反牵张反射的成分的是(3.0分) | A.肌腱 B.Ia类传入纤维 C. 运动神经元 D.抑制性中间神经元 |
B.Ia类传入纤维 |
2 | 5 | 2011 | 29. 亨廷顿综合症的病变部位是(3.0分) | A.中脑黑质 B.网状结构 C.苍白球 D.新纹状体 |
D.新纹状体 |
2 | 5 | 2011 | 30. 脑干的特殊内脏运动核支配(3.0分) | A.咀嚼肌、表情肌、咽喉肌 B.颈肌 C.心肌 D.平滑肌 E.舌肌、眼球外肌 F.疑核 |
A.咀嚼肌、表情肌、咽喉肌 |
2 | 5 | 2011 | 31. 属于背侧丘脑的结构为(3.0分) | A.丘脑枕 B.缰三角 C.松果体 D.乳头体 E.外侧膝状体 |
A.丘脑枕 |
2 | 5 | 2011 | 32. 当颅内压增高时,有可能出现枕骨大孔疝,导致病人生命危险。有关描述错误的是(3.0分) | A.枕骨大孔疝又称小脑扁桃体疝 B.小脑扁桃体被挤入枕骨大孔 C.压迫了脑桥 D.压迫了延髓 E.延髓内有生命中枢 |
C.压迫了脑桥 |
2 | 5 | 2011 | 33. 男性患者,6个月前发现右手活动不灵活,开门时,钥匙不能插进锁内,检查发现右手指鼻尖时颤抖而不准确,提示可能损伤了(3.0分) | A.左侧新小脑 B.右侧旧小脑 C.右侧新小脑 D.左侧旧小脑 E.右侧古小脑 |
C.右侧新小脑 |
2 | 5 | 2012 | 1. 骨显像显示骨病灶与X线诊断相比较,其灵敏度和特异性是:(3.0分) | A.灵敏度和特异性均高 B.灵敏度和特异性均低 C.灵敏度高而特异性低 D.灵敏度低而特异性高 E.灵敏度和特异性均相仿 |
C.灵敏度高而特异性低 |
2 | 5 | 2012 | 2. 骨显像用于原发性骨肿瘤,主要是(3.0分) | A.良恶性鉴别 B.定性诊断 C.排除炎症性骨病变 D.排除外伤性骨病变 E.了解肿瘤确切范围和其他骨组织情况 |
E.了解肿瘤确切范围和其他骨组织情况 |
2 | 5 | 2012 | 3. 骨显像对骨转移性肿瘤的诊断颇具临床价值,是由于(3.0分) | A.灵敏度高 B.可显示全身骨病灶 C.提供放射性核素治疗的依据 D.可显示骨病灶的代谢活性 E.以上均是 |
E.以上均是 |
2 | 5 | 2012 | 4. 67岁,男性患者,肺癌术后3个月,自觉全省疼痛,尤以腰部为甚,X线检查未见明星异常,建议患者首选的检查项目是(3.0分) | A.CT B.全身骨显像 C.MRI D.腰椎活检 E.PET/CT |
B.全身骨显像 |
2 | 5 | 2012 | 5. 骨显像时超级影像最常见于下列哪种情况(3.0分) | A.肺性肥大性骨关节病 B.甲状旁腺功能亢进 C.多发性骨髓瘤 D.骨肉瘤 E.骨质疏松 |
B.甲状旁腺功能亢进 |
2 | 5 | 2012 | 6. 急性骨髓炎时放射性核素骨显像的表现是(3.0分) | A.血流、血池相放射性明显增加,消失缓慢,延迟相放射性增加 B.血流相正常,血池相放射性增高,消失缓慢,延迟相正常 C.血流、血池相正常,延迟相呈放射性明显增加 D.血流相放射性灌注增加,血池相及延迟相呈放射性减低 E.以上均不是 |
A.血流、血池相放射性明显增加,消失缓慢,延迟相放射性增加 |
2 | 5 | 2012 | 7. 骨显像最常采用的显像方式是(3.0分) | A.局部显像 B.全身显像 C.断层显像 D.三相显像 E.动态显像 |
B.全身显像 |
2 | 5 | 2012 | 8. 有关骨骼核素显像的原理以下哪项说法是正确的(3.0分) | A.化学吸附和离子交换 B.选择性排泄 C.微循环栓塞 D.细胞吞噬 E.特异性结合 |
A.化学吸附和离子交换 |
2 | 5 | 2012 | 9. 下列何种脑显像剂能通过血脑屏障(3.0分) | A.Na99mTcO4 B.99mTc-DTPA C.99mTc-葡萄糖酸盐 D.99mTc-ECD E.113mIn-DTPA |
D.99mTc-ECD |
2 | 5 | 2012 | 10. 早老性痴呆SPECT脑灌注显像可见到(3.0分) | A.一侧顶叶和颞叶呈明显的非对称性的血流减低区 B.一侧顶叶和颞叶呈明显的非对称性的血流增高区 C.双侧顶叶和颞叶呈明显的对称性的血流减低区 D.双侧顶叶和颞叶呈明显的对称性的血流增高区 E.以上都不是 |
C.双侧顶叶和颞叶呈明显的对称性的血流减低区 |
2 | 5 | 2012 | 11. 18F-FDG用于葡萄糖代谢显像,主要由于脱氧葡萄糖较葡萄糖(4.0分) | A.进入脑组织快 B.进入脑组织多 C.在脑内代谢为氟脱氧葡萄糖-6-磷酸后能进一步代谢 D.在脑内代谢产物氟脱氧葡萄糖-6-磷酸不能进一步代谢 E.在脑内清除快 |
D.在脑内代谢产物氟脱氧葡萄糖-6-磷酸不能进一步代谢 |
2 | 5 | 2012 | 12. 下列哪一项不是脑血流灌注显像剂的特点(3.0分) | A.良好的脂溶性物质 B.电中性 C.小分子物质 D.水溶性好 E.在脑内滞留时间长,清除慢 |
D.水溶性好 |
2 | 5 | 2012 | 13. 下列哪一项不是脑血流灌注显像介入试验的临床应用(3.0分) | A.短暂性脑缺血发作 B.隐匿性脑缺血病灶的探测 C.精神分裂症的诊断 D.脑血管储备能力的确定 E.失联络现象中血管反应性的判断 |
B.隐匿性脑缺血病灶的探测 |
2 | 5 | 2012 | 14. 下列关于牵张反射的叙述,错误的是(3.0分) | A.骨骼肌受到外力牵拉时能反射性地引起受牵拉的同一肌肉的收缩 B.牵张反射是维持姿势的基本反射 C.牵张反射的感受器是肌梭 D.在脊髓与高位中枢离断后,牵张反射即永远消失 |
D.在脊髓与高位中枢离断后,牵张反射即永远消失 |
2 | 5 | 2012 | 15. 帕金森病以下的哪项表述是不正确的? (3.0分) | A.多在中老年期发病; B.主要表现静止性震颤、运动迟缓和肌强直; C.通常的辅助检查无特殊发现; D.早期发现、早期治疗可治愈; E.抗胆碱能药物适用于震颤明显的较年轻患者 |
D.早期发现、早期治疗可治愈; |
2 | 5 | 2012 | 16. 帕金森病患者常出现齿轮样肌张力障碍,原因是 (3.0分) | A.患者不配合,抗拒检查; B.被检查肢体合并有不自主震颤; C.患者精神紧张、肢体发抖; D.检查者用力过度 |
B.被检查肢体合并有不自主震颤; |
2 | 5 | 2012 | 17. 震颤麻痹的病人禁止使用哪类药物? (3.0分) | A.金刚烷胺 B.抗胆碱能药物 C.单胺氧化酶抑制剂 D.多巴胺受体激动剂 E.吩噻嗪类药 |
E.吩噻嗪类药 |
2 | 5 | 2012 | 18. 男性, 58岁, 逐渐出现四肢震颤,左侧明显,呈"搓丸样",动作缓慢,被动运动时肢体为齿轮样肌张力升高,需使用下列哪种药物治疗 (3.0分) | A.新斯的明 B.左旋多巴 C.卡马西平 D.多巴胺 E.苯妥英钠 |
B.左旋多巴 |
2 | 5 | 2012 | 19. 关于亨廷顿舞蹈病的诊断和治疗错误的是 (3.0分) | A.根据发病年龄,临床表现及家族史即可确诊; B.本病目前尚无特效治疗措施; C.对于舞蹈症状可选用氟哌啶醇; D.头颅MRI一般表现为皮质萎缩,脑室扩大; E.EEG表现一般无特异性 |
A.根据发病年龄,临床表现及家族史即可确诊; |
2 | 5 | 2012 | 20. 关于肝豆状核变性,错误的是 (3.0分) | A.50岁以后也可以起病; B.神经系统可表现为运动减少,也可表现为舞蹈; C.血清铜蓝蛋白正常不能除外肝豆状核变性; D.基因为ATP7B基因; E.本病为遗传性疾病,即使积极诊治,病情也会进行性进展,直至严重致残 |
E.本病为遗传性疾病,即使积极诊治,病情也会进行性进展,直至严重致残 |
2 | 5 | 2012 | 21. 诊断帕金森病最重要的是 (3.0分) | A.脑电图 B.临床表现 C.头颅MRI D.家族史 E.腰椎穿刺加头颅MRI |
B.临床表现 |
2 | 5 | 2012 | 22. L-dopa的特点是 (3.0分) | A.不良反应少见 B.只有在脑内才能转变为多巴胺 C.作用迅速 D.对老年患者疗效较好 E.与carbidopa合用可增强疗效减少不良反应 |
E.与carbidopa合用可增强疗效减少不良反应 |
2 | 5 | 2012 | 23. L-dopa引起的不良反应不包括下列哪项? (3.0分) | A.运动障碍 B.牙龈增生 C.体位性低血压 D.精神障碍 E.胃肠道反应 |
B.牙龈增生 |
2 | 5 | 2012 | 24. 关于苯海索(trihexyphenidy)的叙述错误的是 (3.0分) | A.阻断黑质纹状体多巴胺通路 B.阻断中枢胆碱受体 C.对心脏的影响较阿托品弱 D.抗震颤疗效好 E.能致尿潴留,前列腺肥大者慎用 |
A.阻断黑质纹状体多巴胺通路 |
2 | 5 | 2012 | 25. 司来吉兰(selegiline)的作用机制是 (3.0分) | A.抑制中枢乙酰胆碱酯酶(AChE) B.抑制中枢单胺氧化酶(MAO) C.抑制中枢儿茶酚氧位甲基转移酶(COMT) D.阻断中枢M胆碱受体 E.促进中枢DA的释放 |
B.抑制中枢单胺氧化酶(MAO) |
2 | 5 | 2012 | 26. 鱼滕酮是脂溶性的天然杀虫剂,它在多胺能神经元内的主要作用靶点是? (3.0分) | A.线粒体复合物I B.线粒体复合物II C.蛋白酶体 D.溶酶体 E.细胞膜上的磷脂 |
A.线粒体复合物I |
2 | 5 | 2012 | 27. 在一个病人身上出现下列多个诊断是,在您治疗原则中应该先那种疾病 (3.0分) | A.不安腿综合症 B.(环境) 适应障碍 C.睡行症 D.边缘型人格障碍 |
D.边缘型人格障碍 |
2 | 5 | 2012 | 28. 下列哪种情况下要慎用精神分裂症药物(3.0分) | A.分裂样人格障碍 B.抑郁症 C.强迫型人格障碍 D.夫妻咨询 |
A.分裂样人格障碍 |
2 | 5 | 2012 | 29. 男性患者,6个月前发现右手活动不灵活,开门时,钥匙不能插进锁内,检查发现右手指鼻尖时颤抖而不准确,提示可能损伤了(3.0分) | A.左侧新小脑 B.右侧旧小脑 C.右侧新小脑 D.左侧旧小脑 E.右侧古小脑 |
C.右侧新小脑 |
2 | 5 | 2012 | 30. 中脑内有(3.0分) | A.黑质 B.面神经核 C.三叉神经脊束核 D.展神经核 E.疑核 |
A.黑质 |
2 | 5 | 2012 | 31. 某人得了嗜睡症,工作、进食时都能人睡,是由于损伤了网状上行激活系统,与此系统有关的核团是(3.0分) | A.中线核、网状核和板内核 B.腹前核和背内侧核 C.中线核、枕和板内核 D.后外侧核、网状核和板内核 E.中线核、网状核和背外侧核 |
A.中线核、网状核和板内核 |
2 | 5 | 2012 | 32. 延髓内的非脑神经核为 (3.0分) | A.舌下神经核 B.疑核 C.薄束核 D.迷走神经背核 E.泌涎核 |
C.薄束核 |
2 | 5 | 2012 | 33. 丘脑的腹后外侧核接受 (3.0分) | A.三叉丘系纤维 B.嗅束纤维 C.外侧丘系纤维 D.视束纤维 E.内侧丘系纤维 |
E.内侧丘系纤维 |
2 | 5 | 2013 | 1. 全身骨显像时利用以下哪种机制进行成像?(2.0分) | A.循环通路 B.选择性排泄 C.化学吸附和离子交换 D.细胞吞噬 E.通透弥散 |
C.化学吸附和离子交换 |
2 | 5 | 2013 | 2. 骨显像时超级影像常见于下列哪种情况?(2.0分) | A.全身弥漫性骨转移 B.急性骨髓炎 C.肺性肥大性骨病 D.股骨头无菌性坏死 E.原发性骨肿瘤 |
A.全身弥漫性骨转移 |
2 | 5 | 2013 | 3. 急性骨髓炎时放射性核素骨显像的表现是(2.0分) | A.血流、血池相放射性明显增加,消失缓慢,延迟相放射性增加 B.血流相正常,血池相放射性增高,消失缓慢,延迟相正常 C.血流、血池相正常,延迟相呈放射性明显增加 D.血流相放射性灌注增加,血池相及延迟相呈放射性减低 E.以上均不是 |
A.血流、血池相放射性明显增加,消失缓慢,延迟相放射性增加 |
2 | 5 | 2013 | 4. 全身骨显像时肿瘤骨转移最常见的表现是(2.0分) | A.多发对称性放射性热区 B.超级显像 C.多发无规律放射性热区 D.炸面圈样改变 E.多发性放射性缺损区 |
C.多发无规律放射性热区 |
2 | 5 | 2013 | 5. 不属于特殊内脏运动核的神经核是(2.0分) | A.三叉神经运动核 B.滑车神经核 C.面神经核 D.副神经核 E.疑核 |
B.滑车神经核 |
2 | 5 | 2013 | 6. 当颅内压增高时,有可能出现枕骨大孔疝,导致病人生命危险。有关描述错误的是(2.0分) | A.枕骨大孔疝又称小脑扁桃体疝 B.小脑扁桃体被挤入枕骨大孔 C.压迫了脑桥 D.压迫了延髓 E.延髓内有生命中枢 |
C.压迫了脑桥 |
2 | 5 | 2013 | 7. 男孩,5岁,早晨起床后呕吐。站立不稳,走路时常向后跌倒。检查时发现:患儿站立时两脚叉开,检眼镜检查发现两眼严重视神经盘水肿,表明颅内压过高,提示有肿物存在。根据症状,肿物压迫的结构应该是(2.0分) | A.蚓部小结 B.右侧前叶 C.左侧前叶 D.右侧后叶 E.左侧后叶 |
A.蚓部小结 |
2 | 5 | 2013 | 8. 以下何种疾病不属于心身疾病 (2.0分) | A.冠心病 B.胃溃疡 C.精神分裂症 D.癌症 |
C.精神分裂症 |
2 | 5 | 2013 | 9. 强调心理社会因素通过何种生物学机制引起心身疾病的研究途径是 (2.0分) | A.心理生理学理论 B.学习理论 C.心理动力理论 D.人本主义理论 |
A.心理生理学理论 |
2 | 5 | 2013 | 10. 癌症病人听到癌症的诊断后,出现心理反应的顺序是(2.0分) | A.否认-怀疑期,愤怒-沮丧期,接受-适应期,休克-恐惧期 B.否认-怀疑期,休克-恐惧期,愤怒-沮丧期,接受-适应期 C.休克-恐惧期,愤怒-沮丧期,接受-适应期,否认-怀疑期 D.休克-恐惧期,否认-怀疑期,愤怒-沮丧期,接受-适应期 |
D.休克-恐惧期,否认-怀疑期,愤怒-沮丧期,接受-适应期 |
2 | 5 | 2013 | 11. 某病人拒绝医生有关癌症诊断,先后就诊许多医院,来证实自己没有病。该病人使用的心理防御机制是 (2.0分) | A.否认机制 B.压抑机制 C.转移机制 D.反向机制 |
A.否认机制 |
2 | 5 | 2013 | 12. 失眠的心理社会因素不包括(2.0分) | A.过度疲劳或紧张 B.对健康过度关心 C.不良自我暗示 D.入睡前饮用兴奋性饮料 |
D.入睡前饮用兴奋性饮料 |
2 | 5 | 2013 | 13. 以下不符合疼痛特征的一项描述是 (2.0分) | A.引起疼痛的刺激范围很广 B.刺激与痛觉之间存在必然的关系 C.疼痛具有积极意义 D.疼痛具有消极意义 |
B.刺激与痛觉之间存在必然的关系 |
2 | 5 | 2013 | 14. 关于一般适应综合征(GAS),以下错误的是 (2.0分) | A.分为觉醒期、阻抗期、适应期三个阶段 B.由加拿大生理学家塞里(H.Selye)提出 C.是非特异性的 D.是机体面临有害刺激时表现出的生理生化反应 |
A.分为觉醒期、阻抗期、适应期三个阶段 |
2 | 5 | 2013 | 15. 负性生活事件是指(2.0分) | A.需要付出机体能量应付的事件 B.公认的有害事件 C.与健康成负相关的事件 D.当事人感觉不愉快的事件 |
D.当事人感觉不愉快的事件 |
2 | 5 | 2013 | 16. 关于应激源与健康的关系,以下叙述不正确的是(2.0分) | A.应激源的性质如何与健康关系密切 B.应激源的数量多少与健康没有关系 C.不可控制的的应激源对健康影响更大 D.涉及生活核心部分的应激源对健康影响更大 |
B.应激源的数量多少与健康没有关系 |
2 | 5 | 2013 | 17. 下面不正确的叙述是: (2.0分) | A.在心理神经免疫中介机制中,免疫系统与中枢神经进行双向调节 B.心理应激降低免疫功能 C.慢性应激通过过多分泌皮质激素而抑制免疫功能 D.心理神经免疫中介机制是肿瘤心理病因学关注的对象 |
B.心理应激降低免疫功能 |
2 | 5 | 2013 | 18. 应激中不利于适应的人格特征不包括 (2.0分) | A.A型人格 B.B型人格 C.C型人格 D.D型人格 |
B.B型人格 |
2 | 5 | 2013 | 19. 关于心理应激的概念,下列叙述中不正确的是 (2.0分) | A.应激源涉及生物的、心理的、社会的和文化的 B.应激反应包括生理、心理、行为反应 C.应对方式在心理应激中起关键作用 D.应激的本质就是个体对内外环境的需求的适应 |
C.应对方式在心理应激中起关键作用 |
2 | 5 | 2013 | 20. 关于帕金森病的描述不正确的是(2.0分) | A.帕金森病主要临床症状为运动障碍 B.帕金森病主要表现为运动过度 C.家族性帕金森病只占所有帕金森病的很小比例 D.路易小体是其特征性病理变化 |
B.帕金森病主要表现为运动过度 |
2 | 5 | 2013 | 21. 下面哪些在流行病学调查提示是帕金森病(PD)发病的保护因子(2.0分) | A.衰老 B.杀虫剂接触史 C.服用非甾体类抗炎药物 D.PD家族史 |
C.服用非甾体类抗炎药物 |
2 | 5 | 2013 | 22. 鱼滕酮是脂溶性的天然杀虫剂,它在多胺能神经元内的主要作用靶点是? (2.0分) | A.线粒体复合物I B.线粒体复合物II C.蛋白酶体 D.溶酶体 E.细胞膜上的磷脂 |
A.线粒体复合物I |
2 | 5 | 2013 | 23. 以下哪一项是PD的主要病理特征(2.0分) | A.细胞浆中的路易氏小体 B.细胞外的淀粉样蛋白沉积 C.细胞核中的路易氏小体 D.细胞浆中的Tau蛋白沉积 E.细胞浆中的SOD1蛋白沉积 |
A.细胞浆中的路易氏小体 |
2 | 5 | 2013 | 24. 决定人执行功能的脑区是(2.0分) | A.颞叶 B.额叶 C.顶叶 D.视上回 |
B.额叶 |
2 | 5 | 2013 | 25. 人脑的左半球在下列方面占优势(2.0分) | A.概念、抽象、逻辑 B.空间知觉 C.音乐绘画 D.以上都不是 |
A.概念、抽象、逻辑 |
2 | 5 | 2013 | 26. 知觉的特性包括(2.0分) | A.选择 B.理解 C.恒常 D.三者皆有 |
D.三者皆有 |
2 | 5 | 2013 | 27. 思维的特征除了概括性外,还有(2.0分) | A.间接性 B.情绪性 C.全或无特性 D.不应期特性 |
A.间接性 |
2 | 5 | 2013 | 28. 大五人格特质除了神经质、宜人质,还包括(2.0分) | A.外向性 B.责任心 C.开放性 D.三者皆有 |
D.三者皆有 |
2 | 5 | 2013 | 29. 患者男,58岁。有高血压病史2年,控制不佳。本次因"突发右侧肢体无力伴言语障碍2小时"来急诊科。首选辅助检查为 (2.0分) | A.头颅CT; B.头颅MRI+DWI; C.脑电图; D.腰椎穿刺 |
A.头颅CT; |
2 | 5 | 2013 | 30. 运动神经传导速度反映的是(2.0分) | A.运动神经轴索的功能 B.运动神经髓鞘的功能 C.神经肌肉接头的功能 D.锥体束的功能 |
B.运动神经髓鞘的功能 |
2 | 5 | 2013 | 31. 患者男,30岁。因"发热头痛3天"入院。查体见颈项强直,克氏征阳性。外院头颅CT未见异常。下一步首选检查为 (2.0分) | A.腰椎穿刺; B.头颅核磁共振 C.脑电图 D.颈椎核磁共振 |
A.腰椎穿刺; |
2 | 5 | 2013 | 32. 帕金森病以下的哪项表述是不正确的? (2.0分) | A.多在中老年期发病; B.主要表现静止性震颤、运动迟缓和肌强直; C.通常的辅助检查无特殊发现; D.早期发现、早期治疗可治愈; E.抗胆碱能药物适用于震颤明显的较年轻患者 |
D.早期发现、早期治疗可治愈; |
2 | 5 | 2013 | 33. 帕金森病不会出现的体征是 (2.0分) | A.手的搓丸样震颤; B.齿轮样肌强直; C.面具脸; D.挤奶妇手法; E.慌张步态 |
D.挤奶妇手法; |
2 | 5 | 2013 | 34. 帕金森病患者常出现齿轮样肌张力障碍,原因是 (2.0分) | A.患者不配合,抗拒检查; B.被检查肢体合并有不自主震颤; C.患者精神紧张、肢体发抖; D.检查者用力过度 |
B.被检查肢体合并有不自主震颤; |
2 | 5 | 2013 | 35. 帕金森病震颤的特点是(2.0分) | A.随意动作时明显或加重 B.静止时明显 C.紧张时减轻; D.睡眠时仍存在 E.多双侧上肢远端同时发生 |
B.静止时明显 |
2 | 5 | 2013 | 36. 帕金森病的主要病理改变是(2.0分) | A.脑桥及延髓 B.中脑黑质 C.中脑四叠体; D.小脑 E.尾状核及豆状核 |
B.中脑黑质 |
2 | 5 | 2013 | 37. 帕金森病的主要生化改变是(2.0分) | A.乙酰胆碱含量减少 B.多巴胺含量减少 C.多巴脱羧酶含量减少; D.胆碱酯酶含量减少 E.乙酰胆碱含量增多 |
B.多巴胺含量减少 |
2 | 5 | 2013 | 38. 帕金森病以下哪项诊断及治疗的表述是正确的? (2.0分) | A.脑脊液检查对诊断颇有价值; B.MRI检查有特征性表现; C.帕金森病一经确诊应立刻开始左旋多巴治疗; D.美多巴是由左旋多巴加苄丝肼组成,疗效优于左旋多巴 E.服用左旋多巴出现周围性副作用时应立即停药 |
D.美多巴是由左旋多巴加苄丝肼组成,疗效优于左旋多巴 |
2 | 5 | 2013 | 39. 震颤麻痹的病人禁止使用哪类药物? (2.0分) | A.金刚烷胺 B.抗胆碱能药物 C.单胺氧化酶抑制剂 D.多巴胺受体激动剂 E.吩噻嗪类药 |
E.吩噻嗪类药 |
2 | 5 | 2013 | 40. 男患,69岁,患帕金森病,并患前列腺增生,下列药物中哪一种不宜应用(2.0分) | A.苯海索 B.多巴丝肼 C.苯海拉明 D.金刚烷胺 E.溴隐亭 |
A.苯海索 |
2 | 5 | 2013 | 41. 男,47岁,近3月出现讲话缓慢,声音低沉,右手抖动,右上肢运动迟缓。发病前20天有一氧化碳中毒史。最可能诊断是(2.0分) | A.帕金森病 B.帕金森叠加综合症 C.帕金森综合症 D.特发性震颤 E.脑血栓形成 |
C.帕金森综合症 |
2 | 5 | 2013 | 42. 关于亨廷顿舞蹈病的诊断和治疗错误的是(2.0分) | A.根据发病年龄,临床表现及家族史即可确诊; B.本病目前尚无特效治疗措施; C.对于舞蹈症状可选用氟哌啶醇; D.头颅MRI一般表现为皮质萎缩,脑室扩大; E.EEG表现一般无特异性 |
A.根据发病年龄,临床表现及家族史即可确诊; |
2 | 5 | 2013 | 43. 关于特发性震颤正确的是 (2.0分) | A.可以呈常染色体显性遗传; B.病情一般缓慢进展,最后致残; C.药物治疗效果均不佳; D.患者最终残疾严重,需要行手术治疗,如深部脑刺激; E.饮酒后症状加重 |
A.可以呈常染色体显性遗传; |
2 | 5 | 2013 | 44. 关于肝豆状核变性,错误的是 (2.0分) | A.50岁以后也可以起病; B.神经系统可表现为运动减少,也可表现为舞蹈; C.血清铜蓝蛋白正常不能除外肝豆状核变性; D.基因为ATP7B基因; E.本病为遗传性疾病,即使积极诊治,病情也会进行性进展,直至严重致残 |
E.本病为遗传性疾病,即使积极诊治,病情也会进行性进展,直至严重致残 |
2 | 5 | 2013 | 45. 下面哪种药物不用于治疗帕金森病 (2.0分) | A.Pethidine (哌替啶) B.L-dopa C.Selegiline D.Amantatine E.Trihexyphenidyl(苯海索) |
A.Pethidine (哌替啶) |
2 | 5 | 2013 | 46. 下列药物组合能增强疗效的是 (2.0分) | A.L-dopa与vitamin B6 B.L-dopa与甲氧氯普胺 C.L-dopa与selegiline D.L-dopa与chlorpromazine E.Carbidopa与selegiline |
C.L-dopa与selegiline |
2 | 5 | 2013 | 47. 单用时无抗帕金森病作用的药物是 (2.0分) | A.Pergolide B.Carbidopa C.Trihexyphenidyl D.L-dopa E.Pramipexole |
B.Carbidopa |
2 | 5 | 2013 | 48. 关于苯海索(trihexyphenidy)的叙述错误的是 (2.0分) | A.阻断黑质纹状体多巴胺通路 B.阻断中枢胆碱受体 C.对心脏的影响较阿托品弱 D.抗震颤疗效好 E.能致尿潴留,前列腺肥大者慎用 |
A.阻断黑质纹状体多巴胺通路 |
2 | 5 | 2013 | 49. 普拉克索(pramipexole)治疗帕金森病,是由于 (2.0分) | A.中枢抗胆碱作用 B.激动D2受体 C.激动GABA受体 D.提高脑内DA浓度 E.使DA降解减少 |
B.激动D2受体 |
2 | 5 | 2013 | 50. 司来吉兰(selegiline)的作用机制是 (2.0分) | A.抑制中枢乙酰胆碱酯酶(AChE) B.抑制中枢单胺氧化酶(MAO) C.抑制中枢儿茶酚氧位甲基转移酶(COMT) D.阻断中枢M胆碱受体 E.促进中枢DA的释放 |
B.抑制中枢单胺氧化酶(MAO) |
2 | 5 | 2014 | 1. 骨显像在临床应用广泛,以下哪项不是其适应症(2.0分) | A.判断原发性骨肿瘤的范围 B.诊断隐匿性骨折 C.监测骨移植存活情况 D.诊断骨囊肿 E.早期诊断骨转移瘤 |
D.诊断骨囊肿 |
2 | 5 | 2014 | 2. 急性骨髓炎时全身骨显像的表现是(2.0分) | A.血流相正常,血池相放射性增高,消失缓慢,延迟相正常 B.血流.血池相放射性明显增加,消失缓慢,延迟相放射性增加 C.血流.血池相正常,延迟相呈放射性明显增加 D.血流相放射性灌注增加,血池相及延迟相呈放射性减低 E.血流、血池相放射性降低,延迟相呈放射性明显增加 |
B.血流.血池相放射性明显增加,消失缓慢,延迟相放射性增加 |
2 | 5 | 2014 | 3. 有关核素全身骨显像的原理以下哪项说法是正确的(2.0分) | A.微循环栓塞 B.排泄通路 C.化学吸附和离子交换 D.细胞吞噬 E.特异性结合 |
C.化学吸附和离子交换 |
2 | 5 | 2014 | 4. 骨显像对骨转移性肿瘤的诊断,尤其是原发病灶肯定时寻找骨转移灶颇具临床价值,是由于: (2.0分) | A.灵敏度高 B.特异性高 C.解剖结构清晰 D.软组织侵犯显示清晰 E.可显示髓内侵犯 |
A.灵敏度高 |
2 | 5 | 2014 | 5. 18F-FDG用于脑葡萄糖代谢显像,主要由于脱氧葡萄糖较葡萄糖(2.0分) | A.进入脑组织快 B.进入脑组织多 C.在脑内代谢氟脱氧葡萄糖-6-磷酸能进一步代谢 D.在脑内代谢产物氟脱氧葡萄糖-6-磷酸不能进一步代谢 E.在脑内清除快 |
D.在脑内代谢产物氟脱氧葡萄糖-6-磷酸不能进一步代谢 |
2 | 5 | 2014 | 6. 脑血流灌注显像中脑梗死灶显示灌注减低区范围较X线、CT为(2.0分) | A.范围大,且发现早 B.范围小,且发现早 C.范围大,但发现晚 D.范围小,且发现晚 E.范围一样,但发现晚 |
A.范围大,且发现早 |
2 | 5 | 2014 | 7. 早老性痴呆SPECT脑灌注显像可见到(2.0分) | A.轮圈征 B.分水岭征 C.月晕征 D.新月形征 E.热鼻征 |
B.分水岭征 |
2 | 5 | 2014 | 8. 下列哪一项不是脑血流灌注显像介入试验的临床应用(2.0分) | A.精神分裂症的诊断 B.隐匿性脑缺血病灶的探测 C.短暂性脑缺血发作 D.脑血管储备能力的确定 E.失联络现象中血管反应性的判断像 |
C.短暂性脑缺血发作 |
2 | 5 | 2014 | 9. 关于正性生活事件,以下叙述不正确的是 (2.0分) | A.结婚是正性生活事件 B.是当事人认为积极的事件 C.是当事人认为愉快的事件 D.同样需要付出机体能量应对 |
A.结婚是正性生活事件 |
2 | 5 | 2014 | 10. 关于应激的心理反应,以下叙述不正确的是(2.0分) | A.应激的心理反应涉及"知、情、意、人格"各方面 B.应激的认知反应常常使个体的认知功能受损 C.应激的情绪反应包括焦虑、恐惧、抑郁、愤怒等 D.假如应激过于强烈或应激时间过长,有可能导致人格发生改变 |
B.应激的认知反应常常使个体的认知功能受损 |
2 | 5 | 2014 | 11. 关于应对概念,以下叙述正确的是(2.0分) | A.应对活动涉及应激全过程 B.应对都是个体有意识进行的 C.应对保护个体免受应激损害 D.应对是针对应激性事件采取的行为措施 |
A.应对活动涉及应激全过程 |
2 | 5 | 2014 | 12. 与健康和疾病关系最直接的应激心理反应是(2.0分) | A.认知改变 B.情绪反应 C.个性改变 D.社会适应能力下降 |
B.情绪反应 |
2 | 5 | 2014 | 13. 关于社会支持,以下叙述错误的是(2.0分) | A.具有减轻应激的作用 B.一般而言社会支持对健康具有保护作用 C.肖水源把社会支持分为主观支持和客观支持 D.是指个体来自社会的精神上和物质上的帮助和支援 |
C.肖水源把社会支持分为主观支持和客观支持 |
2 | 5 | 2014 | 14. 以下不属于情绪关注应对的是 (2.0分) | A.积极重新评价 B.面对应对 C.自我控制 D.回避 |
B.面对应对 |
2 | 5 | 2014 | 15. 说明心理社会因素"如何"通过神经内分泌和免疫系统造成躯体疾病的研究领域称为:(2.0分) | A.神经免疫学 B.心理免疫学 C.免疫学 D.心理神经免疫学 |
D.心理神经免疫学 |
2 | 5 | 2014 | 16. 癌症病人听到癌症的诊断后,出现心理反应的顺序是(2.0分) | A.否认-怀疑期,愤怒-沮丧期,接受-适应期,休克-恐惧期 B.否认-怀疑期,休克-恐惧期,愤怒-沮丧期,接受-适应期 C.休克-恐惧期,愤怒-沮丧期,接受-适应期,否认-怀疑期 D.休克-恐惧期,否认-怀疑期,愤怒-沮丧期,接受-适应期 |
D.休克-恐惧期,否认-怀疑期,愤怒-沮丧期,接受-适应期 |
2 | 5 | 2014 | 17. 父母对子女的轻微损伤表示大惊小怪或者泰然处之,影响子女成年后对疼痛的态度属于: (2.0分) | A.社会学习 B.对处境的认知评价 C.注意 D.暗示 |
A.社会学习 |
2 | 5 | 2014 | 18. 下面哪种药物不用于治疗帕金森病 (2.0分) | A.Pethidine (哌替啶) B.L-dopa C.Selegiline D.Amantatine E.Benzhexol(苯海索) |
A.Pethidine (哌替啶) |
2 | 5 | 2014 | 19. 下列药物组合能增强疗效的是 (2.0分) | A.L-dopa与vitamin B6 B.L-dopa与甲氧氯普胺 C.L-dopa与selegiline D.L-dopa与chlorpromazine E.Carbidopa与selegiline |
C.L-dopa与selegiline |
2 | 5 | 2014 | 20. 单用时无抗帕金森病作用的药物是 (2.0分) | A.Pergolide B.Carbidopa C.Trihexyphenidyl D.L-dopa E.Pramipexole |
B.Carbidopa |
2 | 5 | 2014 | 21. 关于苯海索(benzhexol)的叙述错误的是 (2.0分) | A.阻断黑质纹状体多巴胺通路 B.阻断中枢胆碱受体 C.对心脏的影响较阿托品弱 D.抗震颤疗效好 E.能致尿潴留,前列腺肥大者慎用 |
A.阻断黑质纹状体多巴胺通路 |
2 | 5 | 2014 | 22. 司来吉兰(selegiline)的作用机制是 (2.0分) | A.抑制中枢乙酰胆碱酯酶(AChE) B.抑制中枢单胺氧化酶B(MAOB) C.抑制中枢儿茶酚氧位甲基转移酶(COMT) D.阻断中枢M胆碱受体 E.促进中枢DA的释放 |
B.抑制中枢单胺氧化酶B(MAOB) |
2 | 5 | 2014 | 23. 导致帕金森病(PD)中多巴胺能神经元死亡的可能机制包括(2.0分) | A.蛋白错误折叠 B.线粒体功能障碍 C.氧化应激 D.炎性反应 E.以上所有 |
E.以上所有 |
2 | 5 | 2014 | 24. 下列关于牵张反射的叙述,错误的是:(2.0分) | A.骨骼肌受到外力牵拉时能反射性地引起受牵拉的同一肌肉的收缩 B.牵张反射是维持姿势的基本反射 C.牵张反射的感受器是肌梭 D.在脊髓与高位中枢离断后,牵张反射即永远消失 |
D.在脊髓与高位中枢离断后,牵张反射即永远消失 |
2 | 5 | 2014 | 25. 下面哪个不属于去大脑僵直的表现(2.0分) | A.四肢弯曲 B.坚硬如柱 C.头尾昂起 D.脊柱挺硬 |
A.四肢弯曲 |
2 | 5 | 2014 | 26. I型传入纤维的神经末梢是:(2.0分) | A.板状末梢 B.蔓状末梢 C.螺旋形末梢 D.花枝状末梢 |
C.螺旋形末梢 |
2 | 5 | 2014 | 27. 患者男,58岁。有高血压病史2年,控制不佳。本次因"突发右侧肢体无力伴言语障碍2小时"来急诊科。首选辅助检查为(2.0分) | A.头颅CT; B.头颅MRI+DWI; C.脑电图; D.腰椎穿刺 |
A.头颅CT; |
2 | 5 | 2014 | 28. 患者男,30岁。因"发热头痛3天"入院。查体见颈项强直,克氏征阳性。急诊首选检查为(2.0分) | A.腰椎穿刺; B.头颅核磁共振 C.脑电图 D.头颅CT |
D.头颅CT |
2 | 5 | 2014 | 29. 患者男,40岁。因"眼睑下垂20天,吞咽障碍5天"入院。考虑重症肌无力。下面哪项检查为首选 (2.0分) | A.肌电图(周围神经传导速度检查) B.肌电图 (插入电位及静息电位) C.肌电图(重频刺激试验) D.头颅核磁共振 |
C.肌电图(重频刺激试验) |
2 | 5 | 2014 | 30. 年轻男性,因"头疼2天"入院。头颅CT提示蛛网膜下腔出血。头颅CTA未见异常。下面哪项检查可以进一步明确病因 (2.0分) | A.头颅MRA B.头颅增强MRI C.DSA D.腰椎穿刺 |
C.DSA |
2 | 5 | 2014 | 31. 新小脑的传人纤维来自 (2.0分) | A.上橄榄核 B.前庭神经核 C.红核 D.脑桥核 E.下橄榄核 |
D.脑桥核 |
2 | 5 | 2014 | 32. 属于前庭小脑的是(2.0分) | A.小脑前叶 B.小脑后叶 C.小脑蚓部 D.小脑半球 E.绒球小结叶 |
E.绒球小结叶 |
2 | 5 | 2014 | 33. 不属于小脑核的是(2.0分) | A.顶核 B.球状 C.栓状核 D.齿状核 E.下橄榄核 |
E.下橄榄核 |
2 | 5 | 2014 | 34. 决定人执行功能的脑区是(2.0分) | A.颞叶 B.额叶 C.顶叶 D.视上回 |
B.额叶 |
2 | 5 | 2014 | 35. 知觉的特性包括 (2.0分) | A.选择 B.理解 C.恒常 D.三者皆有 |
D.三者皆有 |
2 | 5 | 2014 | 36. 思维的特征除了概括性外,还有(2.0分) | A.间接性 B.情绪性 C.全或无特性 D.不应期特性 |
A.间接性 |
2 | 5 | 2014 | 37. 大五人格特质除了神经质、宜人质,还包括 (2.0分) | A.外向性 B.责任心 C.开放性 D.三者皆有 |
D.三者皆有 |
2 | 5 | 2014 | 38. 与强迫特质对应的正常人格特质是 (2.0分) | A.社交性 B.责任心 C.外向性 D.开放性 |
B.责任心 |
2 | 5 | 2014 | 39. 帕金森病以下的哪项表述是不正确的? (2.0分) | A.多在中老年期发病; B.主要表现静止性震颤、运动迟缓和肌强直; C.通常的辅助检查无特殊发现; D.早期发现、早期治疗可治愈; E.抗胆碱能药物适用于震颤明显的较年轻患者 |
D.早期发现、早期治疗可治愈; |
2 | 5 | 2014 | 40. 帕金森病患者常出现齿轮样肌张力障碍,原因是(2.0分) | A.患者不配合,抗拒检查; B.被检查肢体合并有不自主震颤; C.患者精神紧张、肢体发抖; D.检查者用力过度 |
B.被检查肢体合并有不自主震颤; |
2 | 5 | 2014 | 41. 帕金森病的主要病理改变是(2.0分) | A.脑桥及延髓; B.中脑黑质; C.中脑四叠体; D.小脑; E.尾状核及豆状核 |
B.中脑黑质; |
2 | 5 | 2014 | 42. 帕金森病主要的生化病理改变为黑质变性,在纹状体内可出现(2.0分) | A.多巴胺含量减少; B.多巴胺含量增多; C.乙酰胆碱减少; D.高香草酸含量增多; E.乙酰胆碱含量增多 |
A.多巴胺含量减少; |
2 | 5 | 2014 | 43. 帕金森病以下哪项诊断及治疗的表述是正确的? (2.0分) | A.脑脊液检查对诊断颇有价值; B.MRI检查有特征性表现; C.帕金森病一经确诊应立刻开始左旋多巴治疗; D.美多巴是由左旋多巴加苄丝肼组成,疗效优于左旋多巴 E.服用左旋多巴出现周围性副作用时应立即停药 |
D.美多巴是由左旋多巴加苄丝肼组成,疗效优于左旋多巴 |
2 | 5 | 2014 | 44. 男患,69岁,患帕金森病,并患前列腺增生,下列药物中哪一种不宜应用(2.0分) | A.苯海索 B.多巴丝肼 C.苯海拉明 D.金刚烷胺 E.溴隐亭 |
A.苯海索 |
2 | 5 | 2014 | 45. 男性, 58岁, 逐渐出现四肢震颤,左侧明显,呈"搓丸样",动作缓慢,被动运动时肢体为齿轮样肌张力升高,需使用下列哪种药物治疗(2.0分) | A.新斯的明 B.左旋多巴 C.卡马西平 D.多巴胺 E.苯妥英钠 |
B.左旋多巴 |
2 | 5 | 2014 | 46. 男,47岁,近3月出现讲话缓慢,声音低沉,右手抖动,右上肢运动迟缓。发病前20天有一氧化碳中毒史。最可能诊断是 (2.0分) | A.帕金森病 B.帕金森叠加综合症 C.帕金森综合症 D.特发性震颤 E.脑血栓形成 |
C.帕金森综合症 |
2 | 5 | 2014 | 47. 亨廷顿舞蹈病临床表现不正确的是 (2.0分) | A.锥体外系症状以舞蹈多见; B.有精神症状和痴呆表现; C.不会出现肌强直等帕金森综合征表现; D.有睡眠障碍和癫痫; E.也可老年起病 |
C.不会出现肌强直等帕金森综合征表现; |
2 | 5 | 2014 | 48. 关于特发性震颤临床表现错误的是 (2.0分) | A.震颤是唯一临床表现; B.主要表现为姿势性震颤及动作性震颤; C.头部可以受累; D.下肢较上肢多见; E.情绪紧张时加重 |
D.下肢较上肢多见; |
2 | 5 | 2014 | 49. 关于亨廷顿病的遗传,错误的是(2.0分) | A.CAG重复序列拷贝数增多,症状更重; B.CAG重复序列拷贝数增多,发病越早; C.亨廷顿病后代有发病提早倾向; D.亨廷顿病早发现象在母系遗传更为明显; E.亨廷顿病为完全外显率 |
D.亨廷顿病早发现象在母系遗传更为明显; |
2 | 5 | 2014 | 50. 关于肝豆状核变性,错误的是(2.0分) | A.50岁以后也可以起病; B.神经系统可表现为运动减少,也可表现为舞蹈; C.血清铜蓝蛋白正常不能除外肝豆状核变性; D.基因为ATP7B基因; E.本病为遗传性疾病,即使积极诊治,病情也会进行性进展,直至严重致残 |
E.本病为遗传性疾病,即使积极诊治,病情也会进行性进展,直至严重致残 |
2 | 5 | 2015 | 1. 强调潜意识心理冲突、个体心理特征产生心身疾病的研究途径是(2.0分) | A.学习理论 B.心理动力理论 C.心理生理学理论 D.人本主义理论 |
B.心理动力理论 |
2 | 5 | 2015 | 2. 心身疾病的主要干预方法: (2.0分) | A.消除心理社会刺激因素 B.行为矫正 C.认知干预治疗 D.以上都是 |
D.以上都是 |
2 | 5 | 2015 | 3. 下列有关心理社会因素与癌症关系的叙述不正确的是: (2.0分) | A.不良心理反应和应对方式对癌症病人生活质量有严重影响 B.争强好胜、时间紧迫感、脾气暴躁不是癌症病人的行为特点 C.癌症病人常常有意识疏泄自己的情绪来减轻紧张和痛苦 D.突然得知患癌消息的病人情绪反应强烈,易导致免疫力下降 |
C.癌症病人常常有意识疏泄自己的情绪来减轻紧张和痛苦 |
2 | 5 | 2015 | 4. A 型行为的特征是(2.0分) | A.时间紧迫感 B.竞争性强 C.好斗敌意倾向 D.以上都是 |
D.以上都是 |
2 | 5 | 2015 | 5. 患者夜间易醒,或醒后不能再入睡,属于睡眠障碍中的: (2.0分) | A.失眠 B.保持睡眠困难 C.入睡困难 D.早醒 |
B.保持睡眠困难 |
2 | 5 | 2015 | 6. 父母对子女的轻微损伤表示大惊小怪或者泰然处之,影响子女成年后对疼痛的态度属于: (2.0分) | A.暗示 B.社会学习 C.注意 D.对处境的认知评价 |
B.社会学习 |
2 | 5 | 2015 | 7. "活泼好动"属于下列哪种概念范畴: (2.0分) | A.能力 B.兴趣 C.气质 D.性格 |
C.气质 |
2 | 5 | 2015 | 8. 与人的健康和疾病关系最为直接的心理现象是(2.0分) | A.情绪 B.认知 C.意志 D.人格 |
A.情绪 |
2 | 5 | 2015 | 9. 心理是脑的功能,以下哪个脑区与计划调节和控制复杂心理活动的系统有关: (2.0分) | A.顶叶 B.颞叶 C.枕叶 D.前额叶 |
D.前额叶 |
2 | 5 | 2015 | 10. 医生对病人躯体症状的认识比一般人更为准确,是因为知觉具有(2.0分) | A.选择性 B.整体性 C.理解性 D.恒常性 |
C.理解性 |
2 | 5 | 2015 | 11. 表情的分类不包括(2.0分) | A.面部表情 B.身段表情 C.言语表情 D.动态表情 |
D.动态表情 |
2 | 5 | 2015 | 12. 哪种注意状态下的学习效率最高(2.0分) | A.无意注意 B.有意注意 C.稳定注意 D.有意后注意 |
D.有意后注意 |
2 | 5 | 2015 | 13. 埃里克森的人格发展八阶段理论认为,以下哪一个人生阶段需要解决角色混乱危机建立自我同一性:(2.0分) | A.童年期 B.青少年期 C.成年早期 D.成年中期 |
B.青少年期 |
2 | 5 | 2015 | 14. 感觉的产生过程中,不包括的是(2.0分) | A.感受器 B.神经元 C.中枢 D.肌肉 |
D.肌肉 |
2 | 5 | 2015 | 15. 感受器能把刺激所包含的环境变化信息,转移到动作电位的序列之中,感受器的这种生理特性称为 (2.0分) | A.感受器的适宜刺激 B.感受器的换能作用 C.感受器的编码作用 D.感受器的适应现象 E.感受器的启动电位 |
C.感受器的编码作用 |
2 | 5 | 2015 | 16. 脊髓的何种损伤可导致痛温觉与轻触觉障碍分离? (2.0分) | A.完全横断 B.脊髓空洞症 C.脊髓离断 D.后角受损 E.前根病变 |
B.脊髓空洞症 |
2 | 5 | 2015 | 17. 以下属于丘脑特异性投射系统的核团是 (2.0分) | A.联络核 B.网状核 C.感觉接替核 D.髓板内核群 E.感觉接替核和联络核 |
E.感觉接替核和联络核 |
2 | 5 | 2015 | 18. 以下属于丘脑非特异投射系统的主要细胞群是 (2.0分) | A.联络核和髓板内核群 B.感觉接替核和联络核 C.感觉接替核 D.联络核 E.髓板内核群 |
E.髓板内核群 |
2 | 5 | 2015 | 19. 内侧膝状体接受下列哪种感觉纤维投射? (2.0分) | A.内侧丘系 B.三叉丘系 C.视觉传导束 D.听觉传导束 E.脊髓丘脑前束 |
D.听觉传导束 |
2 | 5 | 2015 | 20. 以下哪种离子通道介导冷觉感知 (2.0分) | A.TRPM2 B.TRPV1 C.TRPV4 D.TRPM8 E.TRPC1 |
D.TRPM8 |
2 | 5 | 2015 | 21. 第一感觉区位于 (2.0分) | A.中央前回 B.中央后回 C.颞叶皮层 D.中央前回与岛叶之间 E.颞下回 |
B.中央后回 |
2 | 5 | 2015 | 22. 交感神经系统不具有的特点是 (2.0分) | A.节前纤维长,节后纤维短 B.支配几乎所有的脏器 C.紧张性活动 D.在应急反应中活动明显加强 |
A.节前纤维长,节后纤维短 |
2 | 5 | 2015 | 23. 下列属于副交感神经的作用的是 (2.0分) | A.瞳孔扩大 B.糖原分解增加 C.逼尿肌收缩 D.骨骼肌血管舒张 E.消化道括约肌收缩 |
C.逼尿肌收缩 |
2 | 5 | 2015 | 24. 下列不属于交感神经兴奋作用的是 (2.0分) | A.心跳加快 B.瞳孔开大 C.支气管平滑肌舒张 D.胃肠平滑肌收缩 E.肾上腺髓质分泌肾上腺素 |
D.胃肠平滑肌收缩 |
2 | 5 | 2015 | 25. 下列关于感觉皮层代表区的叙述,错误的是 (2.0分) | A.本体感觉在中央前回 B.内脏感觉的投射区在额叶 C.体表感觉区在中央后回 D.听觉代表区在颞叶 E.视觉代表区在枕叶 |
B.内脏感觉的投射区在额叶 |
2 | 5 | 2015 | 26. 非条件反射的特点不包括 (2.0分) | A.生来具有 B.无需皮层参与 C.反射弧固定 D.反射形式多变 E.数量有限 |
D.反射形式多变 |
2 | 5 | 2015 | 27. 感觉的最高级中枢是 (2.0分) | A.下丘脑 B.大脑皮层 C.海马 D.丘脑 E.脑干 |
D.丘脑 |
2 | 5 | 2015 | 28. 谈论梅子时引起唾液分泌是 (2.0分) | A.交感神经兴奋所致 B.副交感神经兴奋所致 C.第一信号系统的活动 D.第二信号系统的活动 E.非条件反射 |
D.第二信号系统的活动 |
2 | 5 | 2015 | 29. 下面哪种药物不用于治疗帕金森病 (2.0分) | A.Pethidine (哌替啶) B.L-dopa C.Selegiline D.Amantatine E.Benzhexol(苯海索) |
A.Pethidine (哌替啶) |
2 | 5 | 2015 | 30. 下列药物组合能增强疗效的是 (2.0分) | A.L-dopa与vitamin B6 B.Carbidopa与 entacapone C.L-dopa与selegiline D.L-dopa与chlorpromazine E.Carbidopa与selegiline |
C.L-dopa与selegiline |
2 | 5 | 2015 | 31. 单用时无抗帕金森病作用的药物是 (2.0分) | A.Ropinirole B.Carbidopa C.Selegiline D.L-dopa E.Pramipexole |
B.Carbidopa |
2 | 5 | 2015 | 32. 关于苯海索(benzhexol)的叙述错误的是 (2.0分) | A.阻断黑质纹状体多巴胺通路 B.阻断中枢胆碱受体 C.对心脏的影响较阿托品弱 D.抗震颤疗效好 E.能致尿潴留,前列腺肥大者慎用 |
A.阻断黑质纹状体多巴胺通路 |
2 | 5 | 2015 | 33. 司来吉兰(selegiline)的作用机制是(2.0分) | A.抑制中枢乙酰胆碱酯酶(AChE) B.抑制中枢单胺氧化酶B(MAOB) C.抑制中枢儿茶酚氧位甲基转移酶(COMT) D.阻断中枢M胆碱受体 E.促进中枢DA的释放 |
B.抑制中枢单胺氧化酶B(MAOB) |
2 | 5 | 2015 | 34. 有关多巴胺转运体显像的原理,以下说法正确的是(2.0分) | A.显像剂为水溶性 B.由于血脑屏障受到破坏 C.为代谢显像 D.为血流灌注显像 E.为受体显像 |
E.为受体显像 |
2 | 5 | 2015 | 35. 以下有关多巴胺转运体显像的临床应用正确的是:(2.0分) | A.早期帕金森病的诊断 B.特发性震颤的诊断 C.脑肿瘤的诊断 D.癫痫的诊断 E.短暂性脑缺血的诊断 |
A.早期帕金森病的诊断 |
2 | 5 | 2015 | 36. 早期帕金森病在多巴胺转运体显像中的表现是(2.0分) | A.纹状体局部放射性增高 B.纹状体局部放射性减低 C.纹状体局部放射性正常 D.豆状体局部放射性增高 E.豆状体局部放射性减低 |
B.纹状体局部放射性减低 |
2 | 5 | 2015 | 37. PD患者脑中哪种神经元丢失最严重(2.0分) | A.VTA 多巴胺神经元 B.中脑黑质区多巴胺神经元 C.纹状体MSN神经元 D.海马锥体神经元 |
B.中脑黑质区多巴胺神经元 |
2 | 5 | 2015 | 38. 以下哪些结构不属于基底神经节(2.0分) | A.尾(状)核、壳核 B.下丘脑 C.丘脑底核 D.中脑黑质区 |
B.下丘脑 |
2 | 5 | 2015 | 39. 导致帕金森病(PD)中多巴胺能神经元死亡的可能机制包括(2.0分) | A.蛋白错误折叠 B.线粒体功能障碍 C.氧化应激 D.炎性反应 E.以上所有 |
E.以上所有 |
2 | 5 | 2015 | 40. 男性患者,6个月前发现右手活动不灵活,开门时,钥匙不能插进锁内,检查发现右手指鼻尖时颤抖而不准确,提示可能损伤了 (2.0分) | A.左侧新小脑 B.右侧旧小脑 C.右侧新小脑 D.左侧旧小脑 E.右侧古小脑 |
C.右侧新小脑 |
2 | 5 | 2015 | 41. 中脑内有 (2.0分) | A.黑质 B.面神经核 C.三叉神经脊束核 D.展神经核 E.疑核 |
A.黑质 |
2 | 5 | 2015 | 42. 某人得了嗜睡症,工作、进食时都能人睡,是由于损伤了网状上行激活系统,与此系统有关的核团是(2.0分) | A.中线核、网状核和板内核 B.腹前核和背内侧核 C.中线核、枕和板内核 D.后外侧核、网状核和板内核 E.中线核、网状核和背外侧核 |
A.中线核、网状核和板内核 |
2 | 5 | 2015 | 43. 延髓内的非脑神经核为 (2.0分) | A.舌下神经核 B.疑核 C.薄束核 D.迷走神经背核 E.泌涎核 |
C.薄束核 |
2 | 5 | 2015 | 44. 丘脑的腹后外侧核接受 (2.0分) | A.三叉丘系纤维 B.嗅束纤维 C.外侧丘系纤维 D.视束纤维 E.内侧丘系纤维 |
E.内侧丘系纤维 |
2 | 5 | 2015 | 45. 关于脑脊液循环,以下说法错误的是(2.0分) | A.脑脊液存在于脑室及蛛网膜下腔内 B.脑脊液主要由侧脑室脉络丛产生 C.大部分CSF经脑穹窿面的蛛网膜颗粒吸收至上矢状窦 D.成人CSF总量为300~500ml E.脑脊液压力正常值80-180mmH2O |
D.成人CSF总量为300~500ml |
2 | 5 | 2015 | 46. 腰穿的并发症包括(2.0分) | A.腰穿后头痛 B.出血 C.感染 D.脑疝 E.以上都对 |
E.以上都对 |
2 | 5 | 2015 | 47. 脑电图癫痫样波表现为(2.0分) | A.棘波 (40-80 ms) B.尖波(80-200 ms) C.棘慢综合波 D.多棘波 E.以上都是 |
E.以上都是 |
2 | 5 | 2015 | 48. 若患者怀疑糖尿病周围神经病变,则首选的肌电图检查项目为(2.0分) | A.神经传导速度 B.针极肌电图 C.重复神经电刺激 D.体感诱发电位 E.SSR |
A.神经传导速度 |
2 | 5 | 2015 | 49. 神经活检最常用的取材部位是(2.0分) | A.正中神经 B.尺神经 C.腓肠神经 D.胫神经 E.股神经 |
C.腓肠神经 |
2 | 5 | 2015 | 50. 中枢神经系统PET检查的适应症包括(2.0分) | A.帕金森病 B.痴呆 C.癫痫 D.脑肿瘤 E.以上都对 |
E.以上都对 |
2 | 6 | 2007 | 1. 所谓颅内巨大动脉瘤是指直径大于(3.0分) | A.1.5cm B.1cm C.2cm D.2.5cm E.3cm |
D.2.5cm |
2 | 6 | 2007 | 2. 发生脑栓塞最多的栓子来源是: (3.0分) | A.空气栓子 B.脂肪栓子 C.心源性栓子 D.下肢静脉血栓的脱落 E.肺动脉血栓 |
C.心源性栓子 |
2 | 6 | 2007 | 3. 一儿童患者,突发头痛、呕吐伴癫痫,CT示脑实质内出血约10ml,最可能的出血原因是: (3.0分) | A.动脉瘤 B.MOYAMOYA病 C.脑动静脉畸形 D.海绵状血管瘤 E.高血压脑出血 |
C.脑动静脉畸形 |
2 | 6 | 2007 | 4. 与健康和疾病关系最直接的应激心理反应是: (3.0分) | A.个性改变 B.情绪反应 C.认知改变 D.社会适应能力下降 |
B.情绪反应 |
2 | 6 | 2007 | 5. 基底节出血典型的三偏征是: (3.0分) | A.偏盲、偏侧肌肉痉挛、偏头痛; B.偏瘫、偏盲、偏头痛; C.偏瘫、偏盲、偏侧肌肉痉挛; D.偏瘫、偏盲、偏身感觉障碍; E.以上均不是 |
D.偏瘫、偏盲、偏身感觉障碍; |
2 | 6 | 2007 | 6. DW呈高信号的病变。(3.0分) | A.脑脓肿 B.急性脑梗死 C.蛛网膜囊肿 D.慢性脑梗死 |
A.脑脓肿 |
2 | 6 | 2007 | 7. 诊断动脉瘤的金标准。(3.0分) | A.MRA B.CTA C.TCD D.DSA |
D.DSA |
2 | 6 | 2007 | 8. 以下哪一项不是颅内压增高的常见临床表现(3.0分) | A.头晕 B.呕吐 C.头痛 D.意识障碍 |
A.头晕 |
2 | 6 | 2007 | 9. 判断急性蛛网膜下腔,首选 (3.0分) | A.MR平扫 B.CT增强 C.CT平扫 D.MR增强 |
C.CT平扫 |
2 | 6 | 2007 | 10. 以下属于主观事件的是(3.0分) | A.结婚 B.亲人亡故 C.与同事关系紧张 D.子女出生 |
C.与同事关系紧张 |
2 | 6 | 2007 | 11. 关于应对概念的错误叙述是(3.0分) | A.应对是针对应激性事件采取的任何认知和行为的措施 B.应对的涵义是多维度的 C.应对保护个体免受应激损害 D.应对活动涉及应激全过程 |
C.应对保护个体免受应激损害 |
2 | 6 | 2007 | 12. 负性生活事件是指: (4.0分) | A.个体感觉不愉快的事件 B.对人产生损害的事件 C.与健康成负相关的事件 D. 公认的有害事件 |
A.个体感觉不愉快的事件 |
2 | 6 | 2007 | 13. 脑干一侧损害的瘫痪特点多为: (3.0分) | A.下肢单瘫 B.截瘫 C.交叉性瘫 D.偏瘫 E.上肢单瘫 |
C.交叉性瘫 |
2 | 6 | 2007 | 14. 关于心理应激的概念,下列叙述中正确的是: (3.0分) | A.应激反应包括生理、心理、行为反应 B.应激源涉及生物的、心理的、社会的和文化的 C.认知评价在心理应激中起关键作用 D.应激的本质就是个体对内外环境的需求的不适应 |
D.应激的本质就是个体对内外环境的需求的不适应 |
2 | 6 | 2007 | 15. 以下不属于心理活动应对的是: (3.0分) | A.自责 B.幻想 C.再评价 D.回避 |
D.回避 |
2 | 6 | 2007 | 16. 小脑后下动脉来自(3.0分) | A.锁骨下动脉 B.椎动脉 C.基底动脉 D.大脑后动脉 |
B.椎动脉 |
2 | 6 | 2007 | 17. 下列哪项不是脑血管的常见危险因素: (3.0分) | A.高脂血症 B.2型糖尿病 C.房颤 D.高血压病 E.阑尾炎 |
E.阑尾炎 |
2 | 6 | 2007 | 18. 应激中不利于适应的人格特征不包括: (3.0分) | A. C型人格 B. B型人格 C.A型人格 D.D型人格 |
B. B型人格 |
2 | 6 | 2007 | 19. 一般适应综合征(GAS)分以下三期(3.0分) | A.警戒期、阻抗期、衰竭期 B.警戒期、阻抗期、适应期 C.觉醒期、阻抗期、适应期 D.觉醒期、阻抗期、衰竭期 |
A.警戒期、阻抗期、衰竭期 |
2 | 6 | 2007 | 20. 诊断脑出血最迅速、最可靠的检查是:(3.0分) | A.头颅CT扫描 B.脑脊液检查 C.脑血管造影 D.脑电图检查 E.头颅MRI检查 |
A.头颅CT扫描 |
2 | 6 | 2007 | 21. 患者,女性,57岁。突发右侧肢体偏瘫就诊。影像学检查首选 (3.0分) | A.CT B.MR C.DSA D.平片 |
A.CT |
2 | 6 | 2007 | 22. 颅内动脉瘤破裂出血后最大的危险是 (3.0分) | A.高热 B.并发血管痉挛 C.再出血 D.并发颅神经麻痹 E.合并有动静脉畸形 |
C.再出血 |
2 | 6 | 2007 | 23. 颅内巨大动脉瘤是指大于。(3.0分) | A.25mm B.20mm C.15mm D.30mm |
A.25mm |
2 | 6 | 2007 | 24. 11、T1W和T2W均呈高信号的血肿是哪一期。(3.0分) | A.急性期 B.亚急性晚期 C.亚急性早期 D.慢性晚期 |
B.亚急性晚期 |
2 | 6 | 2007 | 25. 患者,男性,56岁。突发头痛、呕吐就诊。首选 (3.0分) | A.MR B.CT C.DSA D.平片 |
B.CT |
2 | 6 | 2007 | 26. 下面不正确的叙述是:(3.0分) | A.慢性应激通过过多分泌皮质激素而抑制免疫功能 B.心理应激降低免疫功能 C.在心理神经免疫中介机制中,免疫系统与中枢神经进行双向调节 D.心理神经免疫中介机制是肿瘤心理病因学关注的对象 |
B.心理应激降低免疫功能 |
2 | 6 | 2007 | 27. 男性,71岁,突起右上肢麻木乏力,约3分钟后缓解。此后多次发作右上肢麻木乏力,并且有左眼视物模糊,约5-30分钟后缓解。体查无异常。该患者的诊断最可能的是: (3.0分) | A.癫痫发作 B.右侧大脑前动脉的TIA C.左侧大脑前动脉的TIA D.左侧颈内动脉系统的TIA E.右侧颈内动脉系统的TIA |
D.左侧颈内动脉系统的TIA |
2 | 6 | 2007 | 28. 脑血栓形成最常见的病因是:(3.0分) | A.高血压 B.风湿性心脏病 C.动脉粥样硬化 D.颅内动脉瘤 E.高脂血症 |
C.动脉粥样硬化 |
2 | 6 | 2007 | 29. 脑动静脉畸形在CT上一般不会出现。(3.0分) | A.明显占位 B.低密度 C.明显强化 D.高密度 |
A.明显占位 |
2 | 6 | 2007 | 30. 一动脉瘤破裂患者,昏迷,中度瘫痪,请问其Hunt-Hess分级为 (3.0分) | A.四级 B.二级 C.三级 D.一级 E.五级 |
A.四级 |
2 | 6 | 2007 | 31. 以下哪个压力值是属于成人正常颅内压力范围之内的(3.0分) | A.20cmH2O B.100mmH2O C.160mmHgb D.8mmHgb |
D.8mmHgb |
2 | 6 | 2007 | 32. 关于社会支持,以下叙述错误的是: (3.0分) | A.具有减轻应激的作用 B.是指个体来自社会的精神上和物质上的帮助和支援 C.肖水源把社会支持分为主观支持和客观支持 D.社会支持对健康具有保护作用 |
C.肖水源把社会支持分为主观支持和客观支持 |
2 | 6 | 2007 | 33. 哪一种类型的颅内压增高是临床上需要处理的(3.0分) | A.慢性颅内压增高 B.亚急性颅内压增高 C.急性颅内压增高 D.以上所有类型的颅内压增高 |
D.以上所有类型的颅内压增高 |
2 | 6 | 2008 | 1. 一般适应综合征(GAS)分以下三期(2.0分) | A.警戒期、阻抗期、衰竭期 B.觉醒期、阻抗期、适应期 C.警戒期、阻抗期、适应期 D.觉醒期、阻抗期、衰竭期 |
A.警戒期、阻抗期、衰竭期 |
2 | 6 | 2008 | 2. 负性生活事件是指: (2.0分) | A.对人产生损害的事件 B.个体感觉不愉快的事件 C.与健康成负相关的事件 D.公认的有害事件 |
B.个体感觉不愉快的事件 |
2 | 6 | 2008 | 3. 以下属于主观事件的是(2.0分) | A.结婚 B.与同事关系紧张 C.亲人亡故 D.子女出生 |
B.与同事关系紧张 |
2 | 6 | 2008 | 4. 关于应对概念的错误叙述是(2.0分) | A.应对的涵义是多维度的 B.应对是针对应激性事件采取的任何认知和行为的措施 C.应对保护个体免受应激损害 D.应对活动涉及应激全过程 |
C.应对保护个体免受应激损害 |
2 | 6 | 2008 | 5. 下面不正确的叙述是: (2.0分) | A.在心理神经免疫中介机制中,免疫系统与中枢神经进行双向调节 B.心理应激降低免疫功能 C.慢性应激通过过多分泌皮质激素而抑制免疫功能 D.心理神经免疫中介机制是肿瘤心理病因学关注的对象 |
B.心理应激降低免疫功能 |
2 | 6 | 2008 | 6. 与健康和疾病关系最直接的应激心理反应是: (2.0分) | A.认知改变 B.情绪反应 C.个性改变 D.社会适应能力下降 |
B.情绪反应 |
2 | 6 | 2008 | 7. 7-9. 男性,62岁,突然出现剧烈头痛和呕吐8小时。无发热,否认高血压。查体:神清,体温36.9℃,血压120/75mmHg,右侧瞳孔直径3.5mm,对光反射消失,上睑下垂,眼球向上、下及内侧运动不能,颈项强直,克氏征阳性。CT示脑正中裂及右大脑外侧裂、枕大池呈高密度影。该患者受累的脑神经是(2.0分) | A.右侧三叉神经 B.右侧滑车神经 C.右侧动眼神经 D.左侧动眼神经 E.左侧三叉神经 |
C.右侧动眼神经 |
2 | 6 | 2008 | 8. 7-9. 男性,62岁,突然出现剧烈头痛和呕吐8小时。无发热,否认高血压。查体:神清,体温36.9℃,血压120/75mmHg,右侧瞳孔直径3.5mm,对光反射消失,上睑下垂,眼球向上、下及内侧运动不能,颈项强直,克氏征阳性。CT示脑正中裂及右大脑外侧裂、枕大池呈高密度影。该患者最可能的诊断是(2.0分) | A.脑室出血 B.脑干出血 C.内囊出血 D.小脑出血 E.蛛网膜下腔出血 |
E.蛛网膜下腔出血 |
2 | 6 | 2008 | 9. 7-9. 男性,62岁,突然出现剧烈头痛和呕吐8小时。无发热,否认高血压。查体:神清,体温36.9℃,血压120/75mmHg,右侧瞳孔直径3.5mm,对光反射消失,上睑下垂,眼球向上、下及内侧运动不能,颈项强直,克氏征阳性。CT示脑正中裂及右大脑外侧裂、枕大池呈高密度影。为进一步治疗及预防,最重要的检查是(2.0分) | A.脑电图 B.腰穿 C.听觉诱发电位 D.全脑血管造影 E.颅脑X片 |
D.全脑血管造影 |
2 | 6 | 2008 | 10. 颅内压增高病人,发生呕吐常出现在: (2.0分) | A.清晨 B.夜间 C.餐后 D.头痛剧烈时 E.咳嗽时 |
D.头痛剧烈时 |
2 | 6 | 2008 | 11. 枕骨大孔疝与小脑幕切迹疝主要鉴别点在于: (2.0分) | A.头痛剧烈 B.呕吐频繁 C.脉搏加快,血压升高 D.躁动不安 E.呼吸骤停在早期出现 |
E.呼吸骤停在早期出现 |
2 | 6 | 2008 | 12. 形成脑疝的根本条件是下列哪一条:(2.0分) | A.颅内压增高 B.腰穿放液过快过多 C.过量快速输入盐水 D.高位保留灌肠 E.颅腔内各分腔压力失去均衡 |
E.颅腔内各分腔压力失去均衡 |
2 | 6 | 2008 | 13. 脑疝致命的主要原因是: (2.0分) | A.大脑皮层运动区广泛受压 B.颅内血管广泛受压 C.脑干受压 D.小脑受压 E.脑脊液循环严重障碍 |
C.脑干受压 |
2 | 6 | 2008 | 14. 枕骨大孔疝,下列哪一项不是常见症状:(2.0分) | A.意识障碍 B.呼吸突然停止 C.头痛剧烈,呕吐频繁 D.强迫头位 E.早期出现一侧瞳孔散大 |
E.早期出现一侧瞳孔散大 |
2 | 6 | 2008 | 15. 神经外科临床上最常见的病理综合征是 (2.0分) | A.颅内压增高 B.颅内肿瘤 C.颅脑损伤 D.脑积水 E.颅内炎症 |
A.颅内压增高 |
2 | 6 | 2008 | 16. 颅内压调节主要依靠 (2.0分) | A.颅内静脉血被排挤到颅外 B.部分脑脊液被挤入脊髓蛛网膜 C.脑脊液量的增减 D.脑血管收缩 E.脑血管舒张 |
C.脑脊液量的增减 |
2 | 6 | 2008 | 17. 所谓颅内巨大动脉瘤是指直径大于(2.0分) | A.1cm B.1.5cm C.2cm D.2.5cm E.3cm |
D.2.5cm |
2 | 6 | 2008 | 18. 颅内动脉瘤破裂出血后最大的危险是(2.0分) | A.并发颅神经麻痹 B.并发血管痉挛 C.再出血 D.高热 E.合并有动静脉畸形 |
C.再出血 |
2 | 6 | 2008 | 19. 一儿童患者,突发头痛、呕吐伴癫痫,CT示脑实质内出血约10ml,最可能的出血原因是: (2.0分) | A.动脉瘤 B.脑动静脉畸形 C.MOYAMOYA病 D.海绵状血管瘤 E.高血压脑出血 |
B.脑动静脉畸形 |
2 | 6 | 2008 | 20. 一动脉瘤破裂患者,昏迷,中度瘫痪,请问其Hunt-Hess分级为 (2.0分) | A.一级 B.二级 C.三级 D.四级 E.五级 |
D.四级 |
2 | 6 | 2008 | 21. 颈内动脉系统TIA不包括 (2.0分) | A.单侧眼或大脑半球症状 B.一过性黑朦、雾视、视野黑点 C.一侧面部或肢体的无力或麻木 D.失语 E.构音障碍 |
E.构音障碍 |
2 | 6 | 2008 | 22. 椎-基底动脉系统TIA不包括 (2.0分) | A.跌倒发作 B.共济失调 C.异常的眼球运动 D.眼动脉交叉瘫 E.交叉性运动或感觉障碍 |
D.眼动脉交叉瘫 |
2 | 6 | 2008 | 23. 短暂性全面性遗忘症是由于哪部分脑区缺血所致? (2.0分) | A.额叶 B.顶叶 C.颞叶 D.枕叶 E.脑干 |
C.颞叶 |
2 | 6 | 2008 | 24. 跌倒发作是由于哪部分脑区缺血所致? (2.0分) | A.额叶 B.顶叶 C.颞叶 D.枕叶 E.脑干 |
E.脑干 |
2 | 6 | 2008 | 25. 脑出血最常见病因是 (2.0分) | A.动脉瘤 B.血液病 C.脑淀粉样血管病变 D.高血压 E.动静脉畸形 |
D.高血压 |
2 | 6 | 2008 | 26. 脑出血最常见部位 (2.0分) | A.丘脑 B.小脑 C.基底节 D.脑叶 E.脑室 |
C.基底节 |
2 | 6 | 2008 | 27. 针尖样瞳孔常见于(2.0分) | A.延髓出血 B.脑桥出血 C.小脑出血 D.基底节出血 E.脑室出血 |
B.脑桥出血 |
2 | 6 | 2008 | 28. 脑出血最为敏感的影像学检查为 (2.0分) | A.头颅CT B.头颅MRI C.DSA D.CTA E.TCD |
A.头颅CT |
2 | 6 | 2008 | 29. 脑梗死CT早期征象包括(2.0分) | A.岛带征消失 B.灰白质界线不清 C.脑沟变浅 D.动脉高密度征 E.以上都是 |
E.以上都是 |
2 | 6 | 2008 | 30. 溶栓治疗时,血压应控制在(2.0分) | A.收缩压<200mmHg,舒张压<100mmHg B.收缩压<200mmHg,舒张压<110mmHg C.收缩压<180mmHg,舒张压<105mmHg D.收缩压<180mmHg,舒张压<100mmHg E.收缩压<180mmHg,舒张压<110mmHg |
D.收缩压<180mmHg,舒张压<100mmHg |
2 | 6 | 2008 | 31. rtPA溶栓时间窗为(2.0分) | A.3小时内 B.3.5小时内 C.4小时内 D.4.5小时内 E.6小时内 |
D.4.5小时内 |
2 | 6 | 2008 | 32. 脑梗死并发症不包括 (2.0分) | A.脑水肿与颅内压增高 B.出血转化 C.癫痫 D.吞咽困难 E.血管痉挛 |
E.血管痉挛 |
2 | 6 | 2008 | 33. 脑梗死二级预防三大基石为 (2.0分) | A.抗血小板治疗,稳定斑块治疗,降压治疗 B.抗血小板治疗,降糖,降压治疗 C.抗血小板治疗,稳定斑块治疗,降糖治疗 D.抗凝治疗,稳定斑块治疗,降压治疗 E.抗血小板治疗,稳定斑块治疗,降糖治疗 |
A.抗血小板治疗,稳定斑块治疗,降压治疗 |
2 | 6 | 2008 | 34. 蛛网膜下腔出血的并发症包括(2.0分) | A.再出血 B.血管痉挛 C.脑积水 D.癫痫 E.以上都对 |
E.以上都对 |
2 | 6 | 2008 | 35. 原发性蛛网膜下腔出血最常见病因是 (2.0分) | A.动脉瘤 B.高血压 C.动静脉畸形 D.血管炎 E.白血病 |
A.动脉瘤 |
2 | 6 | 2008 | 36. 蛛网膜下腔出血患者,出现中至重度头痛、脑膜刺激征、脑神经麻痹,则Hunt and Hess 分级:(2.0分) | A.1级 B.2级 C.3级 D.4级 E.5级 |
B.2级 |
2 | 6 | 2008 | 37. 颅内动脉瘤初次出血后再出血率最高为(2.0分) | A.1小时内 B.12小时内 C.24小时内 D.72小时内 E.1周内 |
C.24小时内 |
2 | 6 | 2008 | 38. 一蛛网膜下腔出血患者,出现精神障碍、步态异常和尿失禁,则可能发生了以下哪种并发症? (2.0分) | A.再出血 B.脑血管痉挛 C.急性非交通性脑积水 D.正常颅压脑积水 E.脑梗死 |
D.正常颅压脑积水 |
2 | 6 | 2008 | 39. 有关视力、视角的论述,正确的是(2.0分) | A.视力大小与视角大小成正比 B.视角一分角能看清物体时,视力为1.0 C.视锥细胞直径越大,视力越好 D.双眼视物时可以使视力提高 E.同一距离物体越大形成的视角就越小 |
B.视角一分角能看清物体时,视力为1.0 |
2 | 6 | 2008 | 40. 看近物时的视觉调节过程包括(2.0分) | A.晶状体变凸,眼轴会聚,瞳孔散大 B.晶状体变凸,眼轴会聚,瞳孔缩小 C.晶状体扁平,眼轴会聚,瞳孔缩小 D.晶状体扁平,眼轴会聚,瞳孔散大 E.晶状体曲率半径减小,眼轴会聚,瞳孔缩小 |
B.晶状体变凸,眼轴会聚,瞳孔缩小 |
2 | 6 | 2008 | 41. 人眼的调节功能主要是由以下哪种变化来实现的? (2.0分) | A.角膜的曲度变化 B.角膜和晶状体的曲度变化 C.晶状体的曲度变化 D.玻璃体的曲度变化 E.晶状体和玻璃体的曲度变化 |
C.晶状体的曲度变化 |
2 | 6 | 2008 | 42. 人眼看近物时(2.0分) | A.晶状体变扁平,折光力增加 B.晶状体变凸,折光力下降 C.晶状体扁平,折光力下降 D.晶状体变凸,折光力增加 E.晶状体曲率减小,折光力增大 |
D.晶状体变凸,折光力增加 |
2 | 6 | 2008 | 43. 瞳孔对光反射中枢在 (2.0分) | A.大脑皮质 B.下丘脑 C.中脑 D.脑桥 E.延脑 |
C.中脑 |
2 | 6 | 2008 | 44. 关于瞳孔反射的说明,正确的是(2.0分) | A.视近物时瞳孔扩大 B.瞳孔近反射与对光反射的通路相同 C.动眼神经损伤使瞳孔缩小 D.交感神经兴奋使瞳孔扩大 E.副交感神经兴奋使瞳孔扩大 |
D.交感神经兴奋使瞳孔扩大 |
2 | 6 | 2008 | 45. 对暗光敏感的区域是(2.0分) | A.视网膜的中央凹处 B.视盘处 C.视网膜周缘部分 D.整个视网膜 E.视乳头 |
C.视网膜周缘部分 |
2 | 6 | 2008 | 46. 关于视紫红质的特性,错的是(2.0分) | A.只存在于视杆细胞中 B.是一种结合蛋白质 C.在暗处只有合成 D.对光的敏感度与其含量呈正比 E.亮处分解,暗处合成是一个可逆反应 |
C.在暗处只有合成 |
2 | 6 | 2008 | 47. 能产生动作电位的细胞是(2.0分) | A.视锥细胞 B.视杆细胞 C.双极细胞 D.神经节细胞 E.水平细胞 |
D.神经节细胞 |
2 | 6 | 2008 | 48. 维生素A 严重缺乏,可影响人(2.0分) | A.在明处的视力 B.色觉 C.在暗处的视力 D.立体视觉 E.视野范围 |
C.在暗处的视力 |
2 | 6 | 2008 | 49. 按色觉三原色学说,三种视锥细胞分别敏感的颜色是(2.0分) | A.红、蓝、紫 B.红、黄、黑 C.绿、蓝、白 D.红、绿、蓝 E.红、白、青 |
D.红、绿、蓝 |
2 | 6 | 2008 | 50. 下列关于视觉通路的描述,正确的是(2.0分) | A.右眼颞侧视网膜的传入纤维投射至左枕叶皮层 B.左眼鼻侧视网膜的传入纤维投射至右枕叶皮层 C.视网膜上半部的传入纤维投射到距状裂下缘 D.视网膜下半部的传入纤维投射到距状裂上缘 E.视网膜中央黄斑区的传入纤维投射到距状裂前部 |
B.左眼鼻侧视网膜的传入纤维投射至右枕叶皮层 |
2 | 6 | 2009 | 1. 男性,62岁,突然出现剧烈头痛和呕吐8小时。无发热,否认高血压。查体:神清,体温36.9℃,血压120/75mmHg,右侧瞳孔直径3.5mm,对光反射消失,上睑下垂,眼球向上、下及内侧运动不能,颈项强直,克氏征阳性。CT示脑正中裂及右大脑外侧裂、枕大池呈高密度影。该患者受累的脑神经是 (2.0分) | A.右侧三叉神经 B.右侧滑车神经 C.右侧动眼神经 D.左侧动眼神经 E.左侧三叉神经 |
C.右侧动眼神经 |
2 | 6 | 2009 | 2. 枕骨大孔疝与小脑幕切迹疝主要鉴别点在于: (2.0分) | A.头痛剧烈 B.呕吐频繁 C.脉搏加快,血压升高 D.躁动不安 E.呼吸骤停在早期出现 |
E.呼吸骤停在早期出现 |
2 | 6 | 2009 | 3. 脑疝致命的主要原因是: (2.0分) | A.大脑皮层运动区广泛受压 B.颅内血管广泛受压 C.脑干受压 D.小脑受压 E.脑脊液循环严重障碍 |
C.脑干受压 |
2 | 6 | 2009 | 4. 神经外科临床上最常见的病理综合征是 (2.0分) | A.颅内压增高 B.颅内肿瘤 C.颅脑损伤 D.脑积水 E.颅内炎症 |
A.颅内压增高 |
2 | 6 | 2009 | 5. 最易发生小脑幕切迹疝的疾病是 (2.0分) | A.鞍区占位性病变 B.颞叶占位性病变 C.小脑幕下占位性病变 D.梗阻性脑积水 E.弥漫性脑水肿 |
B.颞叶占位性病变 |
2 | 6 | 2009 | 6. Which of the following is non intra-axial haemorrhage ? (2.0分) | A.lobar haemorrhage B.basal ganglia haemorrhage C.pontine haemorrhage D.subarachnoid haemorrhage (SAH) |
D.subarachnoid haemorrhage (SAH) |
2 | 6 | 2009 | 7. To demonstrate small amount of subarachnoid hemorrhage,which modality is the best choice? (2.0分) | A.MRI B.MRA C.CT D.DSA |
C.CT |
2 | 6 | 2009 | 8. Subacute intracerebral hematoma typically presents signal intensity as:(2.0分) | A.Iso T1 intensity and Iso T2 intensity B.Hyper T1 intensity and Hyper T2 intensity C.Hyper T1 intensity and hypo T2 intensity D.Hypo T1 intensity and hyper T2 intensity |
B.Hyper T1 intensity and Hyper T2 intensity |
2 | 6 | 2009 | 9. Which of the following has the highest sensitivity and specificity in the diagnosis of acute ischaemic infarction in the first hour after onset. (2.0分) | A.CT without contrast B.MRI T2WI C.MRI T1WI D.DWI ( Diffiusion weighted imaging ) |
D.DWI ( Diffiusion weighted imaging ) |
2 | 6 | 2009 | 10. In the diagnosis of Cerebral arteriovenous malformation (AVM), which of the following is true.(2.0分) | A.MRA delineate the feeding vessels and the pattern of drainage better then DSA. B.CT is better then MRI in the evalution of an AVM C.CT can easily show three components: nidus of vessels , feeding arteris and drainage veins. D.MRI can easily show three components: nidus of vessels , feeding arteris and drainage veins. |
D.MRI can easily show three components: nidus of vessels , feeding arteris and drainage veins. |
2 | 6 | 2009 | 11. 颈内动脉系统TIA发作的症状,最可能的是(2.0分) | A.失语 B.跌倒发作 C.共济失调 D.异常的眼球运动 E.眼动脉交叉瘫 |
A.失语 |
2 | 6 | 2009 | 12. 椎-基底动脉系统TIA发作的症状,最可能的是 (2.0分) | A.单侧眼或大脑半球症状 B.一过性黑朦、雾视、视野黑点 C.一侧面部或肢体的无力或麻木 D.失语 E.交叉性运动或感觉障碍 |
E.交叉性运动或感觉障碍 |
2 | 6 | 2009 | 13. 以下不符合血流动力学型TIA的描述是 (2.0分) | A.在动脉严重狭窄的基础上发生 B.发作频率密集 C.持续时间短暂 D.临床症状多变 E.临床症状刻板 |
D.临床症状多变 |
2 | 6 | 2009 | 14. 脑叶出血最常见的部位(2.0分) | A.顶叶 B.颞叶 C.枕叶 D.额叶 E.边缘叶 |
A.顶叶 |
2 | 6 | 2009 | 15. 脑出血治疗原则包括以下各项,除了 (2.0分) | A.减轻脑水肿 B.调整血压,防止继续出血 C.促进神经功能恢复 D.降低血脂,稳定斑块 E.防治并发症 |
D.降低血脂,稳定斑块 |
2 | 6 | 2009 | 16. 脑出血需要外科治疗的适应症,除了 (2.0分) | A.基底节区壳核出血≥30ml B.小脑出血量≥10ml,直径≥3cm,脑积水 C.脑干出血≥5cm D.脑叶血肿较大危及生命,血管畸形 E.丘脑出血≥15ml |
C.脑干出血≥5cm |
2 | 6 | 2009 | 17. 延髓背外侧综合征不包括以下那个症状 (2.0分) | A.眩晕,眼震,病灶侧共济失调 B.吞咽困难、构音障碍、同侧软腭低垂及咽反射消失 C.病灶侧面瘫 D.Horner综合征 E.交叉性偏身感觉障碍 |
C.病灶侧面瘫 |
2 | 6 | 2009 | 18. rtPA和尿激酶溶栓时间窗分别为 (2.0分) | A.3小时内和4.5小时内 B.3.5小时内和4.5小时内 C.4小时内和6小时内 D.4.5小时内和6小时内 E.5小时内和6小时内 |
D.4.5小时内和6小时内 |
2 | 6 | 2009 | 19. 溶栓的理论基础和目的是由于(2.0分) | A.在脑缺血超早期治疗的时间窗内存在缺血半暗带,而通过溶栓缺血半暗带有可能被拯救 B.目的是为了拯救梗塞中心的坏死区 C.梗塞区的全部脑组织通过溶栓都有希望被拯救 D.不管是否存在缺血半暗带溶栓都是有效的 E.脑缺血超早期治疗的时间窗为3~6天 |
A.在脑缺血超早期治疗的时间窗内存在缺血半暗带,而通过溶栓缺血半暗带有可能被拯救 |
2 | 6 | 2009 | 20. 男,25岁,突发头部剧烈疼痛,随即出现意识丧失,伴抽搐,醒后仍头剧烈疼痛并呕吐。体检:神志清楚,瞳孔右3mm,左6mm,颈强直、克氏征、布氏征(+),左侧眼底玻璃体下片状出血。体温37 οC,血压145/85 mmHg。根据以上资料,最可能的诊断是 (2.0分) | A.脑出血 B.脑膜脑炎 C.原发性癫痫 D.蛛网膜下腔出血 E.血管性头痛 |
D.蛛网膜下腔出血 |
2 | 6 | 2009 | 21. 动脉瘤破裂的危险因素 (2.0分) | A.吸烟,饮酒 B.高血压,动脉粥样硬化 C.女性,口服避孕药 D.服用刺激性物质 E.以上都是 |
E.以上都是 |
2 | 6 | 2009 | 22. 蛛网膜下腔出血患者,出现嗜睡、脑膜刺激征(+)、脑神经麻痹,则Hunt and Hess 分级:(2.0分) | A.1级 B.2级 C.3级 D.4级 E.5级 |
C.3级 |
2 | 6 | 2009 | 23. 高血压脑出血最易发生的部位是(2.0分) | A.大脑白质 B.小脑 C.丘脑区域 D.桥脑 E.内囊及基底节区域 |
E.内囊及基底节区域 |
2 | 6 | 2009 | 24. 动脉粥样硬化合并血栓形成的最主要的机制是 (2.0分) | A.血流涡流形成 B.局部血流缓慢 C.内膜受损、胶原纤维暴露 D.局部血液凝固性增高 E.以上都不是 |
C.内膜受损、胶原纤维暴露 |
2 | 6 | 2009 | 25. 脑出血的最常见原因是(2.0分) | A.高血压病和脑动脉硬化 B.血液病 C.脑动脉炎 D.血管瘤 E.脑血管畸形 |
A.高血压病和脑动脉硬化 |
2 | 6 | 2009 | 26. 关于一般适应综合征(GAS),以下错误的是 (2.0分) | A.分为觉醒期、阻抗期、适应期三个阶段 B.由加拿大生理学家塞里(H.Selye)提出 C.是非特异性的 D.是机体面临有害刺激时表现出的生理生化反应 |
A.分为觉醒期、阻抗期、适应期三个阶段 |
2 | 6 | 2009 | 27. 负性生活事件是指: (2.0分) | A.需要付出机体能量应付的事件 B.公认的有害事件 C.与健康成负相关的事件 D.当事人感觉不愉快的事件 |
D.当事人感觉不愉快的事件 |
2 | 6 | 2009 | 28. 关于应激源与健康的关系,以下叙述不正确的是(2.0分) | A.应激源的性质如何与健康关系密切 B.应激源的数量多少与健康没有关系 C.不可控制的的应激源对健康影响更大 D.涉及生活核心部分的应激源对健康影响更大 |
B.应激源的数量多少与健康没有关系 |
2 | 6 | 2009 | 29. 下面不正确的叙述是: (2.0分) | A.在心理神经免疫中介机制中,免疫系统与中枢神经进行双向调节 B.心理应激降低免疫功能 C.慢性应激通过过多分泌皮质激素而抑制免疫功能 D.心理神经免疫中介机制是肿瘤心理病因学关注的对象 |
B.心理应激降低免疫功能 |
2 | 6 | 2009 | 30. 关于心理应激的概念,下列叙述中不正确的是: (2.0分) | A.应激源涉及生物的、心理的、社会的和文化的 B.应激反应包括生理、心理、行为反应 C.应对方式在心理应激中起关键作用 D.应激的本质就是个体对内外环境的需求的适应 |
C.应对方式在心理应激中起关键作用 |
2 | 6 | 2009 | 31. 应激中不利于适应的人格特征不包括: (2.0分) | A.A型人格 B.B型人格 C.C型人格 D.D型人格 |
B.B型人格 |
2 | 6 | 2009 | 32. 头皮帽状腱膜下血肿不能吸收者应:(2.0分) | A.切开止血 B.在严密的无菌条件下进行穿刺抽出积血加压包扎 C.穿刺抽血注入抗生素 D.等待血肿自行吸收 E.应用止血药物 |
B.在严密的无菌条件下进行穿刺抽出积血加压包扎 |
2 | 6 | 2009 | 33. 治疗颅底骨折合并脑脊液漏者,以下哪个方案是错误的? (2.0分) | A.绝对卧床休息 B.镇静,止痛,抗炎 C.防治肺部感染 D.将漏脑脊液的鼻孔或外耳道填塞 E.适当控制入水量 |
D.将漏脑脊液的鼻孔或外耳道填塞 |
2 | 6 | 2009 | 34. 颅脑外伤患者,颅骨骨折线与脑膜中动脉沟交叉时,最易发生哪类血肿:(2.0分) | A.头皮下血肿 B.急性硬膜外血肿 C.慢性硬膜下血肿 D.脑内血肿 E.多发性血肿 |
B.急性硬膜外血肿 |
2 | 6 | 2009 | 35. 颅脑损伤后,诊断小脑幕裂孔疝的最可靠的早期临床表现是: (2.0分) | A.头痛,呕吐,进行性意识障碍 B.定位体征 C.癫痫发生 D.血压,脉搏,呼吸改变 E.瞳孔变化 |
E.瞳孔变化 |
2 | 6 | 2009 | 36. 所谓颅内巨大动脉瘤是指直径大于(2.0分) | A.1cm B.1.5cm C.2cm D.2.5cm E.3cm |
D.2.5cm |
2 | 6 | 2009 | 37. 一动脉瘤破裂患者,昏迷,中度瘫痪,请问其Hunt-Hess分级为 (2.0分) | A.一级 B.二级 C.三级 D.四级 E.五级 |
D.四级 |
2 | 6 | 2009 | 38. 一儿童患者,突发头痛、呕吐伴癫痫,CT示脑实质内出血约10ml,最可能的出血原因是(2.0分) | A.动脉瘤 B.脑动静脉畸形 C.MOYAMOYA病 D.海绵状血管瘤 E.高血压脑出血 |
B.脑动静脉畸形 |
2 | 6 | 2009 | 39. 有关视力、视角的论述,正确的是(2.0分) | A.视力大小与视角大小成正比 B.视角一分角能看清物体时,视力为1.0 C.视锥细胞直径越大,视力越好 D.双眼视物时可以使视力提高 E.同一距离物体越大形成的视角就越小 |
B.视角一分角能看清物体时,视力为1.0 |
2 | 6 | 2009 | 40. 下列不属于折光系统的是 (2.0分) | A.角膜 B.房水 C.晶状体 D.玻璃体 E.视网膜 |
E.视网膜 |
2 | 6 | 2009 | 41. 房水的主要功能是 (2.0分) | A.折光成像 B.对玻璃体有保护作用 C.对角膜和晶状体有营养作用 D.对视网膜有保护作用 E.促进睫状肌收缩 |
C.对角膜和晶状体有营养作用 |
2 | 6 | 2009 | 42. 看近物时的视觉调节过程包括 (2.0分) | A.晶状体变凸,眼轴会聚,瞳孔散大 B.晶状体变凸,眼轴会聚,瞳孔缩小 C.晶状体扁平,眼轴会聚,瞳孔缩小 D.晶状体扁平,眼轴会聚,瞳孔散大 E.晶状体曲率半径减小,眼轴会聚,瞳孔缩小 |
B.晶状体变凸,眼轴会聚,瞳孔缩小 |
2 | 6 | 2009 | 43. 瞳孔在弱光下散大,而在强光下缩小,称为 (2.0分) | A.互感性对光反射 B.辐辏反射 C.瞳孔对光反射 D.瞳孔调节反射 E.瞳孔近反射 |
C.瞳孔对光反射 |
2 | 6 | 2009 | 44. 有关晶状体的叙述,错误的是(2.0分) | A.随着年龄的增大,弹性逐渐减退 B.调节能力与弹性无关 C.近点距离的远近受其调节能力所决定 D.所视物体近移时,曲率增大 E.近点越近,调节能力越强 |
B.调节能力与弹性无关 |
2 | 6 | 2009 | 45. 人眼的调节功能主要是由以下哪种变化来实现的? (2.0分) | A.角膜的曲度变化 B.角膜和晶状体的曲度变化 C.晶状体的曲度变化 D.玻璃体的曲度变化 E.晶状体和玻璃体的曲度变化 |
C.晶状体的曲度变化 |
2 | 6 | 2009 | 46. 人眼看近物时(2.0分) | A.晶状体变扁平,折光力增加 B.晶状体变凸,折光力下降 C.晶状体扁平,折光力下降 D.晶状体变凸,折光力增加 E.晶状体曲率减小,折光力增大 |
D.晶状体变凸,折光力增加 |
2 | 6 | 2009 | 47. 眼尽最大能力调节时所能看清物体的最近距离,称为 (2.0分) | A.节点 B.前主焦点 C.远点 D.近点 E.后主焦点 |
D.近点 |
2 | 6 | 2009 | 48. 某人看远物需要调节,看近物也不清楚,提示他可能是(2.0分) | A.远视 B.近视 C.散光 D.老视眼 E.正视眼 |
A.远视 |
2 | 6 | 2009 | 49. 下列哪种非正视眼矫正用凹透镜?(2.0分) | A.近视眼 B.远视眼 C.散光眼 D.老视眼 E.老花眼 |
A.近视眼 |
2 | 6 | 2009 | 50. 瞳孔对光反射中枢在 (2.0分) | A.大脑皮质 B.下丘脑 C.中脑 D.脑桥 E.延脑 |
C.中脑 |
2 | 6 | 2010 | 1. 颈内动脉系统TIA不包括 (2.0分) | A.单侧眼或大脑半球症状 B.一过性黑朦、雾视、视野黑点 C.一侧面部或肢体的无力或麻木 D.失语 E.构音障碍 |
E.构音障碍 |
2 | 6 | 2010 | 2. 椎-基底动脉系统TIA不包括(2.0分) | A.跌倒发作 B.共济失调 C.异常的眼球运动 D.失读 E.交叉性运动或感觉障碍 |
D.失读 |
2 | 6 | 2010 | 3. 哪部分脑区缺血可致双眼视力障碍? (2.0分) | A.视交叉 B.顶叶 C.颞叶 D.枕叶 E.脑干 |
D.枕叶 |
2 | 6 | 2010 | 4. 跌倒发作是由于哪部分脑区缺血所致? (2.0分) | A.额叶 B.顶叶 C.颞叶 D.枕叶 E.脑干 |
E.脑干 |
2 | 6 | 2010 | 5. TIA治疗不包括(2.0分) | A.控制危险因素 B.抗血小板聚集 C.溶栓 D.抗凝 E.稳定斑块 |
C.溶栓 |
2 | 6 | 2010 | 6. 脑出血最常见病因是 (2.0分) | A.动脉瘤 B.血液病 C.脑淀粉样血管病变 D.高血压 E.动静脉畸形 |
D.高血压 |
2 | 6 | 2010 | 7. 高血压脑出血好发于 (2.0分) | A.脑干 B.小脑 C.壳核附近 D.脑叶 E.脑室 |
C.壳核附近 |
2 | 6 | 2010 | 8. 小脑出血患者,剧烈咳嗽后出现意识障碍,需考虑 (2.0分) | A.再出血 B.TIA C.脑疝形成 D.癫痫 E.癔症 |
C.脑疝形成 |
2 | 6 | 2010 | 9. 脑出血最为敏感的影像学检查为 (2.0分) | A.头颅CT B.头颅MRI C.DSA D.CTA E.TCD |
A.头颅CT |
2 | 6 | 2010 | 10. 脑出血患者,血压高于多少时宜开始平稳降压(2.0分) | A.收缩压≥220 mmHg ,舒张压≥110mmHg B.收缩压≥200 mmHg ,舒张压≥110mmHg C.收缩压≥220 mmHg ,舒张压≥105mmHg D.收缩压≥180 mmHg ,舒张压≥100mmHg E.收缩压≥200 mmHg ,舒张压≥100mmHg |
B.收缩压≥200 mmHg ,舒张压≥110mmHg |
2 | 6 | 2010 | 11. 脑梗死后多长时间做头颅CT检查阳性率较高(3.0分) | A.5分钟 B.30分钟 C.1小时 D.12小时 E.24小时 |
E.24小时 |
2 | 6 | 2010 | 12. 脑梗死的体征,一般不包括 (2.0分) | A.视野缺损 B.偏瘫 C.偏身感觉障碍 D.脑膜刺激征 E.病理征阳性 |
D.脑膜刺激征 |
2 | 6 | 2010 | 13. rtPA溶栓时间窗为(2.0分) | A.3小时内 B.3.5小时内 C.4小时内 D.4.5小时内 E.6小时内 |
D.4.5小时内 |
2 | 6 | 2010 | 14. 脑梗死并发症包括 (2.0分) | A.脑水肿与颅内压增高 B.出血转化 C.癫痫 D.吞咽困难 E.以上都对 |
E.以上都对 |
2 | 6 | 2010 | 15. 脑梗死二级预防三大基石为 (2.0分) | A.抗血小板治疗,稳定斑块治疗,降压治疗 B.抗血小板治疗,降糖,降压治疗 C.抗血小板治疗,稳定斑块治疗,降糖治疗 D.抗凝治疗,稳定斑块治疗,降压治疗 E.抗血小板治疗,稳定斑块治疗,降糖治疗 |
A.抗血小板治疗,稳定斑块治疗,降压治疗 |
2 | 6 | 2010 | 16. 蛛网膜下腔出血出现单侧眼睑下垂,需考虑(2.0分) | A.前交通动脉瘤 B.基底动脉瘤 C.椎动脉瘤 D.颈内动脉瘤 E.后交通动脉瘤 |
E.后交通动脉瘤 |
2 | 6 | 2010 | 17. 蛛网膜下腔出血最为可靠的诊断依据 (2.0分) | A.一侧瞳孔散大 B.剧烈头痛、呕吐、肢体抽搐 C.腰穿为均匀一致血性脑脊液 D.脑膜刺激征阳性 E.头颅CT示脑膜高密度 |
C.腰穿为均匀一致血性脑脊液 |
2 | 6 | 2010 | 18. 蛛网膜下腔出血患者,出现深昏迷,去大脑强直,则Hunt and Hess 分级: (2.0分) | A.1级 B.2级 C.3级 D.4级 E.5级 |
E.5级 |
2 | 6 | 2010 | 19. 蛛网膜下腔出血后发生脑血管痉挛的高峰期为 (2.0分) | A.24小时内 B.1周内 C.5-14天 D.2周-1月 E.1月后 |
C.5-14天 |
2 | 6 | 2010 | 20. 一蛛网膜下腔出血患者,出现精神障碍、步态异常和尿失禁,则可能发生了以下哪种并发症? (2.0分) | A.再出血 B.脑血管痉挛 C.急性非交通性脑积水 D.正常颅压脑积水 E.脑梗死 |
D.正常颅压脑积水 |
2 | 6 | 2010 | 21. 颅内压增高病人,发生呕吐常出现在: (2.0分) | A.清晨 B.头痛剧烈时 C.餐后 D.夜间 E.咳嗽时 |
B.头痛剧烈时 |
2 | 6 | 2010 | 22. 诊断颅内压高的可靠依据是:(2.0分) | A.视乳头水肿 B.剧烈头痛 C.频繁呕吐 D.癫痫发作 E.双外展神经麻痹 |
A.视乳头水肿 |
2 | 6 | 2010 | 23. 脑疝致命的主要原因是:(2.0分) | A.大脑皮层运动区广泛受压 B.颅内血管广泛受压 C.脑脊液循环严重障碍 D.小脑受压 E.脑干受压 |
E.脑干受压 |
2 | 6 | 2010 | 24. 颅内压增高"三主症"包括 (2.0分) | A.偏瘫,偏盲,偏身感觉障碍 B.头痛,呕吐,偏瘫 C.头痛,抽搐,意识障碍 D.头痛,呕吐,视乳头水肿 E.头痛,呕吐,血压增高 |
D.头痛,呕吐,视乳头水肿 |
2 | 6 | 2010 | 25. Which of the following is non intra-axial haemorrhage ?(2.0分) | A.lobar haemorrhage B.basal ganglia haemorrhage C.pontine haemorrhage D.subarachnoid haemorrhage (SAH) |
D.subarachnoid haemorrhage (SAH) |
2 | 6 | 2010 | 26. To demonstrate small amount of subarachnoid hemorrhage, which modality is the best choice? (2.0分) | A.MRI B.MRA C.CT D.DSA |
C.CT |
2 | 6 | 2010 | 27. Which of the following has the highest sensitivity and specificity in the diagnosis of acute ischaemic infarction in the first hour after onset. (2.0分) | A.CT without contrast B.MRI T2WI C.MRI T1WI D.DWI ( Diffiusion weighted imaging ) |
D.DWI ( Diffiusion weighted imaging ) |
2 | 6 | 2010 | 28. 关于一般适应综合征(GAS),以下正确的是(2.0分) | A.分为觉醒期、阻抗期、适应期三个阶段 B.由加拿大生理学家塞里(H.Selye)提出 C.是特异性的 D.是机体面临有害刺激时表现出的心理反应 |
B.由加拿大生理学家塞里(H.Selye)提出 |
2 | 6 | 2010 | 29. 关于正性生活事件,以下叙述错误的是: (2.0分) | A.是对健康具有促进作用的事件 B.是当事人认为愉快的事件 C.是当事人认为积极的事件 D.是需要付出机体能量应对的事件 |
A.是对健康具有促进作用的事件 |
2 | 6 | 2010 | 30. 关于负性生活事件,以下叙述正确的是: (2.0分) | A.对人产生损害的事件 B.个体感觉不愉快的事件 C.与健康成负相关的事件 D.公认的有害事件 |
B.个体感觉不愉快的事件 |
2 | 6 | 2010 | 31. 关于应激的心理反应,以下叙述不正确的是(2.0分) | A.应激的心理反应涉及"知、情、意、人格"各方面 B.应激的认知反应常常使个体的认知功能受损 C.应激的情绪反应包括焦虑、恐惧、抑郁、愤怒等 D.假如应激过于强烈或应激时间过长,有可能导致人格发生改变 |
B.应激的认知反应常常使个体的认知功能受损 |
2 | 6 | 2010 | 32. 以下属于主观事件的是(2.0分) | A.结婚 B.与同事关系紧张 C.亲人亡故 D.地震 |
B.与同事关系紧张 |
2 | 6 | 2010 | 33. 关于应对概念的错误叙述是 (2.0分) | A.应对的涵义是多维度的 B.应对是针对应激性事件采取的任何认知和行为的措施 C.应对保护个体免受应激损害 D.应对活动涉及应激全过程 |
C.应对保护个体免受应激损害 |
2 | 6 | 2010 | 34. 下面不正确的叙述是: (2.0分) | A.在心理神经免疫中介机制中,免疫系统与中枢神经进行双向调节 B.心理应激降低免疫功能 C.慢性应激通过过多分泌皮质激素而抑制免疫功能 D.心理神经免疫中介机制是肿瘤心理病因学关注的对象 |
B.心理应激降低免疫功能 |
2 | 6 | 2010 | 35. 与健康和疾病关系最直接的应激心理反应是: (2.0分) | A.认知改变 B.情绪反应 C.个性改变 D.社会适应能力下降 |
B.情绪反应 |
2 | 6 | 2010 | 36. 诊断颅底骨折的确切证据是:(2.0分) | A.头颅X光照片多能显示颅底骨折 B.伤后进行性昏迷伴呕吐 C.脑脊液耳漏,鼻漏,眼睑有淤血斑 D.在受打击的局部立即出现枕下与眼睑或结合膜下淤血 E.头部超声波检查,中线偏移超过0.5厘米 |
C.脑脊液耳漏,鼻漏,眼睑有淤血斑 |
2 | 6 | 2010 | 37. 重症闭合性颅脑外伤病人的急诊,首先应该做到: (2.0分) | A.检查神志,瞳孔 B.测量呼吸,血压,脉搏 C.保持呼吸道通畅 D.躁动者可用吗啡类药物 E.及时输液,输血 |
A.检查神志,瞳孔 |
2 | 6 | 2010 | 38. 颅脑损伤后,诊断小脑幕裂孔疝的最可靠的早期临床表现是:(2.0分) | A.头痛,呕吐,进行性意识障碍 B.定位体征 C.癫痫发生 D.血压,脉搏,呼吸改变 E.瞳孔变化 |
E.瞳孔变化 |
2 | 6 | 2010 | 39. 脑挫裂伤最多见的发生部位是:(2.0分) | A.颞叶后部及其外侧面 B.额极及额叶眶面和颞极 C.头部直接打击处的同侧脑部 D.顶叶前部 E.枕极及其底部 |
B.额极及额叶眶面和颞极 |
2 | 6 | 2010 | 40. 一动脉瘤破裂患者,昏迷,中度瘫痪,请问其Hunt-Hess分级为 (2.0分) | A.一级 B.二级 C.三级 D.四级 E.五级 |
D.四级 |
2 | 6 | 2010 | 41. 颅内动脉瘤破裂出血后最大的危险是(2.0分) | A.并发颅神经麻痹 B.并发血管痉挛 C.再出血 D.高热 E.合并有动静脉畸形 |
C.再出血 |
2 | 6 | 2010 | 42. 神经、肌肉、腺体受阈刺激产生反应的共同表现是 (2.0分) | A.收缩 B.分泌 C.局部电位 D.阈电位 E.动作电位 |
E.动作电位 |
2 | 6 | 2010 | 43. 衡量组织兴奋性高低的指标是 (3.0分) | A.静息电位水平 B.阈电位 C.阈强度 D.动作电位幅度 E.兴奋扩布速度 |
C.阈强度 |
2 | 6 | 2010 | 44. 直接导致神经末梢释放递质的因素是 (2.0分) | A.末梢处的 Na+内流 B.末梢处的 K+外流 C.末梢处的 Cl‐内流 D.末梢处的 Na+‐Ca2+交换 E.末梢处的 Ca2+内流 |
E.末梢处的 Ca2+内流 |
2 | 6 | 2010 | 45. 临床上普鲁卡因局部麻醉药的应用是由于阻断了局部神经冲动的传导,其神经细胞发生的变化是(2.0分) | A.细胞变性 B.结构完整性破坏 C.功能完整性破坏 D.细胞膜电压门控钾通道破坏 E.细胞膜化学门控钠通道破坏 |
C.功能完整性破坏 |
2 | 6 | 2010 | 46. 动作电位沿运动神经纤维传导抵达神经‐肌接头部位时,轴突末梢释放Ach,使终板膜产生终板电位,然后在什么部位引发动作电位 (2.0分) | A.肌细胞膜 B.接头后膜 C.终板膜 D.横管膜 E.三联管膜 |
A.肌细胞膜 |
2 | 6 | 2010 | 47. 高血压脑出血时,哪支血管破裂最常见?(2.0分) | A.基底动脉 B.大脑后动脉 C.大脑中动脉 D.大脑前动脉 E.豆纹动脉 |
E.豆纹动脉 |
2 | 6 | 2010 | 48. 蛛网膜下腔出血的最常见原因是(2.0分) | A.高血压病 B.血液病 C.脑动脉硬化 D.血管瘤 E.脑血管畸形 |
D.血管瘤 |
2 | 6 | 2010 | 49. 下列哪根血管闭塞最容易导致偏瘫 (2.0分) | A.大脑中动脉 B.大脑后动脉 C.大脑前动脉 D.小脑后下动脉 E.小脑前下动脉 |
A.大脑中动脉 |
2 | 6 | 2011 | 1. 颈内动脉系统TIA不包括(2.0分) | A.单侧眼或大脑半球症状 B.一过性黑朦、雾视、视野黑点 C.一侧面部或肢体的无力或麻木 D.失语 E.构音障碍 |
E.构音障碍 |
2 | 6 | 2011 | 2. 椎-基底动脉系统TIA不包括(2.0分) | A.跌倒发作 B.共济失调 C.异常的眼球运动 D.失读 E.交叉性运动或感觉障碍 |
D.失读 |
2 | 6 | 2011 | 3. 哪部分脑区缺血可致双眼视力障碍? (2.0分) | A.视交叉 B.顶叶 C.颞叶 D.枕叶 E.脑干 |
D.枕叶 |
2 | 6 | 2011 | 4. 跌倒发作是由于哪部分脑区缺血所致? (2.0分) | A.额叶 B.顶叶 C.颞叶 D.枕叶 E.脑干 |
E.脑干 |
2 | 6 | 2011 | 5. TIA治疗不包括(2.0分) | A.控制危险因素 B.抗血小板聚集 C.溶栓 D.抗凝 E.稳定斑块 |
C.溶栓 |
2 | 6 | 2011 | 6. 男性,21岁,右侧肢体抽搐2年。突然昏迷1小时。查体:神志浅昏迷,左侧肢体偏瘫。CT显示右额叶脑内血肿,临床诊断最可能是(2.0分) | A.原发性癫痫 B.脑动脉硬化 C.脑动脉瘤 D.脑血管畸形 E.高血压脑出血 |
D.脑血管畸形 |
2 | 6 | 2011 | 7. 高血压动脉硬化性脑出血最好发部位 (2.0分) | A.大脑皮质 B.内囊及基底节 C.丘脑 D.桥脑 E.小脑 |
B.内囊及基底节 |
2 | 6 | 2011 | 8. 小脑出血患者,剧烈咳嗽后出现意识障碍,需考虑(2.0分) | A.再出血 B.TIA C.脑疝形成 D.癫痫 E.癔症 |
C.脑疝形成 |
2 | 6 | 2011 | 9. 脑出血的预后与哪种因素有关(2.0分) | A.出血量 B.并发症严重程度 C.出血部位 D.出血量和部位 E.是否有其它器官疾病 |
D.出血量和部位 |
2 | 6 | 2011 | 10. 脑出血患者,血压高于多少时宜开始平稳降压(2.0分) | A.收缩压≥220 mmHg ,舒张压≥110mmHg B.收缩压≥200 mmHg ,舒张压≥110mmHg C.收缩压≥220 mmHg ,舒张压≥105mmHg D.收缩压≥180 mmHg ,舒张压≥100mmHg E.收缩压≥200 mmHg ,舒张压≥100mmHg |
B.收缩压≥200 mmHg ,舒张压≥110mmHg |
2 | 6 | 2011 | 11. 脑梗死后多长时间做头颅CT检查阳性率较高(2.0分) | A.5分钟 B.30分钟 C.1小时 D.12小时 E.24小时 |
E.24小时 |
2 | 6 | 2011 | 12. Willis环在脑循环中起非常重要的作用,能沟通脑前、后、左、右的血液供应,下列哪条动脉不参与脑底动脉环的组成: (2.0分) | A.大脑前动脉 B.前交通动脉 C.颈内动脉 D.大脑后动脉 E.椎动脉 |
E.椎动脉 |
2 | 6 | 2011 | 13. rtPA溶栓时间窗为(2.0分) | A.3小时内 B.3.5小时内 C.4小时内 D.4.5小时内 E.6小时内 |
D.4.5小时内 |
2 | 6 | 2011 | 14. 脑梗死并发症包括(2.0分) | A.脑水肿与颅内压增高 B.出血转化 C.癫痫 D.吞咽困难 E.以上都对 |
E.以上都对 |
2 | 6 | 2011 | 15. 蛛网膜下腔出血出现单侧眼睑下垂,需考虑(2.0分) | A.前交通动脉瘤 B.基底动脉瘤 C.椎动脉瘤 D.颈内动脉瘤 E.后交通动脉瘤 |
E.后交通动脉瘤 |
2 | 6 | 2011 | 16. 蛛网膜下腔出血最为可靠的诊断依据(2.0分) | A.一侧瞳孔散大 B.剧烈头痛、呕吐、肢体抽搐 C.腰穿为均匀一致血性脑脊液 D.脑膜刺激征阳性 E.头颅CT示脑膜高密度 |
C.腰穿为均匀一致血性脑脊液 |
2 | 6 | 2011 | 17. 蛛网膜下腔出血患者,出现深昏迷,去大脑强直,则Hunt and Hess 分级(2.0分) | A.1级 B.2级 C.3级 D.4级 E.5级 |
E.5级 |
2 | 6 | 2011 | 18. 蛛网膜下腔出血后发生脑血管痉挛的高峰期为(2.0分) | A.24小时内 B.1周内 C.5-14天 D.2周-1月 E.1月后 |
C.5-14天 |
2 | 6 | 2011 | 19. 男,26岁,清晨如厕时突发雷击样头痛,急诊头颅CT考虑蛛网膜下腔出血,经检查发现颅内动脉瘤。为了预防再出血,最根本措施是(2.0分) | A.应用脱水剂 B.控制血压在正常水平 C.应用止血剂 D.避免情绪激动及突然用力 E.及时进行动脉瘤手术 |
E.及时进行动脉瘤手术 |
2 | 6 | 2011 | 20. Which of the following is non intra-axial haemorrhage ?(2.0分) | A.lobar haemorrhage B.basal ganglia haemorrhage C.pontine haemorrhage D.subarachnoid haemorrhage (SAH) |
D.subarachnoid haemorrhage (SAH) |
2 | 6 | 2011 | 21. To demonstrate small amount of subarachnoid hemorrhage, which modality is the best choice? (2.0分) | A.MRI B.MRA C.CT D.DSA |
C.CT |
2 | 6 | 2011 | 22. Subacute intracerebral hematoma typically presents signal intensity as: (2.0分) | A.Iso T1 intensity and Iso T2 intensity B.Hyper T1 intensity and Hyper T2 intensity C.Hyper T1 intensity and hypo T2 intensity D.Hypo T1 intensity and hyper T2 intensity |
B.Hyper T1 intensity and Hyper T2 intensity |
2 | 6 | 2011 | 23. Which of the following has the highest sensitivity and specificity in the diagnosis of acute ischaemic infarction in the first hour after onset(2.0分) | A.CT without contrast B.MRI T2WI C.MRI T1WI D.DWI ( Diffiusion weighted imaging ) |
D.DWI ( Diffiusion weighted imaging ) |
2 | 6 | 2011 | 24. In the diagnosis of Cerebral arteriovenous malformation (AVM), which of the following is true.(2.0分) | A.MRA delineate the feeding vessels and the pattern of drainage better then DSA. B.CT is better then MRI in the evalution of an AVM C.CT can easily show three components: nidus of vessels , feeding arteris and drainage veins. D.MRI can easily show three components: nidus of vessels , feeding arteris and drainage veins. |
D.MRI can easily show three components: nidus of vessels , feeding arteris and drainage veins. |
2 | 6 | 2011 | 25. Which method is the gold standard in imaging a berry aneurysms(2.0分) | A.CTA (CT angiography ) B.MRA (MR angiography ) C.DSA (digital subtraction angiography ) D.MRI ( magnetic resonance imaging ) |
C.DSA (digital subtraction angiography ) |
2 | 6 | 2011 | 26. 关于一般适应综合征(GAS),以下正确的是 (2.0分) | A.分为觉醒期、阻抗期、适应期三个阶段 B.由加拿大生理学家塞里(H.Selye)提出 C.是特异性的 D.是机体面临有害刺激时表现出的心理反应 |
B.由加拿大生理学家塞里(H.Selye)提出 |
2 | 6 | 2011 | 27. 关于正性生活事件,以下叙述错误的是 (2.0分) | A.是对健康具有促进作用的事件 B.是当事人认为愉快的事件 C.是当事人认为积极的事件 D.是需要付出机体能量应对的事件 |
A.是对健康具有促进作用的事件 |
2 | 6 | 2011 | 28. 关于负性生活事件,以下叙述正确的是(2.0分) | A.对人产生损害的事件 B.个体感觉不愉快的事件 C.与健康成负相关的事件 D.公认的有害事件 |
B.个体感觉不愉快的事件 |
2 | 6 | 2011 | 29. 关于应激的心理反应,以下叙述不正确的是(2.0分) | A.应激的心理反应涉及"知、情、意、人格"各方面 B.应激的认知反应常常使个体的认知功能受损 C.应激的情绪反应包括焦虑、恐惧、抑郁、愤怒等 D.假如应激过于强烈或应激时间过长,有可能导致人格发生改变 |
B.应激的认知反应常常使个体的认知功能受损 |
2 | 6 | 2011 | 30. 以下属于主观事件的是(2.0分) | A.结婚 B.与同事关系紧张 C.亲人亡故 D.地震 |
B.与同事关系紧张 |
2 | 6 | 2011 | 31. 关于应对概念的错误叙述是(2.0分) | A.应对的涵义是多维度的 B.应对是针对应激性事件采取的任何认知和行为的措施 C.应对保护个体免受应激损害 D.应对活动涉及应激全过程 |
C.应对保护个体免受应激损害 |
2 | 6 | 2011 | 32. 下面不正确的叙述是(2.0分) | A.在心理神经免疫中介机制中,免疫系统与中枢神经进行双向调节 B.心理应激降低免疫功能 C.慢性应激通过过多分泌皮质激素而抑制免疫功能 D.心理神经免疫中介机制是肿瘤心理病因学关注的对象 |
B.心理应激降低免疫功能 |
2 | 6 | 2011 | 33. 与健康和疾病关系最直接的应激心理反应是(2.0分) | A.认知改变 B.情绪反应 C.个性改变 D.社会适应能力下降 |
B.情绪反应 |
2 | 6 | 2011 | 34. 头皮帽状腱膜下血肿不能吸收者应(2.0分) | A.切开止血 B.在严密的无菌条件下进行穿刺抽出积血加压包扎 C.穿刺抽血注入抗生素 D.等待血肿自行吸收 E.应用止血药物 |
B.在严密的无菌条件下进行穿刺抽出积血加压包扎 |
2 | 6 | 2011 | 35. 诊断颅底骨折的确切证据是(2.0分) | A.头颅X光照片多能显示颅底骨折 B.伤后进行性昏迷伴呕吐 C.脑脊液耳漏,鼻漏,眼睑有淤血斑 D.在受打击的局部立即出现枕下与眼睑或结合膜下淤血 E.头部超声波检查,中线偏移超过0.5厘米 |
C.脑脊液耳漏,鼻漏,眼睑有淤血斑 |
2 | 6 | 2011 | 36. 治疗颅底骨折合并脑脊液漏者,以下哪个方案是错误的?(2.0分) | A.绝对卧床休息 B.镇静,止痛,抗炎 C.防治肺部感染 D.将漏脑脊液的鼻孔或外耳道填塞 E.适当控制入水量 |
D.将漏脑脊液的鼻孔或外耳道填塞 |
2 | 6 | 2011 | 37. 重症闭合性颅脑外伤病人的急诊,首先应该做到(2.0分) | A.检查神志,瞳孔 B.测量呼吸,血压,脉搏 C.保持呼吸道通畅 D.躁动者可用吗啡类药物 E.及时输液,输血 |
A.检查神志,瞳孔 |
2 | 6 | 2011 | 38. 患儿5岁,阵发性头痛3个月,因突然剧烈头痛、反复呕吐半天急诊入院。检查:神志清醒,双侧瞳孔正常,颈项强直,半小时后突然呼吸停止、心跳存在。其诊断最可能是(2.0分) | A.垂体腺瘤 B.急性脑水肿 C.急性脑膜炎 D.枕骨大孔疝 E.小脑幕切迹疝 |
D.枕骨大孔疝 |
2 | 6 | 2011 | 39. 出现小脑幕切迹疝时,有定位意义的瞳孔变化是(2.0分) | A.患侧瞳孔先缩小,后逐渐扩大 B.患侧瞳孔逐渐缩小 C.双侧瞳孔散大 D.双侧瞳孔缩小 E.双侧瞳孔大小多变 |
A.患侧瞳孔先缩小,后逐渐扩大 |
2 | 6 | 2011 | 40. 高血压脑出血患者来院时昏迷,已脑疝,首先应采取的急救措施是(2.0分) | A.开颅手术 B.腰穿释放脑脊液 C.脑室穿刺 D.静脉快速滴注甘露醇 E.静脉注射50%葡萄糖 |
D.静脉快速滴注甘露醇 |
2 | 6 | 2011 | 41. 急性颅内压增高时患者早期生命体征改变为(2.0分) | A.血压升高,脉搏变缓,脉压变小 B.血压升高,脉搏增快,脉压增大 C.血压降低,脉搏变缓,脉压变小 D.血压降低,脉搏增快,脉压变小 E.血压升高,脉搏变缓,脉压增大 |
E.血压升高,脉搏变缓,脉压增大 |
2 | 6 | 2011 | 42. 颅内动脉瘤破裂出血后最大的危险是(2.0分) | A.并发颅神经麻痹 B.并发血管痉挛 C.再出血 D.高热 E.合并有动静脉畸形 |
C.再出血 |
2 | 6 | 2011 | 43. 一儿童患者,突发头痛、呕吐伴癫痫,CT示脑实质内出血约10ml,最可能的出血原因是:(2.0分) | A.脑动静脉畸形 B.MOYAMOYA病 C.海绵状血管瘤 D.高血压脑出血 |
B.MOYAMOYA病 |
2 | 6 | 2011 | 44. 哌替啶的不良反应不包括(2.0分) | A.抑制呼吸 B.便秘 C.心动过速 D.免疫抑制 E.惊厥 |
B.便秘 |
2 | 6 | 2011 | 45. 哌替啶比吗啡应用多的原因是(2.0分) | A.无便秘作用 B.呼吸抑制作用轻 C.作用较慢,维持时间短 D.成瘾性较吗啡轻 E.对支气管平滑肌无影响 |
D.成瘾性较吗啡轻 |
2 | 6 | 2011 | 46. 与脑内阿片受体作用无关镇痛药是 (2.0分) | A.罗通定 B.喷他佐辛 C.曲马朵 D.美沙酮 E.纳洛酮 |
A.罗通定 |
2 | 6 | 2011 | 47. 高血压脑出血最易发生的部位是 (2.0分) | A.大脑白质 B.小脑 C.丘脑区域 D.桥脑 E.内囊及基底节区域 |
E.内囊及基底节区域 |
2 | 6 | 2011 | 48. 动脉粥样硬化合并血栓形成的最主要的机制是 (2.0分) | A.血流涡流形成 B.局部血流缓慢 C.内膜受损、胶原纤维暴露 D.局部血液凝固性增高 E.以上都不是 |
B.局部血流缓慢 |
2 | 6 | 2011 | 49. 脑出血的最常见原因是 (2.0分) | A.高血压病和脑动脉硬化 B.血液病 C.脑动脉炎 D.血管瘤 E.脑血管畸形 |
A.高血压病和脑动脉硬化 |
2 | 6 | 2011 | 50. 下列哪根血管闭塞最容易导致偏瘫(2.0分) | A.大脑中动脉 B.大脑后动脉 C.大脑前动脉 D.小脑后下动脉 E.小脑前下动脉 |
A.大脑中动脉 |
2 | 6 | 2012 | 1. 颈内动脉系统TIA不包括(2.0分) | A.单侧眼或大脑半球症状 B.一过性黑朦、雾视、视野黑点 C.一侧面部或肢体的无力或麻木 D.失语 E.构音障碍 |
E.构音障碍 |
2 | 6 | 2012 | 2. 椎-基底动脉系统TIA不包括(2.0分) | A.跌倒发作 B.共济失调 C.异常的眼球运动 D.失读 E.交叉性运动或感觉障碍 |
D.失读 |
2 | 6 | 2012 | 3. 跌倒发作是由于哪部分脑区缺血所致? (2.0分) | A.额叶 B.顶叶 C.颞叶 D.枕叶 E.脑干 |
E.脑干 |
2 | 6 | 2012 | 4. TIA治疗不包括 (2.0分) | A.控制危险因素 B.抗血小板聚集 C.溶栓 D.抗凝 E.稳定斑块 |
C.溶栓 |
2 | 6 | 2012 | 5. 关于短暂性全面遗忘症下列叙述正确的是? (2.0分) | A.是椎基底动脉系统TIA的一种临床表现 B.累及海马,颞叶内侧 C.发作性短时记忆丧失 D.脑电图正常 E.以上都对 |
E.以上都对 |
2 | 6 | 2012 | 6. 男性,21岁,右侧肢体抽搐2年。突然昏迷1小时。查体:神志浅昏迷,左侧肢体偏瘫。CT显示右额叶脑内血肿,临床诊断最可能是 (2.0分) | A.原发性癫痫 B.脑动脉硬化 C.脑动脉瘤 D.脑血管畸形 E.高血压脑出血 |
D.脑血管畸形 |
2 | 6 | 2012 | 7. 小脑出血患者,剧烈咳嗽后出现意识障碍,需考虑 (2.0分) | A.再出血 B.TIA C.脑疝形成 D.癫痫 E.癔症 |
C.脑疝形成 |
2 | 6 | 2012 | 8. 脑出血患者,血压高于多少时宜开始平稳降压(2.0分) | A.收缩压≥220 mmHg ,舒张压≥110mmHg B.收缩压≥200 mmHg ,舒张压≥110mmHg C.收缩压≥220 mmHg ,舒张压≥105mmHg D.收缩压≥180 mmHg ,舒张压≥100mmHg E.收缩压≥200 mmHg ,舒张压≥100mmHg |
B.收缩压≥200 mmHg ,舒张压≥110mmHg |
2 | 6 | 2012 | 9. 脑淀粉样变性所致脑出血最常见于哪个部位? (2.0分) | A.脑叶 B.基底节 C.延髓 D.小脑 E.脑室 |
A.脑叶 |
2 | 6 | 2012 | 10. 脑出血的预后与哪种因素有关:(2.0分) | A.出血量 B.并发症严重程度 C.出血部位 D.出血量和部位 E.是否有其它器官疾病 |
D.出血量和部位 |
2 | 6 | 2012 | 11. 脑梗死后多长时间做头颅CT检查阳性率较高 (2.0分) | A.5分钟 B.30分钟 C.1小时 D.12小时 E.24小时 |
E.24小时 |
2 | 6 | 2012 | 12. rtPA溶栓时间窗为 (2.0分) | A.3小时内 B.3.5小时内 C.4小时内 D.4.5小时内 E.6小时内 |
D.4.5小时内 |
2 | 6 | 2012 | 13. 脑梗死并发症包括 (2.0分) | A.脑水肿与颅内压增高 B.出血转化 C.癫痫 D.吞咽困难 E.以上都对 |
E.以上都对 |
2 | 6 | 2012 | 14. 脑梗死二级预防三大基石为(2.0分) | A.抗血小板治疗,稳定斑块治疗,降压治疗 B.抗血小板治疗,降糖,降压治疗 C.抗血小板治疗,稳定斑块治疗,降糖治疗 D.抗凝治疗,稳定斑块治疗,降压治疗 E.抗血小板治疗,稳定斑块治疗,降糖治疗 |
A.抗血小板治疗,稳定斑块治疗,降压治疗 |
2 | 6 | 2012 | 15. 男性,右利手,主诉3天来左眼失明多次发作,每次持续5分钟。进而出现语言表达困难,右手抬举不能,右侧鼓腮漏气。最可能的诊断是: (2.0分) | A.左侧大脑中动脉闭塞 B.累及左侧运动皮层区的局灶性癫痫发作 C.左侧颈内动脉闭塞 D.慢性硬脑膜下血肿 E.偏头痛 |
C.左侧颈内动脉闭塞 |
2 | 6 | 2012 | 16. 蛛网膜下腔出血出现单侧眼睑下垂,需考虑 (2.0分) | A.前交通动脉瘤 B.基底动脉瘤 C.椎动脉瘤 D.颈内动脉瘤 E.后交通动脉瘤 |
E.后交通动脉瘤 |
2 | 6 | 2012 | 17. 蛛网膜下腔出血患者,出现深昏迷,去大脑强直,则Hunt and Hess 分级:(2.0分) | A.1级 B.2级 C.3级 D.4级 E.5级 |
E.5级 |
2 | 6 | 2012 | 18. 动脉瘤破裂的危险因素不包括(2.0分) | A.吸烟 B.低血压 C.动脉粥样硬化 D.高龄 E.口服避孕药 |
B.低血压 |
2 | 6 | 2012 | 19. 以下哪个是脑出血与蛛网膜下腔出血最重要的鉴别点 (2.0分) | A.活动中起病 B.头痛呕吐 C.脑脊液含血量多少 D.CT检查发现脑实质内高密度影 E.昏迷 |
D.CT检查发现脑实质内高密度影 |
2 | 6 | 2012 | 20. 男,26岁,清晨如厕时突发雷击样头痛,急诊头颅CT考虑蛛网膜下腔出血,经检查发现颅内动脉瘤。为了预防再出血,最根本措施是 (2.0分) | A.应用脱水剂 B.控制血压在正常水平 C.应用止血剂 D.避免情绪激动及突然用力 E.及时进行动脉瘤手术 |
E.及时进行动脉瘤手术 |
2 | 6 | 2012 | 21. Cushing反应是指:(2.0分) | A.颅内压升高急性期出现血压上升,脉搏加快以及呼吸不规律现象,为皮质缺血缺氧引起 B.颅内压升高急性期出现血压下降,脉搏减慢以及呼吸不规律现象,为脑干缺血缺氧引起 C.颅内压升高急性期出现血压上升,脉搏减慢以及呼吸不规律现象,为脑干缺血缺氧引起 D.颅内压升高急性期出现血压下降,脉搏加快以及呼吸不规律现象,为脑干缺血缺氧引起 E.颅内压升高急性期出现血压上升,脉搏减慢以及呼吸不规律现象,为皮质缺血缺氧引起 |
C.颅内压升高急性期出现血压上升,脉搏减慢以及呼吸不规律现象,为脑干缺血缺氧引起 |
2 | 6 | 2012 | 22. 颅高压的治疗方法中,不包括:(2.0分) | A.头高位,过度换气,镇静镇痛 B.腰穿释放脑脊液 C.脱水药物及激素使用 D.占位性病变切除 E.侧脑室外引流,去骨瓣减压 |
B.腰穿释放脑脊液 |
2 | 6 | 2012 | 23. 出现小脑幕切迹疝时,有定位意义的瞳孔变化是(2.0分) | A.患侧瞳孔先缩小,后逐渐扩大 B.患侧瞳孔逐渐缩小 C.双侧瞳孔散大 D.双侧瞳孔缩小 E.双侧瞳孔大小多变 |
A.患侧瞳孔先缩小,后逐渐扩大 |
2 | 6 | 2012 | 24. 下列哪项不属于标准分: (2.0分) | A.Z分 B.T分 C.标准20 D.划界分 |
D.划界分 |
2 | 6 | 2012 | 25. 离差智商适用于: (2.0分) | A.16岁以上成人 B.18岁以上成人 C.20岁以上成人 D.任何年龄 |
D.任何年龄 |
2 | 6 | 2012 | 26. 卡特尔人格因素测验将人格根源特质分为多少种: (2.0分) | A.8种 B.12种 C.16种 D.18种 |
C.16种 |
2 | 6 | 2012 | 27. 不属于症状评定量表的一项是: (2.0分) | A.SCL-90 B.MMPI C.SDS D.SAS |
B.MMPI |
2 | 6 | 2012 | 28. SDS评估以下什么内容 (2.0分) | A.生活事件 B.抑郁症状 C.心理症状 D.焦虑症状 |
B.抑郁症状 |
2 | 6 | 2012 | 29. 一位20多岁的男青年,与同事相处不好,总觉得他们跟自己过不去,不知如何处理而就诊心理门诊。如果需要作心理评估,应首先考虑选用的量表是: (2.0分) | A.16PF B.EPQ C.MMPI D.SCL-90 |
C.MMPI |
2 | 6 | 2012 | 30. 效度是指测验分数的:(2.0分) | A.有效性 B.可靠性 C.一致性 D.灵敏性 |
A.有效性 |
2 | 6 | 2012 | 31. 关于一般适应综合征(GAS),以下不正确的是(2.0分) | A.分为觉醒期、阻抗期、适应期三个阶段 B.由加拿大生理学家塞里(H.Selye)提出 C.是非特异性的 D.是机体面临有害刺激时表现出的生理生化反应 |
A.分为觉醒期、阻抗期、适应期三个阶段 |
2 | 6 | 2012 | 32. 关于负性生活事件,以下叙述正确的是: (2.0分) | A.对人产生损害的事件 B.个体感觉不愉快的事件 C.与健康成负相关的事件 D.公认的有害事件 |
B.个体感觉不愉快的事件 |
2 | 6 | 2012 | 33. 以下属于主观事件的是 (2.0分) | A.结婚 B.与同事关系紧张 C.亲人亡故 D.地震 |
B.与同事关系紧张 |
2 | 6 | 2012 | 34. 关于应对概念的错误叙述是 (2.0分) | A.应对的涵义是多维度的 B.应对是针对应激性事件采取的任何认知和行为的措施 C.应对保护个体免受应激损害 D.应对活动涉及应激全过程 |
C.应对保护个体免受应激损害 |
2 | 6 | 2012 | 35. 关于心理应激的概念,下列叙述中错误的是: (2.0分) | A.应激源涉及生物的、心理的、社会的和文化的 B.应激反应包括生理、心理、行为反应 C.认知评价在心理应激中起关键作用 D.应激的本质就是个体对内外环境的需求不能适应 |
D.应激的本质就是个体对内外环境的需求不能适应 |
2 | 6 | 2012 | 36. 应激中不利于适应的人格特征不包括: (2.0分) | A.A型人格 B.B型人格 C.C型人格 D.D型人格 |
B.B型人格 |
2 | 6 | 2012 | 37. 以下不属于心理活动应对的是: (2.0分) | A.再评价 B.幻想 C.自责 D.回避 |
D.回避 |
2 | 6 | 2012 | 38. 强调潜意识心理冲突、个体心理特征产生心身疾病的研究途径是(2.0分) | A.心理生理学理论 B.学习理论 C.心理动力理论 D.人本主义理论 |
C.心理动力理论 |
2 | 6 | 2012 | 39. 说明心理社会因素"如何"通过神经内分泌和免疫系统造成躯体疾病的研究领域称为: (2.0分) | A.神经免疫学 B.心理免疫学 C.免疫学 D.心理神经免疫学 |
D.心理神经免疫学 |
2 | 6 | 2012 | 40. 心身疾病的主要干预方法: (2.0分) | A.消除心理社会刺激因素 B.行为矫正 C.认知干预治疗 D.以上都是 |
D.以上都是 |
2 | 6 | 2012 | 41. 失眠的心理社会因素不包括: (2.0分) | A.过度疲劳或紧张 B.对健康过度关心 C.不良自我暗示 D.入睡前饮用兴奋性饮料 |
D.入睡前饮用兴奋性饮料 |
2 | 6 | 2012 | 42. 父母对子女的轻微损伤表示大惊小怪或者泰然处之,影响子女成年后对疼痛的态度属于: (2.0分) | A.社会学习 B.对处境的认知评价 C.注意 D.暗示 |
A.社会学习 |
2 | 6 | 2012 | 43. 下列有关心理社会因素与癌症关系的叙述不正确的是 (2.0分) | A.不良心理反应和应付方式对癌症病人生活质量有严重影响 B.争强好胜、时间紧迫感、脾气暴躁不是癌症病人的行为特点 C.癌症病人常常有意识疏泄自己的情绪来减轻紧张和痛苦 D.突然得知患癌消息的病人情绪反应强烈,易导致免疫力下降 |
C.癌症病人常常有意识疏泄自己的情绪来减轻紧张和痛苦 |
2 | 6 | 2012 | 44. 癌症病人听到癌症的诊断后,出现心理反应的顺序是: (2.0分) | A.否认-怀疑期,愤怒-沮丧期,接受-适应期,休克-恐惧期 B.否认-怀疑期,休克-恐惧期,愤怒-沮丧期,接受-适应期 C.休克-恐惧期,愤怒-沮丧期,接受-适应期,否认-怀疑期 D.休克-恐惧期,否认-怀疑期,愤怒-沮丧期,接受-适应期 |
D.休克-恐惧期,否认-怀疑期,愤怒-沮丧期,接受-适应期 |
2 | 6 | 2012 | 45. 哌替啶的不良反应不包括(2.0分) | A.抑制呼吸 B.便秘 C.心动过速 D.免疫抑制 E.惊厥 |
B.便秘 |
2 | 6 | 2012 | 46. 与脑内阿片受体作用无关镇痛药是 (2.0分) | A.罗通定 B.喷他佐辛 C.曲马朵 D.美沙酮 E.纳洛酮 |
A.罗通定 |
2 | 6 | 2012 | 47. 哌替啶比吗啡应用多的原因是 (2.0分) | A.无便秘作用 B.呼吸抑制作用轻 C.作用较慢,维持时间短 D.成瘾性较吗啡轻 E.对支气管平滑肌无影响 |
D.成瘾性较吗啡轻 |
2 | 6 | 2012 | 48. 高血压脑出血时,哪支血管破裂最常见?(2.0分) | A.基底动脉 B.大脑后动脉 C.大脑中动脉 D.大脑前动脉 E.豆纹动脉 |
E.豆纹动脉 |
2 | 6 | 2012 | 49. 蛛网膜下腔出血的最常见原因是 (2.0分) | A.高血压病 B.血液病 C.脑动脉硬化 D.血管瘤 E.脑血管畸形 |
D.血管瘤 |
2 | 6 | 2012 | 50. 下列哪根血管闭塞最容易导致偏瘫 (2.0分) | A.大脑中动脉 B.大脑后动脉 C.大脑前动脉 D.小脑后下动脉 E.小脑前下动脉 |
A.大脑中动脉 |
2 | 6 | 2013 | 1. 颈内动脉系统TIA不包括(2.5分) | A.单侧眼或大脑半球症状 B.一过性黑朦、雾视、视野黑点 C.一侧面部或肢体的无力或麻木 D.失语 E.构音障碍 |
E.构音障碍 |
2 | 6 | 2013 | 2. 椎-基底动脉系统TIA不包括(2.5分) | A.跌倒发作 B.共济失调 C.异常的眼球运动 D.失读 E.交叉性运动或感觉障碍 |
D.失读 |
2 | 6 | 2013 | 3. 跌倒发作是由于哪部分脑区缺血所致? (2.5分) | A.额叶 B.顶叶 C.颞叶 D.枕叶 E.脑干 |
E.脑干 |
2 | 6 | 2013 | 4. TIA治疗不包括(2.5分) | A.控制危险因素 B.抗血小板聚集 C.溶栓 D.抗凝 E.稳定斑块 |
C.溶栓 |
2 | 6 | 2013 | 5. 关于短暂性全面遗忘症下列叙述正确的是? (2.5分) | A.是椎基底动脉系统TIA的一种临床表现 B.累及海马,颞叶内侧 C.发作性短时记忆丧失 D.脑电图正常 E.以上都对 |
E.以上都对 |
2 | 6 | 2013 | 6. 男性,21岁,右侧肢体抽搐2年。突然昏迷1小时。查体:神志浅昏迷,左侧肢体偏瘫。CT显示右额叶脑内血肿,临床诊断最可能是(2.5分) | A.原发性癫痫 B.脑动脉硬化 C.脑动脉瘤 D.脑血管畸形 E.高血压脑出血 |
D.脑血管畸形 |
2 | 6 | 2013 | 7. 小脑出血患者,剧烈咳嗽后出现意识障碍,需考虑(2.5分) | A.再出血 B.TIA C.脑疝形成 D.癫痫 E.癔症 |
C.脑疝形成 |
2 | 6 | 2013 | 8. 脑出血患者,血压高于多少时宜开始平稳降压(2.5分) | A.收缩压≥220 mmHg ,舒张压≥110mmHg B.收缩压≥200 mmHg ,舒张压≥110mmHg C.收缩压≥220 mmHg ,舒张压≥105mmHg D.收缩压≥180 mmHg ,舒张压≥100mmHg E.收缩压≥200 mmHg ,舒张压≥100mmHg |
B.收缩压≥200 mmHg ,舒张压≥110mmHg |
2 | 6 | 2013 | 9. 脑淀粉样变性所致脑出血最常见于哪个部位? (2.5分) | A.脑叶 B.基底节 C.延髓 D.小脑 E.脑室 |
A.脑叶 |
2 | 6 | 2013 | 10. 脑出血的预后与哪种因素有关(2.5分) | A.出血量 B.并发症严重程度 C.出血部位 D.出血量和部位 E.是否有其它器官疾病 |
D.出血量和部位 |
2 | 6 | 2013 | 11. 脑梗死后多长时间做头颅CT检查阳性率较高(2.5分) | A.5分钟 B.30分钟 C.1小时 D.12小时 E.24小时 |
E.24小时 |
2 | 6 | 2013 | 12. rtPA溶栓时间窗为(2.5分) | A.3小时内 B.3.5小时内 C.4小时内 D.4.5小时内 E.6小时内 |
D.4.5小时内 |
2 | 6 | 2013 | 13. 脑梗死并发症包括 (2.5分) | A.脑水肿与颅内压增高 B.出血转化 C.癫痫 D.吞咽困难 E.以上都对 |
E.以上都对 |
2 | 6 | 2013 | 14. 脑梗死二级预防三大基石为(2.5分) | A.抗血小板治疗,稳定斑块治疗,降压治疗 B.抗血小板治疗,降糖,降压治疗 C.抗血小板治疗,稳定斑块治疗,降糖治疗 D.抗凝治疗,稳定斑块治疗,降压治疗 E.抗血小板治疗,稳定斑块治疗,降糖治疗 |
A.抗血小板治疗,稳定斑块治疗,降压治疗 |
2 | 6 | 2013 | 15. 男性,右利手,主诉3天来左眼失明多次发作,每次持续5分钟。进而出现语言表达困难,右手抬举不能,右侧鼓腮漏气。最可能的诊断是: (2.5分) | A.左侧大脑中动脉闭塞 B.累及左侧运动皮层区的局灶性癫痫发作 C.左侧颈内动脉闭塞 D.慢性硬脑膜下血肿 E.偏头痛 |
C.左侧颈内动脉闭塞 |
2 | 6 | 2013 | 16. 蛛网膜下腔出血出现单侧眼睑下垂,需考虑(2.5分) | A.前交通动脉瘤 B.基底动脉瘤 C.椎动脉瘤 D.颈内动脉瘤 E.后交通动脉瘤 |
E.后交通动脉瘤 |
2 | 6 | 2013 | 17. 蛛网膜下腔出血患者,出现深昏迷,去大脑强直,则Hunt and Hess 分级(2.5分) | A.1级 B.2级 C.3级 D.4级 E.5级 |
E.5级 |
2 | 6 | 2013 | 18. 动脉瘤破裂的危险因素不包括(2.5分) | A.吸烟 B.低血压 C.动脉粥样硬化 D.高龄 E.口服避孕药 |
B.低血压 |
2 | 6 | 2013 | 19. 以下哪个是脑出血与蛛网膜下腔出血最重要的鉴别点 (2.5分) | A.活动中起病 B.头痛呕吐 C.脑脊液含血量多少 D.CT检查发现脑实质内高密度影 E.昏迷 |
D.CT检查发现脑实质内高密度影 |
2 | 6 | 2013 | 20. 男,26岁,清晨如厕时突发雷击样头痛,急诊头颅CT考虑蛛网膜下腔出血,经检查发现颅内动脉瘤。为了预防再出血,最根本措施是(2.5分) | A.应用脱水剂 B.控制血压在正常水平 C.应用止血剂 D.避免情绪激动及突然用力 E.及时进行动脉瘤手术 |
E.及时进行动脉瘤手术 |
2 | 6 | 2013 | 21. Cushing反应是指(2.5分) | A.颅内压升高急性期出现血压上升,脉搏加快以及呼吸不规律现象,为皮质缺血缺氧引起 B.颅内压升高急性期出现血压下降,脉搏减慢以及呼吸不规律现象,为脑干缺血缺氧引起 C.颅内压升高急性期出现血压上升,脉搏减慢以及呼吸不规律现象,为脑干缺血缺氧引起 D.颅内压升高急性期出现血压下降,脉搏加快以及呼吸不规律现象,为脑干缺血缺氧引起 E.颅内压升高急性期出现血压上升,脉搏减慢以及呼吸不规律现象,为皮质缺血缺氧引起 |
C.颅内压升高急性期出现血压上升,脉搏减慢以及呼吸不规律现象,为脑干缺血缺氧引起 |
2 | 6 | 2013 | 22. 颅高压的治疗方法中,不包括(2.5分) | A.头高位,过度换气,镇静镇痛 B.腰穿释放脑脊液 C.脱水药物及激素使用 D.占位性病变切除 E.侧脑室外引流,去骨瓣减压 |
B.腰穿释放脑脊液 |
2 | 6 | 2013 | 23. 出现小脑幕切迹疝时,有定位意义的瞳孔变化是(2.5分) | A.患侧瞳孔先缩小,后逐渐扩大 B.患侧瞳孔逐渐缩小 C.双侧瞳孔散大 D.双侧瞳孔缩小 E.双侧瞳孔大小多变 |
A.患侧瞳孔先缩小,后逐渐扩大 |
2 | 6 | 2013 | 24. 高血压脑出血时,哪支血管破裂最常见?(2.5分) | A.基底动脉 B.大脑后动脉 C.大脑中动脉 D.大脑前动脉 E.豆纹动脉 |
E.豆纹动脉 |
2 | 6 | 2013 | 25. 蛛网膜下腔出血的最常见原因是 (2.5分) | A.高血压病 B.血液病 C.脑动脉硬化 D.血管瘤 E.脑血管畸形 |
D.血管瘤 |
2 | 6 | 2013 | 26. 下列哪根血管闭塞最容易导致偏瘫(2.5分) | A.大脑中动脉 B.大脑后动脉 C.大脑前动脉 D.小脑后下动脉 E.小脑前下动脉 |
A.大脑中动脉 |
2 | 6 | 2013 | 27. 下述哪项不是脑血管造影的适应症(2.5分) | A.脑内或蛛网膜下腔出血病因检查 B.怀疑脑静脉病变 C.头面部富血供肿瘤术前检查 D.并发脑疝或其他危及生命的情况。 |
D.并发脑疝或其他危及生命的情况。 |
2 | 6 | 2013 | 28. 颅内动脉粥样硬化狭窄支架植入术的适应症,目前推荐意见: (2.5分) | A.体检发现颅内动脉粥样硬化狭窄超过70%,目前无症状 B.第一次脑梗死后检查发现同侧颅内动脉粥样硬化狭窄超过70-99% C.第一次脑梗死后检查发现同侧颅内动脉粥样硬化狭窄超过70-99%,经内科药物治疗后仍有短暂性脑缺血发作, D.颅内动脉完全闭塞。 |
C.第一次脑梗死后检查发现同侧颅内动脉粥样硬化狭窄超过70-99%,经内科药物治疗后仍有短暂性脑缺血发作, |
2 | 6 | 2013 | 29. 下述哪项属于颈动脉支架植入术的适应症(2.5分) | A.无症状颈动脉颈动脉粥样硬化伴管腔狭窄60% B.无症状颈动脉粥样硬化伴管腔狭窄75% C.症状性颈动脉颈动脉粥样硬化伴管腔狭窄49%合并溃疡性斑块 D.脑梗后发现同侧颈内动脉夹层伴狭窄80%,目前无脑缺血症状。 |
B.无症状颈动脉粥样硬化伴管腔狭窄75% |
2 | 6 | 2013 | 30. 下述不属于缺血性脑血管病二级预防的是(2.5分) | A.近期发生TIA合并同侧颈动脉颅外段动脉粥样硬化伴管腔严重狭窄(70-99%),进行颈动脉支架植入术 B.6个月内发生缺血性脑卒中合并同侧颈动脉颅外段动脉粥样硬化伴管腔严重狭窄(70-99%),进行颈动脉支架植入术 C.近期发生TIA合并同侧颈动脉颅外段动脉粥样硬化伴管腔严重狭窄(50-69%),进行颈动脉支架植入术 D.体检发现颈动脉颅外段动脉粥样硬化伴管腔严重狭窄70-99%,目前无缺血症状,进行颈动脉支架植入术。 |
D.体检发现颈动脉颅外段动脉粥样硬化伴管腔严重狭窄70-99%,目前无缺血症状,进行颈动脉支架植入术。 |
2 | 6 | 2013 | 31. 以下哪个不是急性脑梗死的早期CT征象(2.5分) | A.高密度血管征 B.豆状核模糊征 C.岛带征 D.烟雾征 E.脑组织密度减低 |
D.烟雾征 |
2 | 6 | 2013 | 32. 发现急性脑梗死最敏感的检查方法是(2.5分) | A.CT脑灌注 B.MR脑灌注 C.MR T2 FLAIR成像 D.MR DWI成像 E.MR SWI 成像 |
D.MR DWI成像 |
2 | 6 | 2013 | 33. 以下不属于脑静脉血栓形成的直接征象的是(2.5分) | A.静脉窦高密度征 B.静脉窦流空信号消失 C.静脉性脑梗死 D.空三角征 E.Δ征 |
C.静脉性脑梗死 |
2 | 6 | 2013 | 34. 脑疝患者的紧急治疗手段不包括:(2.5分) | A.维持呼吸道畅通 B.立即静脉滴注20%甘露醇250-500 ml C.侧脑室穿刺外引流 D.减压手术如去骨瓣减压,颞肌下减压,内减压术等 E.腰穿引流脑脊液 |
E.腰穿引流脑脊液 |
2 | 6 | 2013 | 35. 下列哪项不属于颅内压升高的临床分期(2.5分) | A.早期:超过代偿容积,出现临床症状 B.进展期:内容物持续增加,临床症状加重 C.衰竭期:脑组织几无血流灌注 D.高峰期:典型颅高压三联症 E.代偿期:内容物虽有增加,未超过代偿容积 |
B.进展期:内容物持续增加,临床症状加重 |
2 | 6 | 2013 | 36. 颅内压的主要影响因素不包括(2.5分) | A.病变扩张速度 B.全身系统性疾病 C.伴发水肿程度 D.病变部位 E.患者BMI |
E.患者BMI |
2 | 6 | 2013 | 37. 所谓颅内巨大动脉瘤是指直径大于(2.5分) | A.1cm B.1.5cm C.2cm D.2.5cm E.3cm |
D.2.5cm |
2 | 6 | 2013 | 38. 一儿童患者,突发头痛、呕吐伴癫痫,CT示脑实质内出血约10ml,最可能的出血原因是:(2.5分) | A.动脉瘤 B.脑动静脉畸形 C.MOYAMOYA病 D.海绵状血管瘤 E.高血压脑出血 |
B.脑动静脉畸形 |
2 | 6 | 2013 | 39. 下列哪项不属于枕骨大孔疝的特征性临床表现(2.5分) | A.枕下疼痛 B.剧烈头痛伴喷射性呕吐 C.项强或强迫头 D.早期出现生命体征改变 E.后组脑神经受累:眩晕、听力减退 |
B.剧烈头痛伴喷射性呕吐 |
2 | 6 | 2013 | 40. 以下属于脑小血管病的影像学表现的是(2.5分) | A.皮层下小梗死 B.脑腔隙和血管周围间隙 C.脑白质高信号 D.脑内微出血 E.以上都是 |
E.以上都是 |
2 | 6 | 2014 | 1. 颅骨凹陷性骨折下列哪个情况不需手术治疗(2.5分) | A.明显的颅内压增高表现 B.出现与部位相符的局灶性神经功能障碍 C.颅骨凹陷深度超过1公分 D.颅骨凹陷面积超过25平方厘米 |
D.颅骨凹陷面积超过25平方厘米 |
2 | 6 | 2014 | 2. 关于慢性硬膜下血肿,哪项描述不准确(2.5分) | A.发病与年龄段无明显关系 B.常有轻微外伤史 C.出血机理有多种学说 D.预后好,高龄仍应积极手术 |
A.发病与年龄段无明显关系 |
2 | 6 | 2014 | 3. 关于头皮损伤,哪项描述不准确(2.5分) | A.皮下血肿一般无需特别处理 B.帽状腱膜下血肿因为血肿量大需开颅处理 C.骨膜下血肿应警惕颅内情况 D.头皮撕脱伤应由整形科显微处理 |
B.帽状腱膜下血肿因为血肿量大需开颅处理 |
2 | 6 | 2014 | 4. 关于脑干损伤,哪项描述准确(2.5分) | A.高处坠落多致脑干损伤 B.多伴颅内血肿,应积极手术处理 C.可考虑为DAI第3级 D.伤后即刻意识可以清晰,但随即出现昏迷 |
C.可考虑为DAI第3级 |
2 | 6 | 2014 | 5. 儿童的正常颅内压范围(2.5分) | A.20-80 mmH2O B.50-100 mmH2O C.80-180 mmH2O D.6.0-13.5 mmH2O E.3.7-7.4 mmH2O |
B.50-100 mmH2O |
2 | 6 | 2014 | 6. 枕骨大孔疝与小脑幕切迹疝主要鉴别点在于(2.5分) | A.头痛剧烈 B.躁动不安 C.呼吸骤停在早期出现 D.脉搏加快,血压升高 E.呕吐频繁 |
C.呼吸骤停在早期出现 |
2 | 6 | 2014 | 7. 颅内压超过多少时会引起Cushing反应? (2.5分) | A.15mmH2O B.20mmH2O C.25mmH2O D.30mmH2O E.35mmH2O |
E.35mmH2O |
2 | 6 | 2014 | 8. 枕骨大孔疝,下列哪一项不是常见症状(2.5分) | A.早期出现一侧瞳孔散大 B.呼吸突然停止 C.头痛剧烈,呕吐频繁 D.强迫头位 E.意识障碍 |
A.早期出现一侧瞳孔散大 |
2 | 6 | 2014 | 9. 诊断小脑幕切迹疝的最可靠依据是(2.5分) | A.患侧瞳孔散大,对侧肢体肌力减弱 B.患侧瞳孔不变,同侧肢体肌力减弱 C.患侧瞳孔不变,对侧肢体肌力减弱 D.患侧瞳孔缩小,同侧肢体肌力减弱 E.患侧瞳孔散大,同侧肢体肌力减弱 |
A.患侧瞳孔散大,对侧肢体肌力减弱 |
2 | 6 | 2014 | 10. 关于行为治疗,以下的不正确的是(2.5分) | A.以行为学习理论为指导 B.重视发挥个人的潜能 C.有一定的治疗程序 D.着眼于改变不适应的行为模式 E. |
B.重视发挥个人的潜能 |
2 | 6 | 2014 | 11. 下列心理治疗方法中可能产生立竿见影效果的是 (2.5分) | A.精神分析疗法 B.暗示疗法 C.系统脱敏疗法 D.森田疗法 |
B.暗示疗法 |
2 | 6 | 2014 | 12. 心理治疗不适用于对以下哪类病人的心理问题进行干预(2.5分) | A.肿瘤病人 B.冠心病病人 C.焦虑症病人 D.精神分裂症病人 |
D.精神分裂症病人 |
2 | 6 | 2014 | 13. 来访者中心疗法不主张(2.5分) | A.发展来访者主观能动性 B.对来访者明确指导 C.促进来访者成长 D.治疗是来访者转变和学习的过程 |
B.对来访者明确指导 |
2 | 6 | 2014 | 14. 支持疗法的干预措施中, 不包括(2.5分) | A.挖掘幼年体验 B.善用"资源" C.提高应对能力 D.认知调整 |
A.挖掘幼年体验 |
2 | 6 | 2014 | 15. 在厌恶疗法中不选择作为厌恶刺激的是以下哪一项 (2.5分) | A.电击刺激 B.隔离 C.催吐剂 D.想象被羞辱 |
B.隔离 |
2 | 6 | 2014 | 16. 女,50岁,突发头痛5天,头痛位于整个头部,呈炸裂样,伴恶心,无关节疼痛,无发热,体格检查:颈抵抗,克氏征阳性。头颅CT示鞍上池高密度影。最可能的诊断是:(2.5分) | A.颅内感染 B.紧张性头痛 C.急性脑梗塞 D.蛛网膜下腔出血 E.短暂性脑缺血发作 |
D.蛛网膜下腔出血 |
2 | 6 | 2014 | 17. 男,65岁,突发右侧肢体无力三小时,无发热,查体:神清,右侧中枢性面瘫,右侧肢体肌力4级,右侧偏身感觉减退,查头颅CT未见明显异常,既往2型糖尿病史5年。病变责任血管可能是(2.5分) | A.左侧大脑中动脉 B.右侧大脑中动脉 C.右侧颈内动脉 D.基底动脉 E.右侧大脑后动脉 |
B.右侧大脑中动脉 |
2 | 6 | 2014 | 18. 下列哪项是引起脑梗塞的最常见原因:(2.5分) | A.房颤 B.空气栓塞 C.脑动脉粥样硬化 D.动脉炎 E.血小板增多症 |
C.脑动脉粥样硬化 |
2 | 6 | 2014 | 19. 男,60岁。突发口齿不清伴左侧肢体无力5小时。查体:右侧周围性面瘫,左侧肢体肌力4级,右侧肢体肌力5级。考虑患者病变部位位于(2.5分) | A.脑皮质 B.脑干 C.内囊 D.脊 E.周围神经 |
B.脑干 |
2 | 6 | 2014 | 20. 所谓颅内巨大动脉瘤是指直径大于(2.5分) | A.1cm B.1.5cm C.2cm D.2.5cm E.3cm |
D.2.5cm |
2 | 6 | 2014 | 21. 一儿童患者,突发头痛、呕吐伴癫痫,CT示脑实质内出血约10ml,最可能的出血原因是:(2.5分) | A.脑动静脉畸形 B.MOYAMOYA病 C.海绵状血管瘤 D.高血压脑出血 |
B.MOYAMOYA病 |
2 | 6 | 2014 | 22. 下述哪项不是脑血管造影的适应症(2.5分) | A.脑内或蛛网膜下腔出血病因检查 B.怀疑脑静脉病变 C.头面部富血供肿瘤术前检查 D.并发脑疝或其他危及生命的情况。 |
D.并发脑疝或其他危及生命的情况。 |
2 | 6 | 2014 | 23. 下述哪项属于颈动脉支架植入术的适应症(2.5分) | A.无症状颈动脉颈动脉粥样硬化伴管腔狭窄60% B.无症状颈动脉粥样硬化伴管腔狭窄75% C.症状性颈动脉颈动脉粥样硬化伴管腔狭窄49%合并溃疡性斑块 D.脑梗后发现同侧颈内动脉夹层伴狭窄80%,目前无脑缺血症状。 |
B.无症状颈动脉粥样硬化伴管腔狭窄75% |
2 | 6 | 2014 | 24. 目前通用的颅内动脉狭窄率的测量方法是WASID法,它是100%减去下列哪个数值后的结果:(2.5分) | A.病变血管最窄处残留管腔直径除以病变近端相对正常血管直径所得到的百分数 B.病变血管最窄处残留管腔直径除以病变远端端相对正常血管直径所得到的百分数。 C.病变血管最窄处残留管腔直径除以同一部位血管原有直径所得到的百分数 D.病变血管最窄处残留管腔直径除以对侧同一部位血管直径所得到的百分数 |
A.病变血管最窄处残留管腔直径除以病变近端相对正常血管直径所得到的百分数 |
2 | 6 | 2014 | 25. 以下哪支动脉不属于颈内动脉的分支: (2.5分) | A.眼动脉 B.脑膜中动脉 C.后交通动脉 D.脑膜垂体干 E.脉络膜前动脉。 |
B.脑膜中动脉 |
2 | 6 | 2014 | 26. 阅读CT平扫图像时,下列哪种窗宽和窗位设置有助于发现早期脑梗塞(2.5分) | A.窗宽调宽,窗位调低 B.窗宽调宽,窗位调高 C.窗宽调窄,窗位调高 D.窗宽调窄,窗位调低 E.以上都可以 |
D.窗宽调窄,窗位调低 |
2 | 6 | 2014 | 27. CT脑灌注成像时,计算时间-密度增强曲线下面积得到的血流参数是(2.5分) | A.CBF B.CBV C.MTT D.TTD E.TTP |
B.CBV |
2 | 6 | 2014 | 28. 脑静脉窦血栓形成的间接征象是(2.5分) | A.CT平扫出现高密度血管征 B.磁共振T1加权静脉窦呈高信号 C.磁共振T2加权静脉窦血液流空信号消失 D.增强扫描静脉窦呈现空三角征或δ征 E.静脉性脑梗死和出血 |
E.静脉性脑梗死和出血 |
2 | 6 | 2014 | 29. 下列哪项不是动脉瘤破裂的临床表现?(2.5分) | A.头痛 B.高热 C.癫痫 D.意识障碍 E.呕吐 |
B.高热 |
2 | 6 | 2014 | 30. 一儿童患者,突发头痛、呕吐伴癫痫,CT示脑实质内出血约10ml,最可能的出血原因是:(2.5分) | A.脑动静脉畸形 B.MOYAMOYA病 C.海绵状血管瘤 D.高血压脑出血 |
B.MOYAMOYA病 |
2 | 6 | 2014 | 31. 脑动静脉畸形的主要临床表现有(2.5分) | A.头痛 B.癫痫 C.血肿慢慢增大 D.突发出血 E.呕吐 |
C.血肿慢慢增大 |
2 | 6 | 2014 | 32. 有关烟雾病的描述哪项不对?(2.5分) | A.颈内动脉颅内起始段闭塞 B.其发生与动脉硬化无关 C.脑底出现异常血管网 D.成人易发生脑缺血 E.出血与微小动脉瘤破裂有关 |
D.成人易发生脑缺血 |
2 | 6 | 2014 | 33. 不引起生理性瞳孔缩小的是(2.5分) | A.直接对光反射 B.近反射 C.调节反射 D.间接对光反射 E.惊吓反应 |
E.惊吓反应 |
2 | 6 | 2014 | 34. 暗适应的实质是(2.5分) | A.视锥细胞兴奋性恢复的过程 B.色觉障碍恢复的过程 C.视紫红质浓度恢复的过程 D.瞳孔对光反射调节的过程 E.辐辏反射调节的过程 |
C.视紫红质浓度恢复的过程 |
2 | 6 | 2014 | 35. 下列关于视觉通路的描述,正确的是(2.5分) | A.右眼颞侧视网膜的传入纤维投射至左枕叶皮层 B.左眼鼻侧视网膜的传入纤维投射至右枕叶皮层 C.视网膜上半部的传入纤维投射到距状裂下缘 D.视网膜下半部的传入纤维投射到距状裂上缘 E.视网膜中央黄斑区的传入纤维投射到距状裂前部 |
B.左眼鼻侧视网膜的传入纤维投射至右枕叶皮层 |
2 | 6 | 2014 | 36. 瞳孔在弱光下散大,而在强光下缩小,称为 (2.5分) | A.互感性对光反射 B.辐辏反射 C.瞳孔对光反射 D.瞳孔调节反射 E.瞳孔近反射 |
C.瞳孔对光反射 |
2 | 6 | 2014 | 37. 瞳孔对光反射中枢在(2.5分) | A.大脑皮质 B.下丘脑 C.中脑 D.脑桥 E.延脑 |
C.中脑 |
2 | 6 | 2014 | 38. 瞳孔近反射的作用是(2.5分) | A.增加眼的折光能力 B.可增加球面像差 C.增加入眼的光量 D.增加视觉的清晰度 E.增加色像差 |
D.增加视觉的清晰度 |
2 | 6 | 2014 | 39. 关于视锥细胞的叙述,错误的是(2.5分) | A.其感光色素为视紫红质 B.主要分布在视网膜的中央部分 C.对光的敏感度较差 D.能分辨颜色 E.对物体有高分辨能力 |
A.其感光色素为视紫红质 |
2 | 6 | 2014 | 40. 维生素A严重缺乏,可影响人(2.5分) | A.在明处的视力 B.色觉 C.在暗处的视力 D.立体视觉 E.视野范围 |
C.在暗处的视力 |
2 | 6 | 2015 | 1. 以下不属于心理评估常用方法的是(2.5分) | A.会谈法 B.实验法 C.观察法 D.调查法 |
B.实验法 |
2 | 6 | 2015 | 2. 以下不属于效度检验方法的是: (2.5分) | A.结构效度 B.复本效度 C.内容效度 D.效标效度 |
B.复本效度 |
2 | 6 | 2015 | 3. 信度是指测验分数的: (2.5分) | A.可靠性 B.灵敏性 C.有效性 D.精确性 |
A.可靠性 |
2 | 6 | 2015 | 4. 卡特尔人格因素测验将人格根源特质分为多少种: (2.5分) | A.8种 B.12种 C.16种 D.18种 |
C.16种 |
2 | 6 | 2015 | 5. 以下不属于人格测验的一项是: (2.5分) | A.16PF B.SDS C.MMPI D.EPQ |
B.SDS |
2 | 6 | 2015 | 6. 行为治疗的目的是: (2.5分) | A.纠正不适应的认知模式 B.改变不适应的行为模式 C.重现童年期的心理冲突 D.发挥个人的潜能 |
B.改变不适应的行为模式 |
2 | 6 | 2015 | 7. 系统脱敏疗法的步骤不包括: (2.5分) | A.放松训练 B.列出等级脱敏表 C.暴露于强烈的焦虑环境中 D.放松状态下脱敏 |
C.暴露于强烈的焦虑环境中 |
2 | 6 | 2015 | 8. 心理治疗的基本过程有: (2.5分) | A.探索问题、心理测验、治疗行动、疗效评价和结束巩固 B.建立关系、分析认识、治疗行动、疗效评价和结束巩固 C.建立关系、探索问题、心理测验、治疗行动和疗效评价 D.探索问题、分析认识、治疗行动、疗效评价和结束巩固 |
D.探索问题、分析认识、治疗行动、疗效评价和结束巩固 |
2 | 6 | 2015 | 9. 来访者中心疗法不主张: (2.5分) | A.发展来访者主观能动性 B.对来访者明确指导 C.促进来访者成长 D.医生与来访者是平等"角色" |
B.对来访者明确指导 |
2 | 6 | 2015 | 10. 森田疗法的绝对卧床阶段时间一般为: (2.5分) | A.三天 B.五天 C.七天 D.十天 |
C.七天 |
2 | 6 | 2015 | 11. 支持疗法的理论基础中, 不包括: (2.5分) | A.认知调整 B.善用"资源" C.提高应对能力 D.挖掘幼年体验 |
D.挖掘幼年体验 |
2 | 6 | 2015 | 12. 女,50岁,突发头痛5天,头痛位于整个头部,呈炸裂样,伴恶心,无关节疼痛,无发热,体格检查:颈抵抗,克氏征阳性。头颅CT示鞍上池高密度影。最可能的诊断是: (2.5分) | A.颅内感染 B.紧张性头痛 C.急性脑梗塞 D.蛛网膜下腔出血 E.短暂性脑缺血发作 |
D.蛛网膜下腔出血 |
2 | 6 | 2015 | 13. 男,50岁,与人吵架后出现头痛,为炸裂样痛,伴恶心呕吐,体格检查示神清,右侧肢体肌力3级,右侧病理征阳性,余神经系统体格检查无殊。为明确诊断,首选辅助检查为: (2.5分) | A.肌电图 B.脑电图 C.头颅CT平扫 D.经颅多普勒 E.头颅MR灌注 |
C.头颅CT平扫 |
2 | 6 | 2015 | 14. 男,65岁,突发右侧肢体无力三小时,无发热,查体:神清,右侧中枢性面瘫,右侧肢体肌力4级,右侧偏身感觉减退,查头颅CT未见明显异常,既往2型糖尿病史5年。病变责任血管可能是 (2.5分) | A.左侧大脑中动脉 B.右侧大脑中动脉 C.右侧颈内动脉 D.基底动脉 E.右侧大脑后动脉 |
B.右侧大脑中动脉 |
2 | 6 | 2015 | 15. 下列哪项是引起脑梗塞的最常见原因:(2.5分) | A.房颤 B.空气栓塞 C.脑动脉粥样硬化 D.动脉炎 E.血小板增多症 |
C.脑动脉粥样硬化 |
2 | 6 | 2015 | 16. 男,60岁。突发口齿不清伴左侧肢体无力5小时。查体:右侧周围性面瘫,左侧肢体肌力4级,右侧肢体肌力5级。考虑患者病变部位位于(2.5分) | A.脑皮质 B.脑干 C.内囊 D.脊髓 E.周围神经 |
B.脑干 |
2 | 6 | 2015 | 17. 患者,男,右侧肢体无力2小时。既往高血压与糖尿病史。查体:神清,右侧鼻唇沟浅,口齿含糊,右侧上下肢肌力2级,右侧偏身感觉减退,右侧巴氏征阳性。头颅CT阴性,心电图、血常规、凝血全套均正常。首选的治疗方案是: (2.5分) | A.抗血小板治疗 B.抗凝治疗 C.静脉溶栓 D.降纤治疗 E.神经保护治疗 |
C.静脉溶栓 |
2 | 6 | 2015 | 18. 所谓颅内巨大动脉瘤是指直径大于(2.5分) | A.1cm B.1.5cm C.2cm D.2.5cm E.3cm |
D.2.5cm |
2 | 6 | 2015 | 19. 一儿童患者,突发头痛、呕吐伴癫痫,CT示脑实质内出血约10ml,最可能的出血原因是: (2.5分) | A.动脉瘤 B.脑动静脉畸形 C.MOYAMOYA病 D.海绵状血管瘤 E.高血压脑出血 |
B.脑动静脉畸形 |
2 | 6 | 2015 | 20. 下列哪项不属于颅内压升高的临床分期:(2.5分) | A.早期:超过代偿容积,出现临床症状 B.进展期:内容物持续增加,临床症状加重 C.衰竭期:脑组织几无血流灌注 D.高峰期: 典型颅高压三联症 E.代偿期 :内容物虽有增加,未超过代偿容积 |
B.进展期:内容物持续增加,临床症状加重 |
2 | 6 | 2015 | 21. 脑疝患者的紧急治疗手段不包括:(2.5分) | A.维持呼吸道畅通 B.立即静脉滴注20%甘露醇250-500 ml C.侧脑室穿刺外引流 D.减压手术如去骨瓣减压,颞肌下减压,内减压术等 E.腰穿引流脑脊液 |
E.腰穿引流脑脊液 |
2 | 6 | 2015 | 22. 颅内压增高的常见临床表现不包括:(2.5分) | A.清晨易发生头痛, B.头痛伴喷射性呕吐 C.双眼视物模糊,结膜肿胀 D.用力,咳嗽,大便,弯腰时头痛加重 E.双侧额颞部弥漫性胀痛 |
C.双眼视物模糊,结膜肿胀 |
2 | 6 | 2015 | 23. 下述哪项不是脑血管造影的适应症(2.5分) | A.脑内或蛛网膜下腔出血病因检查 B.怀疑脑静脉病变 C.头面部富血供肿瘤术前检查 D.并发脑疝或其他危及生命的情况。 |
D.并发脑疝或其他危及生命的情况。 |
2 | 6 | 2015 | 24. 颅内动脉粥样硬化狭窄支架植入术的适应症,目前推荐意见: (2.5分) | A.体检发现颅内动脉粥样硬化狭窄超过70%,目前无症状 B.第一次脑梗死后检查发现同侧颅内动脉粥样硬化狭窄超过70-99% C.第一次脑梗死后检查发现同侧颅内动脉粥样硬化狭窄超过70-99%,强化药物治疗无效或脑侧支循环代偿不良,责任血管供血区存在低灌注的患者, D.颅内动脉完全闭塞。 |
C.第一次脑梗死后检查发现同侧颅内动脉粥样硬化狭窄超过70-99%,强化药物治疗无效或脑侧支循环代偿不良,责任血管供血区存在低灌注的患者, |
2 | 6 | 2015 | 25. 下述哪项属于颈动脉支架植入术的适应症(2.5分) | A.无症状颈动脉颈动脉粥样硬化伴管腔狭窄60% B.无症状颈动脉粥样硬化伴管腔狭窄75% C.症状性颈动脉颈动脉粥样硬化伴管腔狭窄49%合并溃疡性斑块 D.脑梗后发现同侧颈内动脉夹层伴狭窄80%,目前无脑缺血症状。 |
B.无症状颈动脉粥样硬化伴管腔狭窄75% |
2 | 6 | 2015 | 26. 以下不属于脑Willis环的结构: (2.5分) | A.颈内动脉 B.大脑后动脉 C.大脑前动脉 D.大脑中动脉 E.后交通动脉 |
D.大脑中动脉 |
2 | 6 | 2015 | 27. 目前通用的颅内动脉狭窄率的测量方法是WASID法,它是100%减去下列哪个数值后的结果: (2.5分) | A.病变血管最窄处残留管腔直径除以病变近端相对正常血管直径所得到的百分数 B.病变血管最窄处残留管腔直径除以病变远端端相对正常血管直径所得到的百分数。 C.病变血管最窄处残留管腔直径除以同一部位血管原有直径所得到的百分数 D.病变血管最窄处残留管腔直径除以对侧同一部位血管直径所得到的百分数 |
A.病变血管最窄处残留管腔直径除以病变近端相对正常血管直径所得到的百分数 |
2 | 6 | 2015 | 28. 下列关于急性缺血性脑卒中患者的治疗,正确的是: (2.5分) | A.发病6h内的患者,无需影像学评估可直接溶栓或采取动脉内治疗 B.发病6h内的患者,明确存在大血管闭塞,必须经灌注成像评估组织状况才能溶栓或采取动脉内治疗 C.发病6h以上的患者不能采用动脉内治疗 D.发病6h以上的患者可以采用灌注成像帮助筛选适合进行动脉内治疗的患者 E.发病时间不明的患者不能采用动脉内治疗 |
D.发病6h以上的患者可以采用灌注成像帮助筛选适合进行动脉内治疗的患者 |
2 | 6 | 2015 | 29. 怀疑急性缺血性脑卒中的患者行弥散加权成像(DWI)检查,下列哪项说法是正确的 (2.5分) | A.急性脑梗死区在DWI上呈高信号 B.急性脑梗死区的ADC值增高 C.DWI高信号区都会转变成软化灶 D.DWI高信号区都代表急性梗死区 E.DWI高信号表示组织含水量增加 |
A.急性脑梗死区在DWI上呈高信号 |
2 | 6 | 2015 | 30. CT脑灌注成像时,血流参数MTT代表的含义是: (2.5分) | A.灌注曲线下的面积 B.灌注曲线的峰值高度 C.灌注曲线从开始到结束的时间 D.灌注曲线从开始到峰值的时间 E.灌注曲线的半高宽 |
D.灌注曲线从开始到峰值的时间 |
2 | 6 | 2015 | 31. 脑静脉窦血栓形成的直接征象是: (2.5分) | A.CT平扫出现岛带征 B.CT平扫出现豆状核模糊征 C.磁共振T2加权静脉窦内出现流空信号 D.静脉引流区域出现脑梗死和出血 E.CT增强扫描静脉窦内出现δ征或空三角征 |
E.CT增强扫描静脉窦内出现δ征或空三角征 |
2 | 6 | 2015 | 32. 不引起生理性瞳孔缩小的是(2.5分) | A.直接对光反射 B.近反射 C.调节反射 D.间接对光反射 E.惊吓反应 |
E.惊吓反应 |
2 | 6 | 2015 | 33. 眼的折光系统中折光率最大的是 (2.5分) | A.晶状体 B.角膜 C.玻璃体 D.前房水 E.后房水 |
B.角膜 |
2 | 6 | 2015 | 34. 按色觉三原色学说,三种视锥细胞分别敏感的颜色是(2.5分) | A.红、蓝、紫 B.红、黄、黑 C.绿、蓝、白 D.红、绿、蓝 E.红、白、青 |
D.红、绿、蓝 |
2 | 6 | 2015 | 35. 看近物时的视觉调节过程包括 (2.5分) | A.晶状体变凸,眼轴会聚,瞳孔散大 B.晶状体变凸,眼轴会聚,瞳孔缩小 C.晶状体扁平,眼轴会聚,瞳孔缩小 D.晶状体扁平,眼轴会聚,瞳孔散大 E.晶状体曲率半径减小,眼轴会聚,瞳孔缩小 |
B.晶状体变凸,眼轴会聚,瞳孔缩小 |
2 | 6 | 2015 | 36. 人眼的调节功能主要是由以下哪种变化来实现的? (2.5分) | A.角膜的曲度变化 B.角膜和晶状体的曲度变化 C.晶状体的曲度变化 D.玻璃体的曲度变化 E.晶状体和玻璃体的曲度变化 |
C.晶状体的曲度变化 |
2 | 6 | 2015 | 37. 瞳孔对光反射中枢在 (2.5分) | A.大脑皮质 B.下丘脑 C.中脑 D.脑桥 E.延脑 |
C.中脑 |
2 | 6 | 2015 | 38. 关于瞳孔反射的说明,正确的是(2.5分) | A.视近物时瞳孔扩大 B.瞳孔近反射与对光反射的通路相同 C.动眼神经损伤使瞳孔缩小 D.交感神经兴奋使瞳孔扩大 E.副交感神经兴奋使瞳孔扩大 |
D.交感神经兴奋使瞳孔扩大 |
2 | 6 | 2015 | 39. 黄斑的中央凹处视敏度最高,是因为 (2.5分) | A.视杆细胞多而集中,单线联系 B.视锥细胞多而集中,单线联系 C.视杆细胞多而集中,聚合联系 D.视杆细胞,视锥细胞都多,单线联系 E.视杆细胞多而集中,辐散式联系 |
B.视锥细胞多而集中,单线联系 |
2 | 6 | 2015 | 40. 关于视锥细胞的叙述,错误的是 (2.5分) | A.其感光色素为视紫红质 B.主要分布在视网膜的中央部分 C.对光的敏感度较差 D.能分辨颜色 E.对物体有高分辨能力 |
A.其感光色素为视紫红质 |
2 | 7 | 2007 | 1. 传导慢痛的外周神经纤维主要是(3.0分) | A.B 类纤维 B.C 类纤维 C.Aδ纤维 D.Aα纤维 E.Aγ纤维 |
B.C 类纤维 |
2 | 7 | 2007 | 2. 下列哪些病人不合适做针电极肌电图(3.0分) | A.乙肝患者 B.肌无力患者 C.肌营养不良患者 D.血友病患者 E.冠心病患者 |
D.血友病患者 |
2 | 7 | 2007 | 3. 疑似心绞痛心肌梗死的病人进入急诊室,除了给予吸氧、含硝酸甘油,阿司匹林外,还可以静脉注射的镇痛药是: (3.0分) | A.哌替啶 B.帕瑞昔布 C.吗啡 D.布比卡因 |
C.吗啡 |
2 | 7 | 2007 | 4. 行为治疗的目的是: (3.0分) | A.纠正不适应的认知模式 B.重现童年期的心理冲突 C.改变不适应的行为模式 D.发挥个人的潜能 |
C.改变不适应的行为模式 |
2 | 7 | 2007 | 5. 慢性关节炎疼痛病人,长期服用布洛芬制剂,最可能发生的药物不良反应是: (3.0分) | A.向心性肥胖 B.白细胞减少 C.药物成瘾 D.胃溃疡 |
D.胃溃疡 |
2 | 7 | 2007 | 6. 塞来昔布的镇痛作用机制是: (3.0分) | A.抑制COX-1 B.抑制神经动作电位传递 C.抑制PGs合成 D.以上都不是 |
C.抑制PGs合成 |
2 | 7 | 2007 | 7. 局麻药中枢毒性反应随中毒程度不同有不同表现。下列哪个症状是中毒症状中最轻的: (3.0分) | A.口唇麻木 B.局部肌肉抽动 C.耳鸣 D.眼球震颤翻白 |
A.口唇麻木 |
2 | 7 | 2007 | 8. 内脏痛的主要特点是(4.0分) | A.刺痛 B.慢痛 C.必有牵涉痛 D.定位不精确 E.对牵拉不敏感 |
D.定位不精确 |
2 | 7 | 2007 | 9. 不属于症状评定量表的一项是: (3.0分) | A.SCL-90 B.SDS C.MMPI D.SAS |
C.MMPI |
2 | 7 | 2007 | 10. 慢性骨骼、软组织疼痛的最常见病因: (3.0分) | A.炎症 B.退行性 C.代谢性 D.外伤性 E.感染性 |
B.退行性 |
2 | 7 | 2007 | 11. 影响疼痛的心理社会因素不包括: (3.0分) | A.人格因素 B.宗教信仰 C.情绪因素 D.经济因素 |
D.经济因素 |
2 | 7 | 2007 | 12. 以下哪项正确: (3.0分) | A.普鲁卡因是酰胺类局麻药,剂量限制为1000mg B.布比卡因是长效局麻药,剂量限制为40mg C.利多卡因可维持2小时左右的局麻效果,剂量限制为400mg D.罗哌卡因可维持4小时以上的局麻效果,使用前先进行皮试 |
C.利多卡因可维持2小时左右的局麻效果,剂量限制为400mg |
2 | 7 | 2007 | 13. 癌症病人听到癌症的诊断后,出现心理反应的顺序是: (3.0分) | A.休克-恐惧期,愤怒-沮丧期,接受-适应期,否认-怀疑期 B.否认-怀疑期,休克-恐惧期,愤怒-沮丧期,接受-适应期 C.否认-怀疑期,愤怒-沮丧期,接受-适应期,休克-恐惧期 D.休克-恐惧期,否认-怀疑期,愤怒-沮丧期,接受-适应期 |
D.休克-恐惧期,否认-怀疑期,愤怒-沮丧期,接受-适应期 |
2 | 7 | 2007 | 14. 心理治疗的基本过程有: (3.0分) | A.建立关系、分析认识、治疗行动、疗效评价和结束巩固 B.探索问题、心理测验、治疗行动、疗效评价和结束巩固 C.建立关系、探索问题、心理测验、治疗行动和疗效评价 D.探索问题、分析认识、治疗行动、疗效评价和结束巩固 |
D.探索问题、分析认识、治疗行动、疗效评价和结束巩固 |
2 | 7 | 2007 | 15. 感受慢痛的痛觉感受器主要为(3.0分) | A.多觉伤害感受器 B.机械伤害感受器 C.低阈值感受器 D.温度伤害感受器 E.高阈值感受器 |
A.多觉伤害感受器 |
2 | 7 | 2007 | 16. 效度是指测验分数的:(3.0分) | A.可靠性 B.有效性 C.一致性 D.灵敏性 |
B.有效性 |
2 | 7 | 2007 | 17. 骨肿瘤的分类是以()分类.(3.0分) | A.间室 B.解剖部位 C.组织来源 D.遗传来源 E.影像学 |
C.组织来源 |
2 | 7 | 2007 | 18. 卡特尔人格因素测验将人格根源特质分为多少种: (3.0分) | A.16种 B.12种 C.8种 D.18种 |
A.16种 |
2 | 7 | 2007 | 19. 有助于帮助诊断重症肌无力的肌电图方法是(3.0分) | A.H 反射测定 B.F 波测定 C.运动神经测定 D.低频重复电刺激 E.高频重复电刺激 |
D.低频重复电刺激 |
2 | 7 | 2007 | 20. 对标准化心理测验应具备条件而言,下列哪项是错误的: (3.0分) | A.随机使用的指导语 B.效度、信度 C.常模、固定测试条件 D.统一的计分标准 |
A.随机使用的指导语 |
2 | 7 | 2007 | 21. 右美托咪定有镇痛作用,其作用的受体是: (3.0分) | A.μ B.δ C.к D.α2 |
D.α2 |
2 | 7 | 2007 | 22. 来访者中心疗法不主张: (3.0分) | A.促进来访者成长 B.对来访者明确指导 C.发展来访者主观能动性 D.医生与来访者是平等"角色" |
B.对来访者明确指导 |
2 | 7 | 2007 | 23. 系统脱敏疗法的步骤不包括: (3.0分) | A.放松训练 B.暴露于强烈的焦虑环境中 C.列出等级脱敏表 D.放松状态下脱敏 |
B.暴露于强烈的焦虑环境中 |
2 | 7 | 2007 | 24. 离差智商适用于: (3.0分) | A.20岁以上成人 B.18岁以上成人 C.16岁以上成人 D.任何年龄 |
D.任何年龄 |
2 | 7 | 2007 | 25. 下列哪项正确:(3.0分) | A.用吗啡后引起恶心呕吐 B.用哌替啶后引起尿失禁 C.纳洛酮有镇咳作用 D.用曲马多后有欣快感 |
A.用吗啡后引起恶心呕吐 |
2 | 7 | 2007 | 26. 下列哪项不属于标准分: (3.0分) | A.Z分 B.标准20 C.T分 D.划界分 |
D.划界分 |
2 | 7 | 2007 | 27. 用较大剂量后会出现瞳孔缩小的药物: (3.0分) | A.曲马多 B.吗啡 C.哌替啶 D.萘普生 |
B.吗啡 |
2 | 7 | 2007 | 28. 说明心理社会因素"如何"通过神经内分泌和免疫系统造成躯体疾病的研究领域称为: (3.0分) | A.心理免疫学 B.神经免疫学 C.免疫学 D.心理神经免疫学 |
D.心理神经免疫学 |
2 | 7 | 2007 | 29. 强调潜意识心理冲突、个体心理特征产生心身疾病的研究途径是: (3.0分) | A.心理动力理论 B.学习理论 C.心理生理学理论 D.人本主义理论 |
A.心理动力理论 |
2 | 7 | 2007 | 30. 失眠的心理社会因素不包括: (3.0分) | A.不良自我暗示 B.对健康过度关心 C.过度疲劳或紧张 D.入睡前饮用兴奋性饮料 |
D.入睡前饮用兴奋性饮料 |
2 | 7 | 2007 | 31. 支持疗法的理论基础中, 不包括: (3.0分) | A.提高应对能力 B.善用"资源" C.认知调整 D.挖掘幼年体验 |
D.挖掘幼年体验 |
2 | 7 | 2007 | 32. 骨折愈合时,通常由下列哪种细胞诱导成骨作用,促进愈合。(3.0分) | A.骨髓间充质干细胞 B.成纤维干细胞 C.成骨细胞 D.成软骨细胞 |
A.骨髓间充质干细胞 |
2 | 7 | 2007 | 33. 儿童骨折的主要方式为:(3.0分) | A.凹陷性骨折 B.压缩性骨折 C.粉碎性骨折 D.青枝骨折 |
D.青枝骨折 |
2 | 7 | 2008 | 1. 内脏痛的主要特点是 (4.0分) | A.刺痛 B.慢痛 C.定位不精确 D.必有牵涉痛 E.对牵拉不敏感 |
C.定位不精确 |
2 | 7 | 2008 | 2. 牵涉痛是指 (4.0分) | A.内脏疾病引起相邻脏器的疼痛 B.手术牵拉脏器引起的疼痛 C.神经疼痛向体表投射 D.按压体表引起部分内脏疼痛 E.内脏疾病引起体表某一部位的疼痛或痛觉过敏 |
E.内脏疾病引起体表某一部位的疼痛或痛觉过敏 |
2 | 7 | 2008 | 3. 传导慢痛的外周神经纤维主要是 (4.0分) | A.Aα纤维 B.Aδ纤维 C.B 类纤维 D.C 类纤维 E.Aγ纤维 |
D.C 类纤维 |
2 | 7 | 2008 | 4. 以会聚学说和易化学说解释牵涉痛的主要依据是 (4.0分) | A.病区与放射部位源于相同胚胎节段和皮节 B.体表局部麻醉不能取消剧烈的牵涉痛 C.体表局部麻醉可消除轻微的牵涉痛 D.脊髓后角存在调控痛觉传入的"闸门" E.感觉传入通路中存在辐散式和聚合式联系 |
A.病区与放射部位源于相同胚胎节段和皮节 |
2 | 7 | 2008 | 5. 感受慢痛的痛觉感受器主要为 (4.0分) | A.机械伤害感受器 B.温度伤害感受器 C.多觉伤害感受器 D.低阈值感受器 E.高阈值感受器 |
C.多觉伤害感受器 |
2 | 7 | 2008 | 6. 闸门学说中调制痛觉的关键细胞位于 (4.0分) | A.背角I 层 B.背角II 层 C.背角III 层 D.背角IV 层 E.背角V 层 |
B.背角II 层 |
2 | 7 | 2008 | 7. 传导快痛的外周神经纤维主要是 (4.0分) | A.Aα纤维 B.Aδ纤维 C.B 类纤维 D.C 类纤维 E.Aγ纤维 |
B.Aδ纤维 |
2 | 7 | 2008 | 8. 闸门学说中抑制SG细胞的是 (4.0分) | A.Aα纤维 B.Aδ纤维 C.B 类纤维 D.C 类纤维 E.Aβ纤维 |
D.C 类纤维 |
2 | 7 | 2008 | 9. 下列哪种药物不是通过阿片受体发挥镇痛作用:(4.0分) | A.吗啡 B.芬太尼 C.瑞芬太尼 D.普鲁卡因 E.美沙酮 |
D.普鲁卡因 |
2 | 7 | 2008 | 10. 下列哪项正确: (4.0分) | A.术中输注瑞芬太尼的病人停药后,出现疼痛、镇痛药物需要量增加的原因与成瘾性有关 B.芬太尼对心血管的影响很大 C.小剂量的阿片类药物即可具有较好的神经病理疼痛治疗效果 D.单用阿片类药物维持镇痛镇静,与心脏手术术中知晓无关 E.芬太尼贴片对术后切口痛无效 |
C.小剂量的阿片类药物即可具有较好的神经病理疼痛治疗效果 |
2 | 7 | 2008 | 11. 对待晚期癌痛病人的疼痛治疗(4.0分) | A.早期就要用强效阿片药物 B.个体化用药 C.阿片药物不会出现恶心呕吐,故治疗中无肠梗阻病人不需要给予止吐药物 D.强啡肽是近年来人工合成的阿片受体激动剂 E.强啡肽是近年来人工合成的阿片受体拮抗剂 |
B.个体化用药 |
2 | 7 | 2008 | 12. 下列哪种药物是阿片受体的拮抗剂,能逆转芬太尼等过量导致的呼吸抑制: (4.0分) | A.吗啡 B.纳洛酮 C.瑞芬太尼 D.普鲁卡因 E.美沙酮 |
B.纳洛酮 |
2 | 7 | 2008 | 13. 下列哪项不正确:(4.0分) | A.阿片受体是G蛋白偶联受体 B.阿片受体能传导跨膜信号,激活MAPK,抑制腺苷酸环化酶等 C.阿片受体活化后,N型钙通道受到抑制,钾通道开放 D.丁丙诺非是μ受体部分激动剂 E.阿片受体部分激动-拮抗剂不会引起呼吸抑制,故临床上低氧血症发生几无。 |
E.阿片受体部分激动-拮抗剂不会引起呼吸抑制,故临床上低氧血症发生几无。 |
2 | 7 | 2008 | 14. 关于阿片类药物的叙述哪项不正确: (4.0分) | A.除杜冷丁外,均有心血管稳定作用,剂量过大,将bradycardia, 需要备用阿托品。 B.芬太尼、舒芬太尼等作用于孤束核和臂旁核控制血管紧张素分泌、稳定血流动力学 C.芬太尼、舒芬太尼等作用于下丘脑-垂体-肾上腺,应激反应,降低交感张力 D.细胞、动物、临床均表明芬太尼、舒芬太尼等具有心机保护作用 E.芬太尼因其心肌的负性肌力作用大,不推荐在心脏手术中使用 |
D.细胞、动物、临床均表明芬太尼、舒芬太尼等具有心机保护作用 |
2 | 7 | 2008 | 15. 地佐辛联合芬太尼使用(4.0分) | A.可减少芬太尼用量 B.术中芬太尼用量不变 C.镇痛作用减弱 D.镇痛作用增强不明显,且部分拮抗 E.不良反应发生率明显增加,尤其是呼吸抑制 |
A.可减少芬太尼用量 |
2 | 7 | 2008 | 16. 下列哪项关于NSAID药物叙述是错误的: (4.0分) | A.NSAID为非甾体抗炎药(Nonsteroidal Antiinflammatory Drugs,NSAIDs)是一类不含有甾体结构的抗炎药 B.NSAIDs通过干预COX酶活性、阻断花生四烯酸的代谢、抑制前列腺素的合成,发挥其解热、镇痛、消炎作用 C.新近的NSAID以选择性作用于COX2为主 D.WHO癌症疼痛三阶梯治疗轻度疼痛:首选第一阶梯非甾体类抗炎药,以阿司匹林为代表 E.个体化给药仅仅指阿片类药,不包括NSAID |
E.个体化给药仅仅指阿片类药,不包括NSAID |
2 | 7 | 2008 | 17. 局麻药的叙述哪项不正确:(4.0分) | A.目前临床的局麻药分为酯类和酰胺类 B.典型的局麻药分子结构的中间链可为酯链或酰胺链 C.足够浓度的局麻药可以阻断相应部位的神经肌肉膜内电冲动的传导 D.全身应用局麻药不会影响心肌、平滑肌等功能,但是,对外周和中枢神经系统及心脏特殊传导系统的冲动传递有作用。 E.局麻药中毒反应中以中枢和心血管系统受累为主 |
C.足够浓度的局麻药可以阻断相应部位的神经肌肉膜内电冲动的传导 |
2 | 7 | 2008 | 18. 下列哪项错误,利多卡因的主要麻醉用途: (4.0分) | A.0.5%-1%局部浸润麻醉 B.5-10%局部浸润麻醉 C.1-2%外周神经阻滞 D.1-2%硬膜外麻醉 E.胶或软膏作表面麻醉 |
B.5-10%局部浸润麻醉 |
2 | 7 | 2008 | 19. 关于常用局麻药布比卡因,哪项叙述不正确: (4.0分) | A.布比卡因是长效麻醉药 B.属于酯类局麻药 C.麻醉效能与其脂溶性密切相关 D.布比卡因PKa值为8.1 E.蛋白结合率高 |
B.属于酯类局麻药 |
2 | 7 | 2008 | 20. 局麻药作用机制,哪项叙述不正确:(4.0分) | A.能渗透过神经轴突膜 B.起效速度与PKa、离子化有关 C.作用持续时间与蛋白结合率有关 D.麻醉效能与其脂溶性密切相关 E.它是通过与Ca离子通道结合发挥神经阻滞作用 |
E.它是通过与Ca离子通道结合发挥神经阻滞作用 |
2 | 7 | 2008 | 21. 局麻药的毒性作用,哪项叙述不正确: (4.0分) | A.局麻药的主要全身中毒反应以中枢和心血管系统受累为主 B.缺氧和酸中 毒不影响其毒性反应 C.布比卡因过量后的复苏与其它局麻药一样容易 D.给药时回抽非常重要再粗大神经阻滞时,要注意分次给药 E.目前临床局麻药物对神经还是有直接毒性作用的 |
B.缺氧和酸中 毒不影响其毒性反应 |
2 | 7 | 2008 | 22. 关于解热抗炎镇痛药物的特点错误的是(4.0分) | A.属于非甾体类抗炎药物 B.大多数药物都有解热、镇痛、抗炎作用 C.对锐痛无效,对炎性所致钝痛疗效较好 D.属于对症治疗 E.可用于低温麻醉 |
E.可用于低温麻醉 |
2 | 7 | 2008 | 23. 下列药物中哪一种药物对风湿性关节炎几无疗效?(4.0分) | A.Aspirin B.Sulindac(舒林酸) C.Acetaminophen D.Indomethacin(吲哚美辛) E.Ibuprofen(布洛芬) |
C.Acetaminophen |
2 | 7 | 2008 | 24. Aspirin的不良反应不包括 (4.0分) | A.诱发胃、十二指肠溃疡出血 B.粒细胞减少 C.诱发哮喘 D.高铁血红蛋白血症 E.凝血障碍 |
D.高铁血红蛋白血症 |
2 | 7 | 2008 | 25. 对类风湿性关节炎,非甾体抗炎药(NSAIDs)的疗效是(4.0分) | A.对因治疗 B.缩短病程 C.对症治疗 D.阻止肉芽增生及瘢痕形成 E.对风湿性心肌炎可根治 |
C.对症治疗 |
2 | 7 | 2009 | 1. 内脏痛的主要特点是 (2.0分) | A.刺痛 B.慢痛 C.定位不精确 D.必有牵涉痛 E.对牵拉不敏感 |
C.定位不精确 |
2 | 7 | 2009 | 2. 牵涉痛是指 (2.0分) | A.内脏疾病引起相邻脏器的疼痛 B.手术牵拉脏器引起的疼痛 C.神经疼痛向体表投射 D.按压体表引起部分内脏疼痛 E.内脏疾病引起体表某一部位的疼痛或痛觉过敏 |
E.内脏疾病引起体表某一部位的疼痛或痛觉过敏 |
2 | 7 | 2009 | 3. 传导慢痛的外周神经纤维主要是 (2.0分) | A.Aα纤维 B.Aδ纤维 C.B 类纤维 D.C 类纤维 E.Aγ纤维 |
D.C 类纤维 |
2 | 7 | 2009 | 4. 以会聚学说和易化学说解释牵涉痛的主要依据是 (2.0分) | A.病区与放射部位源于相同胚胎节段和皮节 B.体表局部麻醉不能取消剧烈的牵涉痛 C.体表局部麻醉可消除轻微的牵涉痛 D.脊髓后角存在调控痛觉传入的"闸门" E.感觉传入通路中存在辐散式和聚合式联系 |
A.病区与放射部位源于相同胚胎节段和皮节 |
2 | 7 | 2009 | 5. 感受慢痛的痛觉感受器主要为 (2.0分) | A.机械伤害感受器 B.温度伤害感受器 C.多觉伤害感受器 D.低阈值感受器 E.高阈值感受器 |
C.多觉伤害感受器 |
2 | 7 | 2009 | 6. 闸门学说中抑制SG细胞的是(2.0分) | A.Aα纤维 B.Aδ纤维 C.B 类纤维 D.C 类纤维 E.Aβ纤维 |
D.C 类纤维 |
2 | 7 | 2009 | 7. 以下不属于心理评估常用方法的是: (2.0分) | A.会谈法 B.实验法 C.观察法 D.调查法 |
B.实验法 |
2 | 7 | 2009 | 8. 下列哪项属于标准分: (2.0分) | A.Z分 B.T分 C.划界分 D.标准20 |
C.划界分 |
2 | 7 | 2009 | 9. 以下不属于效度检验方法的是: (2.0分) | A.结构效度 B.复本效度 C.内容效度 D.效标效度 |
B.复本效度 |
2 | 7 | 2009 | 10. 关于离差智商,下列哪项是错误的: (2.0分) | A.由比奈-西蒙智力量表最早采用 B.其计算公式是:IQ=15(χ- X )/S+100 C.适用于任何年龄 D.是标准分的一种 |
A.由比奈-西蒙智力量表最早采用 |
2 | 7 | 2009 | 11. 信度是指测验分数的(2.0分) | A.可靠性 B.灵敏性 C.有效性 D.精确性 |
A.可靠性 |
2 | 7 | 2009 | 12. 效度是指测验分数的: (2.0分) | A.一致性 B.可靠性 C.有效性 D.灵敏性 |
C.有效性 |
2 | 7 | 2009 | 13. 卡特尔人格因素测验将人格根源特质分为多少种: (2.0分) | A.8种 B.12种 C.16种 D.18种 |
C.16种 |
2 | 7 | 2009 | 14. 关于艾森克人格问卷(EPQ),下列哪项是错误的: (2.0分) | A.包括E、N、P三个分量表 B.E分高反映个性外向 C.N分高反映情绪不稳定,易焦虑 D.P分高反映孤僻、挑衅、不适应环境 |
A.包括E、N、P三个分量表 |
2 | 7 | 2009 | 15. 以下不属于人格测验的一项是: (2.0分) | A.16PF B.SDS C.MMPI D.EPQ |
B.SDS |
2 | 7 | 2009 | 16. 以下何种疾病不属于心身疾病: (2.0分) | A.冠心病 B.胃溃疡 C.精神分裂症 D.癌症 |
C.精神分裂症 |
2 | 7 | 2009 | 17. 强调潜意识心理冲突、个体心理特征产生心身疾病的研究途径是: (2.0分) | A.心理生理学理论 B.学习理论 C.心理动力理论 D.人本主义理论 |
C.心理动力理论 |
2 | 7 | 2009 | 18. 强调心理社会因素通过何种生物学机制引起心身疾病的研究途径是: (2.0分) | A.心理生理学理论 B.学习理论 C.心理动力理论 D.人本主义理论 |
A.心理生理学理论 |
2 | 7 | 2009 | 19. 心身疾病的主要干预方法: (2.0分) | A.消除心理社会刺激因素 B.行为矫正 C.认知干预治疗 D.以上都是 |
D.以上都是 |
2 | 7 | 2009 | 20. 癌症病人听到癌症的诊断后,出现心理反应的顺序是: (2.0分) | A.否认-怀疑期,愤怒-沮丧期,接受-适应期,休克-恐惧期 B.否认-怀疑期,休克-恐惧期,愤怒-沮丧期,接受-适应期 C.休克-恐惧期,愤怒-沮丧期,接受-适应期,否认-怀疑期 D.休克-恐惧期,否认-怀疑期,愤怒-沮丧期,接受-适应期 |
D.休克-恐惧期,否认-怀疑期,愤怒-沮丧期,接受-适应期 |
2 | 7 | 2009 | 21. 下列有关心理社会因素与癌症关系的叙述不正确的是: (2.0分) | A.不良心理反应和应付方式对癌症病人生活质量有严重影响 B.争强好胜、时间紧迫感、脾气暴躁不是癌症病人的行为特点 C.癌症病人常常有意识疏泄自己的情绪来减轻紧张和痛苦 D.突然得知患癌消息的病人情绪反应强烈,易导致免疫力下降 |
C.癌症病人常常有意识疏泄自己的情绪来减轻紧张和痛苦 |
2 | 7 | 2009 | 22. 某病人拒绝医生有关癌症诊断,先后就诊许多医院,来证实自己没有病。该病人使用的心理防御机制是: (2.0分) | A.否认机制 B.压抑机制 C.转移机制 D.反向机制 |
A.否认机制 |
2 | 7 | 2009 | 23. A 型行为的特征是: (2.0分) | A.时间紧迫感 B.竞争性强 C.好斗敌意倾向 D.以上都是 A |
D.以上都是 A |
2 | 7 | 2009 | 24. 失眠的心理社会因素不包括: (2.0分) | A.过度疲劳或紧张 B.对健康过度关心 C.不良自我暗示 D.入睡前饮用兴奋性饮料 |
D.入睡前饮用兴奋性饮料 |
2 | 7 | 2009 | 25. 主观评估疼痛最常用的指标:(2.0分) | A.视觉评估指标(VAS) B.口述描绘评分法(VRS) C.痛阈测定 D.生理生化指标 E.心理测试 |
A.视觉评估指标(VAS) |
2 | 7 | 2009 | 26. 慢性骨骼、软组织疼痛的最常见病因(2.0分) | A.退行性 B.炎症 C.代谢性 D.外伤性 E.感染性 |
A.退行性 |
2 | 7 | 2009 | 27. 慢性骨骼、软组织疼痛治疗的方法: (2.0分) | A.药物治疗: B.神经阻滞 C.硬脊膜外腔注药 D.痛点注射 E.以上都是 |
E.以上都是 |
2 | 7 | 2009 | 28. 骨肿瘤的分类是以( )分类. (2.0分) | A.解剖部位 B.间室 C.组织来源 D.遗传来源 E.影像学 |
C.组织来源 |
2 | 7 | 2009 | 29. 下列药物中,哪一类不属于抗风湿药 (2.0分) | A.解热镇痛药 B.降血糖药 C.某些抗肿瘤药 D.某些抗疟疾药 E.免疫抑制药 |
B.降血糖药 |
2 | 7 | 2009 | 30. 改善病情的抗风湿药(disease-modifying anti-rheumatic drugs, DMARDs)主要包括 (2.0分) | A.镇痛药 B.抗菌药 C.骨代谢调节药 D.免疫增强药 E.免疫抑制药 |
E.免疫抑制药 |
2 | 7 | 2009 | 31. 类风湿关节炎治疗药中,哪一药物作用最好,属于首选药物 (2.0分) | A.甲氨蝶呤(methotrexate, MTX) B.阿司匹林(aspirin) C.可的松(cortisone) D.青霉素G(penicillin G) E.环磷酰胺 (cyclophosphamide, CTX) |
A.甲氨蝶呤(methotrexate, MTX) |
2 | 7 | 2009 | 32. 现在,非甾体抗炎药不作为类风湿关节炎治疗的首选药物,其原因是 (2.0分) | A.疗效有限 B.长期应用不能有效缓解病情 C.有胃肠道和心脏毒性反应 D.已有更好的药物 E.以上均是 |
E.以上均是 |
2 | 7 | 2009 | 33. 神经、肌肉、腺体受阈刺激产生反应的共同表现是(2.0分) | A.收缩 B.分泌 C.局部电位 D.阈电位 E.动作电位 |
E.动作电位 |
2 | 7 | 2009 | 34. 衡量组织兴奋性高低的指标是(2.0分) | A.静息电位水平 B.阈电位 C.阈强度 D.动作电位幅度 E.兴奋扩布速度 |
C.阈强度 |
2 | 7 | 2009 | 35. 直接导致神经末梢释放递质的因素是 (2.0分) | A.末梢处的 Na+内流 B.末梢处的 K+外流 C.末梢处的 Cl‐内流 D.末梢处的 Na+‐Ca2+交换 E.末梢处的 Ca2+内流 |
E.末梢处的 Ca2+内流 |
2 | 7 | 2009 | 36. 临床上普鲁卡因局部麻醉药的应用是由于阻断了局部神经冲动的传导,其神经细胞发生的变化是(2.0分) | A.细胞变性 B.结构完整性破坏 C.功能完整性破坏 D.细胞膜电压门控钾通道破坏 E.细胞膜化学门控钠通道破坏 |
C.功能完整性破坏 |
2 | 7 | 2009 | 37. 动作电位沿运动神经纤维传导抵达神经‐肌接头部位时,轴突末梢释放Ach,使终板膜 产生终板电位,然后在什么部位引发动作电位 (2.0分) | A.肌细胞膜 B.接头后膜 C.终板膜 D.横管膜 E.三联管膜 |
A.肌细胞膜 |
2 | 7 | 2009 | 38. 在神经‐骨骼肌接头中消除乙酰胆碱的酶是(2.0分) | A.ATP 酶 B.胆碱酯酶 C.腺苷酸环化酶 D.磷酸二酯酶 E.单胺氧化酶 |
B.胆碱酯酶 |
2 | 7 | 2009 | 39. 下列哪种药物不是通过阿片受体发挥镇痛作用:(2.0分) | A.吗啡 B.芬太尼 C.瑞芬太尼 D.普鲁卡因 E.美沙酮 |
D.普鲁卡因 |
2 | 7 | 2009 | 40. 下列哪项正确:(2.0分) | A.术中输注瑞芬太尼的病人停药后,出现疼痛、镇痛药物需要量增加的原因与成瘾性有关 B.芬太尼对心血管的影响很大 C.小剂量的阿片类药物即可具有较好的神经病理疼痛治疗效果 D.单用阿片类药物维持镇痛镇静,与心脏手术术中知晓无关 E.芬太尼贴片对术后切口痛无效 |
C.小剂量的阿片类药物即可具有较好的神经病理疼痛治疗效果 |
2 | 7 | 2009 | 41. 对待晚期癌痛病人的疼痛治疗 (2.0分) | A.早期就要用强效阿片药物 B.个体化用药 C.阿片药物不会出现恶心呕吐,故治疗中无肠梗阻病人不需要给予止吐药物 D.强啡肽是近年来人工合成的阿片受体激动剂 E.强啡肽是近年来人工合成的阿片受体拮抗剂 |
B.个体化用药 |
2 | 7 | 2009 | 42. 下列哪种药物是阿片受体的拮抗剂,能逆转芬太尼等过量导致的呼吸抑制:(2.0分) | A.吗啡 B.纳洛酮 C.瑞芬太尼 D.普鲁卡因 E.美沙酮 |
B.纳洛酮 |
2 | 7 | 2009 | 43. 下列哪项不正确:(2.0分) | A.阿片受体是G蛋白偶联受体 B.阿片受体能传导跨膜信号,激活MAPK,抑制腺苷酸环化酶等 C.阿片受体活化后,N型钙通道受到抑制,钾通道开放 D.丁丙诺非是μ受体部分激动剂 E.阿片受体部分激动-拮抗剂不会引起呼吸抑制,故临床上低氧血症发生几无。 |
E.阿片受体部分激动-拮抗剂不会引起呼吸抑制,故临床上低氧血症发生几无。 |
2 | 7 | 2009 | 44. 关于阿片类药物的叙述哪项不正确(2.0分) | A.除杜冷丁外,均有心血管稳定作用,剂量过大,将bradycardia, 需要备用阿托品。 B.芬太尼、舒芬太尼等作用于孤束核和臂旁核控制血管紧张素分泌、稳定血流动力学 C.芬太尼、舒芬太尼等作用于下丘脑-垂体-肾上腺,应激反应,降低交感张力 D.细胞、动物、临床均表明芬太尼、舒芬太尼等具有心机保护作用 E.芬太尼因其心肌的负性肌力作用大,不推荐在心脏手术中使用 |
E.芬太尼因其心肌的负性肌力作用大,不推荐在心脏手术中使用 |
2 | 7 | 2009 | 45. 地佐辛联合芬太尼使用(2.0分) | A.可减少芬太尼用量 B.术中芬太尼用量不变 C.镇痛作用减弱 D.镇痛作用增强不明显,且部分拮抗 E.不良反应发生率明显增加,尤其是呼吸抑制 |
A.可减少芬太尼用量 |
2 | 7 | 2009 | 46. 下列哪项关于NSAID药物叙述是错误的(2.0分) | A.NSAID为非甾体抗炎药(Nonsteroidal Antiinflammatory Drugs,NSAIDs)是一类不含有甾体结构的抗炎药 B.NSAIDs通过干预COX酶活性、阻断花生四烯酸的代谢、抑制前列腺素的合成,发挥其解热、镇痛、消炎作用 C.新近的NSAID以选择性作用于COX2为主 D.WHO癌症疼痛三阶梯治疗轻度疼痛:首选第一阶梯非甾体类抗炎药,以阿司匹林为代表 E.个体化给药仅仅指阿片类药,不包括NSAID |
E.个体化给药仅仅指阿片类药,不包括NSAID |
2 | 7 | 2009 | 47. 局麻药的叙述哪项不正确:(2.0分) | A.目前临床的局麻药分为酯类和酰胺类 B.典型的局麻药分子结构的中间链可为酯链或酰胺链 C.足够浓度的局麻药可以阻断相应部位的神经肌肉膜内电冲动的传导 D.全身应用局麻药不会影响心肌、平滑肌等功能,但是,对外周和中枢神经系统及心脏特殊传导系统的冲动传递有作用。 E.局麻药中毒反应中以中枢和心血管系统受累为主 |
D.全身应用局麻药不会影响心肌、平滑肌等功能,但是,对外周和中枢神经系统及心脏特殊传导系统的冲动传递有作用。 |
2 | 7 | 2009 | 48. 下列哪项错误,利多卡因的主要麻醉用途:(2.0分) | A.0.5%-1%局部浸润麻醉 B.5-10%局部浸润麻醉 C.1-2%外周神经阻滞 D.1-2%硬膜外麻醉 E.胶或软膏作表面麻醉 |
B.5-10%局部浸润麻醉 |
2 | 7 | 2009 | 49. 关于常用局麻药布比卡因,哪项叙述不正确:(2.0分) | A.布比卡因是长效麻醉药 B.属于酯类局麻药 C.麻醉效能与其脂溶性密切相关 D.布比卡因PKa值为8.1 E.蛋白结合率高 |
B.属于酯类局麻药 |
2 | 7 | 2009 | 50. 局麻药作用机制,哪项叙述不正确:(2.0分) | A.能渗透过神经轴突膜 B.起效速度与PKa、离子化有关 C.作用持续时间与蛋白结合率有关 D.麻醉效能与其脂溶性密切相关 E.它是通过与Ca离子通道结合发挥神经阻滞作用 |
E.它是通过与Ca离子通道结合发挥神经阻滞作用 |
2 | 7 | 2010 | 1. 下列药物中哪一种药物对风湿性关节炎几无疗效?(3.0分) | A.Aspirin B.Celecoxib C.Acetaminophen D.Indomethacin E.Ibuprofen F.以上都是 |
C.Acetaminophen |
2 | 7 | 2010 | 2. Aspirin的不良反应不包括 (3.0分) | A.诱发胃、十二指肠溃疡出血 B.粒细胞减少 C.诱发哮喘 D.高铁血红蛋白血症 E.凝血障碍 |
D.高铁血红蛋白血症 |
2 | 7 | 2010 | 3. 对类风湿性关节炎,非甾体抗炎药(NSAIDs)的疗效是(3.0分) | A.对因治疗 B.缩短病程 C.对症治疗 D.阻止肉芽增生及瘢痕形成 E.对风湿性心肌炎可根治 |
C.对症治疗 |
2 | 7 | 2010 | 4. 下列治疗类风湿性关节炎的改善病情抗风湿病药物(DMARDs)是 (3.0分) | A.Indomethacin B.Diclofenac C.Chloroquine D.Celecoxib |
C.Chloroquine |
2 | 7 | 2010 | 5. 强调潜意识心理冲突、个体心理特征产生心身疾病的研究途径是: (3.0分) | A.心理生理学理论 B.学习理论 C.心理动力理论 D.人本主义理论 |
C.心理动力理论 |
2 | 7 | 2010 | 6. 说明心理社会因素"如何"通过神经内分泌和免疫系统造成躯体疾病的研究领域称为:(3.0分) | A.神经免疫学 B.心理免疫学 C.免疫学 D.心理神经免疫学 |
D.心理神经免疫学 |
2 | 7 | 2010 | 7. 失眠的心理社会因素不包括: (3.0分) | A.过度疲劳或紧张 B.对健康过度关心 C.不良自我暗示 D.入睡前饮用兴奋性饮料 |
D.入睡前饮用兴奋性饮料 |
2 | 7 | 2010 | 8. 父母对子女的轻微损伤表示大惊小怪或者泰然处之,影响子女成年后对疼痛的态度属于: (3.0分) | A.社会学习 B.对处境的认知评价 C.注意 D.暗示 |
A.社会学习 |
2 | 7 | 2010 | 9. 癌症病人听到癌症的诊断后,出现心理反应的顺序是: (3.0分) | A.否认-怀疑期,愤怒-沮丧期,接受-适应期,休克-恐惧期 B.否认-怀疑期,休克-恐惧期,愤怒-沮丧期,接受-适应期 C.休克-恐惧期,愤怒-沮丧期,接受-适应期,否认-怀疑期 D.休克-恐惧期,否认-怀疑期,愤怒-沮丧期,接受-适应期 |
D.休克-恐惧期,否认-怀疑期,愤怒-沮丧期,接受-适应期 |
2 | 7 | 2010 | 10. 主观评估疼痛最常用的指标:(4.0分) | A.视觉评估指标(VAS) B.口述描绘评分法(VRS) C.痛阈测定 D.生理生化指标 E.心理测试 |
A.视觉评估指标(VAS) |
2 | 7 | 2010 | 11. 慢性骨骼、软组织疼痛的最常见病因: (3.0分) | A.退行性 B.炎症 C.代谢性 D.外伤性 E.感染性 |
A.退行性 |
2 | 7 | 2010 | 12. 骨肿瘤的分类是以( )分类(3.0分) | A.解剖部位 B.间室 C.组织来源 D.遗传来源 E.影像学 |
C.组织来源 |
2 | 7 | 2010 | 13. 现代骨肉瘤治疗方式是:(3.0分) | A.化疗+放疗+肿瘤边缘切除 B.新辅助化疗+肿瘤广泛切除性保肢 C.化疗+放疗+肿瘤囊内手术 D.新辅助化疗+根治截肢 |
B.新辅助化疗+肿瘤广泛切除性保肢 |
2 | 7 | 2010 | 14. 骨肿瘤的正确诊断需要:(3.0分) | A.病史与体检+家族史+影像学检查 B.家族史+病理检查+影像学检查 C.病史与体检+病理检查+影像学检查 D.影像学检查+穿刺 |
C.病史与体检+病理检查+影像学检查 |
2 | 7 | 2010 | 15. 在神经‐骨骼肌接头中消除乙酰胆碱的酶是 (3.0分) | A.ATP 酶 B.胆碱酯酶 C.腺苷酸环化酶 D.磷酸二酯酶 E.单胺氧化酶 |
B.胆碱酯酶 |
2 | 7 | 2010 | 16. 神经‐骨骼肌接头处的兴奋传递物质是(3.0分) | A.5‐羟色胺 B.乙酰胆碱 C.去甲肾上腺素 D.肾上腺素 E.多巴胺 |
B.乙酰胆碱 |
2 | 7 | 2010 | 17. 细胞膜上不衰减形式传播的电活动是(3.0分) | A.动作电位 B.静息电位 C.终板电位 D.感受器电位 E.突触后电位 |
A.动作电位 |
2 | 7 | 2010 | 18. 人工减小细胞浸浴液中的 Na+浓度,所记录的动作电位出现(3.0分) | A.幅度变小 B.幅度变大 C.时程缩短 D.时程延长 E.复极相延长 |
A.幅度变小 |
2 | 7 | 2010 | 19. 内脏痛的特点不包括 (3.0分) | A.对锐器等皮肤刺激敏感 B.伴有不愉快的情绪反应 C.定位不精确 D.可有牵涉痛 E.对牵拉敏感 |
A.对锐器等皮肤刺激敏感 |
2 | 7 | 2010 | 20. 传导慢痛的外周神经纤维主要是 (3.0分) | A.Aα纤维 B.Aδ纤维 C.B 类纤维 D.C 类纤维 E.Aγ纤维 |
D.C 类纤维 |
2 | 7 | 2010 | 21. 在痛觉传入通路中,调控痛觉信号的"闸门"位于(3.0分) | A.脊髓后索 B.脊髓后角 C.延髓薄束核和楔束核 D.脑干网状结构 E.丘脑髓板内核群 |
B.脊髓后角 |
2 | 7 | 2010 | 22. 传导快痛的外周神经纤维主要是 (3.0分) | A.Aα纤维 B.Aδ纤维 C.B 类纤维 D.C 类纤维 E.Aγ纤维 |
B.Aδ纤维 |
2 | 7 | 2010 | 23. 闸门学说中兴奋SG细胞的是(3.0分) | A.Aα纤维 B.Aδ纤维 C.B 类纤维 D.C 类纤维 E.Aβ纤维 |
E.Aβ纤维 |
2 | 7 | 2010 | 24. 声波感受器是(3.0分) | A.球囊囊斑毛细胞 B.椭圆囊囊斑毛细胞 C.半规管壶腹嵴毛细胞 D.耳蜗基底膜毛细胞 E.耳石 |
D.耳蜗基底膜毛细胞 |
2 | 7 | 2010 | 25. 引起听觉的感受器是什么类型感受器?(3.0分) | A.Mechanoreceptor B.Chemoreceptor C.Electromagnetic receptor D.Thermoreceptor E.Nociceptor |
A.Mechanoreceptor |
2 | 7 | 2010 | 26. 患者主诉左耳比右耳的听力差,检查发现两耳任内试验阳性,韦伯试验偏向右耳(3.0分) | A.左耳传导性耳聋 B.左耳感音性耳聋 C.右耳传导性耳聋 D.右耳神经性耳聋 E.右耳感音性耳聋 |
B.左耳感音性耳聋 |
2 | 7 | 2010 | 27. 下列不属于折光系统的是(3.0分) | A.角膜 B.房水 C.晶状体 D.玻璃体 E.视网膜 |
E.视网膜 |
2 | 7 | 2010 | 28. 房水的主要功能是 (3.0分) | A.折光成像 B.对玻璃体有保护作用 C.对角膜和晶状体有营养作用 D.对视网膜有保护作用 E.促进睫状肌收缩 |
C.对角膜和晶状体有营养作用 |
2 | 7 | 2010 | 29. 看近物时的视觉调节过程包括(3.0分) | A.晶状体变凸,眼轴会聚,瞳孔散大 B.晶状体变凸,眼轴会聚,瞳孔缩小 C.晶状体扁平,眼轴会聚,瞳孔缩小 D.晶状体扁平,眼轴会聚,瞳孔散大 E.晶状体曲率半径减小,眼轴会聚,瞳孔缩小 |
B.晶状体变凸,眼轴会聚,瞳孔缩小 |
2 | 7 | 2010 | 30. 瞳孔对光反射中枢在(3.0分) | A.大脑皮质 B.下丘脑 C.中脑 D.脑桥 E.延脑 |
C.中脑 |
2 | 7 | 2010 | 31. 关于瞳孔反射的说明,正确的是(3.0分) | A.视近物时瞳孔扩大 B.瞳孔近反射与对光反射的通路相同 C.动眼神经损伤使瞳孔缩小 D.交感神经兴奋使瞳孔扩大 E.副交感神经兴奋使瞳孔扩大 |
D.交感神经兴奋使瞳孔扩大 |
2 | 7 | 2010 | 32. 能产生动作电位的细胞是(3.0分) | A.视锥细胞 B.视杆细胞 C.双极细胞 D.神经节细胞 E.水平细胞 |
D.神经节细胞 |
2 | 7 | 2010 | 33. 维生素A 严重缺乏,可影响人(3.0分) | A.在明处的视力 B.色觉 C.在暗处的视力 D.立体视觉 E.视野范围 |
C.在暗处的视力 |
2 | 7 | 2011 | 1. 下列药物中哪一种药物对风湿性关节炎几无疗效?(4.0分) | A.Aspirin B.Celecoxib C.Acetaminophen D.Indomethacin E.Ibuprofen F.以上都是 |
C.Acetaminophen |
2 | 7 | 2011 | 2. Aspirin的不良反应不包括(4.0分) | A.诱发胃、十二指肠溃疡出血 B.粒细胞减少 C.诱发哮喘 D.高铁血红蛋白血症 E.凝血障碍 |
D.高铁血红蛋白血症 |
2 | 7 | 2011 | 3. 对类风湿性关节炎,非甾体抗炎药(NSAIDs)的疗效是(4.0分) | A.对因治疗 B.缩短病程 C.对症治疗 D.阻止肉芽增生及瘢痕形成 E.对风湿性心肌炎可根治 |
C.对症治疗 |
2 | 7 | 2011 | 4. 下列治疗类风湿性关节炎的改善病情抗风湿病药物(DMARDs)是(4.0分) | A.Indomethacin B.Diclofenac C.Chloroquine D.Celecoxib |
C.Chloroquine |
2 | 7 | 2011 | 5. 慢性骨骼、软组织疼痛的最常见病因(4.0分) | A.退行性 B.炎症 C.代谢性 D.外伤性 E.感染性 |
A.退行性 |
2 | 7 | 2011 | 6. 骨肿瘤的分类是以( )分类(4.0分) | A.解剖部位 B.间室 C.组织来源 D.遗传来源 E.影像学 |
C.组织来源 |
2 | 7 | 2011 | 7. 现代骨肉瘤治疗方式是(4.0分) | A.化疗+放疗+肿瘤边缘切除 B.新辅助化疗+肿瘤广泛切除性保肢 C.化疗+放疗+肿瘤囊内手术 D.新辅助化疗+根治截肢 |
B.新辅助化疗+肿瘤广泛切除性保肢 |
2 | 7 | 2011 | 8. 骨肿瘤的正确诊断需要(4.0分) | A.病史与体检+家族史+影像学检查 B.家族史+病理检查+影像学检查 C.病史与体检+病理检查+影像学检查 D.影像学检查+穿刺 |
C.病史与体检+病理检查+影像学检查 |
2 | 7 | 2011 | 9. 下列成分中不是致痛物质的是 (4.0分) | A.葡萄糖 B.组胺 C.P物质 D.前列腺素 E.5-HT |
A.葡萄糖 |
2 | 7 | 2011 | 10. 下列哪种不是内脏痛的主要特点 (4.0分) | A.对锐器切割等刺激不敏感 B.对炎症等刺激不敏感 C.定位不精确 D.可伴有牵涉痛 E.伴有不愉快的情绪反应 |
B.对炎症等刺激不敏感 |
2 | 7 | 2011 | 11. 牵涉痛是指 (4.0分) | A.内脏疾病引起相邻脏器的疼痛 B.手术牵拉脏器引起的疼痛 C.神经疼痛向体表投射 D.按压体表引起部分内脏疼痛 E.内脏疾病引起体表某一部位的疼痛或痛觉过敏 |
E.内脏疾病引起体表某一部位的疼痛或痛觉过敏 |
2 | 7 | 2011 | 12. 传导慢痛的外周神经纤维主要是 (4.0分) | A.Aα纤维 B.Aδ纤维 C.B 类纤维 D.C 类纤维 E.Aγ纤维 |
D.C 类纤维 |
2 | 7 | 2011 | 13. 传导快痛的外周神经纤维主要是 (4.0分) | A.Aα纤维 B.Aδ纤维 C.B 类纤维 D.C 类纤维 E.Aγ纤维 |
B.Aδ纤维 |
2 | 7 | 2011 | 14. 下列哪种药物不是通过阿片受体发挥镇痛作用(4.0分) | A.吗啡 B.芬太尼 C.瑞芬太尼 D.普鲁卡因 E.美沙酮 |
D.普鲁卡因 |
2 | 7 | 2011 | 15. 下列哪项正确(4.0分) | A.术中输注瑞芬太尼的病人停药后,出现疼痛、镇痛药物需要量增加的原因与成瘾性有关 B.芬太尼对心血管的影响很大 C.小剂量的阿片类药物即可具有较好的神经病理疼痛治疗效果 D.单用阿片类药物维持镇痛镇静,与心脏手术术中知晓无关 E.芬太尼贴片对术后切口痛无效 |
C.小剂量的阿片类药物即可具有较好的神经病理疼痛治疗效果 |
2 | 7 | 2011 | 16. 对待晚期癌痛病人的疼痛治疗(4.0分) | A.早期就要用强效阿片药物 B.个体化用药 C.阿片药物不会出现恶心呕吐,故治疗中无肠梗阻病人不需要给予止吐药物 D.强啡肽是近年来人工合成的阿片受体激动剂 E.强啡肽是近年来人工合成的阿片受体拮抗剂 |
B.个体化用药 |
2 | 7 | 2011 | 17. 下列哪种药物是阿片受体的拮抗剂,能逆转芬太尼等过量导致的呼吸抑制(4.0分) | A.吗啡 B.纳洛酮 C.瑞芬太尼 D.普鲁卡因 E.美沙酮 |
B.纳洛酮 |
2 | 7 | 2011 | 18. 下列哪项不正确:(4.0分) | A.阿片受体是G蛋白偶联受体 B.阿片受体能传导跨膜信号,激活MAPK,抑制腺苷酸环化酶等 C.阿片受体活化后,N型钙通道受到抑制,钾通道开放 D.丁丙诺非是μ受体部分激动剂 E.阿片受体部分激动-拮抗剂不会引起呼吸抑制,故临床上低氧血症发生几无。 |
E.阿片受体部分激动-拮抗剂不会引起呼吸抑制,故临床上低氧血症发生几无。 |
2 | 7 | 2011 | 19. 关于阿片类药物的叙述哪项不正确(4.0分) | A.除杜冷丁外,均有心血管稳定作用,剂量过大,将bradycardia, 需要备用阿托品。 B.芬太尼、舒芬太尼等作用于孤束核和臂旁核控制血管紧张素分泌、稳定血流动力学 C.芬太尼、舒芬太尼等作用于下丘脑-垂体-肾上腺,应激反应,降低交感张力 D.细胞、动物、临床均表明芬太尼、舒芬太尼等具有心机保护作用 E.芬太尼因其心肌的负性肌力作用大,不推荐在心脏手术中使用 |
E.芬太尼因其心肌的负性肌力作用大,不推荐在心脏手术中使用 |
2 | 7 | 2011 | 20. 地佐辛联合芬太尼使用(4.0分) | A.可减少芬太尼用量 B.术中芬太尼用量不变 C.镇痛作用减弱 D.镇痛作用增强不明显,且部分拮抗 E.不良反应发生率明显增加,尤其是呼吸抑制 |
A.可减少芬太尼用量 |
2 | 7 | 2011 | 21. 下列哪项关于NSAID药物叙述是错误的(4.0分) | A.NSAID为非甾体抗炎药(Nonsteroidal Antiinflammatory Drugs,NSAIDs)是一类不含有甾体结构的抗炎药 B.NSAIDs通过干预COX酶活性、阻断花生四烯酸的代谢、抑制前列腺素的合成,发挥其解热、镇痛、消炎作用 C.新近的NSAID以选择性作用于COX2为主 D.WHO癌症疼痛三阶梯治疗轻度疼痛:首选第一阶梯非甾体类抗炎药,以阿司匹林为代表 E.个体化给药仅仅指阿片类药,不包括NSAID |
E.个体化给药仅仅指阿片类药,不包括NSAID |
2 | 7 | 2011 | 22. 局麻药的叙述哪项不正确(4.0分) | A.目前临床的局麻药分为酯类和酰胺类 B.典型的局麻药分子结构的中间链可为酯链或酰胺链 C.足够浓度的局麻药可以阻断相应部位的神经肌肉膜内电冲动的传导 D.全身应用局麻药不会影响心肌、平滑肌等功能,但是,对外周和中枢神经系统及心脏特殊传导系统的冲动传递有作用。 E.局麻药中毒反应中以中枢和心血管系统受累为主 |
D.全身应用局麻药不会影响心肌、平滑肌等功能,但是,对外周和中枢神经系统及心脏特殊传导系统的冲动传递有作用。 |
2 | 7 | 2011 | 23. 下列哪项错误,利多卡因的主要麻醉用途(4.0分) | A.0.5%-1%局部浸润麻醉 B.5-10%局部浸润麻醉 C.1-2%外周神经阻滞 D.1-2%硬膜外麻醉 E.胶或软膏作表面麻醉 |
B.5-10%局部浸润麻醉 |
2 | 7 | 2011 | 24. 关于常用局麻药布比卡因,哪项叙述不正确(4.0分) | A.布比卡因是长效麻醉药 B.属于酯类局麻药 C.麻醉效能与其脂溶性密切相关 D.布比卡因PKa值为8.1 E.蛋白结合率高 F. |
B.属于酯类局麻药 |
2 | 7 | 2011 | 25. 哪项叙述正确(4.0分) | A.局麻药中加入肾上腺素的目的是升高血压、避免血管扩张 B.在阴茎等局部神经阻滞时,麻药中要加入一定浓度的肾上腺素 C.利多卡因用于局部浸润麻醉时的浓度是0.5-1%,肾上腺素浓度为1:200000 D.利多卡因用于局部浸润麻醉时的浓度是0.5-1%,肾上腺素浓度为1:2000 E.丁卡因由于心脏毒性大,在局部浸润麻醉时的浓度低为0.25-0.5%,肾上腺素浓度为1:2000 F.临床常用的5种局麻药无神经毒性作用 |
C.利多卡因用于局部浸润麻醉时的浓度是0.5-1%,肾上腺素浓度为1:200000 |
2 | 7 | 2012 | 1. 下列哪项关于NSAID药物叙述是错误的: (4.0分) | A.NSAID为非甾体抗炎药(Nonsteroidal Antiinflammatory Drugs,NSAIDs)是一类不含有甾体结构的抗炎药 B.NSAIDs通过干预COX酶活性、阻断花生四烯酸的代谢、抑制前列腺素的合成,发挥其解热、镇痛、消炎作用 C.新近的NSAID以选择性作用于COX2为主 D.WHO癌症疼痛三阶梯治疗轻度疼痛:首选第一阶梯非甾体类抗炎药,以阿司匹林为代表 E.个体化给药仅仅指阿片类药,不包括NSAID |
E.个体化给药仅仅指阿片类药,不包括NSAID |
2 | 7 | 2012 | 2. 下列药物中哪一种药物对风湿性关节炎几无疗效?(4.0分) | A.Aspirin B.Celecoxib C.Acetaminophen D.Indomethacin E.Ibuprofen F.以上都是 |
C.Acetaminophen |
2 | 7 | 2012 | 3. 下列哪项正确(4.0分) | A.术中输注瑞芬太尼的病人停药后,出现疼痛、镇痛药物需要量增加的原因与成瘾性有关 B.芬太尼对心血管的影响很大 C.小剂量的阿片类药物即可具有较好的神经病理疼痛治疗效果 D.单用阿片类药物维持镇痛镇静,与心脏手术术中知晓无关 E.芬太尼贴片对术后切口痛无效 |
C.小剂量的阿片类药物即可具有较好的神经病理疼痛治疗效果 |
2 | 7 | 2012 | 4. 骨肿瘤的分类是以( )分类.(4.0分) | A.解剖部位 B.间室 C.组织来源 D.遗传来源 E.影像学 |
C.组织来源 |
2 | 7 | 2012 | 5. Aspirin的不良反应不包括 (4.0分) | A.诱发胃、十二指肠溃疡出血 B.粒细胞减少 C.诱发哮喘 D.高铁血红蛋白血症 E.凝血障碍 |
D.高铁血红蛋白血症 |
2 | 7 | 2012 | 6. 对类风湿性关节炎,非甾体抗炎药(NSAIDs)的疗效是 (4.0分) | A.对因治疗 B.缩短病程 C.对症治疗 D.阻止肉芽增生及瘢痕形成 E.对风湿性心肌炎可根治 |
C.对症治疗 |
2 | 7 | 2012 | 7. 下列治疗类风湿性关节炎的改善病情抗风湿病药物(DMARDs)是 (4.0分) | A.Indomethacin B.Diclofenac C.Chloroquine D.Celecoxib |
C.Chloroquine |
2 | 7 | 2012 | 8. 关于阿片类药物的叙述哪项不正确:(4.0分) | A.除杜冷丁外,均有心血管稳定作用,剂量过大,将bradycardia, 需要备用阿托品。 B.芬太尼、舒芬太尼等作用于孤束核和臂旁核控制血管紧张素分泌、稳定血流动力学 C.芬太尼、舒芬太尼等作用于下丘脑-垂体-肾上腺,应激反应,降低交感张力 D.细胞、动物、临床均表明芬太尼、舒芬太尼等具有心机保护作用 E.芬太尼因其心肌的负性肌力作用大,不推荐在心脏手术中使用 |
E.芬太尼因其心肌的负性肌力作用大,不推荐在心脏手术中使用 |
2 | 7 | 2012 | 9. 慢性骨骼、软组织疼痛的最常见病因:(4.0分) | A.退行性 B.炎症 C.代谢性 D.外伤性 E.感染性 |
A.退行性 |
2 | 7 | 2012 | 10. 现代骨肉瘤治疗方式是(4.0分) | A.化疗+放疗+肿瘤边缘切除 B.新辅助化疗+肿瘤广泛切除性保肢 C.化疗+放疗+肿瘤囊内手术 D.新辅助化疗+根治截肢 |
B.新辅助化疗+肿瘤广泛切除性保肢 |
2 | 7 | 2012 | 11. 骨肿瘤的正确诊断需要:(4.0分) | A.病史与体检+家族史+影像学检查 B.家族史+病理检查+影像学检查 C.病史与体检+病理检查+影像学检查 D.影像学检查+穿刺 |
C.病史与体检+病理检查+影像学检查 |
2 | 7 | 2012 | 12. 下列哪种不是内脏痛的主要特点 (4.0分) | A.对锐器切割等刺激不敏感 B.对炎症等刺激不敏感 C.定位不精确 D.可伴有牵涉痛 E.伴有不愉快的情绪反应 |
B.对炎症等刺激不敏感 |
2 | 7 | 2012 | 13. 牵涉痛是指 (4.0分) | A.内脏疾病引起相邻脏器的疼痛 B.手术牵拉脏器引起的疼痛 C.神经疼痛向体表投射 D.按压体表引起部分内脏疼痛 E.内脏疾病引起体表某一部位的疼痛或痛觉过敏 |
E.内脏疾病引起体表某一部位的疼痛或痛觉过敏 |
2 | 7 | 2012 | 14. 传导慢痛的外周神经纤维主要是 (4.0分) | A.Aα纤维 B.Aδ纤维 C.B 类纤维 D.C 类纤维 E.Aγ纤维 |
D.C 类纤维 |
2 | 7 | 2012 | 15. 下列成分中不是致痛物质的是 (4.0分) | A.葡萄糖 B.组胺 C.P物质 D.前列腺素 E.5-HT |
A.葡萄糖 |
2 | 7 | 2012 | 16. 传导快痛的外周神经纤维主要是 (4.0分) | A.Aα纤维 B.Aδ纤维 C.B 类纤维 D.C 类纤维 E.Aγ纤维 |
B.Aδ纤维 |
2 | 7 | 2012 | 17. 下列哪种药物是阿片受体的拮抗剂,能逆转芬太尼等过量导致的呼吸抑制(4.0分) | A.吗啡 B.纳洛酮 C.瑞芬太尼 D.普鲁卡因 E.美沙酮 |
B.纳洛酮 |
2 | 7 | 2012 | 18. 关于常用局麻药布比卡因,哪项叙述不正确(4.0分) | A.布比卡因是长效麻醉药 B.属于酯类局麻药 C.麻醉效能与其脂溶性密切相关 D.布比卡因PKa值为8.1 E.蛋白结合率高 F. |
B.属于酯类局麻药 |
2 | 7 | 2012 | 19. 下列哪种药物不是通过阿片受体发挥镇痛作用: (4.0分) | A.吗啡 B.芬太尼 C.瑞芬太尼 D.普鲁卡因 E.美沙酮 |
D.普鲁卡因 |
2 | 7 | 2012 | 20. 下列哪项错误,利多卡因的主要麻醉用途:(4.0分) | A.0.5%-1%局部浸润麻醉 B.5-10%局部浸润麻醉 C.1-2%外周神经阻滞 D.1-2%硬膜外麻醉 E.胶或软膏作表面麻醉 |
B.5-10%局部浸润麻醉 |
2 | 7 | 2012 | 21. 哪项叙述正确: (4.0分) | A.局麻药中加入肾上腺素的目的是升高血压、避免血管扩张 B.在阴茎等局部神经阻滞时,麻药中要加入一定浓度的肾上腺素 C.利多卡因用于局部浸润麻醉时的浓度是0.5-1%,肾上腺素浓度为1:200000 D.利多卡因用于局部浸润麻醉时的浓度是0.5-1%,肾上腺素浓度为1:2000 E.丁卡因由于心脏毒性大,在局部浸润麻醉时的浓度低为0.25-0.5%,肾上腺素浓度为1:2000 F.临床常用的5种局麻药无神经毒性作用 |
C.利多卡因用于局部浸润麻醉时的浓度是0.5-1%,肾上腺素浓度为1:200000 |
2 | 7 | 2012 | 22. 下列哪项不正确:(4.0分) | A.阿片受体是G蛋白偶联受体 B.阿片受体能传导跨膜信号,激活MAPK,抑制腺苷酸环化酶等 C.阿片受体活化后,N型钙通道受到抑制,钾通道开放 D.丁丙诺非是μ受体部分激动剂 E.阿片受体部分激动-拮抗剂不会引起呼吸抑制,故临床上低氧血症发生几无。 |
E.阿片受体部分激动-拮抗剂不会引起呼吸抑制,故临床上低氧血症发生几无。 |
2 | 7 | 2012 | 23. 对待晚期癌痛病人的疼痛治疗 (4.0分) | A.早期就要用强效阿片药物 B.个体化用药 C.阿片药物不会出现恶心呕吐,故治疗中无肠梗阻病人不需要给予止吐药物 D.强啡肽是近年来人工合成的阿片受体激动剂 E.强啡肽是近年来人工合成的阿片受体拮抗剂 |
B.个体化用药 |
2 | 7 | 2012 | 24. 地佐辛联合芬太尼使用 (4.0分) | A.可减少芬太尼用量 B.术中芬太尼用量不变 C.镇痛作用减弱 D.镇痛作用增强不明显,且部分拮抗 E.不良反应发生率明显增加,尤其是呼吸抑制 |
A.可减少芬太尼用量 |
2 | 7 | 2012 | 25. 局麻药的叙述哪项不正确: (4.0分) | A.目前临床的局麻药分为酯类和酰胺类 B.典型的局麻药分子结构的中间链可为酯链或酰胺链 C.足够浓度的局麻药可以阻断相应部位的神经肌肉膜内电冲动的传导 D.全身应用局麻药不会影响心肌、平滑肌等功能,但是,对外周和中枢神经系统及心脏特殊传导系统的冲动传递有作用。 E.局麻药中毒反应中以中枢和心血管系统受累为主 |
D.全身应用局麻药不会影响心肌、平滑肌等功能,但是,对外周和中枢神经系统及心脏特殊传导系统的冲动传递有作用。 |
2 | 7 | 2013 | 1. 关于离差智商,下列哪项是错误的 (3.0分) | A.由比奈-西蒙智力量表最早采用 B.其计算公式是:IQ=15(χ- X )/S+100 C.适用于任何年龄 D.是标准分的一种 |
A.由比奈-西蒙智力量表最早采用 |
2 | 7 | 2013 | 2. 关于艾森克人格问卷(EPQ),下列哪项是错误的(3.0分) | A.包括E、N、P三个分量表 B.E分高反映个性外向 C.N分高反映情绪不稳定,易焦虑 D.P分高反映孤僻、挑衅、不适应环境 |
A.包括E、N、P三个分量表 |
2 | 7 | 2013 | 3. 下列哪项正确:(3.0分) | A.术中输注瑞芬太尼的病人停药后,出现疼痛、镇痛药物需要量增加的原因与成瘾性有关 B.芬太尼对心血管的影响很大 C.小剂量的阿片类药物即可具有较好的神经病理疼痛治疗效果 D.单用阿片类药物维持镇痛镇静,与心脏手术术中知晓无关 E.芬太尼贴片对术后切口痛无效 |
C.小剂量的阿片类药物即可具有较好的神经病理疼痛治疗效果 |
2 | 7 | 2013 | 4. 骨肿瘤的分类是以( )分类. (3.0分) | A.解剖部位 B.间室 C.组织来源 D.遗传来源 E.影像学 |
C.组织来源 |
2 | 7 | 2013 | 5. Aspirin的不良反应不包括(3.0分) | A.诱发胃、十二指肠溃疡出血 B.粒细胞减少 C.诱发哮喘 D.高铁血红蛋白血症 E.凝血障碍 |
D.高铁血红蛋白血症 |
2 | 7 | 2013 | 6. 以下不属于心理评估常用方法的是: (3.0分) | A.会谈法 B.实验法 C.观察法 D.调查法 |
B.实验法 |
2 | 7 | 2013 | 7. 效度是指测验分数的: (3.0分) | A.一致性 B.可靠性 C.有效性 D.灵敏性 |
C.有效性 |
2 | 7 | 2013 | 8. 关于阿片类药物的叙述哪项不正确(3.0分) | A.除杜冷丁外,均有心血管稳定作用,剂量过大,将bradycardia, 需要备用阿托品。 B.芬太尼、舒芬太尼等作用于孤束核和臂旁核控制血管紧张素分泌、稳定血流动力学 C.芬太尼、舒芬太尼等作用于下丘脑-垂体-肾上腺,应激反应,降低交感张力 D.细胞、动物、临床均表明芬太尼、舒芬太尼等具有心机保护作用 E.芬太尼因其心肌的负性肌力作用大,不推荐在心脏手术中使用 |
E.芬太尼因其心肌的负性肌力作用大,不推荐在心脏手术中使用 |
2 | 7 | 2013 | 9. 慢性骨骼、软组织疼痛的最常见病因(4.0分) | A.退行性 B.炎症 C.代谢性 D.外伤性 E.感染性 |
A.退行性 |
2 | 7 | 2013 | 10. 现代骨肉瘤治疗方式是(3.0分) | A.化疗+放疗+肿瘤边缘切除 B.新辅助化疗+肿瘤广泛切除性保肢 C.化疗+放疗+肿瘤囊内手术 D.新辅助化疗+根治截肢 |
B.新辅助化疗+肿瘤广泛切除性保肢 |
2 | 7 | 2013 | 11. 骨肿瘤的正确诊断需要(3.0分) | A.病史与体检+家族史+影像学检查 B.家族史+病理检查+影像学检查 C.病史与体检+病理检查+影像学检查 D.影像学检查+穿刺 |
C.病史与体检+病理检查+影像学检查 |
2 | 7 | 2013 | 12. 下列哪种不是内脏痛的主要特点 (3.0分) | A.对锐器切割等刺激不敏感 B.对炎症等刺激不敏感 C.定位不精确 D.可伴有牵涉痛 E.伴有不愉快的情绪反应 |
B.对炎症等刺激不敏感 |
2 | 7 | 2013 | 13. 牵涉痛是指 (3.0分) | A.内脏疾病引起相邻脏器的疼痛 B.手术牵拉脏器引起的疼痛 C.神经疼痛向体表投射 D.按压体表引起部分内脏疼痛 E.内脏疾病引起体表某一部位的疼痛或痛觉过敏 |
E.内脏疾病引起体表某一部位的疼痛或痛觉过敏 |
2 | 7 | 2013 | 14. 传导慢痛的外周神经纤维主要是 (3.0分) | A.Aα纤维 B.Aδ纤维 C.B 类纤维 D.C 类纤维 E.Aγ纤维 |
D.C 类纤维 |
2 | 7 | 2013 | 15. 下列成分中不是致痛物质的是 (3.0分) | A.葡萄糖 B.组胺 C.P物质 D.前列腺素 E.5-HT |
A.葡萄糖 |
2 | 7 | 2013 | 16. 传导快痛的外周神经纤维主要是 (3.0分) | A.Aα纤维 B.Aδ纤维 C.B 类纤维 D.C 类纤维 E.Aγ纤维 |
B.Aδ纤维 |
2 | 7 | 2013 | 17. 下列哪种药物是阿片受体的拮抗剂,能逆转芬太尼等过量导致的呼吸抑制(3.0分) | A.吗啡 B.纳洛酮 C.瑞芬太尼 D.普鲁卡因 E.美沙酮 |
B.纳洛酮 |
2 | 7 | 2013 | 18. 关于常用局麻药布比卡因,哪项叙述不正确(3.0分) | A.布比卡因是长效麻醉药 B.属于酯类局麻药 C.麻醉效能与其脂溶性密切相关 D.布比卡因PKa值为8.1 E.蛋白结合率高 |
B.属于酯类局麻药 |
2 | 7 | 2013 | 19. 下列哪种药物不是通过阿片受体发挥镇痛作用(3.0分) | A.吗啡 B.芬太尼 C.瑞芬太尼 D.普鲁卡因 E.美沙酮 |
D.普鲁卡因 |
2 | 7 | 2013 | 20. 下列哪项错误,利多卡因的主要麻醉用途(3.0分) | A.0.5%-1%局部浸润麻醉 B.5-10%局部浸润麻醉 C.1-2%外周神经阻滞 D.1-2%硬膜外麻醉 E.胶或软膏作表面麻醉 |
B.5-10%局部浸润麻醉 |
2 | 7 | 2013 | 21. 哪项叙述正确(3.0分) | A.局麻药中加入肾上腺素的目的是升高血压、避免血管扩张 B.在阴茎等局部神经阻滞时,麻药中要加入一定浓度的肾上腺素 C.利多卡因用于局部浸润麻醉时的浓度是0.5-1%,肾上腺素浓度为1:200000 D.利多卡因用于局部浸润麻醉时的浓度是0.5-1%,肾上腺素浓度为1:2000 E.丁卡因由于心脏毒性大,在局部浸润麻醉时的浓度低为0.25-0.5%,肾上腺素浓度为1:2000 |
C.利多卡因用于局部浸润麻醉时的浓度是0.5-1%,肾上腺素浓度为1:200000 |
2 | 7 | 2013 | 22. 下列哪项不正确(3.0分) | A.阿片受体是G蛋白偶联受体 B.阿片受体能传导跨膜信号,激活MAPK,抑制腺苷酸环化酶等 C.阿片受体活化后,N型钙通道受到抑制,钾通道开放 D.丁丙诺非是μ受体部分激动剂 E.阿片受体部分激动-拮抗剂不会引起呼吸抑制,故临床上低氧血症发生几无。 |
E.阿片受体部分激动-拮抗剂不会引起呼吸抑制,故临床上低氧血症发生几无。 |
2 | 7 | 2013 | 23. 对待晚期癌痛病人的疼痛治疗(3.0分) | A.早期就要用强效阿片药物 B.个体化用药 C.阿片药物不会出现恶心呕吐,故治疗中无肠梗阻病人不需要给予止吐药物 D.强啡肽是近年来人工合成的阿片受体激动剂 E.强啡肽是近年来人工合成的阿片受体拮抗剂 |
B.个体化用药 |
2 | 7 | 2013 | 24. 地佐辛联合芬太尼使用(3.0分) | A.可减少芬太尼用量 B.术中芬太尼用量不变 C.镇痛作用减弱 D.镇痛作用增强不明显,且部分拮抗 E.不良反应发生率明显增加,尤其是呼吸抑制 |
A.可减少芬太尼用量 |
2 | 7 | 2013 | 25. 局麻药的叙述哪项不正确:(3.0分) | A.目前临床的局麻药分为酯类和酰胺类 B.典型的局麻药分子结构的中间链可为酯链或酰胺链 C.足够浓度的局麻药可以阻断相应部位的神经肌肉膜内电冲动的传导 D.全身应用局麻药不会影响心肌、平滑肌等功能,但是,对外周和中枢神经系统及心脏特殊传导系统的冲动传递有作用。 E.局麻药中毒反应中以中枢和心血管系统受累为主 |
D.全身应用局麻药不会影响心肌、平滑肌等功能,但是,对外周和中枢神经系统及心脏特殊传导系统的冲动传递有作用。 |
2 | 7 | 2013 | 26. SAS评估以下什么内容 (3.0分) | A.生活事件 B.抑郁症状 C.心理症状 D.焦虑症状 |
D.焦虑症状 |
2 | 7 | 2013 | 27. 对标准化心理测验应具备的条件而言,下列哪项是错误的(3.0分) | A.效度、信度 B.常模 C.因人制宜的测试条 D.统一的计分标准 |
C.因人制宜的测试条 |
2 | 7 | 2013 | 28. 行为治疗的目的是(3.0分) | A.纠正不适应的认知模式 B.改变不适应的行为模式 C.重现童年期的心理冲突 D.发挥个人的潜能 |
B.改变不适应的行为模式 |
2 | 7 | 2013 | 29. 系统脱敏疗法的步骤不包括 (3.0分) | A.放松训练 B.列出等级脱敏表 C.暴露于强烈的焦虑环境中 D.放松状态下脱敏 |
C.暴露于强烈的焦虑环境中 |
2 | 7 | 2013 | 30. 心理治疗的基本过程有(3.0分) | A.探索问题、心理测验、治疗行动、疗效评价和结束巩固 B.建立关系、分析认识、治疗行动、疗效评价和结束巩固 C.建立关系、探索问题、心理测验、治疗行动和疗效评价 D.探索问题、分析认识、治疗行动、疗效评价和结束巩固 |
D.探索问题、分析认识、治疗行动、疗效评价和结束巩固 |
2 | 7 | 2013 | 31. 来访者中心疗法不主张:(3.0分) | A.发展来访者主观能动性 B.对来访者明确指导 C.促进来访者成长 D.医生与来访者是平等"角色" |
B.对来访者明确指导 |
2 | 7 | 2013 | 32. 支持疗法的理论基础中, 不包括(3.0分) | A.认知调整 B.善用"资源" C.提高应对能力 D.挖掘幼年体验 |
D.挖掘幼年体验 |
2 | 7 | 2013 | 33. 生物反馈疗法的理论基础是 (3.0分) | A.生理学、信息论和控制论 B.行为学习理论、生理学和信息论 C.行为学习理论、信息论和控制论 D.生物学、行为学习理论和信息论 |
C.行为学习理论、信息论和控制论 |
2 | 7 | 2014 | 1. 关于离差智商,下列哪项是错误的: (3.0分) | A.由比奈-西蒙智力量表最早采用 B.其计算公式是:IQ=15(χ- X )/S+100 C.适用于任何年龄 D.是标准分的一种 |
A.由比奈-西蒙智力量表最早采用 |
2 | 7 | 2014 | 2. 关于艾森克人格问卷(EPQ),下列哪项是错误的(3.0分) | A.包括E、N、P三个分量表 B.E分高反映个性外向 C.N分高反映情绪不稳定,易焦虑 D.P分高反映孤僻、挑衅、不适应环境 |
A.包括E、N、P三个分量表 |
2 | 7 | 2014 | 3. 下列哪项正确: (3.0分) | A.术中输注瑞芬太尼的病人停药后,出现疼痛、镇痛药物需要量增加的原因与成瘾性有关芬太尼对心血管的影响很大 B.小剂量的阿片类药物即可具有较好的神经病理疼痛治疗效果 C.单用阿片类药物维持镇痛镇静,与心脏手术术中知晓无关 D.芬太尼贴片对术后切口痛无效 |
C.单用阿片类药物维持镇痛镇静,与心脏手术术中知晓无关 |
2 | 7 | 2014 | 4. 骨肿瘤的分类是以( )分类. (3.0分) | A.解剖部位 B.间室 C.组织来源 D.遗传来源 E.影像学 |
C.组织来源 |
2 | 7 | 2014 | 5. Aspirin的不良反应不包括(3.0分) | A.诱发胃、十二指肠溃疡出血 B.粒细胞减少 C.诱发哮喘 D.高铁血红蛋白血症 E.凝血障碍 |
D.高铁血红蛋白血症 |
2 | 7 | 2014 | 6. 以下不属于心理评估常用方法的是: (3.0分) | A.会谈法 B.实验法 C.察法 D.调查法 |
B.实验法 |
2 | 7 | 2014 | 7. 效度是指测验分数的: (3.0分) | A.一致性 B.可靠性 C.有效性 D.灵敏性 |
C.有效性 |
2 | 7 | 2014 | 8. 关于阿片类药物的叙述哪项不正确:(3.0分) | A.除杜冷丁外,均有心血管稳定作用,剂量过大,将bradycardia, 需要备用阿托品。 B.芬太尼、舒芬太尼等作用于孤束核和臂旁核控制血管紧张素分泌、稳定血流动力学 C.芬太尼、舒芬太尼等作用于下丘脑-垂体-肾上腺,应激反应,降低交感张力 D.细胞、动物、临床均表明芬太尼、舒芬太尼等具有心机保护作用 E.芬太尼因其心肌的负性肌力作用大,不推荐在心脏手术中使用 |
E.芬太尼因其心肌的负性肌力作用大,不推荐在心脏手术中使用 |
2 | 7 | 2014 | 9. 慢性骨骼、软组织疼痛的最常见病因: (4.0分) | A.退行性 B.炎症 C.代谢性 D.外伤性 E.感染性 |
A.退行性 |
2 | 7 | 2014 | 10. 现代骨肉瘤治疗方式是(3.0分) | A.化疗+放疗+肿瘤边缘切除 B.新辅助化疗+肿瘤广泛切除性保肢 C.化疗+放疗+肿瘤囊内手术 D.新辅助化疗+根治截肢 |
B.新辅助化疗+肿瘤广泛切除性保肢 |
2 | 7 | 2014 | 11. 骨肿瘤的正确诊断需要:(3.0分) | A.病史与体检+家族史+影像学检查 B.家族史+病理检查+影像学检查 C.病史与体检+病理检查+影像学检查 D.影像学检查+穿刺 |
C.病史与体检+病理检查+影像学检查 |
2 | 7 | 2014 | 12. 下列哪种不是内脏痛的主要特点 (3.0分) | A.对锐器切割等刺激不敏感 B.对炎症等刺激不敏感 C.定位不精确 D.可伴有牵涉痛 E.伴有不愉快的情绪反应 |
B.对炎症等刺激不敏感 |
2 | 7 | 2014 | 13. 牵涉痛是指 (3.0分) | A.内脏疾病引起相邻脏器的疼痛 B.手术牵拉脏器引起的疼痛 C.神经疼痛向体表投射 D.按压体表引起部分内脏疼痛 E.内脏疾病引起体表某一部位的疼痛或痛觉过敏 |
E.内脏疾病引起体表某一部位的疼痛或痛觉过敏 |
2 | 7 | 2014 | 14. 内脏痛的特点不包括 (3.0分) | A.伴有不愉快的情绪反应 B.对锐器等皮肤刺激敏感 C.定位不精确 D.可有牵涉痛 E.对牵拉敏感 |
B.对锐器等皮肤刺激敏感 |
2 | 7 | 2014 | 15. 传导慢痛的外周神经纤维主要是 (3.0分) | A.Aα纤维 B.Aδ纤维 C.B 类纤维 D.C 类纤维 E.Aγ纤维 |
D.C 类纤维 |
2 | 7 | 2014 | 16. 闸门学说中兴奋SG细胞的是(3.0分) | A.Aα纤维 B.Aδ纤维 C.B 类纤维 D.C 类纤维 E.Aβ纤维 |
E.Aβ纤维 |
2 | 7 | 2014 | 17. 下列哪种药物是阿片受体的拮抗剂,能逆转芬太尼等过量导致的呼吸抑制(3.0分) | A.吗啡 B.纳洛酮 C.瑞芬太尼 D.普鲁卡因 E.美沙酮 |
B.纳洛酮 |
2 | 7 | 2014 | 18. 关于常用局麻药布比卡因,哪项叙述不正确:(3.0分) | A.布比卡因是长效麻醉药 B.属于酯类局麻药 C.麻醉效能与其脂溶性密切相关 D.布比卡因PKa值为8.1 E.蛋白结合率高 |
B.属于酯类局麻药 |
2 | 7 | 2014 | 19. 下列哪种药物不是通过阿片受体发挥镇痛作用(3.0分) | A.吗啡 B.芬太尼 C.瑞芬太尼 D.普鲁卡因 E.美沙酮 |
D.普鲁卡因 |
2 | 7 | 2014 | 20. 下列哪项错误,利多卡因的主要麻醉用途(3.0分) | A.0.5%-1%局部浸润麻醉 B.5-10%局部浸润麻醉 C.1-2%外周神经阻滞 D.1-2%硬膜外麻醉 E.胶或软膏作表面麻醉 |
B.5-10%局部浸润麻醉 |
2 | 7 | 2014 | 21. 哪项叙述正确(3.0分) | A.局麻药中加入肾上腺素的目的是升高血压、避免血管扩张 B.在阴茎等局部神经阻滞时,麻药中要加入一定浓度的肾上腺素 C.利多卡因用于局部浸润麻醉时的浓度是0.5-1%,肾上腺素浓度为1:200000 D.利多卡因用于局部浸润麻醉时的浓度是0.5-1%,肾上腺素浓度为1:2000 E.丁卡因由于心脏毒性大,在局部浸润麻醉时的浓度低为0.25-0.5%,肾上腺素浓度为1:2000 |
C.利多卡因用于局部浸润麻醉时的浓度是0.5-1%,肾上腺素浓度为1:200000 |
2 | 7 | 2014 | 22. 下列哪项不正确: (3.0分) | A.阿片受体是G蛋白偶联受体 B.阿片受体能传导跨膜信号,激活MAPK,抑制腺苷酸环化酶等 C.阿片受体活化后,N型钙通道受到抑制,钾通道开放 D.丁丙诺非是μ受体部分激动剂 E.阿片受体部分激动-拮抗剂不会引起呼吸抑制,故临床上低氧血症发生几无。 |
E.阿片受体部分激动-拮抗剂不会引起呼吸抑制,故临床上低氧血症发生几无。 |
2 | 7 | 2014 | 23. 对待晚期癌痛病人的疼痛治疗 (3.0分) | A.早期就要用强效阿片药物 B.个体化用药 C.阿片药物不会出现恶心呕吐,故治疗中无肠梗阻病人不需要给予止吐药物 D.强啡肽是近年来人工合成的阿片受体激动剂 E.强啡肽是近年来人工合成的阿片受体拮抗剂 |
B.个体化用药 |
2 | 7 | 2014 | 24. 地佐辛联合芬太尼使用(3.0分) | A.可减少芬太尼用量 B.术中芬太尼用量不变 C.镇痛作用减弱 D.镇痛作用增强不明显,且部分拮抗 E.不良反应发生率明显增加,尤其是呼吸抑制 |
A.可减少芬太尼用量 |
2 | 7 | 2014 | 25. 局麻药的叙述哪项不正确:(3.0分) | A.目前临床的局麻药分为酯类和酰胺类 B.典型的局麻药分子结构的中间链可为酯链或酰胺链 C.足够浓度的局麻药可以阻断相应部位的神经肌肉膜内电冲动的传导 D.全身应用局麻药不会影响心肌、平滑肌等功能,但是,对外周和中枢神经系统及心脏特殊传导系统的冲动传递有作用。 E.局麻药中毒反应中以中枢和心血管系统受累为主 |
D.全身应用局麻药不会影响心肌、平滑肌等功能,但是,对外周和中枢神经系统及心脏特殊传导系统的冲动传递有作用。 |
2 | 7 | 2014 | 26. SAS评估以下什么内容 (3.0分) | A.生活事件 B.抑郁症状 C.心理症状 D.焦虑症状 |
D.焦虑症状 |
2 | 7 | 2014 | 27. 对标准化心理测验应具备的条件而言,下列哪项是错误的(3.0分) | A.效度、信度 B.常模 C.因人制宜的测试条 D.统一的计分标准 |
C.因人制宜的测试条 |
2 | 7 | 2014 | 28. 行为治疗的目的是(3.0分) | A.纠正不适应的认知模式 B.改变不适应的行为模式 C.重现童年期的心理冲突 D.发挥个人的潜能 |
B.改变不适应的行为模式 |
2 | 7 | 2014 | 29. 系统脱敏疗法的步骤不包括 (3.0分) | A.放松训练 B.列出等级脱敏表 C.暴露于强烈的焦虑环境中 D.放松状态下脱敏 |
C.暴露于强烈的焦虑环境中 |
2 | 7 | 2014 | 30. 心理治疗的基本过程有(3.0分) | A.探索问题、心理测验、治疗行动、疗效评价和结束巩固 B.建立关系、分析认识、治疗行动、疗效评价和结束巩固 C.建立关系、探索问题、心理测验、治疗行动和疗效评价 D.探索问题、分析认识、治疗行动、疗效评价和结束巩固 |
D.探索问题、分析认识、治疗行动、疗效评价和结束巩固 |
2 | 7 | 2014 | 31. 来访者中心疗法不主张(3.0分) | A.发展来访者主观能动性 B.对来访者明确指导 C.促进来访者成长 D.医生与来访者是平等"角色" |
B.对来访者明确指导 |
2 | 7 | 2014 | 32. 支持疗法的理论基础中, 不包括(3.0分) | A.认知调整 B.善用"资源" C.提高应对能力 D.挖掘幼年体验 |
D.挖掘幼年体验 |
2 | 7 | 2014 | 33. 生物反馈疗法的理论基础是 (3.0分) | A.生理学、信息论和控制论 B.行为学习理论、生理学和信息论 C.行为学习理论、信息论和控制论 D.生物学、行为学习理论和信息论 |
C.行为学习理论、信息论和控制论 |
2 | 7 | 2015 | 1. 一定单位体积内骨组织有机成分正常,而钙盐含量减少,见于(4.0分) | A.骨质破坏 B.骨质疏松 C.骨质软化 D.骨质关节进行性变 E.骨质增生硬化 |
C.骨质软化 |
2 | 7 | 2015 | 2. 转移性骨肿瘤的X线表现,下列哪项错误? (4.0分) | A.骨质破坏可单发或多发 B.一般无骨膜增生,软组织肿块 C.常伴病理性骨折 D.脊柱,骨盘,肋骨多见 E.椎体破坏,椎间隙变窄 |
E.椎体破坏,椎间隙变窄 |
2 | 7 | 2015 | 3. 下列哪项正确(4.0分) | A.术中输注瑞芬太尼的病人停药后,出现疼痛、镇痛药物需要量增加的原因与成瘾性有关 B.芬太尼对心血管的影响很大 C.小剂量的阿片类药物即可具有较好的神经病理疼痛治疗效果 D.单用阿片类药物维持镇痛镇静,与心脏手术术中知晓无关 E.芬太尼贴片对术后切口痛无效 |
C.小剂量的阿片类药物即可具有较好的神经病理疼痛治疗效果 |
2 | 7 | 2015 | 4. 骨肿瘤的分类是以( )分类(4.0分) | A.解剖部位 B.间室 C.组织来源 D.遗传来源 E.影像学 |
C.组织来源 |
2 | 7 | 2015 | 5. Aspirin的不良反应不包括 (4.0分) | A.诱发胃、十二指肠溃疡出血 B.粒细胞减少 C.诱发哮喘 D.高铁血红蛋白血症 E.凝血障碍 |
D.高铁血红蛋白血症 |
2 | 7 | 2015 | 6. 骨关节炎的特点,以下哪项错误? (4.0分) | A.关节面骨质硬化 B.骨质疏松 C.关节面下小囊变 D.关节间隙不对称变窄 |
B.骨质疏松 |
2 | 7 | 2015 | 7. 良性骨巨细胞瘤的X线所见,选出一项错误的 (4.0分) | A.好发于四肢长骨的骨端 B.早期呈偏心性骨质破坏 C.典型者呈皂泡样多房性改变 D.周围可见薄层骨壳形成 E.邻近有针样瘤骨 |
E.邻近有针样瘤骨 |
2 | 7 | 2015 | 8. 关于阿片类药物的叙述哪项不正确:(4.0分) | A.除杜冷丁外,均有心血管稳定作用,剂量过大,将bradycardia, 需要备用阿托品。 B.芬太尼、舒芬太尼等作用于孤束核和臂旁核控制血管紧张素分泌、稳定血流动力学 C.芬太尼、舒芬太尼等作用于下丘脑-垂体-肾上腺,应激反应,降低交感张力 D.细胞、动物、临床均表明芬太尼、舒芬太尼等具有心机保护作用 E.芬太尼因其心肌的负性肌力作用大,不推荐在心脏手术中使用 |
E.芬太尼因其心肌的负性肌力作用大,不推荐在心脏手术中使用 |
2 | 7 | 2015 | 9. 慢性骨骼、软组织疼痛的最常见病因: (4.0分) | A.退行性 B.炎症 C.代谢性 D.外伤性 E.感染性 |
A.退行性 |
2 | 7 | 2015 | 10. 现代骨肉瘤治疗方式是: (4.0分) | A.化疗+放疗+肿瘤边缘切除 B.新辅助化疗+肿瘤广泛切除性保肢 C.化疗+放疗+肿瘤囊内手术 D.新辅助化疗+根治截肢 |
B.新辅助化疗+肿瘤广泛切除性保肢 |
2 | 7 | 2015 | 11. 骨肿瘤的正确诊断需要(4.0分) | A.病史与体检+家族史+影像学检查 B.家族史+病理检查+影像学检查 C.病史与体检+病理检查+影像学检查 D.影像学检查+穿刺 |
C.病史与体检+病理检查+影像学检查 |
2 | 7 | 2015 | 12. 以下不属于致痛物质的是 (4.0分) | A.K+ B.ATP C.AMP D.组胺 E.阿片肽 |
C.AMP |
2 | 7 | 2015 | 13. 牵涉痛是指 (4.0分) | A.内脏疾病引起相邻脏器的疼痛 B.手术牵拉脏器引起的疼痛 C.神经疼痛向体表投射 D.按压体表引起部分内脏疼痛 E.内脏疾病引起体表某一部位的疼痛或痛觉过敏 |
E.内脏疾病引起体表某一部位的疼痛或痛觉过敏 |
2 | 7 | 2015 | 14. 内脏痛的特点不包括 (4.0分) | A.伴有不愉快的情绪反应 B.对锐器等皮肤刺激敏感 C.定位不精确 D.可有牵涉痛 E.对牵拉敏感 |
B.对锐器等皮肤刺激敏感 |
2 | 7 | 2015 | 15. 在痛觉传入通路中,调控痛觉信号的"闸门"位于(4.0分) | A.脊髓后索 B.脊髓后角 C.延髓薄束核和楔束核 D.脑干网状结构 E.丘脑髓板内核群 |
B.脊髓后角 |
2 | 7 | 2015 | 16. 闸门学说中兴奋SG细胞的是 (4.0分) | A.Aα纤维 B.Aδ纤维 C.B 类纤维 D.C 类纤维 E.Aβ纤维 |
E.Aβ纤维 |
2 | 7 | 2015 | 17. 下列哪种药物是阿片受体的拮抗剂,能逆转芬太尼等过量导致的呼吸抑制(4.0分) | A.吗啡 B.纳洛酮 C.瑞芬太尼 D.普鲁卡因 E.美沙酮 |
B.纳洛酮 |
2 | 7 | 2015 | 18. 关于常用局麻药布比卡因,哪项叙述不正确: (4.0分) | A.布比卡因是长效麻醉药 B.属于酯类局麻药 C.麻醉效能与其脂溶性密切相关 D.布比卡因PKa值为8.1 E.蛋白结合率高 F. |
B.属于酯类局麻药 |
2 | 7 | 2015 | 19. 下列哪种药物不是通过阿片受体发挥镇痛作用(4.0分) | A.吗啡 B.芬太尼 C.瑞芬太尼 D.普鲁卡因 E.美沙酮 |
D.普鲁卡因 |
2 | 7 | 2015 | 20. 下列哪项错误,利多卡因的主要麻醉用途:(4.0分) | A.0.5%-1%局部浸润麻醉 B.5-10%局部浸润麻醉 C.1-2%外周神经阻滞 D.1-2%硬膜外麻醉 E.胶或软膏作表面麻醉 |
B.5-10%局部浸润麻醉 |
2 | 7 | 2015 | 21. 哪项叙述正确(4.0分) | A.局麻药中加入肾上腺素的目的是升高血压、避免血管扩张 B.在阴茎等局部神经阻滞时,麻药中要加入一定浓度的肾上腺素 C.利多卡因用于局部浸润麻醉时的浓度是0.5-1%,肾上腺素浓度为1:200000 D.利多卡因用于局部浸润麻醉时的浓度是0.5-1%,肾上腺素浓度为1:2000 E.丁卡因由于心脏毒性大,在局部浸润麻醉时的浓度低为0.25-0.5%,肾上腺素浓度为1:2000 F.临床常用的5种局麻药无神经毒性作用 |
C.利多卡因用于局部浸润麻醉时的浓度是0.5-1%,肾上腺素浓度为1:200000 |
2 | 7 | 2015 | 22. 下列哪项不正确(4.0分) | A.阿片受体是G蛋白偶联受体 B.阿片受体能传导跨膜信号,激活MAPK,抑制腺苷酸环化酶等 C.阿片受体活化后,N型钙通道受到抑制,钾通道开放 D.丁丙诺非是μ受体部分激动剂 E.阿片受体部分激动-拮抗剂不会引起呼吸抑制,故临床上低氧血症发生几无。 |
E.阿片受体部分激动-拮抗剂不会引起呼吸抑制,故临床上低氧血症发生几无。 |
2 | 7 | 2015 | 23. 对待晚期癌痛病人的疼痛治疗(4.0分) | A.早期就要用强效阿片药物 B.个体化用药 C.阿片药物不会出现恶心呕吐,故治疗中无肠梗阻病人不需要给予止吐药物 D.强啡肽是近年来人工合成的阿片受体激动剂 E.强啡肽是近年来人工合成的阿片受体拮抗剂 |
B.个体化用药 |
2 | 7 | 2015 | 24. 地佐辛联合芬太尼使用 (4.0分) | A.可减少芬太尼用量 B.术中芬太尼用量不变 C.镇痛作用减弱 D.镇痛作用增强不明显,且部分拮抗 E.不良反应发生率明显增加,尤其是呼吸抑制 |
A.可减少芬太尼用量 |
2 | 7 | 2015 | 25. 局麻药的叙述哪项不正确(4.0分) | A.目前临床的局麻药分为酯类和酰胺类 B.典型的局麻药分子结构的中间链可为酯链或酰胺链 C.足够浓度的局麻药可以阻断相应部位的神经肌肉膜内电冲动的传导 D.全身应用局麻药不会影响心肌、平滑肌等功能,但是,对外周和中枢神经系统及心脏特殊传导系统的冲动传递有作用。 E.局麻药中毒反应中以中枢和心血管系统受累为主 |
D.全身应用局麻药不会影响心肌、平滑肌等功能,但是,对外周和中枢神经系统及心脏特殊传导系统的冲动传递有作用。 |
2 | 8 | 2007 | 1. 下述阿司匹林作用中哪些是错误的? (3.0分) | A.抑制白三烯合成 B.解热镇痛作用 C.抑制血小板聚集 D.抑制前列腺素合成 |
A.抑制白三烯合成 |
2 | 8 | 2007 | 2. 骨髓炎最常见的致病菌是:(3.0分) | A.表皮葡萄球菌 B.金黄色葡萄球菌 C.乙型链球菌 D.大肠杆菌 |
B.金黄色葡萄球菌 |
2 | 8 | 2007 | 3. 股骨头坏死的早期诊断较可靠的方法是: (3.0分) | A.CT检查 B.X线片检查 C.B超 D.MR检查 |
D.MR检查 |
2 | 8 | 2007 | 4. 关于疼痛下列哪项是错误的?(3.0分) | A.疼痛是-种主观的感觉 B.疼痛是一种不愉快的感觉和情绪上感受,伴随着现有的或潜在的组织损伤。 C.疼痛可依部位、原因和性质等分类 D.慢性疼痛是指疼痛持续时间超过1个月的疼痛 E.疼痛已被列为继呼吸、脉搏、血压、体温之后的第五大生命体征 |
D.慢性疼痛是指疼痛持续时间超过1个月的疼痛 |
2 | 8 | 2007 | 5. 8岁男孩,左髋部肿痛,跛行,伴低热、盗汗、食欲不振3周。查体:体温37.6℃,左髋部活动受限,Thomas征阳性。髋关节x线片见关节间隙略窄,边缘性骨破坏。其诊断首先应考虑为: (3.0分) | A.急性骨髓炎 B.股骨头坏死 C.骨性关节炎 D.髋关节结核 E.急性化脓性关节炎 |
D.髋关节结核 |
2 | 8 | 2007 | 6. 下列不是用于治疗类风湿性关节炎的药物是 (3.0分) | A.甲氨蝶呤 B.秋水仙碱 C.布洛芬 D.阿司匹林 |
B.秋水仙碱 |
2 | 8 | 2007 | 7. 下列腕管综合征首先出现的临床表现中错的有: (3.0分) | A.持物无力 B.桡侧的二个手指指端麻木或疼痛 C.夜间或清晨症状最重 D.适当抖动手腕症状可以减轻 E.有时疼痛科牵涉到前臂。 |
B.桡侧的二个手指指端麻木或疼痛 |
2 | 8 | 2007 | 8. 根据骨折线的形状分类,下述哪一项是错误的:(3.0分) | A.斜形骨折 B.螺旋骨折 C.青枝骨折 D.嵌入骨折 |
C.青枝骨折 |
2 | 8 | 2007 | 9. CT检查的优势不包括下述哪项?(3.0分) | A.显示复杂部位的骨折 B.具有较高的空间分辨力 C.可以作为穿刺活检的介导手段 D.显示软组织结构较好 |
B.具有较高的空间分辨力 |
2 | 8 | 2007 | 10. 下列哪些不是弥漫性结缔组织病的特点: (3.0分) | A.早期治疗可单用非甾体类抗炎药; B.慢性病程,病情迁延反复; C.往往激素和免疫抑制剂治疗有效; D.往往多系统受累,病情复杂难辨; |
A.早期治疗可单用非甾体类抗炎药; |
2 | 8 | 2007 | 11. 急性血源性骨髓炎好发部位: (3.0分) | A.骨干部 B.干骺端 C.骨骺 D.骨膜 |
B.干骺端 |
2 | 8 | 2007 | 12. 20岁女性以1个月的发热、多关节痛和颜面皮疹就诊,无关节肿,伴有口腔溃疡,当地医院查血三系减少伴有血沉、C反应蛋白升高,考虑结缔组织病不能除外,确诊需要的最重要的化验检查是: (3.0分) | A.类风湿因子 B.免疫球蛋白 C.抗核抗体系列 D.补体 |
C.抗核抗体系列 |
2 | 8 | 2007 | 13. 痛风性关节炎与感染性关节炎鉴别的直接有力依据(3.0分) | A.血清尿酸水平 B.关节X线或双源CT C.关节液检查 D.血沉 |
C.关节液检查 |
2 | 8 | 2007 | 14. 关于风湿性疾病的发病机制,下列说法不正确的是 (3.0分) | A.由于风湿性疾病患者体内往往存在大量自身抗体,故该病与体液免疫系统紊乱相关; B.多基因疾病,非遗传疾病; C.多种细胞因子、蛋白酶及血管内皮细胞参与发病机制,可作为治疗靶点; D.自身抗体往往通过直接致病作用直接导致疾病发生,且一种自身抗体导致一种特异性临床症状。 |
D.自身抗体往往通过直接致病作用直接导致疾病发生,且一种自身抗体导致一种特异性临床症状。 |
2 | 8 | 2007 | 15. 65岁男性,1天前劳累后出现右踝关节红肿热痛,伴行走困难。无病前外伤、扭伤史。2年前右第一跖趾关节曾有类似发作,约3天后自行缓解,未予诊治。该患者最可能的诊断(3.0分) | A.细菌性关节炎 B.结核性关节炎 C.骨关节炎 D.痛风性关节炎 |
D.痛风性关节炎 |
2 | 8 | 2007 | 16. The most common tears of menisci in the surgery are: (3.0分) | A.combination tears B.tears with cystic menisci C.transverse and oblique tears D.longitudinal tears |
D.longitudinal tears |
2 | 8 | 2007 | 17. Osgood-Schlatter disease is the osteochondrol disease of the(3.0分) | A.menisci B.head of the femur C.tibial tubercle D.epicondyle of the humerus E.hip joint |
C.tibial tubercle |
2 | 8 | 2007 | 18. 关于影响手术后疼痛程度的因素,下列哪项是不正确的? (3.0分) | A.手术的部位、手术时间 B.麻醉处理质量 C.术前疼痛的治疗 D.手术切口的类型和长度和深度 E.患病时间长短 |
E.患病时间长短 |
2 | 8 | 2007 | 19. 下面哪种病理变化属于类风湿关节炎的基本病理变化: (4.0分) | A.肌腱端炎 B.血管炎 C.滑膜炎 D.软骨退变 |
C.滑膜炎 |
2 | 8 | 2007 | 20. 以下哪种影像检查存在图像重叠的现象?(3.0分) | A.CT B.DR C.超声 D.MRI |
B.DR |
2 | 8 | 2007 | 21. MRI检查最突出的优点是:(3.0分) | A.良好的软组织对比 B.直接三轴成像 C.无电离辐射 D.无需造影剂直接显示血管结构 |
A.良好的软组织对比 |
2 | 8 | 2007 | 22. 急性化脓性骨髓炎早期的MRI征象不包括:(3.0分) | A.死骨形成 B.骨膜下脓肿形成 C.骨髓水肿致信号异常 D.患骨周围软组织肿胀 |
A.死骨形成 |
2 | 8 | 2007 | 23. 股骨头缺血性坏死早期MRI诊断的病理基础是:(3.0分) | A.肉芽组织增生 B.骨质破坏、吸收 C.脂肪细胞坏死 D.骨髓细胞坏死 |
D.骨髓细胞坏死 |
2 | 8 | 2007 | 24. 肩关节撞击综合征常见的原因不包括:(3.0分) | A.肩峰形态异常 B.喙肩韧带肥厚 C.肩峰下滑囊增厚 D.肩锁关节退变 |
C.肩峰下滑囊增厚 |
2 | 8 | 2007 | 25. 下述哪项造影方法属于无创性检查 (3.0分) | A.MRI血管造影 B.CT血管造影 C.关节造影 D.椎间盘造影 |
A.MRI血管造影 |
2 | 8 | 2007 | 26. 骨关节影像技术选择原则,错误的是:(3.0分) | A.需要结合临床材料(如病人年龄、性别、症状及实验室检查等)做出诊断; B.尽可能选择辐射剂量低、安全性高的检查方法; C.选择能对病变作出明确诊断和评价的检查方法,避免不必要的检查; D.在尽可能短的检查经过内,以最小的检查费用以及对最小的损害,作出精细的影像诊断。 |
B.尽可能选择辐射剂量低、安全性高的检查方法; |
2 | 8 | 2007 | 27. 下列哪个不是炎性腰背痛的特点(3.0分) | A.休息后加重 B.晨僵 C.活动后缓解 D.起病急 |
D.起病急 |
2 | 8 | 2007 | 28. 关于疼痛的评估描述错误的是(3.0分) | A.疼痛的评估包括对全过程中不同因素相互作用的测量 B.疼痛的测量是指应用某些测量标准对疼痛的强度进行测量 C.通过评估和测量,可以客观地得到患者的疼痛程度 D.来自患者本人诉说的疼痛是可靠而有效的测量 E.临床工作中以VAS等最为常用 |
C.通过评估和测量,可以客观地得到患者的疼痛程度 |
2 | 8 | 2007 | 29. 关于疼痛的控制下列哪项是错误的? (3.0分) | A.提倡平衡镇痛和多模式镇痛 B.疼痛评分超过7分的疼痛需要及时有效的控制。 C.患者的参与是取得良好镇痛效果的重要条件 D.不需要考虑个体化镇痛 E.疼痛治疗宜尽早进行,彻底镇痛 |
D.不需要考虑个体化镇痛 |
2 | 8 | 2007 | 30. 关于非甾体抗炎药下列哪项是错误的? (3.0分) | A.分为非选择性NSAIDs和选择性COX-2抑制剂 B.此类药物的血浆蛋白结合率高 C.原则上所有NSAIDs药物均可用于可口服患者的术后轻-中度疼痛的镇痛 D.两种非甾体抗炎药可同时使用 E.可用作为多模式镇痛的组成部分 |
D.两种非甾体抗炎药可同时使用 |
2 | 8 | 2007 | 31. 对疼痛患者进行VAS评分法,评分为6,则表示(3.0分) | A.锐痛 B.较痛 C.无痛 D.中度疼痛 E.最痛 |
D.中度疼痛 |
2 | 8 | 2007 | 32. 手指的天鹅颈畸形属于(3.0分) | A.陈旧伸指肌腱远止点断裂 B.痛风关节炎晚期 C.风湿性关节炎晚期 D.骨关节炎晚期 |
C.风湿性关节炎晚期 |
2 | 8 | 2007 | 33. 下列哪项不是镇痛并发症: (3.0分) | A.切口裂开 B.可导致低氧血症和高碳酸血症 C.可导致术后低血压 D.可导致恶心、呕吐 E.可导致便秘 |
A.切口裂开 |
2 | 8 | 2008 | 1. 腰右腿痛1月,加剧1周。体检所见:右侧小腿后侧痛觉略减、同侧足背伸趾、伸拇肌肌力无改变,同侧跟腱反射减弱,病理反射未引出。该患者可能的诊断是(3.0分) | A.腰1/2椎间盘突出症 B.腰2/3极外侧椎间盘突出症 C.腰3/4椎间盘突出症 D.腰4/5椎间盘突出症 E.腰5骶1椎间盘突出症 |
E.腰5骶1椎间盘突出症 |
2 | 8 | 2008 | 2. 该患者进一步常用的辅助检查为: 1.腰椎X线正侧位片 2.腰椎CT检查 3.腰椎MRI检查4.肌电图检查(3.0分) | A.1,2,3 B.1,3 C.2,4 D.1,2,3,4 E.4 |
A.1,2,3 |
2 | 8 | 2008 | 3. 神经根型颈椎病患者的临床表现为单侧肩部、上臂及肘部外侧疼痛,查体有同侧相同区域感觉减退,三角肌、肱二头肌肌力略减退。该患者最有可能的受压神经根是: (3.0分) | A.颈3神经根 B.颈4神经根 C.颈5神经根 D.颈6神经根 E.颈7神经根 |
C.颈5神经根 |
2 | 8 | 2008 | 4. 对该患者需行鉴别诊断的疾患有:1腕管综合症 2运动神经元疾病 3尺管综合症 4腋神经卡压综合症(3.0分) | A.1,2,3 B.1,3 C.2,4 D.1,2,3,4 E.4 |
C.2,4 |
2 | 8 | 2008 | 5. 关于风湿性疾病的发病机制,下列说法不正确的是(3.0分) | A.多基因疾病,并非遗传疾病; B.由于风湿性疾病患者体内往往存在大量自身抗体,故该病与体液免疫系统紊乱相关; C.多种细胞因子、蛋白酶及血管内皮细胞参与发病机制,可作为治疗靶点; D.自身抗体往往通过直接致病作用直接导致疾病发生,且一种自身抗体导致一种特异性临床症状。 |
D.自身抗体往往通过直接致病作用直接导致疾病发生,且一种自身抗体导致一种特异性临床症状。 |
2 | 8 | 2008 | 6. 30岁女性主诉一年双腕及双膝关节痛伴颈肩腰背痛,伴晨僵约60分钟,持重及行走下楼均有困难。查体:双腕关节略肿胀,伴有轻度活动受限及压痛,双膝轻度肿胀,活动度可,浮髌征阳性,未及关节摩擦感。查血沉50mm/第1h, CRP30mg/dl,RF56U/L(正常值<25U/L, 抗CCP抗体79IU/L(正常<25U/L),抗核抗体阴性,她最可能的诊断是: (3.0分) | A.骨关节炎 B.类风湿关节炎 C.脊柱关节病 D.痛风性关节炎 |
B.类风湿关节炎 |
2 | 8 | 2008 | 7. 心理治疗不适用于: (3.0分) | A.心身疾病的病人 B.急性疾病的病人 C.适应困难的病人 D.精神分裂症病人 |
D.精神分裂症病人 |
2 | 8 | 2008 | 8. 系统脱敏疗法的步骤不包括: (3.0分) | A.放松训练 B.列出等级脱敏表 C.直接面对最焦虑的情境 D.脱敏训练 |
C.直接面对最焦虑的情境 |
2 | 8 | 2008 | 9. 精神分析治疗的基本内容不包括: (3.0分) | A.阻抗 B.移情 C.释梦 D.催眠 |
D.催眠 |
2 | 8 | 2008 | 10. 关于厌恶疗法,以下不正确的是: (3.0分) | A.其理论基础是惩罚作用 B.又称"代币法" C.可以选择想象的厌恶刺激 D.刺激要达到明确厌恶的水平 |
B.又称"代币法" |
2 | 8 | 2008 | 11. The Osgood_Schlatter disease is the Osteochondrol disease of the (4.0分) | A.tibial tubercle B.head of the femur C.menisci D.epicondyle of the humerus |
A.tibial tubercle |
2 | 8 | 2008 | 12. The most common tears of menisci in the surgery are:(3.0分) | A.combination tears B.transverse and oblique tears C.tears with cystic menisci D.longitudinal tears |
D.longitudinal tears |
2 | 8 | 2008 | 13. In Lundberg's classification system, Frozen shoulders in patients who report no inciting event and with no abnormality on examination or plain roentgenograms were designated as _________. (3.0分) | A.Primary frozen shoulder B.secondary frozen shoulder C.early frozen shoulder D.late frozen shoulder |
A.Primary frozen shoulder |
2 | 8 | 2008 | 14. 14岁男孩,平素喜踢足球,近一周右小腿上端前侧疼痛。查体:右小腿上端前侧隆起,皮肤无红肿发热及静脉怒张,压痛明显。X线片示胫骨结节骨骺增大、致密。应首先考虑的诊断是胫骨结节: (3.0分) | A.撕脱骨折 B.骨髓炎 C.骨结核 D.骨软骨瘤 E.骨软骨病 |
E.骨软骨病 |
2 | 8 | 2008 | 15. 肩周炎的自然病程一般在______时间左右能自愈。(3.0分) | A.1月 B.2月 C.3月 D.6月 E.12月 |
E.12月 |
2 | 8 | 2008 | 16. 下列Legg-Calve-Perthes病的治疗原则中错的是(3.0分) | A.应使股骨头完全包容在髋臼内 B.避免髋臼外上缘对股骨头的局限性压应力 C.减轻对股骨头的压力 D.维持髋关节有良好的活动范围 E.加强早期负重锻炼 |
E.加强早期负重锻炼 |
2 | 8 | 2008 | 17. 下列腕管综合征首先出现的临床表现中错的有: (3.0分) | A.桡侧的二个手指指端麻木或疼痛 B.持物无力 C.夜间或清晨症状最重 D.适当抖动手腕症状可以减轻 E.有时疼痛科牵涉到前臂。 |
A.桡侧的二个手指指端麻木或疼痛 |
2 | 8 | 2008 | 18. 对运动系统慢性损伤的治疗,临床上最常用的行之有效的方法是: (3.0分) | A.限制致伤动作,纠正不良姿势和增强肌力等 B.理疗,按摩等方法 C.局部注射肾上腺皮质类固醇 D.非甾体类抗炎药 E.手术治疗 |
A.限制致伤动作,纠正不良姿势和增强肌力等 |
2 | 8 | 2008 | 19. 有关滑囊炎,下列哪项是错误的: (3.0分) | A.骨结构异常突出的部位经常受到摩擦和压迫是产生滑囊炎的主要原因 B.瘦弱老妇久坐硬凳致坐骨结节滑囊炎 C.长期穿尖而窄的皮鞋致姆趾滑囊炎 D.包块穿刺常为清晰粘液,如为血性需警惕恶变可能 E.临床上需与结核性、类风湿性滑囊炎相鉴别 |
D.包块穿刺常为清晰粘液,如为血性需警惕恶变可能 |
2 | 8 | 2008 | 20. 关于肱骨外上髁炎的病因及病理变化,下列哪项是不正确的:(3.0分) | A.肱骨外上髁的骨性隆起慢性创伤性摩擦致滑囊炎 B.肱骨外上髁尖部的筋膜、骨膜炎 C.肱骨外上髁与绕骨头之间的肌筋膜炎 D.肱骨外上髁与桡骨小头的桡肱关节滑膜炎 E.伸指总肌腱深处小血管神经束,穿过肌腱和筋膜时被卡压 |
A.肱骨外上髁的骨性隆起慢性创伤性摩擦致滑囊炎 |
2 | 8 | 2008 | 21. 下列哪项检查对明确针对最有帮助: (3.0分) | A.X线摄片检查 B.肌电图检查 C.MR检查 D.关节镜检查 E.同位素扫描检查 |
C.MR检查 |
2 | 8 | 2008 | 22. 桡骨茎突狭窄性腱鞘炎时,哪项试验阳性 (3.0分) | A.Phalen试验 B.Thomas试验 C.Mills试验 D.Finkelstein试验 E.Dugus试验 |
D.Finkelstein试验 |
2 | 8 | 2008 | 23. 股骨头骨软骨病称为:(3.0分) | A.Calve病 B.Scheuermann病 C.Kienbock病 D.De Quervain病 E.Legg-Calve-Perthes病 |
E.Legg-Calve-Perthes病 |
2 | 8 | 2008 | 24. 急性半月板损伤时,下列哪项治疗是不适宜的: (3.0分) | A.石膏托适当限制膝部活动 B.血肿明显时,可抽尽血肿,压力包扎 C.局部用消肿止痛的中药外敷 D.应及时作半月板摘除术 E.关节镜手术 |
D.应及时作半月板摘除术 |
2 | 8 | 2008 | 25. X线摄影的优势不包括下述哪一项? (3.0分) | A.患者接受的辐射剂量较低 B.可以显示骨骼的三维结构 C.具有较高的空间分辨力 D.检查能显示骨膜的病理变化 |
B.可以显示骨骼的三维结构 |
2 | 8 | 2008 | 26. 对于临床怀疑骨质疏松的患者,首选的影像检查方法是: (3.0分) | A.CT B.同位素检查 C.DR D.MRI |
C.DR |
2 | 8 | 2008 | 27. 以下哪一项属于功能位摄片: (3.0分) | A.胸腰椎全长摄片 B.双下肢负重摄片 C.颈椎双侧斜位摄片 D.下颌骨曲面断层摄片 |
B.双下肢负重摄片 |
2 | 8 | 2008 | 28. 临床怀疑患者左下肢腘动脉闭塞,首选的影像学检查方法是: (3.0分) | A.下肢DSA B.下肢CTA C.下肢MRA D.下肢增强MRA |
B.下肢CTA |
2 | 8 | 2008 | 29. MRI应用于骨肿瘤检查,其最重要的价值在于:(3.0分) | A.判断肿瘤侵犯范围 B.了解肿瘤内坏死程度 C.显示肿瘤周围软组织肿块大小 D.明确肿瘤病理性质 |
A.判断肿瘤侵犯范围 |
2 | 8 | 2008 | 30. 肩关节损伤中,最常受累的肩袖结构是: (3.0分) | A.冈上肌腱 B.冈下肌腱 C.小圆肌腱 D.肩胛下肌腱 |
A.冈上肌腱 |
2 | 8 | 2008 | 31. 肩关节前下盂唇撕裂被称为: (3.0分) | A.Hill-sachs B.Bankart C.SLAP D.ALPSA |
B.Bankart |
2 | 8 | 2008 | 32. 显示半月板组织最佳的MRI成像序列是 (3.0分) | A.T1WI B.T2WI C.PdWI D.GRE |
C.PdWI |
2 | 8 | 2008 | 33. 椎间盘造影的主要目的是:(3.0分) | A.诊断椎间盘突出 B.判断脊髓受压程度 C.判断椎间盘纤维环是否完整 D.确定责任椎间盘 |
D.确定责任椎间盘 |
2 | 8 | 2009 | 1. 骨折愈合时,通常由下列哪种细胞诱导成骨作用,促进愈合。(2.0分) | A.成纤维干细胞 B.骨髓间充质干细胞 C.成骨细胞 D. 成软骨细胞 |
B.骨髓间充质干细胞 |
2 | 8 | 2009 | 2. 儿童骨折的主要方式为: (2.0分) | A.粉碎性骨折 B.压缩性骨折 C.凹陷性骨折 D.青枝骨折 |
D.青枝骨折 |
2 | 8 | 2009 | 3. 下列哪项不是儿童骨折的常见并发症? (2.0分) | A.慢性骨髓炎 B.关节僵直 C.感染性骨缺损 D.畸形愈合 |
B.关节僵直 |
2 | 8 | 2009 | 4. 下列哪种检查是普查先天性髋关节脱位的最有效方法? (2.0分) | A. "弹进"和"弹出"试验 B.X线检查 C.B超检查 D. MRI检查 |
A. "弹进"和"弹出"试验 |
2 | 8 | 2009 | 5. 下列哪种不是先天性髋关节脱位的体征? (2.0分) | A. Thomas征 B.Ortolani征 C.Allis征 D. Trendelenburg征 |
A. Thomas征 |
2 | 8 | 2009 | 6. 运动系统检查法,最基本最基础的是(2.0分) | A.X检查 B.物理学检查 C.化验检查 D. 关节镜检查 |
B.物理学检查 |
2 | 8 | 2009 | 7. 关于直腿抬高试验,下列说法错误的是: (2.0分) | A.检查前,患者仰卧位,下肢伸直 B.一般要左右对比,以显示病变部位。 C.腰椎间盘突出症患者可为阳性。 D.膝关节积液患者亦可出现阳性。 |
D.膝关节积液患者亦可出现阳性。 |
2 | 8 | 2009 | 8. Dugas征阳性的临床意义:(2.0分) | A.肩关节脱位 B. 肘关节脱位 C.髋关节后上脱位 D. 髋关节中心脱位 |
A.肩关节脱位 |
2 | 8 | 2009 | 9. 桡骨茎突狭窄性腱鞘炎,下列哪一体征为阳性: (2.0分) | A.Finkelstein B.Laseque氏征 C.Allis征 D.Babinski氏征 |
A.Finkelstein |
2 | 8 | 2009 | 10. 下列关于四肢病理学检查,描述不正确的是: (2.0分) | A.检查时要充分暴露 B.检查时要双侧对比 C.检查时要多次反复用力操作,避免假阴性。 D.客观检查要与主观症状相结合 |
C.检查时要多次反复用力操作,避免假阴性。 |
2 | 8 | 2009 | 11. The Osgood_Schlatter disease is the Osteochondrol disease of the(2.0分) | A.tibial tubercle B.head of the femur C.menisci D.epicondyle of the humerus |
A.tibial tubercle |
2 | 8 | 2009 | 12. Osgood-Schlatter disease is the osteochondrol disease of the _______.(2.0分) | A.tibial tubercle B.head of the femur C.menisci D.epicondyle of the humerus E.hip joint |
A.tibial tubercle |
2 | 8 | 2009 | 13. 8岁男孩,左髋部肿痛,跛行,伴低热、盗汗、食欲不振3周。查体:体温37.6℃,左髋部活动受限,Thomas征阳性。髋关节x线片见关节间隙略窄,边缘性骨破坏。其诊断首先应考虑为: (2.0分) | A.股骨头坏死 B.髋关节结核 C.急性骨髓炎 D.骨性关节炎 E.急性化脓性关节炎 |
B.髋关节结核 |
2 | 8 | 2009 | 14. 股骨头坏死的早期诊断较可靠的方法是: (2.0分) | A.X线片检查 B.CT检查 C.B超 D.MR检查 |
D.MR检查 |
2 | 8 | 2009 | 15. 腕管综合征是______在腕管内受压而表现的一组症状和体征。 (2.0分) | A.正中神经 B.尺神经 C.桡神经 D.肌皮神经 E.前臂皮神经 |
A.正中神经 |
2 | 8 | 2009 | 16. 颈椎病中哪项正确 (2.0分) | A.颈椎病的转归主要取决于颈椎退行性改变 B.颈椎病一般预后良好 C.颈椎病患者均有颈部症状 D.颈椎病在体力劳动者中多见 E.同一节段的颈椎间盘突出也会有不同临床表现 |
E.同一节段的颈椎间盘突出也会有不同临床表现 |
2 | 8 | 2009 | 17. 该患者最有可能受累的颈椎间隙为: (2.0分) | A.颈2/3 B.颈3/4 C.颈4/5 D.颈5/6 E.颈6/7 |
D.颈5/6 |
2 | 8 | 2009 | 18. 该患者若神经根型颈椎病的诊断明确,则首选的治疗为:1 颈椎牵引 2 非甾体类镇痛药 3颈椎制动 4手术减压(2.0分) | A.1,2,3 B.1,3 C.2,4 D.1,2,3,4 E.4 |
A.1,2,3 |
2 | 8 | 2009 | 19. 对该患者的最有意义的辅助检查是: (2.0分) | A.X线片 B.过伸过屈动力位颈椎X线片 C.CT检查 D.MRI检查 E.颈脊髓造影 |
D.MRI检查 |
2 | 8 | 2009 | 20. 腰椎间盘突出症的定义下列哪项错误: (2.0分) | A.腰椎间盘纤维环退行性改变 B.腰椎间盘髓核退行性改变 C.退变髓核突出是导致临床症状的主要原因 D.腰椎间盘突出物大小与临床症状无相关性 E.腰椎间盘突出症突出症可无临床症状 |
E.腰椎间盘突出症突出症可无临床症状 |
2 | 8 | 2009 | 21. 腰右腿痛1月,加剧1周。体检所见:右侧小腿内侧痛觉略减、同侧足背伸趾、伸拇肌肌力无改变,同侧膝反射减弱,病理反射未引出。该患者可能的诊断是 (2.0分) | A.腰1/2椎间盘突出症 B.腰2/3极外侧椎间盘突出症 C.腰3/4椎间盘突出症 D.腰4/5椎间盘突出症 E.腰5骶1椎间盘突出症 |
C.腰3/4椎间盘突出症 |
2 | 8 | 2009 | 22. 腰椎间盘突出症的手术目标为:1.迅速缓解疼痛 2.改善生活质量 3.减少不必要的和长期的非手术治疗 4.减缓椎间盘退变(2.0分) | A.1,2,3 B.1,3 C.2,4 D.1,2,3,4 E.4 |
A.1,2,3 |
2 | 8 | 2009 | 23. 骨髓炎的传播途径:1.血源性 2.创伤性 3.外来性 4.内源性(2.0分) | A.1,2,3 B.1,3 C.2,4 D.1,2,3,4 E.4 |
A.1,2,3 |
2 | 8 | 2009 | 24. 急性骨髓炎的临床表现为:1.病灶经久不愈 2.死骨与窦道 3.局部红肿热痛 4.中毒性休克(2.0分) | A.1,2,3 B.1,3 C.2,4 D.2,3,4 E.1,2,3,4 |
D.2,3,4 |
2 | 8 | 2009 | 25. 急性骨髓炎的治疗原则包括:1.足量足够疗程抗生素 2.延迟局部切开引流术 3.患肢抬高及固定 4.全身支持疗法 5.对症治疗(2.0分) | A.1,2,3 B.1,3 C.2,4 D.1,2,3,4, E.1,2,3,4,5 |
E.1,2,3,4,5 |
2 | 8 | 2009 | 26. X线摄影的优势不包括下述哪一项?(2.0分) | A.患者接受的辐射剂量较低 B.可以显示骨骼的三维结构 C.具有较高的空间分辨力 D.检查能显示骨膜的病理变化 |
B.可以显示骨骼的三维结构 |
2 | 8 | 2009 | 27. 对于临床怀疑骨质疏松的患者,首选的影像检查方法是:(2.0分) | A.CT B.同位素检查 C.DR D.MRI |
C.DR |
2 | 8 | 2009 | 28. 以下哪一项属于功能位摄片:(2.0分) | A.胸腰椎全长摄片 B.双下肢负重摄片 C.颈椎双侧斜位摄片 D.下颌骨曲面断层摄片 |
B.双下肢负重摄片 |
2 | 8 | 2009 | 29. 临床怀疑患者左下肢腘动脉闭塞,首选的影像学检查方法是:(2.0分) | A.下肢DSA B.下肢CTA C.下肢MRA D.下肢增强MRA |
B.下肢CTA |
2 | 8 | 2009 | 30. 评价腰椎间盘突出症,最好的检查方法是:(2.0分) | A.腰椎侧位片 B.CT轴位断层 C.MRI平扫 D.MRI增强 |
C.MRI平扫 |
2 | 8 | 2009 | 31. MRI应用于骨肿瘤检查,其最重要的价值在于:(2.0分) | A.判断肿瘤侵犯范围 B.了解肿瘤内坏死程度 C.显示肿瘤周围软组织肿块大小 D.明确肿瘤病理性质 |
A.判断肿瘤侵犯范围 |
2 | 8 | 2009 | 32. 肩关节损伤中,最常受累的肩袖结构是:(2.0分) | A.冈上肌腱 B.冈下肌腱 C.小圆肌腱 D.肩胛下肌腱 |
A.冈上肌腱 |
2 | 8 | 2009 | 33. 肩关节前下盂唇撕裂被称为:(2.0分) | A.Hill-sachs B.Bankart C.SLAP D.ALPSA |
B.Bankart |
2 | 8 | 2009 | 34. 显示半月板组织最佳的MRI成像序列是(2.0分) | A.T1WI B.T2WI C.PdWI D.GRE |
C.PdWI |
2 | 8 | 2009 | 35. 椎间盘造影的主要目的是: (2.0分) | A.诊断椎间盘突出 B.判断脊髓受压程度 C.判断椎间盘纤维环是否完整 D.确定责任椎间盘 |
D.确定责任椎间盘 |
2 | 8 | 2009 | 36. 下列哪一项是儿童骨折的特征性X线征象: (2.0分) | A.不全性骨折 B.青枝样骨折 C.肱骨髁上骨折 D.螺旋形骨折 |
B.青枝样骨折 |
2 | 8 | 2009 | 37. X线片上骨性强直的诊断依据为: (2.0分) | A.骨小梁通过关节间隙 B.关节间隙狭窄 C.关节面模糊 D.关节间隙消失 |
A.骨小梁通过关节间隙 |
2 | 8 | 2009 | 38. 关于行为治疗,以下的不正确的是: (2.0分) | A.以行为学习理论为指导 B.重视发挥个人的潜能 C.有一定的治疗程序 D.着眼于改变不适应的行为模式 |
B.重视发挥个人的潜能 |
2 | 8 | 2009 | 39. 下列心理治疗方法中可能产生立竿见影效果的是: (2.0分) | A.精神分析疗法 B.暗示疗法 C.系统脱敏疗法 D.森田疗法 |
B.暗示疗法 |
2 | 8 | 2009 | 40. 心理治疗不适用于对以下哪类病人的心理问题进行干预: (2.0分) | A.肿瘤病人 B.冠心病病人 C.焦虑症病人 D.精神分裂症病人 |
D.精神分裂症病人 |
2 | 8 | 2009 | 41. 来访者中心疗法不主张: (2.0分) | A.发展来访者主观能动性 B. 对来访者明确指导 C.促进来访者成长 D. 治疗是来访者转变和学习的过程 |
B. 对来访者明确指导 |
2 | 8 | 2009 | 42. 支持疗法的干预措施中, 不包括:(2.0分) | A.挖掘幼年体验 B. 善用"资源" C.提高应对能力 D.认知调整 |
A.挖掘幼年体验 |
2 | 8 | 2009 | 43. 下列哪个抗体不属于抗ENA系列? (2.0分) | A.抗双链DNA抗体 B.抗SSA抗体 C.抗Scl-70抗体 D.抗Jo-1抗体 |
A.抗双链DNA抗体 |
2 | 8 | 2009 | 44. 以下哪个指标对类风湿关节炎诊断特异性最高 (2.0分) | A.类风湿因子 B.抗环瓜氨酸多肽抗体 C. 抗核抗体 D.抗中性粒细胞胞浆抗体 |
B.抗环瓜氨酸多肽抗体 |
2 | 8 | 2009 | 45. 患者,女性,32岁,双手2-4近端指间关节及右膝关节痛伴颈肩部疼痛1年,伴晨僵约60分钟,持重及行走下楼均有困难。查体:双腕关节略肿胀,伴有轻度活动受限及压痛,双膝轻度肿胀,活动度可,浮髌征阳性,未及关节摩擦感。查血沉59mm/第1h, CRP36mg/dl,RF156U/L(正常值<25U/L, 抗CCP抗体79IU/L(正常<25U/L),抗核抗体阴性,她最可能的诊断是(2.0分) | A.骨关节炎 B.类风湿关节炎 C.脊柱关节病 D.痛风性关节炎 |
C.脊柱关节病 |
2 | 8 | 2009 | 46. 关于风湿性疾病的发病机制,下列说法正确的是 (2.0分) | A.多为遗传性疾病,存在疾病的致病基因; B.由于风湿性疾病患者体内往往存在炎症,故该病多需要抗生素治疗; C.多种细胞因子、蛋白酶及血管内皮细胞参与发病机制,可作为治疗靶点; D.自身抗体往往通过直接致病作用直接导致疾病发生,且一种自身抗体导致一种特异性临床症状。 |
C.多种细胞因子、蛋白酶及血管内皮细胞参与发病机制,可作为治疗靶点; |
2 | 8 | 2009 | 47. 下列哪个疾病不属于弥漫性结缔组织病: (2.0分) | A.痛风 B.类风湿关节炎 C.系统性红斑狼疮 D.系统性血管炎 |
A.痛风 |
2 | 8 | 2009 | 48. 患者,女性,18岁,间断发热、多关节痛、脱发和泡沫尿1个月,无关节肿,伴有口腔溃疡,当地医院查中度贫血伴有血沉升高,考虑结缔组织病不能除外,确诊需要的最重要的化验检查是: (2.0分) | A.类风湿因子 B.免疫球蛋白 C.抗环瓜氨酸多肽抗体 D.抗核抗体系列 |
D.抗核抗体系列 |
2 | 8 | 2009 | 49. 下列哪个不是类风湿关节炎的特点 (2.0分) | A.是一组以外周关节受累为主的慢性关节炎; B.关节受累往往可发生骨质破坏; C.典型的病理表现为附着点炎或肌腱端炎; D.也可有中轴关节如脊柱受累; |
C.典型的病理表现为附着点炎或肌腱端炎; |
2 | 8 | 2009 | 50. 患者,男,65岁1天前劳累后出现右踝关节红肿热痛,伴行走困难。无病前外伤、扭伤史。2年前右第一跖趾关节曾有类似发作,约3天后自行缓解,未予诊治。该患者最可能的诊断: (2.0分) | A.骨关节炎 B. 丹毒 C.细菌性关节炎 D.痛风性关节炎 |
D.痛风性关节炎 |
2 | 8 | 2010 | 1. 骨折愈合时,通常由下列哪种细胞诱导成骨作用,促进愈合。(2.0分) | A.成纤维干细胞 B.骨髓间充质干细胞 C.成骨细胞 D.成软骨细胞 |
B.骨髓间充质干细胞 |
2 | 8 | 2010 | 2. 以下骨骼哪种主要以骨膜内成骨的方式发育?(2.0分) | A.肱骨 B.股骨 C.额骨 D.腓骨 |
C.额骨 |
2 | 8 | 2010 | 3. 儿童骨折的主要方式为:(2.0分) | A.粉碎性骨折 B.压缩性骨折 C.凹陷性骨折 D.青枝骨折 |
D.青枝骨折 |
2 | 8 | 2010 | 4. 下列哪种维生素的缺乏将严重影响骨重吸收和改建的功能?(2.0分) | A.维生素A B.维生素C C.维生素D D.维生素E |
A.维生素A |
2 | 8 | 2010 | 5. 下列哪项不是儿童骨折的常见并发症?(2.0分) | A.慢性骨髓炎 B.关节僵直 C.感染性骨缺损 D.畸形愈合 |
B.关节僵直 |
2 | 8 | 2010 | 6. 下列哪项不是髋关节脱位的体征?(2.0分) | A.患侧肢体常处于屈曲位,不能伸直 B.双下肢不等长,患侧肢体缩短 C.患侧皮皱增多,会阴部增宽 D.患侧肢体皮肤麻木,感觉异常 |
D.患侧肢体皮肤麻木,感觉异常 |
2 | 8 | 2010 | 7. 下列哪种检查是普查先天性髋关节脱位的最有效方法?(2.0分) | A. "弹进"和"弹出"试验 B.X线检查 C.B超检查 D.MRI检查 |
B.X线检查 |
2 | 8 | 2010 | 8. 某患儿出生后一个月发现髋关节活动受限,考虑先髋,以下哪种检查不能使用?(2.0分) | A.髋关节屈曲外展试验 B.Ortolani及Barbow试验 C.B超检查 D.X线检查 |
D.X线检查 |
2 | 8 | 2010 | 9. 下列哪种不是先天性髋关节脱位的体征?(2.0分) | A.Thomas征 B.Ortolani征 C.Allis征 D.Trendelenburg征 |
A.Thomas征 |
2 | 8 | 2010 | 10. 关于先天性髋关节脱位的治疗,以下说法正确的有:(2.0分) | A.先天性髋关节脱位应该尽早治疗,治疗越早,效果越佳。 B.先天性髋关节脱位应在两周岁以后开始治疗,因为部分患儿可自行恢复。 C.先天性髋关节脱位成人期,治疗效果不佳,故主张保守治疗。 D.使用带蹬吊带发治疗,可限制髋关节的所有活动,有利于治疗。 |
A.先天性髋关节脱位应该尽早治疗,治疗越早,效果越佳。 |
2 | 8 | 2010 | 11. 运动系统检查法,最基本最基础的是 (2.0分) | A.X检查 B.物理学检查 C.化验检查 D.关节镜检查 |
B.物理学检查 |
2 | 8 | 2010 | 12. 下列哪种情况下,拾物试验可为阳性?(2.0分) | A.先天性髋关节脱位 B.下肢骨折 C.脊柱结核 D.脊柱侧弯 |
C.脊柱结核 |
2 | 8 | 2010 | 13. The Osgood_Schlatter disease is the Osteochondrol disease of the (2.0分) | A.tibial tubercle B.head of the femur C.menisci D.epicondyle of the humerus |
A.tibial tubercle |
2 | 8 | 2010 | 14. Osgood-Schlatter disease is the osteochondrol disease of the (2.0分) | A.tibial tubercle B.head of the femur C.menisci D.epicondyle of the humerus E.hip joint |
A.tibial tubercle |
2 | 8 | 2010 | 15. 14岁男孩,平素喜踢足球,近一周右小腿上端前侧疼痛。查体:右小腿上端前侧隆起,皮肤无红肿发热及静脉怒张,压痛明显。X线片示胫骨结节骨骺增大、致密。应首先考虑的诊断是胫骨结节: (2.0分) | A.撕脱骨折 B.骨髓炎 C.骨结核 D.骨软骨瘤 E.骨软骨病 |
E.骨软骨病 |
2 | 8 | 2010 | 16. 8岁男孩,左髋部肿痛,跛行,伴低热、盗汗、食欲不振3周。查体:体温37.6℃,左髋部活动受限,Thomas征阳性。髋关节x线片见关节间隙略窄,边缘性骨破坏。其诊断首先应考虑为: (2.0分) | A.股骨头坏死 B.髋关节结核 C.急性骨髓炎 D.骨性关节炎 E.急性化脓性关节炎 |
B.髋关节结核 |
2 | 8 | 2010 | 17. 肩周炎的自然病程一般在 ______时间左右能自愈。(2.0分) | A.1月 B.2月 C.3月 D.6月 E.12月 |
E.12月 |
2 | 8 | 2010 | 18. 股骨头坏死的早期诊断较可靠的方法是:(2.0分) | A.X线片检查 B.CT检查 C.B超 D.MR检查 |
D.MR检查 |
2 | 8 | 2010 | 19. 下列Legg-Calve-Perthes病的治疗原则中错的是: (2.0分) | A.应使股骨头完全包容在髋臼内 B.避免髋臼外上缘对股骨头的局限性压应力 C.减轻对股骨头的压力 D.维持髋关节有良好的活动范围 E.加强早期负重锻炼 |
E.加强早期负重锻炼 |
2 | 8 | 2010 | 20. 颈椎病中哪项正确 (2.0分) | A.颈椎病的转归主要取决于颈椎退行性改变 B.颈椎病一般预后良好 C.颈椎病患者均有颈部症状 D.颈椎病在体力劳动者中多见 E.同一节段的颈椎间盘突出也会有不同临床表现 |
E.同一节段的颈椎间盘突出也会有不同临床表现 |
2 | 8 | 2010 | 21. 1神经根型颈椎病患者的临床表现为单侧前臂桡侧、拇指和食指麻木,查体有同侧拇指和食指感觉减退,肱二头肌、桡侧腕长、短伸肌肌力略减退。该患者最有可能的受压神经根是(2.0分) | A.颈3神经根 B.颈4神经根 C.颈5神经根 D.颈6神经根 E.颈7神经根 |
D.颈6神经根 |
2 | 8 | 2010 | 22. 该患者最有可能受累的颈椎间隙为:(2.0分) | A.颈2/3 B.颈3/4 C.颈4/5 D.颈5/6 E.颈6/7 |
D.颈5/6 |
2 | 8 | 2010 | 23. 对该患者需行鉴别诊断的疾患有:1. 腕管综合症 2.运动神经元疾病 3.网球肘 4.腋神经卡压综合症(2.0分) | A.1,2,3 B.1,3 C.2,4 D.1,2,3,4 E.4 |
B.1,3 |
2 | 8 | 2010 | 24. 该患者若神经根型颈椎病的诊断明确,则首选的治疗为:1. 颈椎牵引 2.非甾体类镇痛药 3. 颈椎制动 4. 手术减压(2.0分) | A.1,2,3 B.1,3 C.2,4 D.1,2,3,4 E.4 |
A.1,2,3 |
2 | 8 | 2010 | 25. 一脊髓型颈椎病患者有一侧上下肢轻偏瘫,受累节段以下平面出现对侧痛温觉减退。该患者最有可能的脊髓损害为:(2.0分) | A.脊髓横贯伤综合征 B.运动综合征 C.中央脊髓综合征 D.Brown-Sequard综合征 E.Brachialgia脊髓综合征 |
D.Brown-Sequard综合征 |
2 | 8 | 2010 | 26. 对该患者的最有意义的辅助检查是:(2.0分) | A.X线片 B.过伸过屈动力位颈椎X线片 C.CT检查 D.MRI检查 E.颈脊髓造影 |
D.MRI检查 |
2 | 8 | 2010 | 27. 对该患者需鉴别的疾患有: 1. 椎动脉疾患 2. 椎管内肿瘤 3. 类风湿性关节炎 4.植物神经功能障碍性疾患(2.0分) | A.1,2,3 B.1,3 C.2,4 D.1,2,3,4 E.4 |
C.2,4 |
2 | 8 | 2010 | 28. 请指出背伸型桡骨远端(Colles)骨折那项是错误的 (2.0分) | A.桡骨远端2~3cm以内骨折 B.远折断向背侧近侧移位 C.骨折段向掌侧成角畸形 D.骨折段向背侧成角畸形 E.常伴尺骨茎突骨折 |
D.骨折段向背侧成角畸形 |
2 | 8 | 2010 | 29. 对于25岁男性的正常小腿照片,下列哪项正确?(2.0分) | A.胫骨近端骨骺清晰可见 B.胫骨近端初期钙化带模糊 C.胫骨近端骨性关节面光滑 D.胫骨近端骨骺板宽达0.5cm E.胫骨近端干骺端轮廓清楚 |
C.胫骨近端骨性关节面光滑 |
2 | 8 | 2010 | 30. 良性骨巨细胞瘤的X线所见,选出一项错误的(2.0分) | A.好发于四肢长骨的骨端 B.早期呈偏心性骨质破坏 C.典型者呈皂泡样多房性改变 D.周围可见薄层骨壳形成 E.邻近有针样瘤骨 |
E.邻近有针样瘤骨 |
2 | 8 | 2010 | 31. 行为治疗的目的是: (2.0分) | A.纠正不适应的认知模式 B.改变不适应的行为模式 C.重现童年期的心理冲突 D.发挥个人的潜能 |
B.改变不适应的行为模式 |
2 | 8 | 2010 | 32. 系统脱敏疗法的步骤不包括: (2.0分) | A.放松训练 B.列出等级脱敏表 C.暴露于强烈的焦虑环境中 D.放松状态下脱敏 |
C.暴露于强烈的焦虑环境中 |
2 | 8 | 2010 | 33. 来访者中心疗法不主张: (2.0分) | A.发展来访者主观能动性 B.对来访者明确指导 C.促进来访者成长 D.医生与来访者是平等"角色" |
B.对来访者明确指导 |
2 | 8 | 2010 | 34. 心理治疗的基本过程有: (2.0分) | A.探索问题、心理测验、治疗行动、疗效评价和结束巩固 B.建立关系、分析认识、治疗行动、疗效评价和结束巩固 C.建立关系、探索问题、心理测验、治疗行动和疗效评价 D.探索问题、分析认识、治疗行动、疗效评价和结束巩固 |
D.探索问题、分析认识、治疗行动、疗效评价和结束巩固 |
2 | 8 | 2010 | 35. 支持疗法的理论基础中, 不包括: (2.0分) | A.认知调整 B.善用"资源" C.提高应对能力 D.挖掘幼年体验 |
D.挖掘幼年体验 |
2 | 8 | 2010 | 36. 下列哪个抗体不属于抗ANA系列?(2.0分) | A.抗双链DNA抗体 B.抗SSA抗体 C.抗中性粒细胞胞浆抗体 D.抗Jo-1抗体 E.抗SCL-70抗体 |
C.抗中性粒细胞胞浆抗体 |
2 | 8 | 2010 | 37. 下面哪个不是类风湿关节炎的发病因素 (2.0分) | A.遗传 B.病毒感染 C.肥胖 D.环境因素 E.免疫因素 |
C.肥胖 |
2 | 8 | 2010 | 38. 以下哪个指标对类风湿关节炎诊断特异性最高(2.0分) | A.类风湿因子 B.抗环瓜氨酸多肽抗体 C.抗核抗体 D.抗中性粒细胞胞浆抗体 E.抗核周因子 |
B.抗环瓜氨酸多肽抗体 |
2 | 8 | 2010 | 39. 下列哪个是炎性腰背痛的特点 (2.0分) | A.休息后加重 B.休息后缓解 C.老年发病 D.有一定诱因 E.一定是晨起出现 |
A.休息后加重 |
2 | 8 | 2010 | 40. 患者,女性,69岁,双手2-4近端指间关节及右膝关节痛伴颈肩部疼痛1年,伴晨僵约20分钟,持重及行走下楼均有困难。查体:双腕关节无肿胀,轻度活动受限及压痛,双膝轻度肿胀,活动度可,浮髌征阳性,右侧明显,未及关节摩擦感。查血沉19mm/第1h, CRP1.6mg/dl,RF156U/L(正常值<25U/L), 抗CCP抗体19 IU/L(正常<25U/L),抗核抗体阴性,她最可能的诊断是:(2.0分) | A.骨关节炎 B.类风湿关节炎 C.脊柱关节病 D.痛风性关节炎 E.反应性关节炎 |
A.骨关节炎 |
2 | 8 | 2010 | 41. 上述患者最需要进一步检查:(2.0分) | A.双手X线 B.双手MRI C.抗双链DNA抗体 D.肿瘤标记物 E.骨骼ECT |
A.双手X线 |
2 | 8 | 2010 | 42. 关于风湿性疾病的发病机制,下列说法错误的是(2.0分) | A.多有遗传性因素参与,部分疾病存在疾病的易感基因; B.由于风湿性疾病患者体内往往存在炎症,故该病多需要抗炎抗感染治疗; C.多种细胞因子、蛋白酶及血管内皮细胞参与发病机制,可作为治疗靶点; D.自身抗体往往通过间接作用导致疾病发生,且一种自身抗体可能导致多种非特异性临床症状。 E.以上均不对 |
B.由于风湿性疾病患者体内往往存在炎症,故该病多需要抗炎抗感染治疗; |
2 | 8 | 2010 | 43. 下列哪些不是风湿性疾病的特点: (2.0分) | A.慢性病程,病情迁延反复; B.往往伴有皮肤、黏膜、关节、肌肉及多系统受累; C.往往需要激素和免疫抑制剂治疗; D.往往多系统受累,病情复杂难辨。 E.以上均是; |
E.以上均是; |
2 | 8 | 2010 | 44. 下列哪个疾病不属于弥漫性结缔组织病:(2.0分) | A.系统性血管炎 B.类风湿关节炎 C.系统性红斑狼疮 D.银屑病关节炎 E.皮肌炎 |
D.银屑病关节炎 |
2 | 8 | 2010 | 45. 下面哪种病理变化属于类风湿关节炎的基本病理变化:(2.0分) | A.滑膜炎 B.肌腱端炎 C.软骨退变 D.血管炎 E.软骨炎 |
A.滑膜炎 |
2 | 8 | 2010 | 46. 患者,女性,18岁,间断发热、多关节痛、脱发和泡沫尿1个月,无关节肿,伴有口腔溃疡,当地医院查中度贫血伴有血沉升高,考虑结缔组织病不能除外,确诊需要的最重要的化验检查是:(2.0分) | A.类风湿因子 B.免疫球蛋白 C.抗核抗体系列 D.抗环瓜氨酸多肽抗体 E.补体 |
C.抗核抗体系列 |
2 | 8 | 2010 | 47. 痛风性关节炎与感染性关节炎鉴别的直接有力依据 (2.0分) | A.关节液检查 B.关节X线或双源CT C.血清尿酸水平 D.C反应蛋白 E.血常规 |
A.关节液检查 |
2 | 8 | 2010 | 48. 下列哪个不是系统性红斑狼疮的特点 (2.0分) | A.是一组以多系统受累为主的自身免疫性疾病; B.往往以发热、皮疹、关节痛及肾脏受累为表现; C.典型的病理表现为继发性血管炎; D.也可有关节受累,表现为关节侵蚀; E.以上均不是; |
D.也可有关节受累,表现为关节侵蚀; |
2 | 8 | 2010 | 49. 患者,男性,45岁,左膝关节肿痛3天就诊。目前左膝关节肿胀伴皮温增高,查体:左膝关节肿胀,不伴局部皮肤发红破溃等,浮髌征阳性。下步最合适的诊断手段是:(2.0分) | A.膝关节B超 B.膝关节穿刺液检查 C.膝关节MRI D.膝关节X线 E.膝关节镜检查 |
B.膝关节穿刺液检查 |
2 | 8 | 2010 | 50. Which of the following autoantibodies is most likely to be present in a patient with systemic lupus erythematosus? (2.0分) | A.anti-dsDNA B.anti-RNP C.anti-Ro D.antiphospholipid E.antiribosomal P |
A.anti-dsDNA |
2 | 8 | 2011 | 1. 股骨头坏死的早期诊断较可靠的方法是(3.0分) | A.X线片检查 B.CT检查 C.B超 D.MR检查 |
D.MR检查 |
2 | 8 | 2011 | 2. 下列Legg-Calve-Perthes病的治疗原则中错的是(3.0分) | A.应使股骨头完全包容在髋臼内 B.避免髋臼外上缘对股骨头的局限性压应力 C.减轻对股骨头的压力 D.维持髋关节有良好的活动范围 E.加强早期负重锻炼 |
E.加强早期负重锻炼 |
2 | 8 | 2011 | 3. 颈椎病中哪项正确 (3.0分) | A.颈椎病的转归主要取决于颈椎退行性改变 B.颈椎病一般预后良好 C.颈椎病患者均有颈部症状 D.颈椎病在体力劳动者中多见 E.同一节段的颈椎间盘突出也会有不同临床表现 |
E.同一节段的颈椎间盘突出也会有不同临床表现 |
2 | 8 | 2011 | 4. 1神经根型颈椎病患者的临床表现为单侧前臂桡侧、拇指和食指麻木,查体有同侧拇指和食指感觉减退,肱二头肌、桡侧腕长、短伸肌肌力略减退。该患者最有可能的受压神经根是: (3.0分) | A.颈3神经根 B.颈4神经根 C.颈5神经根 D.颈6神经根 E.颈7神经根 |
D.颈6神经根 |
2 | 8 | 2011 | 5. 该患者最有可能受累的颈椎间隙为(3.0分) | A.颈2/3 B.颈3/4 C.颈4/5 D.颈5/6 E.颈6/7 |
D.颈5/6 |
2 | 8 | 2011 | 6. 对该患者需行鉴别诊断的疾患有1. 腕管综合症 2.运动神经元疾病 3.网球肘 4.腋神经卡压综合症(3.0分) | A.1,2,3 B.1,3 C.2,4 D.1,2,3,4 E.4 |
B.1,3 |
2 | 8 | 2011 | 7. 该患者若神经根型颈椎病的诊断明确,则首选的治疗为1. 颈椎牵引 2.非甾体类镇痛药 3. 颈椎制动 4. 手术减压(3.0分) | A.1,2,3 B.1,3 C.2,4 D.1,2,3,4 E.4 |
A.1,2,3 |
2 | 8 | 2011 | 8. 一脊髓型颈椎病患者有一侧上下肢轻偏瘫,受累节段以下平面出现对侧痛温觉减退。该患者最有可能的脊髓损害为(3.0分) | A.脊髓横贯伤综合征 B.运动综合征 C.中央脊髓综合征 D.Brown-Sequard综合征 E.Brachialgia脊髓综合征 |
D.Brown-Sequard综合征 |
2 | 8 | 2011 | 9. 对该患者的最有意义的辅助检查是(3.0分) | A.X线片 B.过伸过屈动力位颈椎X线片 C.CT检查 D.MRI检查 E.颈脊髓造影 |
D.MRI检查 |
2 | 8 | 2011 | 10. 对该患者需鉴别的疾患有: 1. 椎动脉疾患 2. 椎管内肿瘤 3. 类风湿性关节炎 4.植物神经功能障碍性疾患(3.0分) | A.1,2,3 B.1,3 C.2,4 D.1,2,3,4 E.4 |
C.2,4 |
2 | 8 | 2011 | 11. 请指出背伸型桡骨远端(Colles)骨折那项是错误的(4.0分) | A.桡骨远端2~3cm以内骨折 B.远折断向背侧近侧移位 C.骨折段向掌侧成角畸形 D.骨折段向背侧成角畸形 E.常伴尺骨茎突骨折 |
D.骨折段向背侧成角畸形 |
2 | 8 | 2011 | 12. 对于25岁男性的正常小腿照片,下列哪项正确?(3.0分) | A.胫骨近端骨骺清晰可见 B.胫骨近端初期钙化带模糊 C.胫骨近端骨性关节面光滑 D.胫骨近端骨骺板宽达0.5cm E.胫骨近端干骺端轮廓清楚 |
C.胫骨近端骨性关节面光滑 |
2 | 8 | 2011 | 13. 良性骨巨细胞瘤的X线所见,选出一项错误的(3.0分) | A.好发于四肢长骨的骨端 B.早期呈偏心性骨质破坏 C.典型者呈皂泡样多房性改变 D.周围可见薄层骨壳形成 E.邻近有针样瘤骨 |
E.邻近有针样瘤骨 |
2 | 8 | 2011 | 14. 行为治疗的目的是(3.0分) | A.纠正不适应的认知模式 B.改变不适应的行为模式 C.重现童年期的心理冲突 D.发挥个人的潜能 |
B.改变不适应的行为模式 |
2 | 8 | 2011 | 15. 下列哪个抗体不属于抗ENA系列? (3.0分) | A.抗线粒体抗体 B.抗SSA抗体 C.抗Scl-70抗体 D.抗Jo-1抗体 |
A.抗线粒体抗体 |
2 | 8 | 2011 | 16. 下面哪个不是类风湿关节炎的发病因素(3.0分) | A.遗传 B.病毒感染 C.负重过多 D.环境因素 |
C.负重过多 |
2 | 8 | 2011 | 17. 以下哪个指标对类风湿关节炎诊断特异性最高(3.0分) | A.类风湿因子 B.抗环瓜氨酸多肽抗体 C.抗双链DNA抗体 D.抗中性粒细胞胞浆抗体 |
B.抗环瓜氨酸多肽抗体 |
2 | 8 | 2011 | 18. 下列哪个不是炎性腰背痛的特点(3.0分) | A.休息后加重 B.活动后缓解 C.老年起病 D.晨僵 |
C.老年起病 |
2 | 8 | 2011 | 19. 患者,女性,42岁,双手3-5近端指间关节及右肩关节痛1年,伴晨僵约2小时,持重及行走下楼均有困难。查体:双腕关节略肿胀,伴有轻度活动受限及压痛,双膝无明显肿胀压痛,活动度可,右肩活动受限。查血沉69mm/第1h, CRP 46mg/dl,RF 156U/L(正常值<25U/L), 抗CCP抗体779 IU/L(正常<25U/L),抗核抗体阴性,她最可能的诊断是:(3.0分) | A.骨关节炎 B.类风湿关节炎 C.脊柱关节病 D.痛风性关节炎 |
C.脊柱关节病 |
2 | 8 | 2011 | 20. 上述患者最需要进一步检查(3.0分) | A.双手X线 B. 双手MRI C.抗双链DNA抗体 D.肿瘤标记物 |
A.双手X线 |
2 | 8 | 2011 | 21. 关于风湿性疾病的发病机制,下列说法不正确的是 (3.0分) | A.有遗传相关性,存在疾病的易感基因; B.由于风湿性疾病患者体内往往存在炎症,故该病多需要抗生素治疗; C.多种细胞因子、蛋白酶及血管内皮细胞参与发病机制,可作为治疗靶点; D.自身抗体参与发病过程,一种自身抗体可导致多种临床症状。 |
B.由于风湿性疾病患者体内往往存在炎症,故该病多需要抗生素治疗; |
2 | 8 | 2011 | 22. 下列哪些不是风湿性疾病的特点 (3.0分) | A.慢性病程,病情迁延反复; B.往往伴有皮肤、黏膜、关节、肌肉受累; C.都需要激素和免疫抑制剂治疗; D.往往多系统受累,病情复杂难辨。 |
C.都需要激素和免疫抑制剂治疗; |
2 | 8 | 2011 | 23. 下列哪个疾病不属于弥漫性结缔组织病(3.0分) | A.强直性脊柱炎 B.类风湿关节炎 C.系统性红斑狼疮 D.系统性血管炎 |
A.强直性脊柱炎 |
2 | 8 | 2011 | 24. 下面哪种病理变化属于系统性红斑狼疮的基本病理变化(3.0分) | A.滑膜炎 B.肌腱端炎 C.软骨退变 D.血管炎 |
D.血管炎 |
2 | 8 | 2011 | 25. 患者,女性,28岁,间断发热、多关节痛、脱发和泡沫尿1个月,无关节肿,伴有口腔溃疡,当地医院查中度贫血伴有血沉升高,考虑结缔组织病不能除外,确诊需要的最重要的化验检查是 (3.0分) | A.类风湿因子 B.免疫球蛋白 C. 抗环瓜氨酸多肽抗体 D.抗核抗体系列 |
D.抗核抗体系列 |
2 | 8 | 2011 | 26. 痛风性关节炎与感染性关节炎鉴别的直接有力依据 (3.0分) | A.C反应蛋白 B.关节X线或双源CT C.血清尿酸水平 D.关节液检查 |
D.关节液检查 |
2 | 8 | 2011 | 27. 关于行为治疗,以下的不正确的是(3.0分) | A.以行为学习理论为指导 B.重视发挥个人的潜能 C.有一定的治疗程序 D.着眼于改变不适应的行为模式 |
B.重视发挥个人的潜能 |
2 | 8 | 2011 | 28. 下列不属于行为治疗的心理治疗方法是(3.0分) | A.示范疗法 B.满灌疗法 C.支持疗法 D.厌恶疗法 |
C.支持疗法 |
2 | 8 | 2011 | 29. 下列心理治疗方法中可能产生立竿见影效果的是 (3.0分) | A.精神分析疗法 B.暗示疗法 C.系统脱敏疗法 D.森田疗法 |
B.暗示疗法 |
2 | 8 | 2011 | 30. 精神分析疗法中,以下哪个做法完全不属于潜意识层面 (3.0分) | A.释梦 B.自由联想 C.移情 D.解释 |
D.解释 |
2 | 8 | 2011 | 31. 心理治疗不适用于对以下哪类病人的心理问题进行干预(3.0分) | A.肿瘤病人 B.冠心病病人 C. 焦虑症病人 D.精神分裂症病人 |
D.精神分裂症病人 |
2 | 8 | 2011 | 32. 关于认知治疗,以下不正确的是 (3.0分) | A.关注个体童年的创伤经历 B.A.T.贝克的认知疗法早期主要用于治疗抑郁症 C.A.艾利斯是其代表人物之一 D.重视个体对现实事件的评价 |
A.关注个体童年的创伤经历 |
2 | 8 | 2011 | 33. 来访者中心疗法不主张(3.0分) | A.发展来访者主观能动性 B.对来访者明确指导 C.促进来访者成长 D.治疗是来访者转变和学习的过程 |
B.对来访者明确指导 |
2 | 8 | 2012 | 1. 下列药物中哪一种药物对风湿性关节炎几无疗效? (3.0分) | A.Aspirin B.Celecoxib C.Acetaminophen D.Indomethacin E.Ibuprofen |
C.Acetaminophen |
2 | 8 | 2012 | 2. 对类风湿性关节炎,非甾体抗炎药(NSAIDs)的疗效是(3.0分) | A.对因治疗 B.缩短病程 C.对症治疗 D.阻止肉芽增生及瘢痕形成 E.对风湿性心肌炎可根治 |
C.对症治疗 |
2 | 8 | 2012 | 3. 女性,40岁,双腕、掌指关节近端指间关节肿痛3月余,诊断为类风湿性关节炎。可以控制病情发展,缓解病情治疗的首选药物是 (3.0分) | A.Ibuprofen B.Sulfasalazine C.Prednisone D.D-penicillamine E.Methotrexate |
E.Methotrexate |
2 | 8 | 2012 | 4. 下面哪个不是类风湿关节炎的发病因素 (3.0分) | A.肥胖 B.病毒感染 C.遗传 D.环境因素 |
A.肥胖 |
2 | 8 | 2012 | 5. 下列哪个不是晨僵的特点(3.0分) | A.休息后加重 B.活动后缓解 C.老年起病 D.与负重无关 |
C.老年起病 |
2 | 8 | 2012 | 6. 患者,女性,69岁,双手2-4近端指间关节及右膝关节痛伴颈肩部疼痛1年,伴晨僵约20分钟,持重及行走下楼均有困难。查体:双腕关节无肿胀,轻度活动受限及压痛,双膝轻度肿胀,活动度可,浮髌征阳性,右侧明显,未及关节摩擦感。查血沉19mm/第1h, CRP1.6mg/dl,RF156U/L(正常值<25U/L), 抗CCP抗体19 IU/L(正常<25U/L),抗核抗体阴性,她最可能的诊断是:(3.0分) | A.骨关节炎 B.类风湿关节炎 C.脊柱关节病 D.痛风性关节炎 |
A.骨关节炎 |
2 | 8 | 2012 | 7. 上述患者最需要进一步检查:(4.0分) | A.双手MRI B.双手X线 C.抗双链DNA抗体 D.肿瘤标记物 |
B.双手X线 |
2 | 8 | 2012 | 8. 下列哪些不是风湿性疾病的特点:(3.0分) | A.慢性病程,病情迁延反复; B.往往伴有皮肤、黏膜、关节、肌肉受累; C.往往多系统受累,病情复杂难辨。 D.都需要激素和免疫抑制剂治疗; |
D.都需要激素和免疫抑制剂治疗; |
2 | 8 | 2012 | 9. 下面哪种病理变化属于类风湿关节炎的基本病理变化: (3.0分) | A.肌腱端炎 B.滑膜炎 C.软骨退变 D.血管炎 |
B.滑膜炎 |
2 | 8 | 2012 | 10. 痛风性关节炎与感染性关节炎鉴别的直接有力依据 (3.0分) | A.血清尿酸水平 B.关节X线或双源CT C.关节液检查 D.C反应蛋白 |
C.关节液检查 |
2 | 8 | 2012 | 11. 患者,男性,45岁,左膝关节肿痛3天就诊。目前左膝关节肿胀伴皮温增高,查体:左膝关节肿胀,不伴局部皮肤发红破溃等,浮髌征阳性。下步最合适的诊断手段是:(3.0分) | A.膝关节B超 B.膝关节穿刺液检查 C.膝关节MRI D.膝关节X线 |
B.膝关节穿刺液检查 |
2 | 8 | 2012 | 12. 颈椎病中哪项正确 (3.0分) | A.颈椎病的转归主要取决于颈椎退行性改变 B.颈椎病一般预后良好 C.颈椎病患者均有颈部症状 D.颈椎病在体力劳动者中多见 E.同一节段的颈椎间盘突出也会有不同临床表现 |
E.同一节段的颈椎间盘突出也会有不同临床表现 |
2 | 8 | 2012 | 13. 1神经根型颈椎病患者的临床表现为单侧前臂桡侧、拇指和食指麻木,查体有同侧拇指和食指感觉减退,肱二头肌、桡侧腕长、短伸肌肌力略减退。该患者最有可能的受压神经根是: (3.0分) | A.颈3神经根 B.颈4神经根 C.颈5神经根 D.颈6神经根 E.颈7神经根 |
D.颈6神经根 |
2 | 8 | 2012 | 14. 该患者最有可能受累的颈椎间隙为:(3.0分) | A.颈2/3 B.颈3/4 C.颈4/5 D.颈5/6 E.颈6/7 |
D.颈5/6 |
2 | 8 | 2012 | 15. 对该患者需行鉴别诊断的疾患有: 1. 腕管综合症 2.运动神经元疾病 3.网球肘 4.腋神经卡压综合症(3.0分) | A.1,2,3 B.1,3 C.2,4 D.1,2,3,4 E.4 |
B.1,3 |
2 | 8 | 2012 | 16. 该患者若神经根型颈椎病的诊断明确,则首选的治疗为:1. 颈椎牵引 2.非甾体类镇痛药 3. 颈椎制动 4. 手术减压(3.0分) | A.1,2,3 B.1,3 C.2,4 D.1,2,3,4 E.4 |
A.1,2,3 |
2 | 8 | 2012 | 17. 一脊髓型颈椎病患者有一侧上下肢轻偏瘫,受累节段以下平面出现对侧痛温觉减退。该患者最有可能的脊髓损害为:(3.0分) | A.脊髓横贯伤综合征 B.运动综合征 C.中央脊髓综合征 D.Brown-Sequard综合征 E.Brachialgia脊髓综合征 |
D.Brown-Sequard综合征 |
2 | 8 | 2012 | 18. 对该患者的最有意义的辅助检查是:(3.0分) | A.X线片 B.过伸过屈动力位颈椎X线片 C.CT检查 D.MRI检查 E.颈脊髓造影 |
D.MRI检查 |
2 | 8 | 2012 | 19. 对该患者需鉴别的疾患有:1. 椎动脉疾患 2. 椎管内肿瘤 3. 类风湿性关节炎 4.植物神经功能障碍性疾患(3.0分) | A.1,2,3 B.1,3 C.2,4 D.1,2,3,4 E.4 |
C.2,4 |
2 | 8 | 2012 | 20. MR的优势,以下哪个是错的?(3.0分) | A.显示软组织最好(椎间盘、肿瘤、骨髓、脓肿等); B.直接三轴成像; C.无电离辐射; D.能显示微小钙化和骨片 |
D.能显示微小钙化和骨片 |
2 | 8 | 2012 | 21. 转移性骨肿瘤的X线表现,下列哪项错误?(3.0分) | A.骨质破坏可单发或多发 B.一般无骨膜增生,软组织肿块 C.常伴病理性骨折 D.脊柱,骨盘,肋骨多见 E.椎体破坏,椎间隙变窄 F.软组织肿块 |
E.椎体破坏,椎间隙变窄 |
2 | 8 | 2012 | 22. 良性骨巨细胞瘤的X线所见,选出一项错误的 (3.0分) | A.好发于四肢长骨的骨端 B.早期呈偏心性骨质破坏 C.典型者呈皂泡样多房性改变 D.周围可见薄层骨壳形成 E.邻近有针样瘤骨 |
E.邻近有针样瘤骨 |
2 | 8 | 2012 | 23. 成骨肉瘤的X线表现下列哪些是错误的(3.0分) | A.好发于长骨的骨干 B.不规则的骨质破坏 C.骨膜反应 D.有瘤骨形成 |
A.好发于长骨的骨干 |
2 | 8 | 2012 | 24. 股骨头坏死的早期诊断较可靠的方法是:(3.0分) | A.X线片检查 B.CT检查 C.B超 D.MR检查 |
D.MR检查 |
2 | 8 | 2012 | 25. 下列Legg-Calve-Perthes病的治疗原则中错的是:(3.0分) | A.应使股骨头完全包容在髋臼内 B.避免髋臼外上缘对股骨头的局限性压应力 C.减轻对股骨头的压力 D.维持髋关节有良好的活动范围 E.加强早期负重锻炼 |
E.加强早期负重锻炼 |
2 | 8 | 2012 | 26. 请指出背伸型桡骨远端(Colles)骨折那项是错误的 (3.0分) | A.桡骨远端2~3cm以内骨折 B.远折断向背侧近侧移位 C.骨折段向掌侧成角畸形 D.骨折段向背侧成角畸形 E.常伴尺骨茎突骨折 |
D.骨折段向背侧成角畸形 |
2 | 8 | 2012 | 27. 对于25岁男性的正常小腿照片,下列哪项正确?(3.0分) | A.胫骨近端骨骺清晰可见 B.胫骨近端初期钙化带模糊 C.胫骨近端骨性关节面光滑 D.胫骨近端骨骺板宽达0.5cm E.胫骨近端干骺端轮廓清楚 |
C.胫骨近端骨性关节面光滑 |
2 | 8 | 2012 | 28. The Osgood_Schlatter disease is the Osteochondrol disease of the (3.0分) | A.tibial tubercle B.head of the femur C.menisci D.epicondyle of the humerus |
A.tibial tubercle |
2 | 8 | 2012 | 29. Osgood-Schlatter disease is the osteochondrol disease of the _____.(3.0分) | A.tibial tubercle B.head of the femur C.menisci D.epicondyle of the humerus E.hip joint |
A.tibial tubercle |
2 | 8 | 2012 | 30. 8岁男孩,左髋部肿痛,跛行,伴低热、盗汗、食欲不振3周。查体:体温37.6℃,左髋部活动受限,Thomas征阳性。髋关节x线片见关节间隙略窄,边缘性骨破坏。其诊断首先应考虑为: (3.0分) | A.股骨头坏死 B.髋关节结核 C.急性骨髓炎 D.骨性关节炎 E.急性化脓性关节炎 |
B.髋关节结核 |
2 | 8 | 2012 | 31. 股骨头坏死的早期诊断较可靠的方法是(3.0分) | A.X线片检查 B.CT检查 C.B超 D.MR检查 |
D.MR检查 |
2 | 8 | 2012 | 32. 腕管综合征是______在腕管内受压而表现的一组症状和体征(3.0分) | A.正中神经 B.尺神经 C.桡神经 D.肌皮神经 E.前臂皮神经 |
A.正中神经 |
2 | 8 | 2012 | 33. 下列腕管综合征首先出现的临床表现中错的有:(3.0分) | A.桡侧的二个手指指端麻木或疼痛 B.持物无力 C.夜间或清晨症状最重 D.适当抖动手腕症状可以减轻 E.有时疼痛科牵涉到前臂。 |
A.桡侧的二个手指指端麻木或疼痛 |
2 | 8 | 2013 | 1. Aspirin的不良反应不包括 (3.0分) | A.诱发胃、十二指肠溃疡出血 B.粒细胞减少 C.诱发哮喘 D.高铁血红蛋白血症 E.凝血障碍 |
D.高铁血红蛋白血症 |
2 | 8 | 2013 | 2. 下列治疗类风湿性关节炎的改善病情抗风湿病药物(DMARDs)是 (3.0分) | A.Indomethacin B.Diclofenac C.Chloroquine D.Celecoxib E.以上都是 |
C.Chloroquine |
2 | 8 | 2013 | 3. 下列关于生物反应调节剂(生物制剂)的表述,哪个是错误的? (3.0分) | A.依那西普可以2周内起效 B.结核病患者不能长期使用英夫利昔单抗 C.儿童长期使用阿达木单抗可能会增加患淋巴瘤的风险 D.生物制剂可以和甲氨蝶呤一起口服给药 E.依那西普容易引起过敏反应 |
D.生物制剂可以和甲氨蝶呤一起口服给药 |
2 | 8 | 2013 | 4. 以下哪个指标对类风湿关节炎诊断特异性最高 (3.0分) | A.类风湿因子 B.抗中性粒细胞胞浆抗体 C.抗核抗体 D.抗环瓜氨酸多肽抗体 |
D.抗环瓜氨酸多肽抗体 |
2 | 8 | 2013 | 5. 下列哪个不是系统性红斑狼疮的特点(3.0分) | A.是一组以多系统受累为主的自身免疫性疾病; B.往往以发热、皮疹、关节痛及肾脏受累为表现; C.典型的病理表现为继发性血管炎; D.也可有关节受累,表现为关节侵蚀; |
D.也可有关节受累,表现为关节侵蚀; |
2 | 8 | 2013 | 6. 患者,男,55岁,1天前劳累后出现右踝关节红肿热痛,夜间发作伴晨起行走困难。无病前外伤、扭伤史。2年前右第一跖趾关节曾有类似发作,约3天后自行缓解,未予诊治。该患者最可能的诊断: (3.0分) | A.骨关节炎 B.丹毒 C.痛风性关节炎 D.细菌性关节炎 |
C.痛风性关节炎 |
2 | 8 | 2013 | 7. 患者,女性,69岁,双手2-4近端指间关节及右膝关节痛伴颈肩部疼痛1年,伴晨僵约20分钟,持重及行走下楼均有困难。查体:双腕关节无肿胀,轻度活动受限及压痛,双膝轻度肿胀,活动度可,浮髌征阳性,右侧明显,未及关节摩擦感。查血沉19mm/第1h, CRP1.6mg/dl,RF156U/L(正常值<25U/L), 抗CCP抗体19 IU/L(正常<25U/L),抗核抗体阴性,她最可能的诊断是:(3.0分) | A.骨关节炎 B.类风湿关节炎 C.脊柱关节病 D.痛风性关节炎 |
A.骨关节炎 |
2 | 8 | 2013 | 8. 下列哪些不是风湿性疾病的特点:(3.0分) | A.慢性病程,病情迁延反复; B.往往伴有皮肤、黏膜、关节、肌肉受累; C.往往多系统受累,病情复杂难辨。 D.都需要激素和免疫抑制剂治疗; |
D.都需要激素和免疫抑制剂治疗; |
2 | 8 | 2013 | 9. 下面哪种病理变化属于类风湿关节炎的基本病理变化(3.0分) | A.肌腱端炎 B.滑膜炎 C.软骨退变 D.血管炎 |
B.滑膜炎 |
2 | 8 | 2013 | 10. 痛风性关节炎与感染性关节炎鉴别的直接有力依据(4.0分) | A.血清尿酸水平 B.关节X线或双源CT C.关节液检查 D.C反应蛋白 |
C.关节液检查 |
2 | 8 | 2013 | 11. 患者,男性,45岁,左膝关节肿痛3天就诊。目前左膝关节肿胀伴皮温增高,查体:左膝关节肿胀,不伴局部皮肤发红破溃等,浮髌征阳性。下步最合适的诊断手段是(3.0分) | A.膝关节B超 B.膝关节穿刺液检查 C.膝关节MRI D.膝关节X线 |
B.膝关节穿刺液检查 |
2 | 8 | 2013 | 12. 颈椎病中与预后相关的哪项正确 (3.0分) | A.年龄大于60岁预后较差 B.轻症患者较好,中重度患者较差 C.病程大于6月预后较差 D.椎管椎体矢状径比值小于0.8较差(Torg-Pavlov比值) E.以上都正确 |
E.以上都正确 |
2 | 8 | 2013 | 13. 1神经根型颈椎病患者的临床表现为单侧前臂桡侧、拇指和食指麻木,查体有同侧拇指和食指感觉减退,肱二头肌、桡侧腕长、短伸肌肌力略减退。该患者最有可能的受压神经根是: (3.0分) | A.颈3神经根 B.颈4神经根 C.颈5神经根 D.颈6神经根 E.颈7神经根 |
D.颈6神经根 |
2 | 8 | 2013 | 14. 该患者最有可能受累的颈椎间隙为:(3.0分) | A.颈2/3 B.颈3/4 C.颈4/5 D.颈5/6 E.颈6/7 |
D.颈5/6 |
2 | 8 | 2013 | 15. 对该患者需行鉴别诊断的疾患有1. 腕管综合症 2.运动神经元疾病 3.网球肘 4.腋神经卡压综合症(3.0分) | A.1,2,3 B.1,3 C.2,4 D.1,2,3,4 E.4 |
B.1,3 |
2 | 8 | 2013 | 16. 该患者若神经根型颈椎病的诊断明确,则首选的治疗为1. 颈椎牵引 2.非甾体类镇痛药 3. 颈椎制动 4. 手术减压(3.0分) | A.1,2,3 B.1,3 C.2,4 D.1,2,3,4 E.4 |
A.1,2,3 |
2 | 8 | 2013 | 17. 一脊髓型颈椎病患者有一侧上下肢轻偏瘫,受累节段以下平面出现对侧痛温觉减退。该患者最有可能的脊髓损害为(3.0分) | A.脊髓横贯伤综合征 B.运动综合征 C.中央脊髓综合征 D.Brown-Sequard综合征 E.Brachialgia脊髓综合征 |
D.Brown-Sequard综合征 |
2 | 8 | 2013 | 18. 腰椎间盘突出症的定义下列哪项错误(3.0分) | A.腰椎间盘纤维环退行性改变 B.腰椎间盘髓核退行性改变 C.退变髓核突出是导致临床症状的主要原因 D.该疾患的转归之一是突出的髓核回纳入椎间盘内 E.腰椎间盘突出症突出症常合并有腰椎管狭窄症 |
D.该疾患的转归之一是突出的髓核回纳入椎间盘内 |
2 | 8 | 2013 | 19. 神经根型颈椎病的手术治疗适应症(3.0分) | A.持续性或反复发作性的上肢疼痛,经一段时间保守治疗无效(6~12周) B.神经功能障碍进行性加重 C.神经功能障碍静止但神经根放射性疼痛加重 D.影像学检查结果与临床症状一致 E.以上都正确 |
E.以上都正确 |
2 | 8 | 2013 | 20. 一定单位体积内骨组织有机成分正常,而钙盐含量减少,见于(3.0分) | A.骨质破坏 B.骨质疏松 C.骨质软化 D.骨质关节进行性变 E.骨质增生硬化 |
C.骨质软化 |
2 | 8 | 2013 | 21. 转移性骨肿瘤的X线表现,下列哪项错误? (3.0分) | A.骨质破坏可单发或多发 B.一般无骨膜增生,软组织肿块 C.常伴病理性骨折 D.脊柱,骨盘,肋骨多见 E.椎体破坏,椎间隙变窄 |
E.椎体破坏,椎间隙变窄 |
2 | 8 | 2013 | 22. 良性骨巨细胞瘤的X线所见,选出一项错误的(3.0分) | A.好发于四肢长骨的骨端 B.早期呈偏心性骨质破坏 C.典型者呈皂泡样多房性改变 D.周围可见薄层骨壳形成 E.邻近有针样瘤骨 |
E.邻近有针样瘤骨 |
2 | 8 | 2013 | 23. 骨肉瘤的X线表现下列哪些是错误的(3.0分) | A.好发于长骨的骨干 B.不规则的骨质破坏 C.骨膜反应 D.有瘤骨形成 E.软组织肿块 |
A.好发于长骨的骨干 |
2 | 8 | 2013 | 24. 股骨头坏死的早期诊断较可靠的方法是(3.0分) | A.X线片检查 B.CT检查 C.B超 D.MR检查 |
D.MR检查 |
2 | 8 | 2013 | 25. 下列Legg-Calve-Perthes病的治疗原则中错的是(3.0分) | A.应使股骨头完全包容在髋臼内 B.避免髋臼外上缘对股骨头的局限性压应力 C.减轻对股骨头的压力 D.维持髋关节有良好的活动范围 E.加强早期负重锻炼 |
E.加强早期负重锻炼 |
2 | 8 | 2013 | 26. 请指出背伸型桡骨远端(Colles)骨折那项是错误的(3.0分) | A.桡骨远端2~3cm以内骨折 B.远折断向背侧近侧移位 C.骨折段向掌侧成角畸形 D.骨折段向背侧成角畸形 E.常伴尺骨茎突骨折 |
D.骨折段向背侧成角畸形 |
2 | 8 | 2013 | 27. 对于25岁男性的正常小腿照片,下列哪项正确? (3.0分) | A.胫骨近端骨骺清晰可见 B.胫骨近端初期钙化带模糊 C.胫骨近端骨性关节面光滑 D.胫骨近端骨骺板宽达0.5cm E.胫骨近端干骺端轮廓清楚 |
C.胫骨近端骨性关节面光滑 |
2 | 8 | 2013 | 28. The Osgood_Schlatter disease is the Osteochondrol disease of the(3.0分) | A.tibial tubercle B.head of the femur C.menisci D.epicondyle of the humerus |
A.tibial tubercle |
2 | 8 | 2013 | 29. Osgood-Schlatter disease is the osteochondrol disease of the _____.(3.0分) | A.tibial tubercle B.head of the femur C.menisci D.epicondyle of the humerus E.hip joint |
A.tibial tubercle |
2 | 8 | 2013 | 30. 肩周炎的自然病程一般在E______时间左右能自愈(3.0分) | A.1月 B.2月 C.3月 D.6月 E.12月 |
E.12月 |
2 | 8 | 2013 | 31. 股骨头坏死的早期诊断较可靠的方法是(3.0分) | A.X线片检查 B.CT检查 C.B超 D.MR检查 |
D.MR检查 |
2 | 8 | 2013 | 32. 腕管综合征是______在腕管内受压而表现的一组症状和体征(3.0分) | A.正中神经 B.尺神经 C.桡神经 D.肌皮神经 E.前臂皮神经 |
A.正中神经 |
2 | 8 | 2013 | 33. 下列Legg-Calve-Perthes病的治疗原则中错的是(3.0分) | A.应使股骨头完全包容在髋臼内 B.避免髋臼外上缘对股骨头的局限性压应力 C.减轻对股骨头的压力 D.维持髋关节有良好的活动范围 E.加强早期负重锻炼 |
E.加强早期负重锻炼 |
2 | 8 | 2014 | 1. 下列药物中哪一种药物对风湿性关节炎几无疗效? (2.5分) | A.Aspirin B.Celecoxib C.Acetaminophen D.Indomethacin E.Ibuprofen |
C.Acetaminophen |
2 | 8 | 2014 | 2. 对类风湿性关节炎,非甾体抗炎药(NSAIDs)的疗效是 (2.5分) | A.对因治疗 B.缩短病程 C.对症治疗 D.阻止肉芽增生及瘢痕形成 E.对风湿性心肌炎可根治 |
C.对症治疗 |
2 | 8 | 2014 | 3. 患者女性,40岁,双腕、掌指关节近端指间关节肿痛3月余,诊断为类风湿性关节炎。可以控制病情发展,缓解病情治疗的首选药物是 (2.5分) | A.Ibuprofen B.Sulfasalazine C.Prednisone D.D-penicillamine E.Methotrexate |
E.Methotrexate |
2 | 8 | 2014 | 4. 下列关于风湿性疾病治疗的描述,正确的是? (2.5分) | A.非甾体类抗炎药可以有效控制炎症,延缓疾病进展 B.布洛芬可用于缓解关节炎的疼痛,不影响血小板功能 C.阿司匹林对COX的抑制是不可逆的 D.对乙酰氨基酚安全性高,可长期用于治疗各种慢性疼痛 E.COX2的抑制剂容易引起胃肠道损害 |
C.阿司匹林对COX的抑制是不可逆的 |
2 | 8 | 2014 | 5. 下列哪个抗体不属于ANA系列? (2.5分) | A.抗双链DNA抗体 B.抗SSA抗体 C.抗ENA抗体 D.抗CCP抗体 |
D.抗CCP抗体 |
2 | 8 | 2014 | 6. 患者,女性,49岁,双手3-4近端指间关节及右踝关节痛伴颈肩部肿痛1年,伴晨僵约80分钟,持重及行走下楼均有困难。查体:双腕关节无肿胀,轻度活动受限及压痛,双膝轻度肿胀,活动度可,浮髌征阳性,右侧明显,未及关节摩擦感。查血沉39mm/第1h(正常值<20mm/h), CRP 20.6mg/L(正常值<10mg/L),RF 156U/L(正常值<25U/L), 抗CCP抗体19 IU/L(正常<25U/L),抗核抗体阴性,她最可能的诊断是(2.5分) | A.骨关节炎 B.类风湿关节炎 C.脊柱关节病 D.痛风性关节炎 |
B.类风湿关节炎 |
2 | 8 | 2014 | 7. 关于风湿性疾病的发病机制,下列说法错误的是 (2.5分) | A.多有遗传性因素参与,部分疾病存在疾病的易感基因; B.由于风湿性疾病患者体内往往存在炎症,故该病多需要抗炎抗感染治疗; C.多种细胞因子、蛋白酶及血管内皮细胞参与发病机制,可作为治疗靶点; D.自身抗体往往通过间接作用导致疾病发生,且一种自身抗体可能导致多种非特异性临床症状。 |
B.由于风湿性疾病患者体内往往存在炎症,故该病多需要抗炎抗感染治疗; |
2 | 8 | 2014 | 8. 患者,男性,45岁,左膝关节肿痛3天就诊。目前左膝关节肿胀伴皮温增高,查体:左膝关节肿胀,不伴局部皮肤发红破溃等,浮髌征阳性。下步最合适的辅助检查是:(2.5分) | A.膝关节B超 B.膝关节穿刺液检查 C.膝关节MRI D.膝关节X线 |
B.膝关节穿刺液检查 |
2 | 8 | 2014 | 9. 下列哪些不是风湿性疾病的特点(2.5分) | A.慢性病程,病情迁延反复; B.往往伴有皮肤、黏膜、关节、肌肉受累; C.往往多系统受累,病情复杂难辨。 D.都需要激素和免疫抑制剂治疗; |
D.都需要激素和免疫抑制剂治疗; |
2 | 8 | 2014 | 10. 下面哪种病理变化属于类风湿关节炎的基本病理变化:(2.5分) | A.肌腱端炎 B.滑膜炎 C.软骨退变 D.血管炎 |
B.滑膜炎 |
2 | 8 | 2014 | 11. 痛风性关节炎与感染性关节炎鉴别的直接有力依据(2.5分) | A.血清尿酸水平 B.关节X线或双源CT C.关节液检查 D.C反应蛋白 |
C.关节液检查 |
2 | 8 | 2014 | 12. 1神经根型颈椎病患者的临床表现为单侧前臂桡侧、拇指和食指麻木,查体有同侧拇指和食指感觉减退,肱二头肌、桡侧腕长、短伸肌肌力略减退。该患者最有可能的受压神经根是: (2.5分) | A.颈3神经根 B.颈4神经根 C.颈5神经根 D.颈6神经根 E.颈7神经根 |
D.颈6神经根 |
2 | 8 | 2014 | 13. 该患者最有可能受累的颈椎间隙为: (2.5分) | A.颈2/3 B.颈3/4 C.颈4/5 D.颈5/6 E.颈6/7 |
D.颈5/6 |
2 | 8 | 2014 | 14. 对该患者需行鉴别诊断的疾患有:1. 腕管综合症 2.运动神经元疾病 3.网球肘 4.腋神经卡压综合症(2.5分) | A.1,2,3 B.1,3 C.2,4 D.1,2,3,4 E.4 |
B.1,3 |
2 | 8 | 2014 | 15. 该患者若神经根型颈椎病的诊断明确,则首选的治疗为:1. 颈椎牵引 2.非甾体类镇痛药 3. 颈椎制动 4. 手术减压(2.5分) | A.1,2,3 B.1,3 C.2,4 D.1,2,3,4 E.4 |
A.1,2,3 |
2 | 8 | 2014 | 16. 腰右腿痛1月,加剧1周。体检所见:右侧小腿前外侧痛觉略减、同侧足背伸趾、伸拇肌肌力稍有下降,双下肢反射无异常。该患者可能的诊断是 (2.5分) | A.腰3/4椎间盘突出症 B.腰3/4极外侧椎间盘突出症 C.腰4/5椎间盘突出症 D.腰4/5极外侧型椎间盘突出症 E.腰5骶1椎间盘突出症 |
C.腰4/5椎间盘突出症 |
2 | 8 | 2014 | 17. 该患者进一步首选的辅助检查为:1肌电图检查 2腰椎CT检查 3腰椎X线正侧位片 4腰椎MRI检查(2.5分) | A.1,2,3 B.1,3 C.2,4 D.1,2,3,4 E.4 |
E.4 |
2 | 8 | 2014 | 18. 该患者需要鉴别的疾患有: 1腰椎管狭窄症 2椎管内肿瘤 3腰椎结核 4腰椎肿瘤(2.5分) | A.1,2,3 B.1,3 C.2,4 D.1,2,3,4 E.4 |
D.1,2,3,4 |
2 | 8 | 2014 | 19. 该患者可选择的治疗:1 卧硬板床休息 2非甾体类镇痛药 3开窗髓核摘除 4椎管内注射治疗(2.5分) | A.1,2,3 B.1,3 C.2,4 D.1,2,3,4 E.4 |
D.1,2,3,4 |
2 | 8 | 2014 | 20. 一定单位体积内骨组织有机成分正常,而钙盐含量减少,见于 (2.5分) | A.骨质破坏 B.骨质疏松 C.骨质软化 D.骨质关节进行性变 E.骨质增生硬化 |
C.骨质软化 |
2 | 8 | 2014 | 21. 转移性骨肿瘤的X线表现,下列哪项错误? (2.5分) | A.骨质破坏可单发或多发 B.一般无骨膜增生,软组织肿块 C.常伴病理性骨折 D.脊柱,骨盘,肋骨多见 E.椎体破坏,椎间隙变窄 |
E.椎体破坏,椎间隙变窄 |
2 | 8 | 2014 | 22. 良性骨巨细胞瘤的X线所见,选出一项错误的 (2.5分) | A.好发于四肢长骨的骨端 B.早期呈偏心性骨质破坏 C.典型者呈皂泡样多房性改变 D.周围可见薄层骨壳形成 E.邻近有针样瘤骨 |
E.邻近有针样瘤骨 |
2 | 8 | 2014 | 23. 骨肉瘤的X线表现下列哪些是错误的 (2.5分) | A.好发于长骨的骨干 B.不规则的骨质破坏 C.骨膜反应 D.有瘤骨形成 E.软组织肿块 |
A.好发于长骨的骨干 |
2 | 8 | 2014 | 24. 股骨头坏死的早期诊断较可靠的方法是(2.5分) | A.X线片检查 B.CT检查 C.B超 D.MR检查 |
D.MR检查 |
2 | 8 | 2014 | 25. 下列Legg-Calve-Perthes病的治疗原则中错的是(2.5分) | A.应使股骨头完全包容在髋臼内 B.避免髋臼外上缘对股骨头的局限性压应力 C.减轻对股骨头的压力 D.维持髋关节有良好的活动范围 E.加强早期负重锻炼 |
E.加强早期负重锻炼 |
2 | 8 | 2014 | 26. 请指出背伸型桡骨远端(Colles)骨折那项是错误的(2.5分) | A.桡骨远端2~3cm以内骨折 B.远折断向背侧近侧移位 C.骨折段向掌侧成角畸形 D.骨折段向背侧成角畸形 E.常伴尺骨茎突骨折 |
D.骨折段向背侧成角畸形 |
2 | 8 | 2014 | 27. 对于25岁男性的正常小腿照片,下列哪项正确? (2.5分) | A.胫骨近端骨骺清晰可见 B.胫骨近端初期钙化带模糊 C.胫骨近端骨性关节面光滑 D.胫骨近端骨骺板宽达0.5cm E.胫骨近端干骺端轮廓清楚 |
C.胫骨近端骨性关节面光滑 |
2 | 8 | 2014 | 28. The Osgood_Schlatter disease is the Osteochondrol disease of the (2.5分) | A.tibial tubercle B.head of the femur C.menisci D.epicondyle of the humerus |
A.tibial tubercle |
2 | 8 | 2014 | 29. Osgood-Schlatter disease is the osteochondrol disease of the _____.(2.5分) | A.tibial tubercle B.head of the femur C.menisci D.epicondyle of the humerus E.hip joint |
A.tibial tubercle |
2 | 8 | 2014 | 30. 肩周炎的自然病程一般在E______时间左右能自愈(2.5分) | A.1月 B.2月 C.3月 D.6月 E.12月 |
E.12月 |
2 | 8 | 2014 | 31. 股骨头坏死的早期诊断较可靠的方法是(2.5分) | A.X线片检查 B.CT检查 C.B超 D.MR检查 |
D.MR检查 |
2 | 8 | 2014 | 32. 腕管综合征是______在腕管内受压而表现的一组症状和体征。(2.5分) | A.正中神经 B.尺神经 C.桡神经 D.肌皮神经 E.前臂皮神经 |
A.正中神经 |
2 | 8 | 2014 | 33. 下列Legg-Calve-Perthes病的治疗原则中错的是(2.5分) | A.应使股骨头完全包容在髋臼内 B.避免髋臼外上缘对股骨头的局限性压应力 C.减轻对股骨头的压力 D.维持髋关节有良好的活动范围 E.加强早期负重锻炼 |
E.加强早期负重锻炼 |
2 | 8 | 2014 | 34. 急性骨髓炎的诊断金标准: (2.5分) | A.化验示血象升高 B.分层穿刺细菌培养 C.X线示骨破坏、分层骨膜增生 D.局部肿胀、压痛、皮温升高 |
B.分层穿刺细菌培养 |
2 | 8 | 2014 | 35. 急性骨髓炎发病两周内 的x线表现下列那一项不常见?(2.5分) | A.皮下脂肪层内出现致密的条纹影 B.皮下组织与肌间的分界模糊 C.肌间隙模糊或消失 D.局限性骨质疏松 |
D.局限性骨质疏松 |
2 | 8 | 2014 | 36. 骨质软化见于下列哪一种情况?(2.5分) | A.佝偻病 B.化脓性骨髓炎 C.骨结核 D.老年性骨质疏松 |
A.佝偻病 |
2 | 8 | 2014 | 37. 软组织肿胀见于下列何种疾病?(2.5分) | A.骨关节感染 B.骨恶性肿瘤 C.良性肿瘤 D.转移性肿瘤 |
A.骨关节感染 |
2 | 8 | 2014 | 38. 下列哪个属于骨肿瘤样病变? (2.5分) | A.骨囊肿 B.骨气鼓 C.慢性骨脓肿 D.巨细胞瘤 |
B.骨气鼓 |
2 | 8 | 2014 | 39. 骨气鼓为下列哪种骨病的特征性表现 (2.5分) | A.慢性骨脓肿 B.骨结核 C.骨囊肿 D.软骨瘤 |
B.骨结核 |
2 | 8 | 2014 | 40. 下列关于桡骨远端(柯氏)骨折哪项是错误的 (2.5分) | A.桡骨远端3cm以内骨折 B.远折段向背侧及桡侧移位 C.远折段向掌侧成角畸形 D.远折段向背侧成角畸形 E.常伴尺骨茎突骨折 |
D.远折段向背侧成角畸形 |
2 | 8 | 2015 | 1. 颈椎病中哪项正确 1.颈椎病的转归主要取决于颈椎退行性改变 2.神经根型颈椎病一般预后良好3.脊髓型颈椎病一般预后良好 4.同一节段的颈椎间盘突出也会有不同临床表现(2.5分) | A.1,2,3 B.1,3 C.2,4 D.1,2,3,4 E.4 |
C.2,4 |
2 | 8 | 2015 | 2. 1神经根型颈椎病患者的临床表现为单侧前臂外侧,拇指和示指的疼痛麻木,检查显示相应区域的感觉减退,同侧肱二头肌、桡侧腕长、短伸肌肌力减退及同侧肱二头肌、肱桡肌反射减弱,该患者最有可能的受压神经根是:(2.5分) | A.颈3神经根 B.颈4神经根 C.颈5神经根 D.颈6神经根 E.颈7神经根 |
D.颈6神经根 |
2 | 8 | 2015 | 3. 该患者最有可能受累的颈椎间隙为(2.5分) | A.颈2/3 B.颈3/4 C.颈4/5 D.颈5/6 E.颈6/7 |
D.颈5/6 |
2 | 8 | 2015 | 4. 1神经根型颈椎病患者的临床表现为单侧中指感觉异常,查体有(2.5分) | A.颈3神经根 B.颈4神经根 C.颈5神经根 D.颈6神经根 E.颈7神经根 |
E.颈7神经根 |
2 | 8 | 2015 | 5. 该患者最有可能受累的颈椎间隙为(2.5分) | A.颈2/3 B.颈3/4 C.颈4/5 D.颈5/6 E.颈6/7 |
E.颈6/7 |
2 | 8 | 2015 | 6. 对该患者需行鉴别诊断的疾患有1.腕管综合症 2.运动神经元疾病 3.腋神经卡压综合症 4.尺管综合症(2.5分) | A.1,2,3 B.1,3 C.2,4 D.1,2,3,4 E.4 |
C.2,4 |
2 | 8 | 2015 | 7. 神经根型颈椎病诊断明确,则首选的治疗为 1.颈椎牵引 2.非甾体类镇痛药 3.颈椎制动 4.手术减压(2.5分) | A.1,2,3 B.1,3 C.2,4 D.1,2,3,4 E.4 |
A.1,2,3 |
2 | 8 | 2015 | 8. 1脊髓型颈椎病患者有明显双侧上肢感觉异常,检查显示双侧上肢感觉减退区,双上肢肌力可,双下肢感觉,肌力无明显减退。该患者最有可能的脊髓损害为:(2.5分) | A.脊髓横贯伤综合征 B.运动综合征 C.中央脊髓综合征 D.Brown-Sequard综合征 E.Brachialgia脊髓综合征 |
C.中央脊髓综合征 |
2 | 8 | 2015 | 9. 对该患者的最有意义的辅助检查是(2.5分) | A.X线片 B.过伸过屈动力位颈椎X线片 C.CT检查 D.MRI检查 E.颈脊髓造影 |
D.MRI检查 |
2 | 8 | 2015 | 10. 对该患者需鉴别的疾患有 1. 椎动脉疾患 2. 椎管内肿瘤 3. 类风湿性关节炎 4.植物神经功能障碍性疾患(2.5分) | A.1,2,3 B.1,3 C.2,4 D.1,2,3,4 E.4 |
C.2,4 |
2 | 8 | 2015 | 11. 桡骨茎突腱鞘炎的特有体征是(2.5分) | A.Finkelstein征 B.Mills征 C.Dugas征 D.Tinel征 E.Phalen征 |
A.Finkelstein征 |
2 | 8 | 2015 | 12. "网球肘"的特有体征是(2.5分) | A.Finkelstein征 B.Mills征 C. Dugas征 D.Tinel征 E. Phalen征 |
B.Mills征 |
2 | 8 | 2015 | 13. 腕管综合征主要累及的神经是(2.5分) | A.桡神经 B.正中神经 C.尺神经 D.前臂外侧皮神经 E.前臂内侧皮神经 |
B.正中神经 |
2 | 8 | 2015 | 14. 患者30岁男性,某IT企业职员,近期出现右手拇指、示指、环指桡侧半麻木,拇指对掌无力,其诊断首先考虑(2.5分) | A.腕管综合征 B.桡骨茎突腱鞘炎 C.腱鞘囊肿 D.肱骨外上髁炎 E.指屈肌腱狭窄性腱鞘炎 |
A.腕管综合征 |
2 | 8 | 2015 | 15. 患者52岁男性,体力劳动者,3月前出现右肘外侧疼痛,拧毛巾时疼痛明显加重,保守治疗效果欠佳,目前其首选治疗方案是(2.5分) | A.口服NSAIDs治疗 B.激素局部注射 C.手术治疗 D.石膏固定制动 E.加强肘关节活动 |
B.激素局部注射 |
2 | 8 | 2015 | 16. 下列哪一个不是肩袖的组成部分(2.5分) | A.肩胛下肌 B.冈上肌 C.冈下肌 D.大圆肌 E.小圆肌 |
D.大圆肌 |
2 | 8 | 2015 | 17. 临床疑有肩袖损伤时,首选影像学检查方法是(2.5分) | A.CT检查 B.X线平片 C.B超检查 D.MRI检查 E.PET检查 |
D.MRI检查 |
2 | 8 | 2015 | 18. 冻结肩好发于哪个年龄段(2.5分) | A.30岁左右 B.40岁左右 C.50岁左右 D.60岁左右 E.70岁左右 |
C.50岁左右 |
2 | 8 | 2015 | 19. 肩周炎一般多长时间可以自愈(2.5分) | A.3月 B.6月 C.12月 D.18月 E.24月 |
D.18月 |
2 | 8 | 2015 | 20. 以下最容易引起骨坏死的骨折类型是(2.5分) | A.Colles骨折 B.股骨颈骨折 C.肱骨髁上骨折 D.胫腓骨骨折 E.脊柱骨折 |
B.股骨颈骨折 |
2 | 8 | 2015 | 21. 急性骨髓炎转为慢性骨髓炎的主要原因是(2.5分) | A.机体抵抗力低 B.细菌毒力过于强大 C.治疗不及时和不恰当 D.局部血运不好 E.肢体活动过早 |
C.治疗不及时和不恰当 |
2 | 8 | 2015 | 22. 急性血源性骨髓炎最常见的致病菌是(2.5分) | A.白色葡萄球菌 B.霉菌 C.金黄色葡萄球菌 D.大肠杆菌 E.肺炎双球菌 |
C.金黄色葡萄球菌 |
2 | 8 | 2015 | 23. 急性骨髓炎,在骨膜下或骨髓内抽得脓液后,最关键的治疗措施是(2.5分) | A.多次抽脓并注入抗生素 B.进行脓液细菌培养及药敏试验,据结果调整用药 C.联合使用大量抗生素 D.局部引流 E.局部固定防止病理性骨折 |
D.局部引流 |
2 | 8 | 2015 | 24. 慢性骨髓炎死骨摘除术指征,是(2.5分) | A.有死骨、死腔骨包壳薄弱 B.发热,局部红肿,有死骨、死腔 C.开放性骨折感染,骨折尚未愈合,有大块死骨 D.骨包壳充分形成,有死骨、死腔 |
D.骨包壳充分形成,有死骨、死腔 |
2 | 8 | 2015 | 25. 化脓性关节炎经穿刺及关节内注入抗生素治疗后,未能控制症状时,应当(2.5分) | A.调整抗生素种类 B.调整并加大抗生素全身用量 C.增加关节穿刺次数 D.增加关节内抗生素用量 E.切开引流 |
E.切开引流 |
2 | 8 | 2015 | 26. 伴有膝关节畸形、伸屈活动明显受限的骨性关节炎最佳治疗方案是(2.5分) | A.关节镜微创技术 B.局部封闭止痛 C.关节置换术 D.保守治疗。 |
C.关节置换术 |
2 | 8 | 2015 | 27. 骨关节炎最常累及的关节是(2.5分) | A.腕关节,踝关节,远端指间关节 B.膝关节,肩关节,近端指间关节 C.腕关节,肘关节,近端指间关节 D.膝关节,髋关节,远端指间关节 E.掌指关节,远端指间关节,近端指间关节 |
D.膝关节,髋关节,远端指间关节 |
2 | 8 | 2015 | 28. 骨性关节炎最基本的病理改变是(2.5分) | A.关节软骨微小裂隙 B.软骨下骨板裸露 C.软骨变性 D.关节鼠 E.关节软骨局灶性软化 |
C.软骨变性 |
2 | 8 | 2015 | 29. 下述哪一项表现不符合骨关节炎的特点(2.5分) | A.表现为关节疼痛、骨性膨大 B.慢性起病,进展缓慢 C.膝关节在活动时有骨擦感 D.休息时关节疼痛明显,活动时疼痛减轻 E.晨僵可达20~30分钟 |
D.休息时关节疼痛明显,活动时疼痛减轻 |
2 | 8 | 2015 | 30. 关于骨关节炎的临床表现描述错误的是(2.5分) | A.多见于老年女性 B.多累及少数关节 C.最常受累的是膝、髋关节 D.指间关节骨关节炎多为原发性 |
A.多见于老年女性 |
2 | 8 | 2015 | 31. 下列药物中哪一种药物对几无抗炎作用?(2.5分) | A.Aspirin B.Celecoxib C.Acetaminophen D.Indomethacin E.Ibuprofen |
C.Acetaminophen |
2 | 8 | 2015 | 32. Aspirin的不良反应不包括(2.5分) | A.诱发胃、十二指肠溃疡出血 B.粒细胞减少 C.诱发哮喘 D.高铁血红蛋白血症 E.凝血障碍 |
D.高铁血红蛋白血症 |
2 | 8 | 2015 | 33. 对类风湿性关节炎,非甾体抗炎药(NSAIDs)的疗效是(2.5分) | A.对因治疗 B.缩短病程 C.对症治疗 D.阻止肉芽增生及瘢痕形成 E.对风湿性心肌炎可根治 |
C.对症治疗 |
2 | 8 | 2015 | 34. 下列治疗类风湿性关节炎的改善病情抗风湿病药物(DMARDs)是(2.5分) | A.Indomethacin B.Diclofenac C.Chloroquine D.Celecoxib E.Corticosteroids |
C.Chloroquine |
2 | 8 | 2015 | 35. 患者女性,40岁,双腕、掌指关节近端指间关节肿痛3月余,诊断为类风湿性关节炎。可以控制病情发展,缓解病情治疗的首选药物是(2.5分) | A.Ibuprofen B.Sulfasalazine C.Prednisone D.D-penicillamine E.Methotrexate |
E.Methotrexate |
2 | 8 | 2015 | 36. 下面哪个不是类风湿关节炎的发病因素 (2.5分) | A.受湿 B.病毒感染 C.遗传 D.环境因素 E.吸烟 |
A.受湿 |
2 | 8 | 2015 | 37. 以下哪个指标对类风湿关节炎诊断特异性最高(2.5分) | A.类风湿因子 B.抗中性粒细胞胞浆抗体 C.抗核抗体 D.抗磷脂抗体 E.抗环瓜氨酸多肽抗体 |
E.抗环瓜氨酸多肽抗体 |
2 | 8 | 2015 | 38. 下列哪个不是晨僵的特点 (2.5分) | A.休息后加重 B.活动后缓解 C.老年起病 D.与负重无关 E.久坐后加重 |
C.老年起病 |
2 | 8 | 2015 | 39. 下列哪个疾病不属于弥漫性结缔组织病(2.5分) | A.系统性血管炎 B.类风湿关节炎 C.系统性红斑狼疮 D.银屑病关节炎 E.系统性硬化 |
D.银屑病关节炎 |
2 | 8 | 2015 | 40. 下面哪种病理变化属于类风湿关节炎的基本病理变化(2.5分) | A.肌腱端炎 B.滑膜炎 C.软骨退变 D.血管炎 E.附着点炎 |
B.滑膜炎 |